You are on page 1of 321

G.R. No.

18081 March 3, 1922


IN THE MATTER OF THE ESTATE OF CHEONG BOO, deceased.
MORA ADONG, petitioner-appellant,
vs.
CHEONG SENG GEE, opponent-appellant.
Kincaid, Perkins & Kincaid and P. J. Moore for petitioner-appellant.
Carlos A. Sobral for opponent-appellant.
MALCOLM, J .:
The two question presented for determination by these appeals may be framed as follows: Is a marriage
contracted in China and proven mainly by an alleged matrimonial letter, valid in the Philippines? Are the
marriage performed in the Philippines according to the rites of the Mohammedan religion valid? As the
decision of the Supreme Court on the last point will affect marriages consummated by not less than one
hundred and fifty thousand Moros who profess the Mohammedan faith, the transcendental importance of
the cause can be realized. We proposed to give to the subject the serious consideration which it
deserves.
Cheong Boo, a native of China, died intestate in Zamboanga, Philippine Islands, on August 5, 1919. He
left property worth nearly P100,000. The estate of the deceased was claimed, on the one hand, by
Cheong Seng Gee, who alleged that he was a legitimate child by a marriage contracted by Cheong Boo
with Tan Dit in China in 1895. The estate was claimed, on the other hand, by the Mora Adong who
alleged that she had been lawfully married to Cheong Boo in 1896 in Basilan, Philippine Islands, and her
daughters, Payang, married to Cheng Bian Chay, and Rosalia Cheong Boo, unmarried.
The conflicting claims to the estate of Cheong Boo were ventilated in the Court of First Instance of
Zamboanga. The trial judge, the Honorable Quirico Abeto, after hearing the evidence presented by both
sides, reached the conclusion, with reference to the allegations of Cheong Seng Gee, that the proof did
not sufficiently establish the Chinese marriage, but that because Cheong Seng Gee had been admitted to
the Philippine Islands as the son of the deceased, he should share in the estate as a natural child. With
reference to the allegations of the Mora Adong and her daughters Payang and Rosalia, the trial judge
reached the conclusion that the marriage between the Mora Adong and the deceased had been
adequately proved but that under the laws of the Philippine Islands it could not be held to be a lawful
marriage; accordingly, the daughters Payang and Rosalia would inherit as natural children. The order of
the trial judge, following these conclusions, was that there should be a partition of the property of the
deceased Cheong Boo between the natural children, Cheong Seng Gee, Payang, and Rosalia.
From the judgment of the Judge of First Instance both parties perfected appeals. As to the facts, we can
say that we agree in substance with the findings of the trial court. As to the legal issues submitted for
decision by the numerous assignments of error, these can best be resolved under two heads, namely: (1)
The validity of the Chinese marriage; and (2) the validity of the Mohammedan marriage.
1. Validity of the Chinese Marriage
The theory advanced on behalf of the claimant Cheong Seng Gee was that Cheong Boo was married in
the city of Amoy, China, during the second moon of the twenty-first year of the Emperor Quang Su, or,
according to the modern count, on February 16, 1985, to a young lady named Tan Dit. Witnesses were
presented who testified to having been present at the marriage ceremony. There was also introduced in
evidence a document in Chinese which in translation reads as follows:
One hundred
years of life and
health for both.
Your nephew, Tan Chao, respecfully
answers the venerable Chiong Ing, father
of the bridegroom, accepting his offer of
marriage, and let this document serve as
proof of the acceptance of said marriage
which is to be celebrated during the merry
season of the flowers.
I take advantage of this occasion to wish
for your and the spouses much happiness,
a long life, and prolific issue, as noble and
great as that which you brought forth. I
consider the marriage of your son Boo with
my sister Lit Chia as a mandate of God
and I hope that they treat each other with
great love and mutual courtesy and that
both they and their parents be very happy.
Given during the second moon of the
twenty-first year of the reign of the
Emperor Quang Su.
Cheong Boo is said to have remained in China for one year and four months after his marriage during
which time there was born to him and his wife a child named Cheong Seng Gee. Cheong Boo then left
China for the Philippine Islands and sometime thereafter took to himself a concubine Mora by whom he
had two children. In 1910, Cheong Boo was followed to the Philippines by Cheong Seng Gee who, as
appears from documents presented in evidence, was permitted to land in the Philippine Islands as the
son of Cheong Boo. The deceased, however, never returned to his native hearth and seems never to
have corresponded with his Chinese wife or to have had any further relations with her except once when
he sent her P10.
The trial judge found, as we have said, that the proof did not sustain the allegation of the claimant
Cheong Seng Gee, that Cheong Boo had married in China. His Honor noted a strong inclination on the
part of the Chinese witnesses, especially the brother of Cheong Boo, to protect the interests of the
alleged son, Cheong Seng Gee, by overstepping the limits of truthfulness. His Honor also noted that
reliable witnesses stated that in the year 1895, when Cheong Boo was supposed to have been in China,
he was in reality in Jolo, in the Philippine Islands. We are not disposed to disturb this appreciation of fact
by the trial court. The immigration documents only go to show the relation of parent and child existing
between the deceased Cheong Boo and his son Cheong Seng Gee and do not establish the marriage
between the deceased and the mother of Cheong Seng Gee.
Section IV of the Marriage Law (General Order No. 68) provides that "All marriages contracted without
these Islands, which would be valid by the laws of the country in which the same were contracted, are
valid in these Islands." To establish a valid foreign marriage pursuant to this comity provision, it is first
necessary to prove before the courts of the Islands the existence of the foreign law as a question of fact,
and it is then necessary to prove the alleged foreign marriage by convincing evidence.
As a case directly in point is the leading one of Sy Joc Lieng vs. Encarnacion ([1910]), 16 Phil., 137;
[1913], 228 U.S., 335). Here, the courts of the Philippines and the Supreme Court of the United States
were called upon to decide, as to the conflicting claims to the estate of a Chinese merchant, between the
descendants of an alleged Chinese marriage and the descendants of an alleged Philippine marriage. The
Supreme Courts of the Philippine Islands and the United States united in holding that the Chinese
marriage was not adequately proved. The legal rule was stated by the United States Supreme Court to be
this: A Philippine marriage, followed by forty years of uninterrupted marital life, should not be impugned
and discredited, after the death of the husband and administration of his estate, though an alleged prior
Chinese marriage, "save upon proof so clear, strong, and unequivocal as to produce a moral conviction of
the existence of such impediment." Another case in the same category is that of Son Cui vs. Guepangco
([1912], 22 Phil., 216).
In the case at bar there is no competent testimony as to what the laws of China in the Province of Amoy
concerning marriage were in 1895. As in the Encarnacion case, there is lacking proof so clear, strong,
and unequivocal as to produce a moral conviction of the existence of the alleged prior Chinese marriage.
Substitute twenty-three years for forty years and the two cases are the same.
The lower court allowed the claimant, Cheong Seng Gee, the testamentary rights of an acknowledged
natural child. This finding finds some support in Exhibit 3, the affidavit of Cheong Boo before the
American Vice-Consul at Sandakan, British North Borneo. But we are not called upon to make a
pronouncement on the question, because the oppositor-appellant indicates silent acquiescence by
assigning no error.
2. Validity of the Mohammedan Marriage
The biographical data relating to the Philippine odyssey of the Chinaman Cheong Boo is fairly complete.
He appears to have first landed on Philippine soil sometime prior to the year 1896. At least, in the year las
mentioned, we find him in Basilan, Philippine Islands. There he was married to the Mora Adong according
to the ceremonies prescribed by the book on marriage of the Koran, by the Mohammedan Iman (priest)
Habubakar. That a marriage ceremony took place is established by one of the parties to the marriage, the
Mora Adong, by the Iman who solemnized the marriage, and by other eyewitnesses, one of whom was
the father of the bride, and another, the chief of the rancheria, now a municipal councilor. The groom
complied with Quranic law by giving to the bride a dowry of P250 in money and P250 in goods.
The religious rites began with the bride and groom seating themselves in the house of the father of the
bride, Marahadja Sahibil. The Iman read from the Koran. Then the Iman asked the parents if they had any
objection to the marriage. The marital act was consummated by the groom entering the woman's
mosquito net.
From the marriage day until the death of Cheong Boo, twenty-three years later, the Chinaman and the
Mora Adong cohabited as husband and wife. To them were born five children, two of whom, Payang and
Rosalia, are living. Both in his relations with Mora Adong and with third persons during his lifetime,
Cheong Boo treated Adong as his lawful wife. He admitted this relationship in several private and public
documents. Thus, when different legal documents were executed, including decrees of registration,
Cheong Boo stated that he was married to the Mora Adong while as late as 1918, he gave written
consent to the marriage of his minor daughter, Payang.
Notwithstanding the insinuation of counsel for the Chinese appellant that the custom is prevalent among
the Moros to favor in their testimony, a relative or friend, especially when they do not swear on the Koran
to tell the truth, it seems to us that proof could not be more convincing of the fact that a marriage was
contracted by the Chinaman Cheong Boo and the Mora Adong, according to the ceremonies of the
Mohammedan religion.
It is next incumbent upon us to approach the principal question which we announced in the very
beginning of this decision, namely, Are the marriages performed in the Philippines according to the rites
of the Mohammedan religion valid? Three sections of the Marriage Law (General Order No. 68) must be
taken into consideration.
Section V of the Marriage Law provides that "Marriage may be solemnized by either a judge of any court
inferior to the Supreme Court, justice of the peace, or priest or minister of the Gospel of any
denomination . . ." Counsel, failing to take account of the word "priest," and only considering the phrase
"minister of the Gospel of any denomination" would limit the meaning of this clause to ministers of the
Christian religion. We believe this is a strained interpretation. "Priest," according to the lexicographers,
means one especially consecrated to the service of a divinity and considered as the medium through
whom worship, prayer, sacrifice, or other service is to be offered to the being worshipped, and pardon,
blessing, deliverance, etc., obtained by the worshipper, as a priest of Baal or of Jehovah; a Buddhist
priest. "Minister of the Gospel" means all clergymen of every denomination and faith. A "denomination" is
a religious sect having a particular name. (Haggin vs. Haggin [1892], 35 Neb., 375; In reReinhart, 9 O.
Dec., 441; Hale vs. Everett [1868], 53 N. H. 9.) A Mohammedan Iman is a "priest or minister of the
Gospel," and Mohammedanism is a "denomination," within the meaning of the Marriage Law.
The following section of the Marriage Law, No. VI, provides that "No particular form for the ceremony of
marriage is required, but the parties must declare, in the presence of the person solemnizing the
marriage, that they take each other as husband and wife." The law is quite correct in affirming that no
precise ceremonial is indispensable requisite for the creation of the marriage contract. The two essentials
of a valid marriage are capacity and consent. The latter element may be inferred from the ceremony
performed, the acts of the parties, and habit or repute. In this instance, there is no question of capacity.
Nor do we think there can exist any doubt as to consent. While it is true that during the Mohammedan
ceremony, the remarks of the priest were addressed more to the elders than to the participants, it is
likewise true that the Chinaman and the Mora woman did in fact take each other to be husband and wife
and did thereafter live together as husband and wife. (Travers vs. Reinhardt [1907], 205 U.S., 423.
It would be possible to leave out of view altogether the two sections of the Marriage Law which have just
been quoted and discussed. The particular portion of the law which, in our opinion, is controlling, is
section IX, reading as follows: "No marriage heretofore solemnized before any person professing to have
authority therefor shall be invalid for want of such authority or on account of any informality, irregularity, or
omission, if it was celebrated with the belief of the parties, or either of them, that he had authority and that
they have been lawfully married."
The trial judge in construing this provision of law said that he did not believe that the legislative intention
in promulgating it was to validate marriages celebrated between Mohammedans. To quote the judge:
This provisions relates to marriages contracted by virtue of the provisions of the Spanish law
before revolutionary authorized to solemnized marriages, and it is not to be presumed that the
legislator intended by this law to validate void marriages celebrated during the Spanish
sovereignty contrary to the laws which then governed.
What authority there is for this statement, we cannot conceive. To our mind, nothing could be clearer than
the language used in section IX. Note for a moment the all embracing words found in this section:
"No marriage" Could more inclusive words be found? "Heretofore solemnized" Could any other
construction than that of retrospective force be given to this phrase? "Before any person professing to
have authority therefor shall be invalid for want of such authority" Could stronger language than this be
invoked to announce legislative intention? "Or on account of any informality, irregularity, or omission"
Could the legislative mind frame an idea which would more effectively guard the marriage relation against
technicality? "If it was celebrated with the belief of the parties, or either of them, that he had authority and
that they have been lawfully married" What was the purpose of the legislator here, if it was not to
legalize the marriage, if it was celebrated by any person who thought that he had authority to perform the
same, and if either of the parties thought that they had been married? Is there any word or hint of any
word which would restrict the curative provisions of section IX of the Marriage Law to Christian
marriages? By what system of mental gymnastics would it be possible to evolve from such precise
language the curious idea that it was restricted to marriages performed under the Spanish law before the
revolutionary authorities?
In view of the importance of the question, we do not desire to stop here but would ascertain from other
sources the meaning and scope of Section IX of General Order No. 68.
The purpose of the government toward the Mohammedan population of the Philippines has, time and
again, been announced by treaty, organic law, statutory law, and executive proclamation. The Treaty of
Paris in its article X, provided that "The inhabitants of the territories over which Spain relinquishes or
cedes her sovereignty shall be secured Instructions to the Philippine Commission imposed on every
branch of the Government of the Philippine Islands the inviolable rule "that no law shall be made
respecting an establishment of religion or prohibiting the free exercise thereof, and that the free exercise
and enjoyment of religious profession and worship, without discrimination or preference, shall forever be
allowed ... That no form of religion and no minister of religion shall be forced upon any community or upon
any citizen of the Islands; that, upon the other hand, no minister of religion shall be interfered with or
molested in following his calling, and that the separation between state and church shall be real, entire,
and absolute." The notable state paper of President McKinley also enjoined the Commission, "to bear in
mind that the Government which they are establishing is designed . . . for the happiness, peace, and
prosperity of the people of the Philippine Islands" and that, therefore, "the measures adopted should be
made to conform to their customs, their habits, and even their prejudices. . . . The Philippine Bill and the
Jones Law reproduced the main constitutional provisions establishing religious toleration and equality.
Executive and legislative policy both under Spain and the United States followed in the same path. For
instance, in the Treaty of April 30, 1851, entered into by the Captain General of the Philippines and the
Sultan of Sulu, the Spanish Government guaranteed "with all solemnity to the Sultan and other
inhabitants of Sulu the free exercise of their religion, with which it will not interfere in the slightest way,
and it will also respect their customs." (See furtherDecree of the Governor-General of January 14, 1881.)
For instance, Act No. 2520 of the Philippine Commission, section 3, provided that "Judges of the Court of
First Instance and justices of the peace deciding civil cases in which the parties are Mohammedans or
pagans, when such action is deemed wise, may modify the application of the law of the Philippine
Islands, except laws of the United States applicable to the Philippine Islands, taking into account local
laws and customs. . . ." (See further Act No. 787, sec. 13 [ j]; Act No. 1283, sec. 6 [b]; Act No. 114 of the
Legislative Council amended and approved by the Philippine Commission; Cacho vs. Government of the
United States [1914], 28 Phil., 616.) Various responsible officials have so oft announced the purpose of
the Government not to interfere with the customs of the Moros, especially their religious customs, as to
make quotation of the same superfluous.
The retrospective provisions of the Philippine Marriage Law undoubtedly were inspired by the
governmental policy in the United States, with regard to the marriages of the Indians, the Quakers, and
the Mormons. The rule as to Indians marriages is, that a marriage between two Indians entered into
according to the customs and laws of the people at a place where such customs and laws are in force,
must be recognized as a valid marriage. The rule as to the Society of Quakers is, that they will be left to
their own customs and that their marriages will be recognized although they use no solemnization. The
rule as to Mormon marriages is that the sealing ceremony entered into before a proper official by
members of that Church competent to contract marriage constitutes a valid marriage.
The basis of human society throughout the civilized world is that of marriage. Marriage in this jurisdiction
is not only a civil contract, but, it is a new relation, an institution in the maintenance of which the public is
deeply interested. Consequently, every intendment of the law leans toward legalizing matrimony. Persons
dwelling together in apparent matrimony are presumed, in the absence of any counter-presumption or
evidence special to the case, to be in fact married. The reason is that such is the common order of
society, and if the parties were not what they thus hold themselves out as being, they would be living in
the constant violation of decency and of law. A presumption established by our Code of Civil Procedure is
"that a man and woman deporting themselves as husband and wife have entered into a lawful contract of
marriage.:" (Sec. 334, No. 28.) Semper praesumitur pro matrimonio Always presume marriage. (U.
S. vs. Villafuerte and Rabano [1905], 4 Phil., 476; Son Cui vs.Guepangco, supra; U.S. vs. Memoracion
and Uri [1916], 34 Phil., 633; Teter vs. Teter [1884], 101 Ind., 129.)
Section IX of the Marriage Law is in the nature of a curative provision intended to safeguard society by
legalizing prior marriages. We can see no substantial reason for denying to the legislative power the right
to remove impediments to an effectual marriage. If the legislative power can declare what shall be valid
marriages, it can render valid, marriages which, when they took place, were against the law. Public policy
should aid acts intended to validate marriages and should retard acts intended to invalidate marriages.
(Coghsen vs. Stonington [1822], 4 Conn, 209; Baity vs. Cranfill [1884], 91 N. C., 273.)
The courts can properly incline the scales of their decisions in favors of that solution which will mot
effectively promote the public policy. That is the true construction which will best carry legislative intention
into effect. And here the consequences, entailed in holding that the marriage of the Mora Adong and the
deceased Cheong Boo, in conformity with the Mohammedan religion and Moro customs, was void, would
be far reaching in disastrous result. The last census shows that there are at least one hundred fifty
thousand Moros who have been married according to local custom. We then have it within our power
either to nullify or to validate all of these marriages; either to make all of the children born of these unions
bastards or to make them legitimate; either to proclaim immorality or to sanction morality; either to block
or to advance settled governmental policy. Our duty is a obvious as the law is plain.
In moving toward our conclusion, we have not lost sight of the decisions of this court in the cases of
United Statesvs. Tubban ([1915]), 29 Phil., 434) and United States vs. Verzola ([1916, 33 Phil., 285). We
do not, however, believe these decisions to be controlling. In the first place, these were criminal actions
and two Justice dissented.. In the second place, in the Tubban case, the marriage in question was a tribal
marriage of the Kalingas, while in the Verzola case, the marriage had been performed during the
Spanish regime by a lieutenant of the Guardia Civil. In neither case, in deciding as to whether or not the
accused should be given the benefit of the so-called unwritten law, was any consideration given to the
provisions of section IX of General Order No. 68. We are free to admit that, if necessary, we would
unhesitatingly revoke the doctrine announced in the two cases above mentioned.
We regard the evidence as producing a moral conviction of the existence of the Mohammedan marriage.
We regard the provisions of section IX of the Marriage law as validating marriages performed according
to the rites of the Mohammedan religion.
There are other questions presented in the various assignments of error which it is unnecessary to
decide. Inresume, we find the Chinese marriage not to be proved and that the Chinaman Cheong Seng
Gee has only the rights of a natural child, and we find the Mohammedan marriage to be proved and to be
valid, thus giving to the widow and the legitimate children of this union the rights accruing to them under
the law.
Judgment is reversed in part, and the case shall be returned to the lower court for a partition of the
property in accordance with this decision, and for further proceedings in accordance with law. Without
special findings as to costs in this instance, it is so ordered.
Araullo, C.J., Johnson, Street, Avancea, Villamor, Ostrand, Johns and Romualdez, JJ., concur.









Republic of the Philippines
SUPREME COURT
Manila
EN BANC
G.R. No. L-10533 November 11, 1915
THE UNITED STATES, plaintiff-appellee,
vs.
DIONISIO ENRIQUEZ, defendant-appellant.
Jose R. Varela for appellant.
Attorney-General Avancea for appellee.

ARAULLO, J .:
In a judgment rendered by the Court of First Instance of Bataan on September 30, 1914, the herein
defendant was sentenced for the crime of illegal marriage to six years and one day of prision mayor, with
the accessory penalties of the law and costs. From the judgment he has appealed.
It was proved that the defendant was united in marriage to Juliana Marcelo before the parish priest
of Binondo of this city of Manila on July 21, 1886; that without this marriage having been legally dissolved
he contracted another marriage, on February 1, 1905, with Joaquina Trajano, before the parish priest of
Orion, Province of Bataan. But from the other evidence presented at the trial, the following facts also
appear, as set forth in the judgment appealed from:lawph!1.net
"It appears by the record that the defendant, Dionisio Enriquez, contracted a canonical marriage
with Juliana Marcelo in the parish of Binondo, Manila, on July 21, 1886. He left his wife and children in the
barrio of Bantan, municipality of Orion, Province of Bataan, in the year 1895, going to the Province of
Laguna as a postal employee; and, after experiencing various vicissitudes during the revolution of `96
and `98, on returning to the said pueblo of Orion in 1901 he did not find his wife, nor could be obtain the
slightest information of her or her whereabouts, notwithstanding his persistent and diligent search.
Therefore, believing her dead, he determined to contract a canonical marriage with Joaquina Trajano, in
the parish of Orion, Bataan, on February 1, 1905. In December, 1913, his first wife, Juliana Marcelo,
made her appearance in the pueblo of Orion. In her testimony she stated that she was actually absent
and in Manila, Tarlac and Victoria, from 1895 to 1913, when she returned to the pueblo of Orion, and that
during this period she had had no news of her husband." .
The trial court recognized that the defendant, when he contracted his second marriage with
Joaquina Trajano, acted on the reasonable and well founded belief that his first wife, Juliana Marcelo,
was dead, in view of the fact that for 19 years he had had no news whatever of her whereabouts and in
view of the fruitless result of his endeavors to find her. In other words, he believed in good faith that his
marriage with his first wife, Juliana Marcelo, had been dissolved by her death, consequently fraudulent
intent, which constitutes one of the essential elements of the crime of illegal marriage, cannot be charged
to the defendant and he cannot, therefore, be considered guilty of the said crime.
The fact that, on contracting his second marriage, he stated to the priest who solemnized it that he
was single cannot be considered sufficient proof of bad faith on the part of defendant if we take into
account, on the one hand, that the defendant satisfactorily explained that statement, and on the other,
that he could not have intended to deceive the said priest, for it was proved at the trial that the
defendant's children by his first marriage, in company with the defendant's mother, had resided for a long
time in the barrio of Bantan of the said municipality of Orion, when the second marriage took place, and
that after they were married, these children had houses of their own in Limay, also a barrio of Orion.
Moreover, it was also shown, by the testimony of the priest who celebrated the second marriage, that it
was solemnized after due information and the publication of banns.
Therefore, in conformity with the recommendation made by the Attorney-General in his brief, we
hereby reverse the judgment appealed from and freely acquit the defendant, with the costs of both
instances de oficio.
Arellano, C.J., Torres, Johnson, Carson, and Trent, JJ., concur.

















Republic of the Philippines
SUPREME COURT
Manila
SECOND DIVISION
G.R. No. 70479 February 27, 1987
FIRESTONE TIRE AND RUBBER COMPANY OF THE PHILIPPINES, petitioner,
vs.
CARLOS LARIOSA and NATIONAL LABOR RELATIONS COMMISSION, respondents.

FERNAN, J :
In this petition for certiorari, petitioner Firestone Tire and Rubber Company of the Philippines [Firestone
for brevity] assails the decision of public respondent National Labor Relations Commission which ordered
the reinstatement without backwages of Carlos Lariosa, a dismissed tire builder of petitioner, as having
been rendered with grave abuse of discretion amounting to lack of jurisdiction.
The facts are as follows:
Carlos Lariosa started working with Firestone on January 3, 1972 as a factory worker. At the time of his
dismissal, he was a tire builder.
At around 2:00 o'clock in the afternoon of July 27, 1983, as he was about to leave the company premises
Lariosa submitted himself to a routine check by the security guards at the west gate. He was frisked by
Security Guard Ambrosio Liso [Lizo] while his personal bag was inspected by Security Guard Virgilio
Olvez. In the course of the inspection, sixteen [16] wool flannel swabs, all belonging to the company,
were found inside his bag, tucked underneath his soiled clothes.
As a result of the incident, Firestone terminated Lariosa's services on August 2, 1983, citing as grounds
therefor: "stealing company property and loss of trust."
1
Firestone also filed a criminal complaint against
him with the Rizal provincial fiscal for attempted theft [IS No. 83-436-M].
2

Lariosa, on the other hand, sued Firestone before the Ministry of Labor and Employment for illegal
dismissal, violation of Batas Pambansa Blg. 130 and its related rules and regulations, and damages. The
Labor Arbiter, in his decision dated May 8, 1984, found Lariosa's dismissal justified.
3
However, on
appeal, the National Labor Relations Commission on December 28, 1984 reversed the decision of the
Labor Arbiter [with one commissioner voting for affirmance] and held that the dismissal of Lariosa was too
severe a penalty. It therefore ordered Lariosa's reinstatement but without backwages, the period when he
was out of work to be considered a suspension.
4

Petitioner Firestone, in this special civil action for certiorari, contends that the NLRC erred in not
dismissing Lariosa's appeal for being late, in finding that Lariosa was not accorded due process and in
reversing the Labor Arbiter.
We shall deal first with the timeliness of the appeal. It is admitted that Lariosa filed his appeal on June 7,
1984 or after the lapse of fourteen days from notice of the decision of the Labor Arbiter. Article 223 of the
Labor Code clearly provides for a reglementary period of ten days within which to appeal decision of the
Labor Arbiter to the NLRC. The ten-day period has been interpreted by this Court in the case of Vir-jen
Shipping and Marine Services, Inc. vs. NLRC, G.R. No. 58011-12, July 20, 1982, 115 SCRA 347, 361, to
mean ten "calendar" days and not ten "working" days. However, the "Notice of Decision" which Lariosa's
lawyer received together with a copy of the arbiter's decision advised them that an appeal could be taken
to the NLRC within ten "working" days from receipt of the said decision.
5

Mindful of the fact that Lariosa's counsel must have been misled by the implementing rules of the labor
commission and considering that the shortened period for appeal is principally intended more for the
employees' benefit, rather than that of the employer, We are inclined to overlook this particular procedural
lapse and to proceed with the resolution of the instant case.
A review of the record shows that Lariosa was indubitably involved in the attempted theft of the flannel
swabs. During the investigation called by the company's industrial relations manager Ms. Villavicencio on
July 28, 1983, or one day after the incident, Security Guards Liso and Olvez contradicted Lariosa's bare
claim that he had no intention to bring home the swabs and that he had simply overlooked that he had
earlier placed them inside his bag after they were given to him by his shift supervisor while he was busy
at work. Guard Olvez stated that when he confronted Lariosa with the swabs, the latter replied that they
were for "home use." And when he requested Lariosa to stay behind while he reported the matter to the
authorities, Lariosa refused and hurriedly left the premises and boarded a passing jeepney.
6

From the records, it is likewise clear that Firestone did not act arbitrarily in terminating Lariosa's services.
On the contrary, there are transcripts to prove that an investigation of the incident was promptly
conducted in the presence of the employee concerned, the union president and the security guards who
witnessed the attempted asportation. Records also belie the allegation that Lariosa was shown his
walking papers on the very day of the incident. The letter of Ms. Villavicencio to Lariosa dated August 1,
1983 informing the latter of his dismissal effective August 2, 1983 conclusively shows that he was
discharged only on August 2, 1983, after an investigation was held to ventilate the truth about the July 27
incident.
7
Thus, we cannot agree with the NLRC's conclusion that even if Firestone had found substantial
proof of Lariosa's misconduct, it did not observe the statutory requirements of due process.
There is no gainsaying that theft committed by an employee constitutes a valid reason for his dismissal by
the employer. Although as a rule this Court leans over backwards to help workers and employees
continue with their employment or to mitigate the penalties imposed on them, acts of dishonesty in the
handling of company property are a different matter.
8

Thus, under Article 283 of the Labor Code, an employer may terminate an employment for "serious
misconduct" or for "fraud or willful breach by the employee of the trust reposed in him by his employer or
representative."
If there is sufficient evidence that an employee has been guilty of a breach of trust or that his employer
has ample reasons to distrust him, the labor tribunal cannot justly deny to the employer the authority to
dismiss such an employee.
9

As a tire builder, Lariosa was entrusted with certain materials for use in his job. On the day in question, he
was given two bundles of wool flannel swabs [ten pieces per bundle] for cleaning disks. He used four
swabs from one pack and kept the rest [sixteen pieces] in his "blue travelling bag."
10
Why he placed the
swabs in his personal bag, which is not the usual receptacle for company property, has not been
satisfactorily explained.
If Lariosa, by his own wrong-doing, could no longer be trusted, it would be an act of oppression to compel
the company to retain him, fully aware that such an employee could, in the long run, endanger its very
viability.
The employer's obligation to give his workers just compensation and treatment carries with it the corollary
right to expect from the workers adequate work, diligence and good conduct.
11

In view of the foregoing, We rule that Firestone had valid grounds to dispense with the services of Lariosa
and that the NLRC acted with grave abuse of discretion in ordering his reinstatement. However,
considering that Lariosa had worked with the company for eleven years with no known previous bad
record, the ends of social and compassionate justice would be served if he is paid full separation pay but
not reinstatement without backages as decreed by the NLRC.
12

WHEREFORE, the petition is granted. The decision of the National Labor Relations Commission dated
December 28, 1984 is reversed and set aside. Petitioner Firestone Tire and Rubber Company of the
Philippines is directed to pay its dismissed worker Carlos Lariosa the separation pay to which he may be
entitled under the law, or any collective bargaining agreement or company rules or practice, whichever is
higher.
SO ORDERED.
Alampay, Gutierrez, Jr., Paras, Padilla and Bidin JJ., concur.
Cortes, J., took no part.
Footnotes
1 Rollo, P. 43.
2 Rollo, p. 82.
3 Rollo, p. 27.
4 Rollo, pp. 20-21.
5 Rollo, p. 88.
6 Rollo p. 43.
7 Rollo, pp. 42-43.
8 Metro Drug Corporation vs. NLRC, G.R. No. 72248, July 22, 1986, 143 SCRA 132,
citing Dole Philippines, Inc. vs. NLRC, G.R. 55413, July 25, 1983, 123 SCRA 673.
9 Philippine Geothermal Inc. vs. NLRC, G.R. Nos. 55249-50, October 19, 1982, 117
SCRA 692, 695; Reynolds Philippines Corp. vs. Eslava, G.R. No. L-48814, June 27,
1985, 137 SCRA 259.
10 Rollo, p. 93.
11 Jacinto vs. Standard-Vacuum Oil Co., 70 Phil. 501; San Miguel Corporation vs. NLRC,
G.R. No. 56554, July 20, 1982, 115 SCRA 329.
12 Engineering Equipment Inc. vs. NLRC, G.R. No. 59221, December 26, 1984, 133
SCRA 752; National Service Corporation vs. Leogardo, Jr., G.R. No. 64296, July 20,
1984, 130 SCRA 502.


SECOND DIVISION


JOSELITO R. PIMENTEL, G.R. No. 172060
Petitioner,
Present:

CARPIO, J., Chairperson,
- versus - PERALTA,
BERSAMIN,
*

ABAD, and
VILLARAMA, JR.,
**
JJ.

MARIA CHRYSANTINE
L. PIMENTEL and PEOPLE Promulgated:
OF THE PHILIPPINES,
Respondents. September 13, 2010
x- - - - - - - - - - - - - - - - - - - - - - - - - - - - - - - - - - - - - - - - - - - - - - - - - - - x


D E C I S I O N

CARPIO, J .:

The Case

Before the Court is a petition for review
[1]
assailing the Decision
[2]
of the Court of Appeals,
promulgated on 20 March 2006, in CA-G.R. SP No. 91867.

The Antecedent Facts

The facts are stated in the Court of Appeals decision:
On 25 October 2004, Maria Chrysantine Pimentel y Lacap (private respondent) filed an action for
frustrated parricide against Joselito R. Pimentel (petitioner), docketed as Criminal Case No. Q-04-130415,
before the Regional Trial Court of Quezon City, which was raffled to Branch 223 (RTC Quezon City).

On 7 February 2005, petitioner received summons to appear before the Regional Trial Court of
Antipolo City, Branch 72 (RTC Antipolo) for the pre-trial and trial of Civil Case No. 04-7392 (Maria
Chrysantine Lorenza L. Pimentel v. Joselito Pimentel) for Declaration of Nullity of Marriage under Section
36 of the Family Code on the ground of psychological incapacity.

On 11 February 2005, petitioner filed an urgent motion to suspend the proceedings before the RTC
Quezon City on the ground of the existence of a prejudicial question. Petitioner asserted that since the
relationship between the offender and the victim is a key element in parricide, the outcome of Civil Case
No. 04-7392 would have a bearing in the criminal case filed against him before the RTC Quezon City.

The Decision of the Trial Court

The RTC Quezon City issued an Order dated 13 May 2005
[3]
holding that the pendency of the case
before the RTC Antipolo is not a prejudicial question that warrants the suspension of the criminal case
before it. The RTC Quezon City held that the issues in Criminal Case No. Q-04-130415 are the injuries
sustained by respondent and whether the case could be tried even if the validity of petitioners marriage
with respondent is in question. The RTC Quezon City ruled:


WHEREFORE, on the basis of the foregoing, the Motion to Suspend Proceedings
On the [Ground] of the Existence of a Prejudicial Question is, for lack of merit, DENIED.

SO ORDERED.
[4]



Petitioner filed a motion for reconsideration. In its 22 August 2005 Order,
[5]
the RTC Quezon City
denied the motion.

Petitioner filed a petition for certiorari with application for a writ of preliminary injunction and/or
temporary restraining order before the Court of Appeals, assailing the 13 May 2005 and 22 August 2005
Orders of the RTC Quezon City.

The Decision of the Court of Appeals

In its 20 March 2006 Decision, the Court of Appeals dismissed the petition. The Court of Appeals
ruled that in the criminal case for frustrated parricide, the issue is whether the offender commenced the
commission of the crime of parricide directly by overt acts and did not perform all the acts of execution by
reason of some cause or accident other than his own spontaneous desistance. On the other hand, the
issue in the civil action for annulment of marriage is whether petitioner is psychologically incapacitated to
comply with the essential marital obligations. The Court of Appeals ruled that even if the marriage
between petitioner and respondent would be declared void, it would be immaterial to the criminal
case because prior to the declaration of nullity, the alleged acts constituting the crime of frustrated
parricide had already been committed. The Court of Appeals ruled that all that is required for the charge
of frustrated parricide is that at the time of the commission of the crime, the marriage is still subsisting.

Petitioner filed a petition for review before this Court assailing the Court of Appeals decision.

The Issue

The only issue in this case is whether the resolution of the action for annulment of marriage is a
prejudicial question that warrants the suspension of the criminal case for frustrated parricide against
petitioner.

The Ruling of this Court

The petition has no merit.

Civil Case Must be Instituted
Before the Criminal Case

Section 7, Rule 111 of the 2000 Rules on Criminal Procedure
[6]
provides:

Section 7. Elements of Prejudicial Question. - The elements of a prejudicial question are:
(a) the previously instituted civil action involves an issue similar or intimately related to the
issue raised in the subsequent criminal action and (b) the resolution of such issue
determines whether or not the criminal action may proceed.

The rule is clear that the civil action must be instituted first before the filing of the criminal action. In
this case, the Information
[7]
for Frustrated Parricide was dated 30 August 2004. It was raffled to RTC
Quezon City on 25 October 2004 as per the stamped date of receipt on the Information. The RTC
Quezon City set Criminal Case No. Q-04-130415 for pre-trial and trial on 14 February 2005. Petitioner
was served summons in Civil Case No. 04-7392 on 7 February 2005.
[8]
Respondents petition
[9]
in Civil
Case No. 04-7392 was dated 4 November 2004 and was filed on 5 November 2004. Clearly, the civil
case for annulment was filed after the filing of the criminal case for frustrated parricide. As such, the
requirement of Section 7, Rule 111 of the 2000 Rules on Criminal Procedure was not met since the civil
action was filed subsequent to the filing of the criminal action.

Annulment of Marriage is not a Prejudicial Question
in Criminal Case for Parricide

Further, the resolution of the civil action is not a prejudicial question that would warrant the
suspension of the criminal action.

There is a prejudicial question when a civil action and a criminal action are both pending, and there
exists in the civil action an issue which must be preemptively resolved before the criminal action may
proceed because howsoever the issue raised in the civil action is resolved would be determinative of the
guilt or innocence of the accused in the criminal case.
[10]
A prejudicial question is defined as:

x x x one that arises in a case the resolution of which is a logical antecedent of the issue
involved therein, and the cognizance of which pertains to another tribunal. It is a question
based on a fact distinct and separate from the crime but so intimately connected with it
that it determines the guilt or innocence of the accused, and for it to suspend the criminal
action, it must appear not only that said case involves facts intimately related to those
upon which the criminal prosecution would be based but also that in the resolution of the
issue or issues raised in the civil case, the guilt or innocence of the accused would
necessarily be determined.
[11]



The relationship between the offender and the victim is a key element in the crime of
parricide,
[12]
which punishes any person who shall kill his father, mother, or child, whether legitimate or
illegitimate, or any of his ascendants or descendants, or his spouse.
[13]
The relationship between the
offender and the victim distinguishes the crime of parricide from murder
[14]
or homicide.
[15]
However, the
issue in the annulment of marriage is not similar or intimately related to the issue in the criminal case for
parricide. Further, the relationship between the offender and the victim is not determinative of the guilt or
innocence of the accused.

The issue in the civil case for annulment of marriage under Article 36 of the Family Code is
whether petitioner is psychologically incapacitated to comply with the essential marital obligations. The
issue in parricide is whether the accused killed the victim. In this case, since petitioner was charged with
frustrated parricide, the issue is whether he performed all the acts of execution which would have killed
respondent as a consequence but which, nevertheless, did not produce it by reason of causes
independent of petitioners will.
[16]
At the time of the commission of the alleged crime, petitioner and
respondent were married. The subsequent dissolution of their marriage, in case the petition in Civil Case
No. 04-7392 is granted, will have no effect on the alleged crime that was committed at the time of the
subsistence of the marriage. In short, even if the marriage between petitioner and respondent is
annulled, petitioner could still be held criminally liable since at the time of the commission of the alleged
crime, he was still married to respondent.


We cannot accept petitioners reliance on Tenebro v. Court of Appeals
[17]
that the judicial
declaration of the nullity of a marriage on the ground of psychological incapacity retroacts to the date of
the celebration of the marriage insofar as the vinculum between the spouses is concerned x x
x. First, the issue in Tenebro is the effect of the judicial declaration of nullity of a second or subsequent
marriage on the ground of psychological incapacity on a criminal liability for bigamy. There was no issue
of prejudicial question in that case. Second, the Court ruled in Tenebro that [t]here is x x x a
recognition written into the law itself that such a marriage, although void ab initio, may still produce legal
consequences.
[18]
In fact, the Court declared in that case that a declaration of the nullity of the second
marriage on the ground of psychological incapacity is of absolutely no moment insofar as the States
penal laws are concerned.
[19]


In view of the foregoing, the Court upholds the decision of the Court of Appeals. The trial
in Criminal Case No. Q-04-130415 may proceed as the resolution of the issue in Civil Case No. 04-7392
is not determinative of the guilt or innocence of petitioner in the criminal case.

WHEREFORE, we DENY the petition. We AFFIRM the 20 March 2006 Decision of the Court of
Appeals in CA-G.R. SP No. 91867.

SO ORDERED.




ANTONIO T. CARPIO
Associate Justice










FIRST DIVISION

KRIZIA KATRINA TY-DE ZUZUARREGUI,
Petitioner,


- versus -


THE HON. JOSELITO C. VILLAROSA, in his
capacity as Presiding Judge of Branch 66 of
the RTC of Makati City, and FANNIE
TORRES-TY,
Respondents.
G.R. No. 183788

Present:

PUNO, C.J., Chairperson,
CARPIO MORALES,
LEONARDO-DE CASTRO,
BERSAMIN, and
VILLARAMA, JR., JJ.

Promulgated:

April 5, 2010
x- - - - - - - - - - - - - - - - - - - - - - - - - - - - - - - - - - - - - - - - - - - - - - - - - - -x

DECISION

VILLARAMA, JR., J .:
This is a petition for review on certiorari
[1]
under Rule 45 of the 1997 Rules of Civil Procedure, as
amended, assailing the Resolutions dated August 23, 2007
[2]
and July 14, 2008
[3]
of the Court of Appeals
in CA-G.R. SP No. 98978. The Court of Appeals dismissed the petition for certiorari and prohibition filed
by petitioner seeking the reversal of the November 16, 2006 and March 9, 2007 Orders
[4]
of the Regional
Trial Court (RTC) of Makati City, Branch 66, which found that there was no prejudicial question to warrant
the suspension of the criminal actions against petitioner.
The following facts are established:
Sometime in August 2000, Rosemary Torres Ty-Rasekhi (Rosemary), the sister of petitioners late
father Alexander Torres Ty, filed a petition for the issuance of letters of administration of the estate of her
mother, Bella Torres (Bella), before the RTC of Pasig City.
[5]
Petitioner initially opposed
[6]
Rosemarys
petition, but they eventually reached an amicable settlement and entered into a compromise agreement
which they submitted to the RTC for approval.
[7]
In a Decision
[8]
dated November 19, 2002, the RTC
approved the compromise agreement.
Subsequently, two (2) of Rosemarys alleged siblings, Peter Torres Ty (Peter) and Catherine Torres
Ty-Chavez (Catherine), filed with the Court of Appeals a Petition to Annul Judgment Approving Compromise
Agreement, docketed as CA-G.R. SP No. 87222.
[9]
Peter and Catherine claimed that they are also
biological children of the late Bella, and are entitled to participate in the settlement of the latters
estate. Later, private respondent Fannie Torres-Ty (Fannie), who likewise claimed to be a biological child of
the late Bella and therefore also entitled to inherit from her, filed a petition-in-intervention in the action for
annulment of judgment.
[10]

Peter, Catherine, and Fannie alleged that upon the death of Bella, they held a number of discussions
pertaining to the settlement of the latters estate. Rosemary, their elder sister, promised to take care of the
processing of papers so that the estate may be divided among them in the manner provided by
law. However, in subsequent discussions, Rosemary made known to them her intention to get a
disproportionately larger share of the estate, but they did not agree. No agreement was reached and as far
as they know, no progress was made towards the settlement of Bellas estate. They were not aware that
Rosemary had filed a petition for the issuance of letters of administration and that a judgment by
compromise agreement was rendered by the RTC of Pasig City. Rosemary had falsely averred that aside
from herself, petitioner, who was her niece, was the only other heir of Bella. In petitioners opposition, it was
likewise averred that petitioner and Rosemary were the only heirs of Bella. The subsequent compromise
agreement contained similar averments, and it was not disclosed that Peter, Catherine, and Fannie were
also Bellas heirs. It was only sometime in June 2004 that they came to know of the decision by
compromise agreement of the Pasig City RTC.
Petitioner and Rosemary filed their answers
[11]
to the petition for annulment of judgment and the
petition-in-intervention. They raised similar defenses. They denied that Peter, Catherine, and Fannie
were heirs of Bella for, as far as they knew, the three (3) were literally purchased from third persons who
represented to Bella and the latters common-law husband, Alejandro Ty, that they were abandoned
children. Bella and Alejandro took pity on the three (3) and brought them up as their own. This was
known within the family circle, but was not disclosed to Peter, Catherine, and Fannie in order to protect
them from the stigma of knowing they were unwanted children. However, Alejandro and Bella did not
legally adopt them; hence, they were never conferred the rights of legitimate children.
While the action for annulment of judgment was pending before the Court of Appeals, Fannie filed a
complaint
[12]
for falsification and perjury against petitioner and Rosemary. Fannie alleged that petitioner and
Rosemary falsely and maliciously stated in the pertinent pleadings filed before the RTC of Pasig City that
the late Bella had only two (2) heirs, namely the two (2) of them. Petitioner and Rosemary forthwith filed a
joint motion to suspend the preliminary investigation on the ground of a pending prejudicial question before
the Court of Appeals.
[13]
They argued that the issue of whether Peter, Catherine, and Fannie are related to
Bella and therefore legal heirs of the latter was pending before the Court of Appeals. The investigating
prosecutor denied the joint motion and found probable cause against petitioner and Rosemary for two (2)
counts each of falsification of public documents.
[14]
The prosecutor held that the issue before the Court of
Appeals is the validity of the compromise agreement which is not determinative of the criminal case which
involves the liability of petitioner and Rosemary for falsification, allegedly for willfully making the false
statements in the opposition to the petition for letters of administration and in the subsequent compromise
agreement filed before the RTC of Pasig City.
On December 20, 2005, three (3) informations
[15]
against petitioner and Rosemary were thus filed
with the Metropolitan Trial Court (MeTC) of Makati City, Branch 61.
Petitioner filed a petition for review
[16]
with the Department of Justice (DOJ) and a motion to defer
proceedings
[17]
before the MeTC on the ground of the pending appeal before the DOJ. Also, petitioner and
Rosemary filed with the MeTC separate motions to suspend proceedings on the ground of prejudicial
question.
[18]
However, petitioners appeal was dismissed by the DOJ,
[19]
while her motions before the MeTC
were denied by the said court.
[20]
The MeTC agreed with the prosecutor that the issue before the Court of
Appeals in the action for annulment of judgment is the validity of the compromise agreement while the
criminal case involves their liability for falsification of public documents. The MeTC also denied petitioners
motion for reconsideration.
[21]

Aggrieved, petitioner filed a petition for certiorari and prohibition
[22]
with the RTC of Makati City,
Branch 66. In an Order
[23]
dated November 16, 2006, the RTC denied the petition on the ground that there
was no prejudicial question; hence, the MeTC did not act with grave abuse of discretion in denying
petitioners motion to suspend proceedings. The RTC held that there was no prejudicial question as the
quantum of evidence in the civil action for annulment of judgment differs from the quantum of evidence
required in the criminal action for falsification of public documents. Petitioners motion for
reconsideration
[24]
was also denied by the RTC in its Order
[25]
dated March 9, 2007.
Undaunted, petitioner filed a petition for certiorari and prohibition before the Court of Appeals
assailing the RTCs orders. In its August 23, 2007 Resolution,
[26]
the appellate court dismissed the petition
on the ground that the certification of non-forum shopping was signed only by petitioners counsel and not
by petitioner herself. Petitioners motion for reconsideration was also denied in the July 14,
2008 Resolution
[27]
of the Court of Appeals.
Hence, the present recourse.
Petitioner alleges that:
I

THE HONORABLE COURT OF APPEALS GRAVELY ERRED IN DISMISSING THE
PETITION FOR CERTIORARI ON THE GROUND THAT THE CERTIFICATION OF
NON-FORUM SHOPPING WAS SIGNED BY COUNSEL ALLEGEDLY IN VIOLATION
OF SEC. 3, RULE 46, IN RELATION TO SEC. 1 RULE 65, 1997 RULES OF CIVIL
PROCEDURE.

II

THE HONORABLE COURT OF APPEALS GRAVELY ERRED IN NOT NULLIFYING
THE ASSAILED ORDERS OF PUBLIC RESPONDENT JOSELITO VILLAROSA ON THE
GROUND THAT THE SAME WAS ISSUED WITH GRAVE ABUSE OF DISCRETION
AMOUNTING TO LACK OF OR EXCESS OF JURISDICTION.
[28]

The petition is meritorious.
Under Rule 46, Section 3, paragraph 3 of the 1997 Rules of Civil Procedure, as amended,
petitions for certiorari must be verified and accompanied by a sworn certification of non-forum
shopping.
[29]
The primary question that has to be resolved in this case is whether the verification and
certification of non-forum shopping, erroneously signed by counsel, may be cured by subsequent
compliance.
[30]

Generally, subsequent compliance with the requirement of a certification of non-forum shopping
does not excuse a party from failure to comply in the first instance.
[31]
A certification of the plaintiffs
counsel will not suffice for the reason that it is the petitioner, and not the counsel, who is in the best
position to know whether he actually filed or caused the filing of a petition.
[32]
A certification against forum
shopping signed by counsel is a defective certification that is equivalent to non-compliance with the
requirement and constitutes a valid cause for the dismissal of the petition.
[33]

However, there are instances when we treated compliance with the rule with relative liberality,
especially when there are circumstances or compelling reasons making the strict application of the rule
clearly unjustified.
[34]

In the case of Far Eastern Shipping Company v. Court of Appeals,
[35]
while we said that, strictly, a
certification against forum shopping by counsel is a defective certification, the verification, signed by
petitioners counsel in said case, is substantial compliance inasmuch as it served the purpose of the
Rules of informing the Court of the pendency of another action or proceeding involving the same issues.
We then explained that procedural rules are instruments in the speedy and efficient administration of
justice which should be used to achieve such end and not to derail it.
[36]

In Sy Chin v. Court of Appeals,
[37]
we categorically stated that while the petition was flawed as the
certification of non-forum shopping was signed only by counsel and not by the party, such procedural
lapse may be overlooked in the interest of substantial justice.
[38]
Finally, the Court has also on occasion
held that the party need not sign the verification; a partys representative, lawyer or any person who
personally knows the truth of the facts alleged in the pleading may sign the verification.
[39]

Here, the verification and certification of non-forum shopping was signed by petitioners counsel.
Upon receipt of the resolution of the Court of Appeals dismissing her petition for non-compliance with the
rules, petitioner submitted, together with her motion for reconsideration, a verification and certification
signed by her in compliance with the said rule.
[40]
We deem this to be sufficient compliance especially in
view of the merits of the case, which may be considered as a special circumstance or a compelling
reason that would justify tempering the hard consequence of the procedural requirement on non-forum
shopping.
[41]

On the second assignment of error that the Court of Appeals erred in denying petitioners prayer
for a writ of certiorari and prohibition, we likewise find for petitioner.
Under Rule 111 of the Revised Rules of Criminal Procedure, as amended, a criminal action may
be suspended upon the pendency of a prejudicial question in a civil action, to wit:
SEC. 6. Suspension by reason of prejudicial question. - A petition for suspension
of the criminal action based upon the pendency of a prejudicial question in a civil action
may be filed in the office of the prosecutor or the court conducting the preliminary
investigation. When the criminal action has been filed in court for trial, the petition to
suspend shall be filed in the same criminal action at any time before the prosecution
rests.
For a prejudicial question in a civil case to suspend a criminal action, it must appear not only that
said civil case involves facts intimately related to those upon which the criminal prosecution would be based,
but also that in the resolution of the issue or issues raised in the civil case, the guilt or innocence of the
accused would necessarily be determined.
[42]
The rationale behind the principle of prejudicial question is to
avoid two (2) conflicting decisions.
Thus, for a civil action to be considered prejudicial to a criminal case as to cause the suspension
of the criminal proceedings until the final resolution of the civil case, the following requisites must be
present: (1) the civil case involves facts intimately related to those upon which the criminal prosecution
would be based; (2) in the resolution of the issue or issues raised in the civil action, the guilt or innocence
of the accused would necessarily be determined; and (3) jurisdiction to try said question must be lodged
in another tribunal.
[43]

If the resolution of the issue in the civil action will not determine the criminal responsibility of the
accused in the criminal action based on the same facts, or there is no necessity that the civil case be
determined first before taking up the criminal case, the civil case does not involve a prejudicial
question.
[44]
Neither is there a prejudicial questionif the civil and the criminal action can, according to law,
proceed independently of each other.
[45]

As stated, the determination of whether the proceedings may be suspended on the basis of a
prejudicial question rests on whether the facts and issues raised in the pleadings in the civil case are so
related with the issues raised in the criminal case such that the resolution of the issues in the civil case
would also determine the judgment in the criminal case.
A perusal of the allegations in the petition to annul judgment shows that CA-G.R. SP No. 87222
pending before the Court of Appeals is principally for the determination of the validity of the compromise
agreement which did not include Peter, Catherine, and Fannie as heirs of Bella. Peter, Catherine, and
Fannie presented evidence to prove that they are also biological children of Bella and Alejandro. On the
other hand, Criminal Case Nos. 343812 to 343814 before the MeTC involve the determination of whether
petitioner committed falsification of public documents in executing pleadings containing untruthful
statements that she and Rosemary were the only legal heirs of Bella.
It is evident that the result of the civil case will determine the innocence or guilt of the petitioner in
the criminal cases for falsification of public documents. The criminal cases arose out of the claim of Peter,
Catherine, and Fannie that they are also the legal heirs of Bella. If it is finally adjudged in the civil case
that they are not biological children of the late Bella and consequently not entitled to a share in her estate
as heirs, there is no more basis to proceed with the criminal cases against petitioner who could not have
committed falsification in her pleadings filed before the RTC of Pasig City, the truth of her statements
regarding the filiation of Peter, Catherine and Fannie having been judicially settled.
WHEREFORE, the petition is GRANTED. The Resolutions dated August 23, 2007 and July 14,
2008 of the Court of Appeals in CA-G.R. SP No. 98978 are herebyREVERSED and SET ASIDE. The
criminal proceedings against petitioner Krizia Katrina Ty-De Zuzuarregui in Criminal Case Nos. 343812 to
343814 before the MetropolitanTrial Court of Makati City, Branch 61 are hereby
ordered SUSPENDED until the final resolution of CA-G.R. SP No. 87222.
No costs.
SO ORDERED.
MARTIN S. VILLARAMA, JR.
Associate Justice


Republic of the Philippines
Supreme Court
Manila


THIRD DIVISION


GERBERT R. CORPUZ,
Petitioner,




- versus -





DAISYLYN TIROL STO. TOMAS and The
SOLICITOR GENERAL,
Respondents. -- -
G.R. No. 186571

Present:

CARPIO MORALES, J., Chairperson,
BRION,
BERSAMIN,

*
ABAD, and
VILLARAMA, JR., JJ.



Promulgated:

August 11, 2010
x--------------------------------------------------------------------------------------------------------------x

D E C I S I O N

BRION, J .:


Before the Court is a direct appeal from the decision
[1]
of the Regional Trial Court (RTC)
of Laoag City, Branch 11, elevated via a petition for review on certiorari
[2]
under Rule 45 of the Rules of
Court (present petition).

Petitioner Gerbert R. Corpuz was a former Filipino citizen who acquired Canadian citizenship
through naturalization on November 29, 2000.
[3]
On January 18, 2005, Gerbert married respondent
Daisylyn T. Sto. Tomas, a Filipina, in Pasig City.
[4]
Due to work and other professional commitments,
Gerbert left for Canada soon after the wedding. He returned to the Philippines sometime in April 2005 to
surprise Daisylyn, but was shocked to discover that his wife was having an affair with another man. Hurt
and disappointed, Gerbert returned to Canada and filed a petition for divorce. The Superior Court of
Justice, Windsor, Ontario, Canada granted Gerberts petition for divorce onDecember 8, 2005. The
divorce decree took effect a month later, on January 8, 2006.
[5]


Two years after the divorce, Gerbert has moved on and has found another Filipina to
love. Desirous of marrying his new Filipina fiance in the Philippines, Gerbert went to the Pasig City Civil
Registry Office and registered the Canadian divorce decree on his and Daisylyns marriage
certificate. Despite the registration of the divorce decree, an official of the National Statistics Office (NSO)
informed Gerbert that the marriage between him and Daisylyn still subsists under Philippine law; to be
enforceable, the foreign divorce decree must first be judicially recognized by a competent Philippine
court, pursuant to NSO Circular No. 4, series of 1982.
[6]


Accordingly, Gerbert filed a petition for judicial recognition of foreign divorce and/or
declaration of marriage as dissolved (petition) with the RTC. Although summoned, Daisylyn did not file
any responsive pleading but submitted instead a notarized letter/manifestation to the trial court. She
offered no opposition to Gerberts petition and, in fact, alleged her desire to file a similar case herself but
was prevented by financial and personal circumstances. She, thus, requested that she be considered as
a party-in-interest with a similar prayer to Gerberts.

In its October 30, 2008 decision,
[7]
the RTC denied Gerberts petition. The RTC concluded that
Gerbert was not the proper party to institute the action for judicial recognition of the foreign divorce
decree as he is a naturalized Canadian citizen. It ruled that only the Filipino spouse can avail of the
remedy, under the second paragraph of Article 26 of the Family Code,
[8]
in order for him or her to be able
to remarry under Philippine law.
[9]
Article 26 of the Family Code reads:

Art. 26. All marriages solemnized outside the Philippines, in accordance with the
laws in force in the country where they were solemnized, and valid there as such, shall
also be valid in this country, except those prohibited under Articles 35(1), (4), (5) and (6),
36, 37 and 38.

Where a marriage between a Filipino citizen and a foreigner is validly
celebrated and a divorce is thereafter validly obtained abroad by the alien spouse
capacitating him or her to remarry, the Filipino spouse shall likewise have capacity
to remarry under Philippine law.

This conclusion, the RTC stated, is consistent with the legislative intent behind the enactment of the
second paragraph of Article 26 of the Family Code, as determined by the Court in Republic v. Orbecido
III;
[10]
the provision was enacted to avoid the absurd situation where the Filipino spouse remains married
to the alien spouse who, after obtaining a divorce, is no longer married to the Filipino spouse.
[11]


THE PETITION

From the RTCs ruling,
[12]
Gerbert filed the present petition.
[13]


Gerbert asserts that his petition before the RTC is essentially for declaratory relief, similar to that
filed in Orbecido; he, thus, similarly asks for a determination of his rights under the second paragraph of
Article 26 of the Family Code. Taking into account the rationale behind the second paragraph of Article 26
of the Family Code, he contends that the provision applies as well to the benefit of the alien spouse. He
claims that the RTC ruling unduly stretched the doctrine in Orbecido by limiting the standing to file the
petition only to the Filipino spouse an interpretation he claims to be contrary to the essence of the
second paragraph of Article 26 of the Family Code. He considers himself as a proper party, vested with
sufficient legal interest, to institute the case, as there is a possibility that he might be prosecuted for
bigamy if he marries his Filipina fiance in the Philippines since two marriage certificates, involving him,
would be on file with the Civil Registry Office. The Office of the Solicitor General and Daisylyn, in their
respective Comments,
[14]
both support Gerberts position.

Essentially, the petition raises the issue of whether the second paragraph of Article 26 of the
Family Code extends to aliens the right to petition a court of this jurisdiction for the recognition of
a foreign divorce decree.

THE COURTS RULING

The alien spouse can claim no right under the
second paragraph of Article 26 of the Family Code
as the substantive right it establishes is in favor of
the Filipino spouse

The resolution of the issue requires a review of the legislative history and intent behind the second
paragraph of Article 26 of the Family Code.

The Family Code recognizes only two types of defective marriages void
[15]
and
voidable
[16]
marriages. In both cases, the basis for the judicial declaration of absolute nullity or annulment
of the marriage exists before or at the time of the marriage. Divorce, on the other hand, contemplates the
dissolution of the lawful union for cause arising afterthe marriage.
[17]
Our family laws do not recognize
absolute divorce between Filipino citizens.
[18]


Recognizing the reality that divorce is a possibility in marriages between a Filipino and an alien,
President Corazon C. Aquino, in the exercise of her legislative powers under the Freedom
Constitution,
[19]
enacted Executive Order No. (EO) 227, amending Article 26 of the Family Code to its
present wording, as follows:

Art. 26. All marriages solemnized outside the Philippines, in accordance with the
laws in force in the country where they were solemnized, and valid there as such, shall
also be valid in this country, except those prohibited under Articles 35(1), (4), (5) and (6),
36, 37 and 38.

Where a marriage between a Filipino citizen and a foreigner is validly
celebrated and a divorce is thereafter validly obtained abroad by the alien spouse
capacitating him or her to remarry, the Filipino spouse shall likewise have capacity
to remarry under Philippine law.

Through the second paragraph of Article 26 of the Family Code, EO 227 effectively incorporated into the
law this Courts holding in Van Dorn v. Romillo, Jr.
[20]
and Pilapil v. Ibay-Somera.
[21]
In both cases, the
Court refused to acknowledge the alien spouses assertion of marital rights after a foreign courts divorce
decree between the alien and the Filipino. The Court, thus, recognized that the foreign divorce had
already severed the marital bond between the spouses. The Court reasoned in Van Dorn v. Romillo that:

To maintain x x x that, under our laws, [the Filipino spouse] has to be
considered still married to [the alien spouse] and still subject to a wife's
obligations x x x cannot be just. [The Filipino spouse] should not be obliged to live
together with, observe respect and fidelity, and render support to [the alien spouse]. The
latter should not continue to be one of her heirs with possible rights to conjugal
property. She should not be discriminated against in her own country if the ends of
justice are to be served.
[22]



As the RTC correctly stated, the provision was included in the law to avoid the absurd situation
where the Filipino spouse remains married to the alien spouse who, after obtaining a divorce, is no longer
married to the Filipino spouse.
[23]
The legislative intent is for the benefit of the Filipino spouse, by
clarifying his or her marital status, settling the doubts created by the divorce decree. Essentially, the
second paragraph of Article 26 of the Family Code provided the Filipino spouse a substantive
right to have his or her marriage to the alien spouse considered as dissolved, capacitating him or
her to remarry.
[24]
Without the second paragraph of Article 26 of the Family Code, the judicial recognition
of the foreign decree of divorce, whether in a proceeding instituted precisely for that purpose or as a
related issue in another proceeding, would be of no significance to the Filipino spouse since our laws do
not recognize divorce as a mode of severing the marital bond;
[25]
Article 17 of the Civil Code provides that
the policy against absolute divorces cannot be subverted by judgments promulgated in a foreign
country. The inclusion of the second paragraph in Article 26 of the Family Code provides the direct
exception to this rule and serves as basis for recognizing the dissolution of the marriage between the
Filipino spouse and his or her alien spouse.

Additionally, an action based on the second paragraph of Article 26 of the Family Code is not
limited to the recognition of the foreign divorce decree. If the court finds that the decree capacitated the
alien spouse to remarry, the courts can declare that the Filipino spouse is likewise capacitated to contract
another marriage. No court in this jurisdiction, however, can make a similar declaration for the alien
spouse (other than that already established by the decree), whose status and legal capacity are generally
governed by his national law.
[26]


Given the rationale and intent behind the enactment, and the purpose of the second paragraph of
Article 26 of the Family Code, the RTC was correct in limiting the applicability of the provision for the
benefit of the Filipino spouse. In other words, only the Filipino spouse can invoke the second paragraph
of Article 26 of the Family Code; the alien spouse can claim no right under this provision.


The foreign divorce decree is presumptive evidence
of a right that clothes the party with legal interest to
petition for its recognition in this jurisdiction

We qualify our above conclusion i.e., that the second paragraph of Article 26 of the Family
Code bestows no rights in favor of aliens with the complementary statement that this conclusion is not
sufficient basis to dismiss Gerberts petition before the RTC. In other words, the unavailability of the
second paragraph of Article 26 of the Family Code to aliens does not necessarily strip Gerbert of legal
interest to petition the RTC for the recognition of his foreign divorce decree. The foreign divorce decree
itself, after its authenticity and conformity with the aliens national law have been duly proven according to
our rules of evidence, serves as a presumptive evidence of right in favor of Gerbert, pursuant to Section
48, Rule 39 of the Rules of Court which provides for the effect of foreign judgments. This Section states:

SEC. 48. Effect of foreign judgments or final orders.The effect of a judgment
or final order of a tribunal of a foreign country, having jurisdiction to render the
judgment or final order is as follows:

(a) In case of a judgment or final order upon a specific thing, the
judgment or final order is conclusive upon the title of the thing; and

(b) In case of a judgment or final order against a person, the
judgment or final order is presumptive evidence of a right as
between the parties and their successors in interest by a
subsequent title.

In either case, the judgment or final order may be repelled by evidence of a want
of jurisdiction, want of notice to the party, collusion, fraud, or clear mistake of law or fact.

To our mind, direct involvement or being the subject of the foreign judgment is sufficient to clothe a party
with the requisite interest to institute an action before our courts for the recognition of the foreign
judgment. In a divorce situation, we have declared, no less, that the divorce obtained by an alien abroad
may be recognized in the Philippines, provided the divorce is valid according to his or her national law.
[27]


The starting point in any recognition of a foreign divorce judgment is the acknowledgment that our
courts do not take judicial notice of foreign judgments and laws.

Justice Herrera explained that, as a rule,
no sovereign is bound to give effect within its dominion to a judgment rendered by a tribunal of another
country.
[28]
This means that the foreign judgment and its authenticity must be proven as facts under our
rules on evidence, together with the aliens applicable national law to show the effect of the judgment on
the alien himself or herself.
[29]
The recognition may be made in an action instituted specifically for the
purpose or in another action where a party invokes the foreign decree as an integral aspect of his claim or
defense.

In Gerberts case, since both the foreign divorce decree and the national law of the alien,
recognizing his or her capacity to obtain a divorce, purport to be official acts of a sovereign authority,
Section 24, Rule 132 of the Rules of Court comes into play. This Section requires proof, either by (1)
official publications or (2) copies attested by the officer having legal custody of the documents. If the
copies of official records are not kept in the Philippines, these must be (a) accompanied by a certificate
issued by the proper diplomatic or consular officer in the Philippine foreign service stationed in the foreign
country in which the record is kept and (b) authenticated by the seal of his office.

The records show that Gerbert attached to his petition a copy of the divorce decree, as well as
the required certificates proving its authenticity,
[30]
but failed to include a copy of the Canadian law on
divorce.
[31]
Under this situation, we can, at this point, simply dismiss the petition for insufficiency of
supporting evidence, unless we deem it more appropriate to remand the case to the RTC to determine
whether the divorce decree is consistent with the Canadian divorce law.

We deem it more appropriate to take this latter course of action, given the Article 26 interests that
will be served and the Filipina wifes (Daisylyns) obvious conformity with the petition. A remand, at the
same time, will allow other interested parties to oppose the foreign judgment and overcome a petitioners
presumptive evidence of a right by proving want of jurisdiction, want of notice to a party, collusion, fraud,
or clear mistake of law or fact. Needless to state, every precaution must be taken to ensure conformity
with our laws before a recognition is made, as the foreign judgment, once recognized, shall have the
effect of res judicata
[32]
between the parties, as provided in Section 48, Rule 39 of the Rules of Court.
[33]


In fact, more than the principle of comity that is served by the practice of reciprocal recognition of
foreign judgments between nations, the res judicata effect of the foreign judgments of divorce serves as
the deeper basis for extending judicial recognition and for considering the alien spouse bound by its
terms. This same effect, as discussed above, will not obtain for the Filipino spouse were it not for the
substantive rule that the second paragraph of Article 26 of the Family Code provides.

Considerations beyond the recognition of the
foreign divorce decree

As a matter of housekeeping concern, we note that the Pasig City Civil Registry Office has
already recorded the divorce decree on Gerbert and Daisylyns marriage certificate based on the
mere presentation of the decree.
[34]
We consider the recording to be legally improper; hence, the need
to draw attention of the bench and the bar to what had been done.

Article 407 of the Civil Code states that [a]cts, events and judicial decrees concerning the civil
status of persons shall be recorded in the civil register. The law requires the entry in the civil registry of
judicial decrees that produce legal consequences touching upon a persons legal capacity and status, i.e.,
those affecting all his personal qualities and relations, more or less permanent in nature, not ordinarily
terminable at his own will, such as his being legitimate or illegitimate, or his being married or not.
[35]


A judgment of divorce is a judicial decree, although a foreign one, affecting a persons legal
capacity and status that must be recorded. In fact, Act No. 3753 or the Law on Registry of Civil Status
specifically requires the registration of divorce decrees in the civil registry:

Sec. 1. Civil Register. A civil register is established for recording the civil
status of persons, in which shall be entered:

(a) births;
(b) deaths;
(c) marriages;
(d) annulments of marriages;
(e) divorces;
(f) legitimations;
(g) adoptions;
(h) acknowledgment of natural children;
(i) naturalization; and
(j) changes of name.

x x x x

Sec. 4. Civil Register Books. The local registrars shall keep and preserve in their
offices the following books, in which they shall, respectively make the proper entries
concerning the civil status of persons:

(1) Birth and death register;

(2) Marriage register, in which shall be entered not only the marriages
solemnized but also divorces and dissolved marriages.

(3) Legitimation, acknowledgment, adoption, change of name and naturalization
register.


But while the law requires the entry of the divorce decree in the civil registry, the law and the submission
of the decree by themselves do not ipso facto authorize the decreesregistration. The law should be
read in relation with the requirement of a judicial recognition of the foreign judgment before it can be
given res judicata effect. In the context of the present case, no judicial order as yet exists recognizing the
foreign divorce decree. Thus, the Pasig City Civil Registry Office acted totally out of turn and without
authority of law when it annotated the Canadian divorce decree on Gerbert and Daisylyns marriage
certificate, on the strength alone of the foreign decree presented by Gerbert.

Evidently, the Pasig City Civil Registry Office was aware of the requirement of a court recognition,
as it cited NSO Circular No. 4, series of 1982,
[36]
and Department of Justice Opinion No. 181, series of
1982
[37]
both of which required a final order from a competent Philippine court before a foreign
judgment, dissolving a marriage, can be registered in the civil registry, but it, nonetheless, allowed the
registration of the decree. For being contrary to law, the registration of the foreign divorce decree without
the requisite judicial recognition is patently void and cannot produce any legal effect.

Another point we wish to draw attention to is that the recognition that the RTC may extend to the
Canadian divorce decree does not, by itself, authorize the cancellation of the entry in the civil registry. A
petition for recognition of a foreign judgment is not the proper proceeding, contemplated under the Rules
of Court, for the cancellation of entries in the civil registry.

Article 412 of the Civil Code declares that no entry in a civil register shall be changed or
corrected, without judicial order. The Rules of Court supplements Article 412 of the Civil Code by
specifically providing for a special remedial proceeding by which entries in the civil registry may be
judicially cancelled or corrected. Rule 108 of the Rules of Court sets in detail the jurisdictional and
procedural requirements that must be complied with before a judgment, authorizing the cancellation or
correction, may be annotated in the civil registry. It also requires, among others, that the verified petition
must be filed with the RTC of the province where the corresponding civil registry is located;
[38]
that the civil
registrar and all persons who have or claim any interest must be made parties to the proceedings;
[39]
and
that the time and place for hearing must be published in a newspaper of general circulation.
[40]
As these
basic jurisdictional requirements have not been met in the present case, we cannot consider the petition
Gerbert filed with the RTC as one filed under Rule 108 of the Rules of Court.

We hasten to point out, however, that this ruling should not be construed as requiring two separate
proceedings for the registration of a foreign divorce decree in the civil registry one for recognition of the
foreign decree and another specifically for cancellation of the entry under Rule 108 of the Rules of
Court. The recognition of the foreign divorce decree may be made in a Rule 108 proceeding itself, as the
object of special proceedings (such as that in Rule 108 of the Rules of Court) is precisely to establish the
status or right of a party or a particular fact. Moreover, Rule 108 of the Rules of Court can serve as the
appropriate adversarial proceeding
[41]
by which the applicability of the foreign judgment can be measured
and tested in terms of jurisdictional infirmities, want of notice to the party, collusion, fraud, or clear mistake
of law or fact.

WHEREFORE, we GRANT the petition for review on certiorari, and REVERSE the October 30,
2008 decision of the Regional Trial Court of Laoag City, Branch 11, as well as its February 17,
2009 order. We order the REMAND of the case to the trial court for further proceedings in accordance
with our ruling above. Let a copy of this Decision be furnished the Civil Registrar General. No costs.
SO ORDERED.


ARTURO D. BRION
Associate Justice


















THIRD DIVISION
[G.R. No. 121397. April 17, 1997]
RADIO COMMUNICATIONS OF THE PHILIPPINES, INC. (RCPI), petitioner, vs. COURT OF
APPEALS, DAITY SALVOSA, and RAY DEAN SALVOSA,respondents.
D E C I S I O N
FRANCISCO, J .:
Assailed in this petition for certiorari under Rule 65 is the decision of the Court of Appeals dated May
31, 1995 and its resolution of July 26, 1995 in CA-G. R. SP No. 33962, upholding private respondents'
right to file an amended complaint for damages against herein petitioner.
As a result of its failure to transmit private respondent Daity E. Salvosa's telegram on time, petitioner,
Radio Communications of the Philippines, Inc. (RCPI), was sued by private respondents Spouses
Salvosa for damages before the Regional Trial Court of Baguio City. RCPI moved for the dismissal of the
complaint on the ground of failure to state a cause of action, alleging that in the absence of any allegation
of fraud or bad faith, petitioner cannot be held liable for moral and exemplary damages and attorney's
fees.
[1]

In its order dated August 5, 1992, the trial court dismissed the complaint for failure to state a cause
of action.
[2]

Before receipt of their copy of the order of dismissal, private respondents filed an amended
complaint, this time alleging bad faith on petitioner's part.
[3]
Upon receipt of the order of dismissal on
August 24, 1992, private respondents then filed a motion for reconsideration on September 4,
1992.
[4]
Petitioner opposed the motion for reconsideration on the ground that the same was filed out of
time and argued that amendment of the complaint is improper because the purpose is to introduce a
subsequently accrued action.
[5]

On January 13, 1993, the trial court issued an order granting private respondents' motion for
reconsideration and set aside its previous order of dismissal. The trial court ruled that notwithstanding
the previous order of dismissal, private respondents are still entitled as a matter of right to amend their
complaint since the order has not become final and there was yet no responsive pleading served upon
them.
[6]

Petitioner in turn moved for reconsideration but the trial court denied the same in its order dated
March 18, 1994.
[7]

Thereafter, petitioner proceeded to the Court of Appeals through a petition for certiorari imputing
grave abuse of discretion on the trial court for setting aside its initial order dismissing private respondents'
complaint and in admitting their amended complaint. Petitioner maintained that private respondents
cannot amend their complaint by changing the theory of the case from one of negligence to a case of bad
faith so as to confer jurisdiction to the trial court.
[8]

On May 31, 1995, the Court of Appeals rendered the assailed decision dismissing petitioner's
petition and upheld private respondents' right to file an amended complaint under Section 2 of Rule 10 of
the Rules of Court. It also declared that the allegation of bad faith in the amended complaint did not in any
way change private respondents' theory of the case or their cause of action, that is, for breach of
contract.
[9]
Petitioner sought reconsideration but respondent court denied the same in its resolution dated
August 18, 1995.
[10]
Hence this petition.
Petitioner's objection to the admission of private respondents' amended complaint is anchored on the
pronouncement laid out in the case of Torres vs. Tomacruz,
[11]
which imposed a limitation on the
procedural right to amend a complaint, that is, the cause of action or defense shall not be substantially
changed, or that the theory of the case shall not be altered.
On the other hand, private respondents denied that they changed their theory in their amended
complaint which has consistently been an action for breach of contract. Nevertheless, even assuming
that there was any switch in their theory of the case, private respondents argued that the same is allowed
under Section 2 of Rule 10 of the Rules of Court because no responsive pleading has yet been filed by
petitioner.
There is no merit in the petition. Section 2, Rule 10 of the Rules of Court is very explicit as to when
amendments are allowed as a matter of right, to wit:
"Sec. 2. When amendments allowed as a matter of right. A party may amend his pleading once as a
matter of course at any time before a responsive pleading is served or, if the pleading is one to which no
responsive pleading is permitted and the action has not been placed upon the trial calendar, he may so
amend it at any time within ten (10) days after it is served."
Undoubtedly, no responsive pleading has been filed prior to the submission by private respondents
of an amended complaint. The motion to dismiss previously filed by petitioner is definitely not a
responsive pleading,
[12]
hence the admission of the amended complaint was properly made. Before the
filing of any responsive pleading, a party has the absolute right to amend his pleading whether a new
cause of action or change in theory is introduced.
[13]
Contrary to petitioner's assertion, the limitation
imposed in the Torres case (supra) does not apply herein because of the difference in the factual
circumstances. In the aforecited case, the amended complaint which introduced a new cause of action
was discarded because it was filed after an answer has already been adduced. However, in the instant
case, the amended complaint was filed prior to the submission of any responsive pleading.
Furthermore, anent petitioner's apprehension that a party's right to amend a pleading will be subject
to abuse if the same can be done without leave of court in cases like this one, the same is clearly
baseless in view of the limitation imposed in Sec. 2 of Rule 10 which allows the amendment "only once as
a matter of course."
WHEREFORE, for lack of merit, the petition is hereby DENIED.
SO ORDERED.
Narvasa, C.J., (Chairman), Davide, Jr., Melo, and Panganiban, JJ., concur.









EN BANC
[A.M. No. P-03-1697. October 1, 2003]
JOCELYN S. PAISTE, complainant, vs. APRONIANO V. MAMENTA, JR., Clerk of Court II, Municipal
Circuit Trial Court, Tayug-San Nicolas, Pangasinan, respondent.
[A.M. No. P-03-1699. October 1, 2003]
JOANNE S. GOLTIAO, complainant, vs. APRONIANO V. MAMENTA, JR., Clerk of Court II,
Municipal Circuit Trial Court, Tayug-San Nicolas, Pangasinan, respondent.
D E C I S I O N
PER CURIAM:
These consolidated administrative cases arose from two different complaints filed against
respondent Aproniano V. Mamenta, Jr., Clerk of Court II, Municipal Circuit Trial Court of Tayug-San
Nicolas, Pangasinan. In an affidavit-complaint filed with the Office of the Chief Justice,
[1]
dated August
10, 2001, complainant Joanne S. Goltiao charged him with gambling and drinking liquor during office
hours, sexual harassment, arrogance and acts unbecoming of a government official. In another affidavit-
complaint filed with the Office of the Court Administrator,
[2]
dated August 28, 2001, the other complainant
Jocelyn C. Paiste charged him with conduct unbecoming of a public officer and with violation of the Anti-
Graft and Corrupt Practices Act
[3]
for his failure to issue official receipt.
After respondent denied the accusations against him in separate letters,
[4]
we resolved
[5]
to refer both
cases to Hon. Ulysses Butuyan, Executive Judge of the Regional Trial Court of Tayug, Pangasinan for
investigation, report and recommendation.
[6]
Separate investigations of the complaints were conducted.
In A.M. No. P-03-1699 (formerly OCA I.P.I. No. 01-1202-P), evidence shows that complainant
Goltiao is a Stenographer I of the MCTC of Tayug-San Nicolas since 1997. She testified
[7]
that on August
7, 2000, at about 3:00 p.m., a representative from the Plaridel Insurance Co. came to their office seeking
clearance. She immediately prepared the necessary form and, together with the representative, went to
see respondent in the courtroom to obtain his signature. When she asked him to sign the document,
respondent, who was at that time playing tong-its (a card game) at the lawyers table with unnamed
individuals, got angry and threw his cards. He shouted at her: Why did you bring them with you? Did you
like them to bring me to the Supreme Court? She responded that such was not her intention and
reminded him of his requirement that he must first see the applicants before he sign their clearance. He
did not sign the clearance, sent then out and shouted Bullshit ka! at her thrice. They all then went out of
the courtroom and proceeded back to the staff room. She went to her table and buried her face in her
hands, crying. Respondent followed her and continued uttering unsavory remarks: Bullshit ka! Vulva of
your mother! Why did you take the client there and even raised your voice? (Bullshit ka! Okinnam nga
babai! Apay ta innalam dagita kliyente idiay sanak to rinayawan!) She replied that her parents taught
them not to answer back at older people. He still shouted: Vulva of your mother! I wish you will die now!
Whom are you bragging of? We will try each other. (Okinnam nga babai! Matay ka koma itattan! Apay
sinno aya ti paglaslastog mo? Sige, agpipinnadas tayo.) Thereafter, he asked the utility aide to buy him
four bottles of beer.
Goltiao declared that her working relationship with the respondent is sometimes good and
sometimes bad because of his ill temper.
[8]
He easily gets mad at her even for small, trivial
mistakes. This situation started, according to her, when she told him to stop courting and sending her
love notes as she is already a married woman. She related an incident which happened early one
morning when he asked her to see him inside the judges chamber. At that time, the designated judge
was not around. Once inside, she was told to sit in one of the chairs in front of the judges table. The
respondent, who was sitting at the judges chair, then extended his hand to her, as if he wanted to shake
her hands. She reciprocated by extending her hands and jokingly put his hands on her forehead
(agmanmano). She afterwards tried to free her hands off his but he would no let her. Instead, he told
her, Wait for a while, I would just like to tell you something. I love you, is that okay? Tell me that you
love me too. No strings attached. She retorted, As if you are my father. Spurned, he got mad. This
kind of incident happened at least ten more times.
Furthermore, respondent sent her love notes. He wrote his love messages on pieces of paper in
front of her and handed them to her.
[9]
She knew that they were intended for her as there was nobody
else present when he gave the notes to her. He would also call her at her mothers house. She did not
respond affirmatively to his display of affection as both of them are married. Finally, she explained that
she filed a complaint against him only on August 10, 2001 because of the August 7, 2001 incident, when
she got fed up at the way he treated her.
Marilyn de Leon, also a court stenographer of MCTC Tayug-San Nicolas, corroborated the testimony
of Goltiao.
[10]
She stated that she witnessed the incident on August 7, 2001, when respondent became
angry at Goltiao after the latter went to ask for his signature while he was playing tong-its. She heard
respondent shout foul words at Goltiao. She added that respondent gets mad at everyone at the office on
trivial matter. She, too, received love notes from the respondent.
Court stenographer Glenda C. Ramirez also corroborated the testimony of Goltiao.
[11]
She declared
that she witnessed the incident on August 7, 2001 at their office. She observed that respondent was
fuming mad at Goltiao, when the latter asked him to sign a clearance form. She heard respondent shout
at Goltiao inside the courtroom and at the staff room. Respondent got mad when Goltiao allowed the
representatives of the insurance company to enter the courtroom, where he was playing tong-its. She
related that she also received love notes from him. One time, after respondent signed her travel order
she noticed the following message written on the order: 7:30 in the evening, Jollibee. She did not go out
with him. She chose him as a principal sponsor on her wedding in the hope that he would treat her like
his own child and that he would stop sending notes to her. This did not happen even as respondent
continued to easily get mad at her for simple lapses.
Renato Rombawa, a utility worker at the court, saw the incident on August 7, 2001, when respondent
got mad at Goltiao inside the courtroom.
[12]
He did not know why he became furious. At the time of the
incident, respondent was playing tong-its.
Estifanio Acosta, Clerk III at the MCTC, stated that he likewise saw respondent got mad when
Goltiao tried to obtain his signature for a clearance form.
[13]
Respondent, he recalled, was at that time
playing tong-its. He knew that respondent gives love notes to Goltiao and Glenda Ramirez, as the two
revealed this secret to him.
Lastly, David Kagaoan testified that at about 3:00 oclock in the afternoon of August 7, 2001,
somebody came to get a clearance from the court.
[14]
The clearance form was prepared by Goltiao who
went to see the respondent at the courtroom. He then heard shouts of vulva of your mother, get out of
here! from the room. He thought it came from the respondent, who was then playing tong-
its. Rombawa, the utility worker, went inside the courtroom and brought the sobbing Goltiao out. He
stated that complainant showed him some of the love notes respondent gave her. He knows it came from
respondent as he is familiar with his signature.
Respondent denied the allegations against him.
[15]
He admitted shouting at Goltiao because she, too,
shouted at him. He declared that the complaint was intended to harass him and put him into shame and
to remove him from office. He said that Goltiao and her witnesses resent his strictness and they want to
continue with their bad ways in the office. Goltiao, Ramirez and de Leon come to office late but Goltiaos
daily time record does not reflect her tardiness. She sells stenographic notes without remitting a single
centavo to the Judiciary Development Fund (JDF). Witness Rombawa does not work in the office from
2:00 to 5:00 oclock in the afternoon. Witness Acosta is a habitual absentee, reports at 10:00 oclock and
leaves after only an hour. He then returns at 2:00 oclock in the afternoon. He is hired by one Bobby Go
as a driver. Witness Kagaoan is a habitual latecomer as he attends to his agricultural business first
before going to the court. He adds that witnesses Rombawa, Acosta and Kagaoan all play tong-its either
at the session hall of the court or the first floor of the building.
On rebuttal, complainant Goltiao revealed that respondent invited her several times for a date at
Jollibee, instructing her not to tell her husband about it.
[16]
Respondent countered that every time he
invites her, it was with de Leon, Ramirez and other court personnel.
[17]

In A.M. No. P-03-1697 (formerly OCA I.P.I. No. 01-1196-P), complainant Paiste is the owner of
JCP72 Insurance Agency, and as its proprietor, has transactions with the MCTC Tayug-San
Nicolas.
[18]
She testified that on April 3, 2001 at about 2:00 p.m. she went to the court, together with a
client, to post bail. When they arrived, they could not find the respondent. An employee named Marilyn
accompanied then to one of the rooms at the hall of justice where the respondent was holding game
cards and playing tong-its with some people. Marilyn called respondents attention telling him that
somebody wants to post bail. Respondent retorted, You just go ahead upstairs. Why, could you not
wait? They went ahead and respondent followed fifteen minutes after. She noticed that he was
mad. They told him they want to post bail and he asked for the records of the case. After examining the
records, he slammed it and said, You go to Rosales! That is not our work! He also became angry when
they followed him.
On April 20, 2001, Paiste, an officemate and a client again went to the MCTC in Tayug-San Nicolas
to post bail in connection with Criminal Case No. 7461. The respondent instructed them to go to the
house of Judge Pastor. They went there as told but the judge rejected their papers. They returned the
next day, a Saturday, after completing their documents. They came from the court where they met
respondent, who accompanied them to the judges house. The judge approved the surety bond.
[19]
Later,
they handed three thousand pesos (P3,000.00) to the respondent, two thousand four hundred pesos
(P2,400.00) of which was for the JDF. They gave him the remaining six hundred pesos (P600.00) after
he asked them, Paano naman yung sa akin? They understood that to mean that he was expecting some
form of compensation as he accompanied them outside the office on a non-working day. They then
demanded an official receipt for the two thousand four hundred pesos (P2,400.00) they paid him
corresponding to the amount of the JDF. He told them to go to the court and get it the following
Monday. That Monday, she sent one of her staff to get the receipt, but the latter reported to her that he
did not issue one. She added that this is not the only time that he failed to issue her a receipt. In other
cases, he also neglected to do so even after she has paid the mandatory JDF fees.
A staff member of the MCTC Tayug-San Nicolas, Estifanio Acosta, verified the records of Criminal
Case No. 7461 and stated that no official receipt appears on file although the personal bail bonds issued
by the JCP72 Insurance Agency, bearing the date April 21, 2001, in favor of the several accused in the
case were approved by the acting presiding judge of the court.
[20]

For his part, respondent merely denied the foregoing allegations in open court and waived
presentation of evidence.
[21]

After investigation, the investigating judge rendered a Consolidated Report and
Recommendation
[22]
finding respondent guilty of serious misconduct and accordingly proposed that he be
sternly reprimanded and fined ten thousand pesos (P10,000.00), thus:
Nonetheless, on the basis of the evidence adduced, barely refuted by the respondent, the undersigned
finds him guilty of serious misconduct in office, consisting of abundantly substantiated offenses thus:
1. Making undue advances through love notes to female subordinates;
2. Indulging in card games during office hours;
3. Using abusive language in dealing with subordinates as well as with third persons
seeking his services as clerk of court; and
4. Receiving payments for court fees without promptly issuing the corresponding
receipts therefore.
WHEREFORE, all of the foregoing premises and circumstances considered, the undersigned hereby
respectfully recommends that respondent be sternly reprimanded, and fined in the amount of P10,000.00,
for serious misconduct.
[23]

The report was forwarded to the Office of the Court Administrator (OCA). After evaluating the report
and the records, the OCA proposed that a stiffer penalty must be imposed on respondent. On the basis
of Sections 52, 54 and 55 of Memorandum Circular No. 19, series of 1999 of the Civil Service
Commission,
[24]
it recommended that the two complaints be consolidated and docketed as a regular
administrative case and that respondent be SUSPENDED for a period of one (1) year having been found
Guilty of various offenses including the (sic) Grave Offenses and disgraceful and immoral conduct.
[25]

A painstaking examination of the records of the cases convinces us that the respondent is guilty of
the offenses charged against him.
The evidence in A.M. No. P-03-1699 adequately establishes that he was guilty of gambling, using
abusive language and sexually harassing his female subordinates. We find credible the categorical and
straightforward testimonies of complainant Goltiao and her witnesses. Their frank and candid
testimonies, unshaken by cross-examination and unflawed by inconsistencies or contradictions in their
material points, deserve our full faith and belief.
[26]

In contrast, we are not persuaded by the denial put forward by the respondent. The purported
motive he imputes against complainant and her witnesses for filing this case is not worthy of
attention. He asserts that they just want him removed so they can continue with their bad ways in the
office. It strikes us strange that he allowed these bad ways for a long time without taking any measures
or reporting them to the OCA.
Time and again, we have emphasized that court officers, circumscribed with heavy responsibility,
must be the paragon of propriety and good behavior.
[27]
This is especially true for a clerk of court like the
respondent. As a ranking officer of the court, it is incumbent upon him to set an example to his co-
employees as to how they should conduct themselves in office; to see to it that his subordinates work
efficiently in accordance with the rules and regulations of the civil service and the judiciary; and to provide
then with a healthy working atmosphere wherein co-workers treat each other with respect, courtesy and
cooperation, so that in the end public interest will be benefited.
[28]

Respondent failed to measure up to these standards. His use of offensive language towards
complainant Goltiao reflects his impoliteness and lack of decorum. He shouted profanity at her, verbally
abused her and even disrespected her mother. He did this in the presence of so many employees and
clients of the court. Worse, there was no reason for him to unleash to her a fury of this kind. She was
merely asking him to sign a clearance form and to examine the applicant pursuant to his specific
instruction that he should first see the applicant for the clearance before he sign the form. In fine, she
was just doing her job. The truth is, he got mad at her out of his fear that the client she brought along
might report him to this court when they caught him in flagrante delicto gambling.
Similarly objectionable is respondents penchant for playing tong-its (a card game) and gambling with
other employees of the court. Gambling is illegal and is absolutely forbidden at court premises during
office hours.
[29]
It generates unwholesome consequences on the gambler as it diverts his attention from
the more important responsibilities of his job.
[30]
Respondent himself was very much aware of this
prohibition and this is exactly the reason why he castigated the complainant for approaching him while he
was playing cards, thus: Why did you bring then with you? Did you like them to bring me to the Supreme
Court?
[31]

Worse, respondent was playing cards and gambling during office hours at the session hall of the
courtroom, which is hardly the place for such undesirable activities. A courtroom is generally looked upon
by people with high respect and regarded as a sacred place where witnesses testify under oath, where
conflicts are resolved, rights adjudicated, and justice solemnly dispensed.
[32]
Making it a game room and a
casino area diminishes its sanctity and dignity.
[33]

The undue advances respondent made to complainant Goltiao betrays his twisted sense of
propriety. Many times, he declared his feelings for her and handed her love notes. He would then
beseech her to say the same things to him. He proposed to have dinner dates with her at Jollibee. There
were times that he cornered her at the judges chamber and unnecessarily held her hand. While
professing ones amorous intention is not something that usually causes a hullabaloo, it becomes
indecent and improper in this case considering he is complainant Goltiaos superior and both of them are
married. His dissoluteness told itself when he went to the extent of calling her at her mothers house and
persuading her not to tell her husband about these incidents.
It appears too that the complainant was not the exclusive object of respondents
advances. Witnesses Marilyn de Leon and Glenda Ramirez testified that they also received love notes
and invitations for dinner from him. Like a hunter out on the prowl, he victimized other female workers
unabashedly professing his alleged feelings for them in utter disregard of the fact that they were his
subordinates, they were married and they were young enough to be his daughters. Instead of he being in
loco parentis over his subordinate employees, he preyed on them as he took advantage of his superior
position.
[34]

Under the circumstances, we find respondent guilty of sexual harassment. His severely outrageous
acts, which are an affront to women, constitute sexual harassment because they necessarily result in an
intimidating, hostile, and offensive working environment for his female subordinates.
[35]
He abused the
power and authority he exercises over them, which is the gravamen of the offense in sexual
harassment.
[36]
Sexual harassment in the workplace is not about a man taking advantage of a woman by
reason of sexual desire it is about power being exercised by a superior over his women
subordinates.
[37]
That power emanates from the fact that he can remove them if they refuse his amorous
advances.
[38]

There is likewise sufficient evidence in A.M. No. P-03-1697 to hold respondent administratively liable
for his failure to issue official receipt after receiving court fees and for discourtesy. The testimony of
complainant Paiste is worthy of belief. We find her categorical and consistent declarations credible,
especially when viewed in the light of the fact that no ill-motive on her part was established.
[39]
Against her
positive testimony, supported by documentary evidence based on official court records, the mere
uncorroborated denial of the respondent deserves scant considerations.
[40]

The evidence shows that on April 21, 2001, complainant Paiste gave to the respondent the amount
of two thousand four hundred pesos (P2,400.00) as bail bond of the accused in Criminal Case No.
7461. When she asked for an official receipt, he told her to get it the following Monday at the court. She
sent an office representative that Monday as told but respondent did not issue any receipt. Further
verification from the records of Criminal Case No. 7461 reveals that no copy of the official receipt appears
on file indicating that no receipt was ever issued to complainant Paiste.
Undoubtedly, respondent should have issued an official receipt when he received the sum of money
from complainant Paiste. His failure to do so is a violation of the National Accounting and Auditing
Manual which mandates that no payment of any nature shall be received by a collecting officer without
immediately issuing an official receipt, in acknowledgment thereof.
[41]
It bears emphasis that there is no
valid reason for his non-issuance of a receipt. Even if he argues that he could not issue an official receipt
to the complainant Paiste as the transaction transpired on a Saturday and outside court premises, still, his
failure to issue a receipt the following Monday is totally unjustified.
We also find the rude and boorish manner respondent treated complainant Paiste and her client on
April 3, 2001 uncalled for. His demeanor bordered on arrogance. He made them wait for fifteen minutes
before he attended to their concern as they found him playing cards and gambling. He was very irritable
the whole time he was talking to them. After he examined the record, he slammed it an told them to go to
the court at Rosales. He then got annoyed when they followed him. To be sure, this is no way to treat
court users.
High strung and belligerent behavior has no place in government, especially in the judiciary, where
the personnel are enjoined to act with self-restraint and civility at all times, even when confronted with
rudeness and insolence.
[42]
They are expected to extend prompt, courteous and adequate service to the
people.
[43]
Such conduct is exacted from them so that they will earn and keep societys high regard for
and confidence in the judicial service.
[44]
Conduct violative of this standard quickly and surely corrodes
respect for the courts. It is the imperative and sacred duty of everyone charged with the dispensation of
justice, from the judge to the lowliest clerk, to uphold the courts good name and standing as true temples
of justice.
[45]

We do not agree with both the investigating judge and the OCA when they imposed a single penalty
on the respondent for two cases. Although consolidated, the two cases are distinct and each one
contains different charges. Respondent, as a consequence, must be separately held liable and penalized
for the charges in each case.
This brings us to the individual penalties which must be imposed on the respondent for each
case. In A.M. No. P-03-1699, we hold that the gross discourtesy, gambling and sexual harassment he
committed constituted serious misconduct and conduct prejudicial to the interest of the service which
warrant his dismissal from office. He has been shown to lack the requisite professional and moral
qualifications to continue as an employee in the judiciary. In A.M. No. P-03-1697, his failure to issue a
receipt amounted to gross neglect of duty while his grossly discourteous behavior is conduct unbecoming
of a public officer. For this reason, we impose on him a suspension of one (1) month.
IN VIEW WHEREOF, RESPONDENT Aproniano V. Mamenta, Jr. is found Guilty of gross
discourtesy, gambling, and sexual harassment in A.M. No. P-03-1699 and is accordingly DISMISSED
from the service with prejudice to re-employment in any branch, agency, or instrumentality of the
Government, including government-owned and controlled corporations.
He is likewise found Guilty of gross discourtesy and failure to issue an official receipt in A.M. No. P-
03-1697 and is meted the penalty of suspension for a period of One (1) Month. Considering, however,
that we already ordered his dismissal in A.M. No. P-03-1699, his suspension is moot.
SO ORDERED.
Davide, Jr., C.J., Bellosillo, Puno, Vitug, Panganiban, Quisumbing, Ynares-Santiago, Sandoval-
Gutierrez, Carpio, Austria-Martinez, Corona, Carpio-Morales, Callejo, Sr., and Tinga, JJ.,concur.
Azcuna, J., on leave.










FIRST DIVISION


MAY D. AONUEVO,ALEXANDER
BLEE DESANTIS and JOHN DESANTIS
NERI,
Petitioners,




-versus-



INTESTATE ESTATE OF RODOLFO G.
JALANDONI,represented
by BERNARDINO G. JALANDONI as
Special Administrator,
Respondent.

G.R. No. 178221


Present:

CORONA, C.J.,
Chairperson,
LEONARDO-DE CASTRO,*
PERALTA,**
ABAD,*** and
PEREZ, JJ.





Promulgated:

December 1, 2010
x----------------------------------------------------------------------------------------------- x

D E C I S I O N


PEREZ, J .:


On appeal
[1]
is the Decision
[2]
dated 31 May 2007 of the Court of Appeals in CA-G.R. SP No.
00576. In the said decision, the Court of Appeals nullified, on certiorari, the Orders
[3]
of the Regional Trial
Court, Branch 40, of Negros Occidental (intestate court) allowing herein petitioners and their siblings
[4]
to
intervene in the estate proceedings of the late Rodolfo G. Jalandoni.
[5]
The decretal portion of the
decision of the appellate court reads:

ACCORDINGLY, the petition for certiorari is hereby GRANTED, the assailed
Orders dated July 2, 2004 and January 26, 2005, of the Regional Trial Court in Spec.
Proc. No. 338 are herebySET ASIDE and NULLIFIED, and a permanent injunction is
hereby issued enjoining respondents [petitioners], their agents and anyone acting for
and in their behalves, from enforcing the assailed Orders. No costs.
[6]


The antecedents are:

Rodolfo G. Jalandoni (Rodolfo) died intestate on 20 December 1966.
[7]
He died without issue.
[8]


On 28 April 1967, Bernardino G. Jalandoni (Bernardino), the brother of Rodolfo, filed a petition for
the issuance of letters of administration
[9]
with the Court of First Instance of Negros Occidental, to
commence the judicial settlement of the latters estate. The petition was docketed as Spec. Proc. No.
338 and is currently pending before the intestate court.
[10]


On 17 January 2003, the petitioners and their siblings filed a Manifestation
[11]
before the intestate
court. In the Manifestation, they introduced themselves as the children of Sylvia Blee Desantis (Sylvia)
who, in turn, was revealed to be the daughter of Isabel Blee (Isabel) with one John Desantis.
[12]


The petitioners and their siblings contend that their grandmotherIsabelwas, at the time of
Rodolfos death, the legal spouse of the latter.
[13]
For which reason, Isabel is entitled to a share in the
estate of Rodolfo.

Seeking to enforce the right of Isabel, the petitioners and their siblings pray that they be allowed
to intervene on her behalf in the intestate proceedings of the late Rodolfo G. Jalandoni.
[14]
As it was, by
the time the Manifestation was filed, both Sylvia and Isabel have already passed away with the former
predeceasing the latter.
[15]


To support their cause, the petitioners and their siblings appended in their Manifestation, the
following documents:

a.) Two (2) marriage certificates between Isabel and Rodolfo;
[16]


b.) The birth certificate of their mother, Sylvia;
[17]
and

c.) Their respective proof of births.
[18]


It is the assertion of the petitioners and their siblings that the foregoing pieces of evidence
sufficiently establish that Isabel was the spouse of Rodolfo, and that they are her lawful representatives.

The respondent intestate estate of Rodolfo G. Jalandoni, now represented by Bernardino as its
Special Administrator, however, begged to differ. It opposed the intervention on the ground that the
petitioners and their siblings have failed to establish the status of Isabel as an heir of Rodolfo. The very
evidence presented by the petitioners and their siblings showed that Isabel had a previous and subsisting
marriage with John Desantis at the time she was purportedly married to Rodolfo.

In its Comment to the Manifestation,
[19]
the respondent called attention to the entries in the birth
certificate of Sylvia, who was born on 14 February 1946.
[20]
As it turned out, the record of birth of Sylvia
states that she was a legitimate child of Isabel and John Desantis.
[21]
The document also certifies the
status of both Isabel and John Desantis as married.
[22]
The respondent posits that the foregoing entries,
having been made in an official registry, constitute prima facie proof of a prior marriage between Isabel
and John Desantis.
[23]


According to the respondent, Isabels previous marriage, in the absence of any proof that it was
dissolved, made her subsequent marriage with Rodolfo bigamous and voidab initio.
[24]


On 2 July 2004, the intestate court issued an order allowing the petitioners and their siblings to take
part in the settlement proceedings.
[25]
The intestate court was convinced that the evidence at hand
adequately establish Isabels status as the legal spouse of Rodolfo and, by that token, permitted the
petitioners and their siblings to intervene in the proceedings on her behalf.
[26]


The intestate court also held that the birth certificate of Sylvia was insufficient to prove that there
was a previous marriage between Isabel and John Desantis.
[27]
It ventured on the possibility that the
entries in the birth record of Sylvia regarding her legitimacy and the status of her parents, may have been
made only in order to save Isabel and her family from the social condemnation of having a child out of
wedlock.
[28]


The respondent sought for reconsideration, but was denied by the intestate court in its order
dated 26 January 2006.
[29]
Undeterred, the respondent hoisted a petition forcertiorari before the Court of
Appeals.

On 31 May 2007, the Court of Appeals granted the petition and nullified the orders of the intestate
court.
[30]


In coming to its conclusion, the Court of Appeals found that it was an error on the part of the
intestate court to have disregarded the probative value of Sylvias birth certificate.
[31]
The appellate court,
siding with the respondent, held that Sylvias birth certificate serves as prima facie evidence of the facts
therein statedwhich includes the civil status of her parents.
[32]
Hence, the previous marriage of Isabel
with John Desantis should have been taken as established.

The Court of Appeals added that since the petitioners and their siblings failed to offer any other
evidence proving that the marriage of Isabel with John Desantis had been dissolved by the time she was
married to Rodolfo, it then follows that the latter marriagethe Isabel-Rodolfo unionis a nullity for being
bigamous.
[33]
From that premise, Isabel cannot be considered as the legal spouse of Rodolfo. The
petitioners and their siblings, therefore, failed to show that Isabel has any interest in the estate of Rodolfo.

Hence, the instant appeal.
[34]


The sole issue in this appeal is whether the Court of Appeals erred when it nullified the orders of
the intestate court allowing the petitioners and their siblings to intervene in the settlement proceedings.

The petitioners answer in the affirmative. They proffer the following arguments:

One. The Court of Appeals exceeded the limits of review under a writ of certiorari.
[35]
In nullifying
the intestate courts order, the appellate court did not confine itself to the issue of whether the same was
issued with grave abuse of discretion.
[36]
Rather, it chose to re-assess the evidence and touch upon the
issue pertaining to Isabels right to inherit from Rodolfo.
[37]


Had the appellate court limited itself to the issue of whether grave abuse of discretion exists, it
would have found that the intestate court did not act whimsically or capriciously in issuing its assailed
orders.
[38]
Grave abuse of discretion on the part of the intestate court is belied by the fact that the said
orders may be supported by the two (2) marriage certificates between Isabel and Rodolfo.
[39]


Second. Assuming ex-gratia argumenti that the Court of Appeals was correct in addressing the
issue of whether there was sufficient evidence to prove that Isabel has a right to inherit from Rodolfo, it
nevertheless erred in finding that there was none.
[40]
A proper evaluation of the evidence at hand does
not support the conclusion that Isabel had a previous marriage with John Desantis.
[41]


To begin with, the respondent was not able to produce any marriage certificate executed between
Isabel and John Desantis.
[42]
The conspicuous absence of such certificate can, in turn, only lend
credibility to the position that no such marriage ever took place.

Moreover, the entries in the birth certificate of Sylvia do not carry the necessary weight to be able
to prove a marriage between Isabel and John Desantis.
[43]
In assessing the probative value of such
entries, the Court of Appeals should have taken note of a typical practice among unwed Filipino couples
who, in order to save face and not to embarrass their families, concoct the illusion of marriage and
make it appear that a child begot by them is legitimate.
[44]


Since the alleged previous marriage of Isabel with John Desantis was not satisfactorily proven,
the Court of Appeals clearly erred in finding that her marriage with Rodolfo is bigamous.

We are not impressed.

First Argument

The first argument raised by the petitioners is specious at best. The question of whether the
intestate court gravely abused its discretion is intricately linked with the issue of whether there was
sufficient evidence to establish Isabels status as the legal spouse of Rodolfo.

A courts power to allow or deny intervention, albeit discretionary in nature, is circumscribed by
the basic demand of sound judicial procedure that only a person withinterest in an action or proceeding
may be allowed to intervene.
[45]
Otherwise stated, a court has no authority to allow a person, who has no
interest in an action or proceeding, to intervene therein.
[46]


Consequently, when a court commits a mistake and allows an uninterested person to intervene in
a casethe mistake is not simply an error of judgment, but one of jurisdiction. In such event, the
allowance is made in excess of the courts jurisdiction and can only be the product of an exercise of
discretion gravely abused. That kind of error may be reviewed in a special civil action for certiorari.

Verily, the Court of Appeals was acting well within the limits of review under a writ of certiorari,
when it examined the evidence proving Isabels right to inherit from Rodolfo. The sufficiency or
insufficiency of such evidence determines whether the petitioners and their siblings have successfully
established Isabels interest in Rodolfos estatewhich, as already mentioned, is an indispensable
requisite to justify any intervention. Ultimately, the re-assessment of the evidence presented by the
petitioners and their siblings will tell if the assailed orders of the intestate court were issued in excess of
the latters jurisdiction or with grave abuse of discretion.

We now proceed to the second argument of the petitioners.


Second Argument

The second argument of the petitioners is also without merit. We agree with the finding of the
Court of Appeals that the petitioners and their siblings failed to offer sufficient evidence to establish that
Isabel was the legal spouse of Rodolfo. The very evidence of the petitioners and their siblings negates
their claim that Isabel has interest in Rodolfos estate.

Contrary to the position taken by the petitioners, the existence of a previous marriage between
Isabel and John Desantis was adequately established. This holds true notwithstanding the fact that no
marriage certificate between Isabel and John Desantis exists on record.

While a marriage certificate is considered the primary evidence of a marital union, it is not
regarded as the sole and exclusive evidence of marriage.
[47]
Jurisprudence teaches that the fact of
marriage may be proven by relevant evidence other than the marriage certificate.
[48]
Hence, even a
persons birth certificate may be recognized as competent evidence of the marriage between his
parents.
[49]


In the present case, the birth certificate of Sylvia precisely serves as the competent evidence of
marriage between Isabel and John Desantis. As mentioned earlier, it contains the following notable
entries: (a) that Isabel and John Desantis were married and (b) that Sylvia is their legitimate
child.
[50]
In clear and categorical language, Sylvias birth certificate speaks of a subsisting marriage
between Isabel and John Desantis.

Pursuant to existing laws,
[51]
the foregoing entries are accorded prima facie weight. They are
presumed to be true. Hence, unless rebutted by clear and convincing evidence, they can, and will, stand
as proof of the facts attested.
[52]
In the case at bench, the petitioners and their siblings offered no such
rebuttal.

The petitioners did no better than to explain away the entries in Sylvias birth certificate as
untruthful statements made only in order to save face.
[53]
They urge this Court to take note of a typical
practice among unwed Filipino couples to concoct the illusion of marriage and make it appear that a child
begot by them is legitimate. That, the Court cannot countenance.

The allegations of the petitioners, by themselves and unsupported by any other evidence, do not
diminish the probative value of the entries. This Court cannot, as the petitioners would like Us to do,
simply take judicial notice of a supposed folkway and conclude therefrom that the usage was in fact
followed. It certainly is odd that the petitioners would themselves argue that the document on which they
based their interest in intervention contains untruthful statements in its vital entries.
Ironically, it is the evidence presented by the petitioners and their siblings themselves which,
properly appreciated, supports the finding that Isabel was, indeed, previously married to John
Desantis. Consequently, in the absence of any proof that such marriage had been dissolved by the time
Isabel was married to Rodolfo, the inescapable conclusion is that the latter marriage is bigamous and,
therefore, void ab initio.

The inability of the petitioners and their siblings to present evidence to prove that Isabels prior
marriage was dissolved results in a failure to establish that she has interest in the estate of
Rodolfo. Clearly, an intervention by the petitioners and their siblings in the settlement proceedings
cannot be justified. We affirm the Court of Appeals.

WHEREFORE, the instant appeal is DENIED. Accordingly, the decision dated 31 May 2007 of
the Court of Appeals in CA-G.R. SP No. 00576 is hereby AFFIRMED.

Costs against the petitioners.

SO ORDERED.

JOSE PORTUGAL PEREZ
Associate Justice





SECOND DIVISION
[A.M. No. MTJ-96-1088. July 19, 1996]
RODOLFO G. NAVARRO, complainant, vs. JUDGE HERNANDO C. DOMAGTOY, respondent.
D E C I S I O N
ROMERO, J .:
The complainant in this administrative case is the Municipal Mayor of Dapa, Surigao del Norte,
Rodolfo G. Navarro. He has submitted evidence in relation to two specific acts committed by respondent
Municipal Circuit Trial Court Judge Hernando Domagtoy, which, he contends, exhibits gross misconduct
as well as inefficiency in office and ignorance of the law.
First, on September 27, 1994, respondent judge solemnized the wedding between Gaspar A.
Tagadan and Arlyn F. Borga, despite the knowledge that the groom is merely separated from his first
wife.
Second, it is alleged that he performed a marriage ceremony between Floriano Dador Sumaylo and
Gemma G. del Rosario outside his court's jurisdiction on October 27, 1994. Respondent judge holds
office and has jurisdiction in the Municipal Circuit Trial Court of Sta. Monica-Burgos, Surigao del
Norte. The wedding was solemnized at the respondent judge's residence in the municipality of Dapa,
which does not fall within his jurisdictional area of the municipalities of Sta. Monica and Burgos, located
some 40 to 45 kilometers away from the municipality of Dapa, Surigao del Norte.
In his letter-comment to the Office of the Court Administrator, respondent judge avers that the office
and name of the Municipal Mayor of Dapa have been used by someone else, who, as the mayor's
"lackey," is overly concerned with his actuations both as judge and as a private person. The same person
had earlier filed Administrative Matter No. 94-980-MTC, which was dismissed for lack of merit on
September 15, 1994, and Administrative Matter No. OCA-IPI-95-16, "Antonio Adapon v. Judge Hernando
C. Domagtoy," which is still pending.
In relation to the charges against him, respondent judge seeks exculpation from his act of having
solemnized the marriage between Gaspar Tagadan, a married man separated from his wife, and Arlyn F.
Borga by stating that he merely relied on the Affidavit issued by the Municipal Trial Judge of Basey,
Samar, confirming the fact that Mr. Tagadan and his first wife have not seen each other for almost seven
years.
[1]
With respect to the second charge, he maintains that in solemnizing the marriage between
Sumaylo and del Rosario, he did not violate Article 7, paragraph 1 of the Family Code which states
that: "Marriage may be solemnized by: (1) Any incumbent member of the judiciary within the court's
jurisdiction; and that Article 8 thereof applies to the case in question.
The complaint was not referred, as is usual, for investigation, since the pleadings submitted were
considered sufficient for a resolution of the case.
[2]

Since the countercharges of sinister motives and fraud on the part of complainant have not been
sufficiently proven, they will not be dwelt upon. The acts complained of and respondent judge's answer
thereto will suffice and can be objectively assessed by themselves to prove the latter's malfeasance.
The certified true copy of the marriage contract between Gaspar Tagadan and Arlyn Borga states
that Tagadan's civil status is "separated." Despite this declaration, the wedding ceremony was
solemnized by respondent judge. He presented in evidence a joint affidavit by Maurecio A. Labado, Sr.
and Eugenio Bullecer, subscribed and sworn to before Judge Demosthenes C. Duquilla, Municipal Trial
Judge of Basey, Samar.
[3]
The affidavit was not issued by the latter judge, as claimed by respondent
judge, but merely acknowledged before him. In their affidavit, the affiants stated that they knew Gaspar
Tagadan to have been civilly married to Ida D. Pearanda in September 1983; that after thirteen years of
cohabitation and having borne five children, Ida Pearanda left the conjugal dwelling in Valencia,
Bukidnon and that she has not returned nor been heard of for almost seven years, thereby giving rise to
the presumption that she is already dead.
In effect, Judge Domagtoy maintains that the aforementioned joint affidavit is sufficient proof of Ida
Pearanda's presumptive death, and ample reason for him to proceed with the marriage ceremony. We
do not agree.
Article 41 of the Family Code expressly provides:
"A marriage contracted by any person during the subsistence of a previous marriage shall be null and
void, unless before the celebration of the subsequent marriage, the prior spouse had been absent for four
consecutive years and the spouse present had a well-founded belief that the absent spouse was already
dead. In case of disappearance where there is danger of death under the circumstances set forth in the
provisions of Articles 391 of the Civil Code, an absence of only two years shall be sufficient.
For the purpose of contracting the subsequent marriage under the preceding paragraph, the spouse
present must institute a summary proceeding as provided in this Code for the declaration of
presumptive death of the absentee, without prejudice to the effect of reappearance of the absent
spouse." (Emphasis added.)
There is nothing ambiguous or difficult to comprehend in this provision. In fact, the law is clear and
simple. Even if the spouse present has a well-founded belief that the absent spouse was already dead, a
summary proceeding for the declaration of presumptive death is necessary in order to contract a
subsequent marriage, a mandatory requirement which has been precisely incorporated into the Family
Code to discourage subsequent marriages where it is not proven that the previous marriage has been
dissolved or a missing spouse is factually or presumptively dead, in accordance with pertinent provisions
of law.
In the case at bar, Gaspar Tagadan did not institute a summary proceeding for the declaration of his
first wife's presumptive death. Absent this judicial declaration, he remains married to Ida
Pearanda. Whether wittingly, or unwittingly, it was manifest error on the part of respondent judge to
have accepted the joint affidavit submitted by the groom. Such neglect or ignorance of the law has
resulted in a bigamous, and therefore void, marriage. Under Article 35 of the Family Code, "The following
marriage shall be void from the beginning: (4) Those bigamous x x x marriages not falling under Article
41."
The second issue involves the solemnization of a marriage ceremony outside the court's jurisdiction,
covered by Articles 7 and 8 of the Family Code, thus:
"Art. 7. Marriage may be solemnized by:
(1) Any incumbent member of the judiciary within the court's jurisdiction;
x x x x x x xxx (Emphasis supplied.)
Art. 8. The marriage shall be solemnized publicly in the chambers of the judge or in open court, in the
church, chapel or temple, or in the office of the consul-general, consul or vice-consul, as the case may
be, and not elsewhere, except in cases of marriages contracted on the point of death or in remote
places in accordance with Article 29 of this Code, or where both parties request the solemnizing
officer in writing in which case the marriage may be solemnized at a house or place designated by
them in a sworn statement to that effect."
Respondent judge points to Article 8 and its exceptions as the justifications for his having solemnized
the marriage between Floriano Sumaylo and Gemma del Rosario outside of his court's jurisdiction. As
the aforequoted provision states, a marriage can be held outside of the judge's chambers or courtroom
only in the following instances: (1) at the point of death, (2) in remote places in accordance with Article
29 or (3) upon request of both parties in writing in a sworn statement to this effect. There is no pretense
that either Sumaylo or del Rosario was at the point of death or in a remote place. Moreover, the written
request presented addressed to the respondent judge was made by only one party, Gemma del
Rosario.
[4]

More importantly, the elementary principle underlying this provision is the authority of the
solemnizing judge. Under Article 3, one of the formal requisites of marriage is the "authority of the
solemnizing officer." Under Article 7, marriage may be solemnized by, among others, "any incumbent
member of the judiciary within the court's jurisdiction." Article 8, which is a directory provision, refers only
to the venue of the marriage ceremony and does not alter or qualify the authority of the solemnizing
officer as provided in the preceding provision. Non-compliance herewith will not invalidate the marriage.
A priest who is commissioned and allowed by his local ordinary to marry the faithful, is authorized to
do so only within the area of the diocese or place allowed by his Bishop. An appellate court Justice or a
Justice of this Court has jurisdiction over the entire Philippines to solemnize marriages, regardless of the
venue, as long as the requisites of the law are complied with. However, judges who are appointed to
specific jurisdictions, may officiate in weddings only within said areas and not beyond. Where a judge
solemnizes a marriage outside his court's jurisdiction, there is a resultant irregularity in the formal
requisite laid down in Article 3, which while it may not affect the validity of the marriage, may subject the
officiating official to administrative liability.
[5]

Inasmuch as respondent judge's jurisdiction covers the municipalities of Sta. Monica and Burgos, he
was not clothed with authority to solemnize a marriage in the municipality of Dapa, Surigao del Norte. By
citing Article 8 and the exceptions therein as grounds for the exercise of his misplaced authority,
respondent judge again demonstrated a lack of understanding of the basic principles of civil law.
Accordingly, the Court finds respondent to have acted in gross ignorance of the law. The legal
principles applicable in the cases brought to our attention are elementary and uncomplicated, prompting
us to conclude that respondent's failure to apply them is due to a lack of comprehension of the law.
The judiciary should be composed of persons who, if not experts, are at least, proficient in the law
they are sworn to apply, more than the ordinary laymen. They should be skilled and competent in
understanding and applying the law. It is imperative that they be conversant with basic legal principles
like the ones involved in instant case.
[6]
It is not too much to expect them to know and apply the law
intelligently.
[7]
Otherwise, the system of justice rests on a shaky foundation indeed, compounded by the
errors committed by those not learned in the law. While magistrates may at times make mistakes in
judgment, for which they are not penalized, the respondent judge exhibited ignorance of elementary
provisions of law, in an area which has greatly prejudiced the status of married persons.
The marriage between Gaspar Tagadan and Arlyn Borga is considered bigamous and void, there
being a subsisting marriage between Gaspar Tagadan and Ida Pearanda.
The Office of the Court Administrator recommends, in its Memorandum to the Court, a six-month
suspension and a stern warning that a repetition of the same or similar acts will be dealt with more
severely. Considering that one of the marriages in question resulted in a bigamous union and therefore
void, and the other lacked the necessary authority of respondent judge, the Court adopts said
recommendation. Respondent is advised to be more circumspect in applying the law and to cultivate a
deeper understanding of the law.
IN VIEW OF THE FOREGOING, respondent Judge Hernando C. Domagtoy is hereby SUSPENDED
for a period of six (6) months and given a STERN WARNING that a repetition of the same or similar acts
will be dealt with more severely.
SO ORDERED.
Regalado (Chairman), Puno, Mendoza, and Torres, Jr., JJ., concur.

THIRD DIVISION
[G.R. No. 138322. October 2, 2001]
GRACE J. GARCIA, a.k.a. GRACE J. GARCIA-RECIO, petitioner, vs. REDERICK A.
RECIO, respondent.
D E C I S I O N
PANGANIBAN, J .:
A divorce obtained abroad by an alien may be recognized in our jurisdiction, provided such decree is
valid according to the national law of the foreigner. However, the divorce decree and the governing
personal law of the alien spouse who obtained the divorce must be proven. Our courts do not take
judicial notice of foreign laws and judgments; hence, like any other facts, both the divorce decree and the
national law of the alien must be alleged and proven according to our law on evidence.
The Case
Before us is a Petition for Review under Rule 45 of the Rules of Court, seeking to nullify the January
7, 1999 Decision
[1]
and the March 24, 1999 Order
[2]
of the Regional Trial Court of Cabanatuan City,
Branch 28, in Civil Case No. 3026AF. The assailed Decision disposed as follows:
WHEREFORE, this Court declares the marriage between Grace J. Garcia and Rederick A. Recio
solemnized on January 12, 1994 at Cabanatuan City as dissolved and both parties can now remarry
under existing and applicable laws to any and/or both parties.
[3]

The assailed Order denied reconsideration of the above-quoted Decision.
The Facts
Rederick A. Recio, a Filipino, was married to Editha Samson, an Australian citizen, in Malabon,
Rizal, on March 1, 1987.
[4]
They lived together as husband and wife in Australia. On May 18, 1989,
[5]
a
decree of divorce, purportedly dissolving the marriage, was issued by an Australian family court.
On June 26, 1992, respondent became an Australian citizen, as shown by a Certificate of Australian
Citizenship issued by the Australian government.
[6]
Petitioner -- a Filipina -- and respondent were married
on January 12, 1994 in Our Lady of Perpetual Help Church in Cabanatuan City.
[7]
In their application for a
marriage license, respondent was declared as single and Filipino.
[8]

Starting October 22, 1995, petitioner and respondent lived separately without prior judicial dissolution
of their marriage. While the two were still in Australia, their conjugal assets were divided on May 16,
1996, in accordance with their Statutory Declarations secured in Australia.
[9]

On March 3, 1998, petitioner filed a Complaint for Declaration of Nullity of Marriage
[10]
in the court a
quo, on the ground of bigamy -- respondent allegedly had a prior subsisting marriage at the time he
married her on January 12, 1994. She claimed that she learned of respondents marriage to Editha
Samson only in November, 1997.
In his Answer, respondent averred that, as far back as 1993, he had revealed to petitioner his prior
marriage and its subsequent dissolution.
[11]
He contended that his first marriage to an Australian citizen
had been validly dissolved by a divorce decree obtained in Australia in 1989;
[12]
thus, he was legally
capacitated to marry petitioner in 1994.
On July 7, 1998 -- or about five years after the couples wedding and while the suit for the declaration
of nullity was pending -- respondent was able to secure a divorce decree from a family court in Sydney,
Australia because the marriage ha[d] irretrievably broken down.
[13]

Respondent prayed in his Answer that the Complaint be dismissed on the ground that it stated no
cause of action.
[14]
The Office of the Solicitor General agreed with respondent.
[15]
The court marked and
admitted the documentary evidence of both parties.
[16]
After they submitted their respective memoranda,
the case was submitted for resolution.
[17]

Thereafter, the trial court rendered the assailed Decision and Order.
Ruling of the Trial Court
The trial court declared the marriage dissolved on the ground that the divorce issued in Australia was
valid and recognized in the Philippines. It deemed the marriage ended, but not on the basis of any defect
in an essential element of the marriage; that is, respondents alleged lack of legal capacity to
remarry. Rather, it based its Decision on the divorce decree obtained by respondent. The Australian
divorce had ended the marriage; thus, there was no more marital union to nullify or annul.
Hence, this Petition.
[18]

Issues
Petitioner submits the following issues for our consideration:
1
The trial court gravely erred in finding that the divorce decree obtained in Australia by the respondent ipso
facto terminated his first marriage to Editha Samson thereby capacitating him to contract a second
marriage with the petitioner.
2
The failure of the respondent, who is now a naturalized Australian, to present a certificate of legal
capacity to marry constitutes absence of a substantial requisite voiding the petitioners marriage to the
respondent
3
The trial court seriously erred in the application of Art. 26 of the Family Code in this case.
4
The trial court patently and grievously erred in disregarding Arts. 11, 13, 21, 35, 40, 52 and 53 of the
Family Code as the applicable provisions in this case.
5
The trial court gravely erred in pronouncing that the divorce decree obtained by the respondent in
Australia ipso facto capacitated the parties to remarry, without first securing a recognition of the judgment
granting the divorce decree before our courts.
[19]

The Petition raises five issues, but for purposes of this Decision, we shall concentrate on two pivotal
ones: (1) whether the divorce between respondent and Editha Samson was proven, and (2) whether
respondent was proven to be legally capacitated to marry petitioner. Because of our ruling on these two,
there is no more necessity to take up the rest.
The Courts Ruling
The Petition is partly meritorious.
First Issue:
Proving the Divorce Between Respondent and Editha Samson
Petitioner assails the trial courts recognition of the divorce between respondent and Editha
Samson. Citing Adong v. Cheong Seng Gee,
[20]
petitioner argues that the divorce decree, like any other
foreign judgment, may be given recognition in this jurisdiction only upon proof of the existence of (1) the
foreign law allowing absolute divorce and (2) the alleged divorce decree itself. She adds that respondent
miserably failed to establish these elements.
Petitioner adds that, based on the first paragraph of Article 26 of the Family Code, marriages
solemnized abroad are governed by the law of the place where they were celebrated (the lex loci
celebrationis). In effect, the Code requires the presentation of the foreign law to show the conformity of
the marriage in question to the legal requirements of the place where the marriage was performed.
At the outset, we lay the following basic legal principles as the take-off points for our
discussion. Philippine law does not provide for absolute divorce; hence, our courts cannot grant it.
[21]
A
marriage between two Filipinos cannot be dissolved even by a divorce obtained abroad, because of
Articles 15
[22]
and 17
[23]
of the Civil Code.
[24]
In mixed marriages involving a Filipino and a foreigner, Article
26
[25]
of the Family Code allows the former to contract a subsequent marriage in case the divorce is
validly obtained abroad by the alien spouse capacitating him or her to remarry.
[26]
A divorce obtained
abroad by a couple, who are both aliens, may be recognized in the Philippines, provided it is consistent
with their respective national laws.
[27]

A comparison between marriage and divorce, as far as pleading and proof are concerned, can be
made. Van Dorn v. Romillo Jr. decrees that aliens may obtain divorces abroad, which may be
recognized in the Philippines, provided they are valid according to their national law.
[28]
Therefore, before
a foreign divorce decree can be recognized by our courts, the party pleading it must prove the divorce as
a fact and demonstrate its conformity to the foreign law allowing it.
[29]
Presentation solely of the divorce
decree is insufficient.
Divorce as a Question of Fact
Petitioner insists that before a divorce decree can be admitted in evidence, it must first comply with
the registration requirements under Articles 11, 13 and 52 of the Family Code. These articles read as
follows:
ART. 11. Where a marriage license is required, each of the contracting parties shall file separately a
sworn application for such license with the proper local civil registrar which shall specify the following:
x x x x x x x x x
(5) If previously married, how, when and where the previous marriage was dissolved or annulled;
x x x x x x x x x
ART. 13. In case either of the contracting parties has been previously married, the applicant shall be
required to
ART. 13. In case either of the contracting parties has been previously married, the applicant shall be
required to furnish, instead of the birth or baptismal certificate required in the last preceding article, the
death certificate of the deceased spouse or the judicial decree of the absolute divorce, or the judicial
decree of annulment or declaration of nullity of his or her previous marriage. x x x.
ART. 52. The judgment of annulment or of absolute nullity of the marriage, the partition and distribution
of the properties of the spouses, and the delivery of the childrens presumptive legitimes shall be
recorded in the appropriate civil registry and registries of property; otherwise, the same shall not affect
their persons.
Respondent, on the other hand, argues that the Australian divorce decree is a public document -- a
written official act of an Australian family court. Therefore, it requires no further proof of its authenticity
and due execution.
Respondent is getting ahead of himself. Before a foreign judgment is given presumptive evidentiary
value, the document must first be presented and admitted in evidence.
[30]
A divorce obtained abroad is
proven by the divorce decree itself. Indeed the best evidence of a judgment is the judgment itself.
[31]
The
decree purports to be a written act or record of an act of an official body or tribunal of a foreign country.
[32]

Under Sections 24 and 25 of Rule 132, on the other hand, a writing or document may be proven as a
public or official record of a foreign country by either (1) an official publication or (2) a copy thereof
attested
[33]
by the officer having legal custody of the document. If the record is not kept in the Philippines,
such copy must be (a) accompanied by a certificate issued by the proper diplomatic or consular officer in
the Philippine foreign service stationed in the foreign country in which the record is kept and (b)
authenticated by the seal of his office.
[34]

The divorce decree between respondent and Editha Samson appears to be an authentic one issued
by an Australian family court.
[35]
However, appearance is not sufficient; compliance with the
aforementioned rules on evidence must be demonstrated.
Fortunately for respondents cause, when the divorce decree of May 18, 1989 was submitted in
evidence, counsel for petitioner objected, not to its admissibility, but only to the fact that it had not been
registered in the Local Civil Registry of Cabanatuan City.
[36]
The trial court ruled that it was admissible,
subject to petitioners qualification.
[37]
Hence, it was admitted in evidence and accorded weight by the
judge. Indeed, petitioners failure to object properly rendered the divorce decree admissible as a written
act of the Family Court of Sydney, Australia.
[38]

Compliance with the quoted articles (11, 13 and 52) of the Family Code is not necessary; respondent
was no longer bound by Philippine personal laws after he acquired Australian citizenship in
1992.
[39]
Naturalization is the legal act of adopting an alien and clothing him with the political and civil
rights belonging to a citizen.
[40]
Naturalized citizens, freed from the protective cloak of their former states,
don the attires of their adoptive countries. By becoming an Australian, respondent severed his allegiance
to the Philippines and the vinculum juris that had tied him to Philippine personal laws.
Burden of Proving Australian Law
Respondent contends that the burden to prove Australian divorce law falls upon petitioner, because
she is the party challenging the validity of a foreign judgment. He contends that petitioner was satisfied
with the original of the divorce decree and was cognizant of the marital laws of Australia, because she
had lived and worked in that country for quite a long time. Besides, the Australian divorce law is allegedly
known by Philippine courts; thus, judges may take judicial notice of foreign laws in the exercise of sound
discretion.
We are not persuaded. The burden of proof lies with the party who alleges the existence of a fact or
thing necessary in the prosecution or defense of an action.
[41]
In civil cases, plaintiffs have the burden of
proving the material allegations of the complaint when those are denied by the answer; and defendants
have the burden of proving the material allegations in their answer when they introduce new
matters.
[42]
Since the divorce was a defense raised by respondent, the burden of proving the pertinent
Australian law validating it falls squarely upon him.
It is well-settled in our jurisdiction that our courts cannot take judicial notice of foreign laws.
[43]
Like
any other facts, they must be alleged and proved. Australian marital laws are not among those matters
that judges are supposed to know by reason of their judicial function.
[44]
The power of judicial notice must
be exercised with caution, and every reasonable doubt upon the subject should be resolved in the
negative.
Second Issue: Respondents Legal Capacity to Remarry
Petitioner contends that, in view of the insufficient proof of the divorce, respondent was legally
incapacitated to marry her in 1994. Hence, she concludes that their marriage was void ab initio.
Respondent replies that the Australian divorce decree, which was validly admitted in evidence,
adequately established his legal capacity to marry under Australian law.
Respondents contention is untenable. In its strict legal sense, divorce means the legal dissolution of
a lawful union for a cause arising after marriage. But divorces are of different types. The two basic ones
are (1) absolute divorce or a vinculo matrimonii and (2) limited divorce or a mensa et thoro. The first kind
terminates the marriage, while the second suspends it and leaves the bond in full force.
[45]
There is no
showing in the case at bar which type of divorce was procured by respondent.
Respondent presented a decree nisi or an interlocutory decree -- a conditional or provisional
judgment of divorce. It is in effect the same as a separation from bed and board, although an absolute
divorce may follow after the lapse of the prescribed period during which no reconciliation is effected.
[46]

Even after the divorce becomes absolute, the court may under some foreign statutes and practices,
still restrict remarriage. Under some other jurisdictions, remarriage may be limited by statute; thus, the
guilty party in a divorce which was granted on the ground of adultery may be prohibited from marrying
again. The court may allow a remarriage only after proof of good behavior.
[47]

On its face, the herein Australian divorce decree contains a restriction that reads:
1. A party to a marriage who marries again before this decree becomes absolute
(unless the other party has died) commits the offence of bigamy.
[48]

This quotation bolsters our contention that the divorce obtained by respondent may have been
restricted. It did not absolutely establish his legal capacity to remarry according to his national
law. Hence, we find no basis for the ruling of the trial court, which erroneously assumed that the
Australian divorce ipso facto restored respondents capacity to remarry despite the paucity of evidence on
this matter.
We also reject the claim of respondent that the divorce decree raises a disputable presumption or
presumptive evidence as to his civil status based on Section 48, Rule 39
[49]
of the Rules of Court, for the
simple reason that no proof has been presented on the legal effects of the divorce decree obtained under
Australian laws.
Significance of the Certificate of Legal Capacity
Petitioner argues that the certificate of legal capacity required by Article 21 of the Family Code was
not submitted together with the application for a marriage license. According to her, its absence is proof
that respondent did not have legal capacity to remarry.
We clarify. To repeat, the legal capacity to contract marriage is determined by the national law of the
party concerned. The certificate mentioned in Article 21 of the Family Code would have been sufficient to
establish the legal capacity of respondent, had he duly presented it in court. A duly authenticated and
admitted certificate is prima facie evidence of legal capacity to marry on the part of the alien applicant for
a marriage license.
[50]

As it is, however, there is absolutely no evidence that proves respondents legal capacity to marry
petitioner. A review of the records before this Court shows that only the following exhibits were presented
before the lower court: (1) for petitioner: (a) Exhibit A Complaint;
[51]
(b) Exhibit B Certificate of
Marriage Between Rederick A. Recio (Filipino-Australian) and Grace J. Garcia (Filipino) on January 12,
1994 in Cabanatuan City, Nueva Ecija;
[52]
(c) Exhibit C Certificate of Marriage Between Rederick A.
Recio (Filipino) and Editha D. Samson (Australian) on March 1, 1987 in Malabon, Metro Manila;
[53]
(d)
Exhibit D Office of the City Registrar of Cabanatuan City Certification that no information of annulment
between Rederick A. Recio and Editha D. Samson was in its records;
[54]
and (e) Exhibit E Certificate of
Australian Citizenship of Rederick A. Recio;
[55]
(2) for respondent: (a) Exhibit 1 -- Amended
Answer;
[56]
(b) Exhibit 2 Family Law Act 1975 Decree Nisi of Dissolution of Marriage in the Family
Court of Australia;
[57]
(c) Exhibit 3 Certificate of Australian Citizenship of Rederick A. Recio;
[58]
(d)
Exhibit 4 Decree Nisi of Dissolution of Marriage in the Family Court of Australia Certificate;
[59]
and
Exhibit 5 -- Statutory Declaration of the Legal Separation Between Rederick A. Recio and Grace J.
Garcia Recio since October 22, 1995.
[60]

Based on the above records, we cannot conclude that respondent, who was then a naturalized
Australian citizen, was legally capacitated to marry petitioner on January 12, 1994. We agree with
petitioners contention that the court a quo erred in finding that the divorce decree ipso facto clothed
respondent with the legal capacity to remarry without requiring him to adduce sufficient evidence to show
the Australian personal law governing his status; or at the very least, to prove his legal capacity to
contract the second marriage.
Neither can we grant petitioners prayer to declare her marriage to respondent null and void on the
ground of bigamy. After all, it may turn out that under Australian law, he was really capacitated to marry
petitioner as a direct result of the divorce decree. Hence, we believe that the most judicious course is to
remand this case to the trial court to receive evidence, if any, which show petitioners legal capacity to
marry petitioner. Failing in that, then the court a quo may declare a nullity of the parties marriage on the
ground of bigamy, there being already in evidence two existing marriage certificates, which were both
obtained in the Philippines, one in Malabon, Metro Manila dated March 1, 1987 and the other, in
Cabanatuan City dated January 12, 1994.
WHEREFORE, in the interest of orderly procedure and substantial justice, we REMAND the case to
the court a quo for the purpose of receiving evidence which conclusively show respondents legal
capacity to marry petitioner; and failing in that, of declaring the parties marriage void on the ground of
bigamy, as above discussed. No costs.
SO ORDERED.
Melo, (Chairman), Vitug, and Sandoval-Gutierrez, JJ., concur.
FIRST DIVISION
[G.R. No. 133778. March 14, 2000]
ENGRACE NIAL for Herself and as Guardian ad Litem of the minors BABYLINE NIAL, INGRID
NIAL, ARCHIE NIAL & PEPITO NIAL, JR., petitioners, vs. NORMA
BAYADOG, respondent. Ncmmis
D E C I S I O N
YNARES_SANTIAGO, J .:
May the heirs of a deceased person file a petition for the declaration of nullity of his marriage after his
death?
Pepito Nial was married to Teodulfa Bellones on September 26, 1974. Out of their marriage were born
herein petitioners. Teodulfa was shot by Pepito resulting in her death on April 24, 1985. One year and 8
months thereafter or on December 11, 1986, Pepito and respondent Norma Badayog got married without
any marriage license. In lieu thereof, Pepito and Norma executed an affidavit dated December 11, 1986
stating that they had lived together as husband and wife for at least five years and were thus exempt from
securing a marriage license. On February 19, 1997, Pepito died in a car accident. After their fathers
death, petitioners filed a petition for declaration of nullity of the marriage of Pepito to Norma alleging that
the said marriage was void for lack of a marriage license. The case was filed under the assumption that
the validity or invalidity of the second marriage would affect petitioners successional rights. Norma filed a
motion to dismiss on the ground that petitioners have no cause of action since they are not among the
persons who could file an action for "annulment of marriage" under Article 47 of the Family Code.
Judge Ferdinand J. Marcos of the Regional Trial Court of Toledo City, Cebu, Branch 59, dismissed the
petition after finding that the Family Code is "rather silent, obscure, insufficient" to resolve the following
issues:
(1) Whether or not plaintiffs have a cause of action against defendant in asking for the
declaration of the nullity of marriage of their deceased father, Pepito G. Nial, with her
specially so when at the time of the filing of this instant suit, their father Pepito G. Nial is
already dead;
(2) Whether or not the second marriage of plaintiffs deceased father with defendant is
null and void ab initio;
(3) Whether or not plaintiffs are estopped from assailing the validity of the second
marriage after it was dissolved due to their fathers death.
[1]

Thus, the lower court ruled that petitioners should have filed the action to declare null and void their
fathers marriage to respondent before his death, applying by analogy Article 47 of the Family Code which
enumerates the time and the persons who could initiate an action for annulment of marriage.
[2]
Hence,
this petition for review with this Court grounded on a pure question of law. Scnc m
This petition was originally dismissed for non-compliance with Section 11, Rule 13 of the 1997 Rules of
Civil Procedure, and because "the verification failed to state the basis of petitioners averment that the
allegations in the petition are true and correct." It was thus treated as an unsigned pleading which
produces no legal effect under Section 3, Rule 7, of the 1997 Rules.
[3]
However, upon motion of
petitioners, this Court reconsidered the dismissal and reinstated the petition for review.
[4]

The two marriages involved herein having been solemnized prior to the effectivity of the Family Code
(FC), the applicable law to determine their validity is the Civil Code which was the law in effect at the time
of their celebration.
[5]
A valid marriage license is a requisite of marriage under Article 53 of the Civil
Code,
[6]
the absence of which renders the marriage void ab initio pursuant to Article 80(3)
[7]
in relation to
Article 58.
[8]
The requirement and issuance of marriage license is the States demonstration of its
involvement and participation in every marriage, in the maintenance of which the general public is
interested.
[9]
This interest proceeds from the constitutional mandate that the State recognizes the sanctity
of family life and of affording protection to the family as a basic "autonomous social
institution."
[10]
Specifically, the Constitution considers marriage as an "inviolable social institution," and is
the foundation of family life which shall be protected by the State.
[11]
This is why the Family Code
considers marriage as "a special contract of permanent union"
[12]
and case law considers it "not just an
adventure but a lifetime commitment."
[13]

However, there are several instances recognized by the Civil Code wherein a marriage license is
dispensed with, one of which is that provided in Article 76,
[14]
referring to the marriage of a man and a
woman who have lived together and exclusively with each other as husband and wife for a continuous
and unbroken period of at least five years before the marriage. The rationale why no license is required in
such case is to avoid exposing the parties to humiliation, shame and embarrassment concomitant with
the scandalous cohabitation of persons outside a valid marriage due to the publication of every
applicants name for a marriage license. The publicity attending the marriage license may discourage
such persons from legitimizing their status.
[15]
To preserve peace in the family, avoid the peeping and
suspicious eye of public exposure and contain the source of gossip arising from the publication of their
names, the law deemed it wise to preserve their privacy and exempt them from that requirement. Sdaa
miso
There is no dispute that the marriage of petitioners father to respondent Norma was celebrated without
any marriage license. In lieu thereof, they executed an affidavit stating that "they have attained the age of
majority, and, being unmarried, have lived together as husband and wife for at least five years, and that
we now desire to marry each other."
[16]
The only issue that needs to be resolved pertains to what nature
of cohabitation is contemplated under Article 76 of the Civil Code to warrant the counting of the five year
period in order to exempt the future spouses from securing a marriage license. Should it be a cohabitation
wherein both parties are capacitated to marry each other during the entire five-year continuous period or
should it be a cohabitation wherein both parties have lived together and exclusively with each other as
husband and wife during the entire five-year continuous period regardless of whether there is a legal
impediment to their being lawfully married, which impediment may have either disappeared or intervened
sometime during the cohabitation period?
Working on the assumption that Pepito and Norma have lived together as husband and wife for five years
without the benefit of marriage, that five-year period should be computed on the basis of a cohabitation as
"husband and wife" where the only missing factor is the special contract of marriage to validate the union.
In other words, the five-year common-law cohabitation period, which is counted back from the date of
celebration of marriage, should be a period of legal union had it not been for the absence of the marriage.
This 5-year period should be the years immediately before the day of the marriage and it should be a
period of cohabitation characterized by exclusivity meaning no third party was involved at any time
within the 5 years and continuity that is unbroken. Otherwise, if that continuous 5-year cohabitation is
computed without any distinction as to whether the parties were capacitated to marry each other during
the entire five years, then the law would be sanctioning immorality and encouraging parties to have
common law relationships and placing them on the same footing with those who lived faithfully with their
spouse. Marriage being a special relationship must be respected as such and its requirements must be
strictly observed. The presumption that a man and a woman deporting themselves as husband and wife
is based on the approximation of the requirements of the law. The parties should not be afforded any
excuse to not comply with every single requirement and later use the same missing element as a pre-
conceived escape ground to nullify their marriage. There should be no exemption from securing a
marriage license unless the circumstances clearly fall within the ambit of the exception. It should be noted
that a license is required in order to notify the public that two persons are about to be united in matrimony
and that anyone who is aware or has knowledge of any impediment to the union of the two shall make it
known to the local civil registrar.
[17]
The Civil Code provides:
Article 63: "x x x. This notice shall request all persons having knowledge of any
impediment to the marriage to advice the local civil registrar thereof. x x x."
Article 64: "Upon being advised of any alleged impediment to the marriage, the local civil
registrar shall forthwith make an investigation, examining persons under oath. x x
x" Sdaad
This is reiterated in the Family Code thus:
Article 17 provides in part: "x x x. This notice shall request all persons having knowledge
of any impediment to the marriage to advise the local civil registrar thereof. x x x."
Article 18 reads in part: "x x x. In case of any impediment known to the local civil registrar
or brought to his attention, he shall note down the particulars thereof and his findings
thereon in the application for a marriage license. x x x."
This is the same reason why our civil laws, past or present, absolutely prohibited the concurrence of
multiple marriages by the same person during the same period. Thus, any marriage subsequently
contracted during the lifetime of the first spouse shall be illegal and void,
[18]
subject only to the exception
in cases of absence or where the prior marriage was dissolved or annulled. The Revised Penal Code
complements the civil law in that the contracting of two or more marriages and the having of extramarital
affairs are considered felonies, i.e., bigamy and concubinage and adultery.
[19]
The law sanctions
monogamy.
In this case, at the time of Pepito and respondents marriage, it cannot be said that they have lived with
each other as husband and wife for at least five years prior to their wedding day. From the time Pepitos
first marriage was dissolved to the time of his marriage with respondent, only about twenty months had
elapsed. Even assuming that Pepito and his first wife had separated in fact, and thereafter both Pepito
and respondent had started living with each other that has already lasted for five years, the fact remains
that their five-year period cohabitation was not the cohabitation contemplated by law. It should be in the
nature of a perfect union that is valid under the law but rendered imperfect only by the absence of the
marriage contract. Pepito had a subsisting marriage at the time when he started cohabiting with
respondent. It is immaterial that when they lived with each other, Pepito had already been separated in
fact from his lawful spouse. The subsistence of the marriage even where there was actual severance of
the filial companionship between the spouses cannot make any cohabitation by either spouse with any
third party as being one as "husband and wife". Scs daad
Having determined that the second marriage involved in this case is not covered by the exception to the
requirement of a marriage license, it is void ab initio because of the absence of such element.
The next issue to be resolved is: do petitioners have the personality to file a petition to declare their
fathers marriage void after his death?
Contrary to respondent judges ruling, Article 47 of the Family Code
[20]
cannot be applied even by analogy
to petitions for declaration of nullity of marriage. The second ground for annulment of marriage relied
upon by the trial court, which allows "the sane spouse" to file an annulment suit "at any time before the
death of either party" is inapplicable. Article 47 pertains to the grounds, periods and persons who can file
an annulment suit, not a suit for declaration of nullity of marriage. The Code is silent as to who can file a
petition to declare the nullity of a marriage. Voidable and void marriages are not identical. A marriage that
is annulable is valid until otherwise declared by the court; whereas a marriage that is void ab initio is
considered as having never to have taken place
[21]
and cannot be the source of rights. The first can be
generally ratified or confirmed by free cohabitation or prescription while the other can never be ratified. A
voidable marriage cannot be assailed collaterally except in a direct proceeding while a void marriage can
be attacked collaterally. Consequently, void marriages can be questioned even after the death of either
party but voidable marriages can be assailed only during the lifetime of the parties and not after death of
either, in which case the parties and their offspring will be left as if the marriage had been perfectly
valid.
[22]
That is why the action or defense for nullity is imprescriptible, unlike voidable marriages where
the action prescribes. Only the parties to a voidable marriage can assail it but any proper interested party
may attack a void marriage. Void marriages have no legal effects except those declared by law
concerning the properties of the alleged spouses, regarding co-ownership or ownership through actual
joint contribution,
[23]
and its effect on the children born to such void marriages as provided in Article 50 in
relation to Article 43 and 44 as well as Article 51, 53 and 54 of the Family Code. On the contrary, the
property regime governing voidable marriages is generally conjugal partnership and the children
conceived before its annulment are legitimate. Sup rema
Contrary to the trial courts ruling, the death of petitioners father extinguished the alleged marital bond
between him and respondent. The conclusion is erroneous and proceeds from a wrong premise that there
was a marriage bond that was dissolved between the two. It should be noted that their marriage was void
hence it is deemed as if it never existed at all and the death of either extinguished nothing.
Jurisprudence under the Civil Code states that no judicial decree is necessary in order to establish the
nullity of a marriage.
[24]
"A void marriage does not require a judicial decree to restore the parties to their
original rights or to make the marriage void but though no sentence of avoidance be absolutely
necessary, yet as well for the sake of good order of society as for the peace of mind of all concerned, it is
expedient that the nullity of the marriage should be ascertained and declared by the decree of a court of
competent jurisdiction."
[25]
"Under ordinary circumstances, the effect of a void marriage, so far as
concerns the conferring of legal rights upon the parties, is as though no marriage had ever taken place.
And therefore, being good for no legal purpose, its invalidity can be maintained in any proceeding in
which the fact of marriage may be material, either direct or collateral, in any civil court between any
parties at any time, whether before or after the death of either or both the husband and the wife, and upon
mere proof of the facts rendering such marriage void, it will be disregarded or treated as non-existent by
the courts." It is not like a voidable marriage which cannot be collaterally attacked except in direct
proceeding instituted during the lifetime of the parties so that on the death of either, the marriage cannot
be impeached, and is made good ab initio.
[26]
But Article 40 of the Family Code expressly provides that
there must be a judicial declaration of the nullity of a previous marriage, though void, before a party can
enter into a second marriage
[27]
and such absolute nullity can be based only on a final judgment to that
effect.
[28]
For the same reason, the law makes either the action or defense for the declaration of absolute
nullity of marriage imprescriptible.
[29]
Corollarily, if the death of either party would extinguish the cause of
action or the ground for defense, then the same cannot be considered imprescriptible. Juris
However, other than for purposes of remarriage, no judicial action is necessary to declare a marriage an
absolute nullity. For other purposes, such as but not limited to determination of heirship, legitimacy or
illegitimacy of a child, settlement of estate, dissolution of property regime, or a criminal case for that
matter, the court may pass upon the validity of marriage even in a suit not directly instituted to question
the same so long as it is essential to the determination of the case. This is without prejudice to any issue
that may arise in the case. When such need arises, a final judgment of declaration of nullity is necessary
even if the purpose is other than to remarry. The clause "on the basis of a final judgment declaring such
previous marriage void" in Article 40 of the Family Code connotes that such final judgment need not be
obtained only for purpose of remarriage.
WHEREFORE, the petition is GRANTED. The assailed Order of the Regional Trial Court, Toledo City,
Cebu, Branch 59, dismissing Civil Case No. T-639, is REVERSED and SET ASIDE. The said case is
ordered REINSTATED.
SO ORDERED.Davide, Jr., (Chairman), Puno, and Kapunan, JJ., concur. Sc jurisPardo, J., on official
business abroad.
FIRST DIVISION
[A.M. No. MTJ-00-1329. March 8, 2001]
HERMINIA BORJA-MANZANO, petitioner, vs. JUDGE ROQUE R. SANCHEZ, MTC, Infanta,
Pangasinan, respondent.
R E S O L U T I O N
DAVIDE, JR., C.J .:
The solemnization of a marriage between two contracting parties who were both bound by a prior
existing marriage is the bone of contention of the instant complaint against respondent Judge Roque R.
Sanchez, Municipal Trial Court, Infanta, Pangasinan. For this act, complainant Herminia Borja-Manzano
charges respondent Judge with gross ignorance of the law in a sworn Complaint-Affidavit filed with the
Office of the Court Administrator on 12 May 1999.
Complainant avers that she was the lawful wife of the late David Manzano, having been married to
him on 21 May 1966 in San Gabriel Archangel Parish, Araneta Avenue, Caloocan City.
[1]
Four children
were born out of that marriage.
[2]
On 22 March 1993, however, her husband contracted another marriage
with one Luzviminda Payao before respondent Judge.
[3]
When respondent Judge solemnized said
marriage, he knew or ought to know that the same was void and bigamous, as the marriage contract
clearly stated that both contracting parties were separated.
Respondent Judge, on the other hand, claims in his Comment that when he officiated the marriage
between Manzano and Payao he did not know that Manzano was legally married. What he knew was
that the two had been living together as husband and wife for seven years already without the benefit of
marriage, as manifested in their joint affidavit.
[4]
According to him, had he known that the late Manzano
was married, he would have advised the latter not to marry again; otherwise, he (Manzano) could be
charged with bigamy. He then prayed that the complaint be dismissed for lack of merit and for being
designed merely to harass him.
After an evaluation of the Complaint and the Comment, the Court Administrator recommended that
respondent Judge be found guilty of gross ignorance of the law and be ordered to pay a fine of P2,000,
with a warning that a repetition of the same or similar act would be dealt with more severely.
On 25 October 2000, this Court required the parties to manifest whether they were willing to submit
the case for resolution on the basis of the pleadings thus filed. Complainant answered in the affirmative.
For his part, respondent Judge filed a Manifestation reiterating his plea for the dismissal of the
complaint and setting aside his earlier Comment. He therein invites the attention of the Court to two
separate affidavits
[5]
of the late Manzano and of Payao, which were allegedly unearthed by a member of
his staff upon his instruction. In those affidavits, both David Manzano and Luzviminda Payao expressly
stated that they were married to Herminia Borja and Domingo Relos, respectively; and that since their
respective marriages had been marked by constant quarrels, they had both left their families and had
never cohabited or communicated with their spouses anymore. Respondent Judge alleges that on the
basis of those affidavits, he agreed to solemnize the marriage in question in accordance with Article 34 of
the Family Code.
We find merit in the complaint.
Article 34 of the Family Code provides:
No license shall be necessary for the marriage of a man and a woman who have lived together as
husband and wife for at least five years and without any legal impediment to marry each other. The
contracting parties shall state the foregoing facts in an affidavit before any person authorized by law to
administer oaths. The solemnizing officer shall also state under oath that he ascertained the
qualifications of the contracting parties and found no legal impediment to the marriage.
For this provision on legal ratification of marital cohabitation to apply, the following requisites must
concur:
1. The man and woman must have been living together as husband and wife for at least five
years before the marriage;
2. The parties must have no legal impediment to marry each other;
3. The fact of absence of legal impediment between the parties must be present at the time
of marriage;
4. The parties must execute an affidavit stating that they have lived together for at least five
years [and are without legal impediment to marry each other]; and
5. The solemnizing officer must execute a sworn statement that he had ascertained the
qualifications of the parties and that he had found no legal impediment to their marriage.
[6]

Not all of these requirements are present in the case at bar. It is significant to note that in their
separate affidavits executed on 22 March 1993 and sworn to before respondent Judge himself, David
Manzano and Luzviminda Payao expressly stated the fact of their prior existing marriage. Also, in their
marriage contract, it was indicated that both were separated.
Respondent Judge knew or ought to know that a subsisting previous marriage is a diriment
impediment, which would make the subsequent marriage null and void.
[7]
In fact, in his Comment, he
stated that had he known that the late Manzano was married he would have discouraged him from
contracting another marriage. And respondent Judge cannot deny knowledge of Manzanos and Payaos
subsisting previous marriage, as the same was clearly stated in their separate affidavits which were
subscribed and sworn to before him.
The fact that Manzano and Payao had been living apart from their respective spouses for a long time
already is immaterial. Article 63(1) of the Family Code allows spouses who have obtained a decree of
legal separation to live separately from each other, but in such a case the marriage bonds are not
severed. Elsewise stated, legal separation does not dissolve the marriage tie, much less authorize the
parties to remarry. This holds true all the more when the separation is merely de facto, as in the case at
bar.
Neither can respondent Judge take refuge on the Joint Affidavit of David Manzano and Luzviminda
Payao stating that they had been cohabiting as husband and wife for seven years. Just like separation,
free and voluntary cohabitation with another person for at least five years does not severe the tie of a
subsisting previous marriage. Marital cohabitation for a long period of time between two individuals who
are legally capacitatedto marry each other is merely a ground for exemption from marriage license. It
could not serve as a justification for respondent Judge to solemnize a subsequent marriage vitiated by the
impediment of a prior existing marriage.
Clearly, respondent Judge demonstrated gross ignorance of the law when he solemnized a void and
bigamous marriage. The maxim ignorance of the law excuses no one has special application to
judges,
[8]
who, under Rule 1.01 of the Code of Judicial Conduct, should be the embodiment of
competence, integrity, and independence. It is highly imperative that judges be conversant with the law
and basic legal principles.
[9]
And when the law transgressed is simple and elementary, the failure to know
it constitutes gross ignorance of the law.
[10]

ACCORDINGLY, the recommendation of the Court Administrator is hereby ADOPTED, with
the MODIFICATION that the amount of fine to be imposed upon respondent Judge Roque Sanchez is
increased to P20,000.SO ORDERED.Puno, Kapunan, Pardo, and Ynares-Santiago, JJ., concur.
EN BANC
[G.R. No. 108763. February 13, 1997]
REPUBLIC OF THE PHILIPPINES, petitioner, vs. COURT OF APPEALS and RORIDEL OLAVIANO
MOLINA, respondents.
D E C I S I O N
PANGANIBAN, J .:
The Family Code of the Philippines provides an entirely new ground (in addition to those enumerated
in the Civil Code) to assail the validity of a marriage, namely, psychological incapacity. Since the Codes
effectivity, our courts have been swamped with various petitions to declare marriages void based on this
ground. Although this Court had interpreted the meaning of psychological incapacity in the recent case
of Santos vs. Court of Appeals, still many judges and lawyers find difficulty in applying said novel
provision in specific cases. In the present case and in the context of the herein assailed Decision of the
Court of Appeals, the Solicitor General has labelled -- exaggerated to be sure but nonetheless expressive
of his frustration -- Article 36 as the most liberal divorce procedure in the world. Hence, this Court in
addition to resolving the present case, finds the need to lay down specific guidelines in the interpretation
and application of Article 36 of the Family Code.
Before us is a petition for review on certiorari under Rule 45 challenging the January 25, 1993
Decision
[1]
of the Court of Appeals
[2]
in CA-G.R. CV No. 34858 affirming in toto the May 14, 1991 decision
of the Regional Trial Court of La Trinidad,
[3]
Benguet, which declared the marriage of respondent Roridel
Olaviano Molina to Reynaldo Molina void ab initio, on the ground of psychological incapacity under
Article 36 of the Family Code.
The Facts
This case was commenced on August 16, 1990 with the filing by respondent Roridel O. Molina of a
verified petition for declaration of nullity of her marriage to Reynaldo Molina. Essentially, the petition
alleged that Roridel and Reynaldo were married on April 14, 1985 at the San Agustin Church
[4]
in Manila;
that a son, Andre O. Molina was born; that after a year of marriage, Reynaldo showed signs of
immaturity and irresponsibility as a husband and a father since he preferred to spend more time with his
peers and friends on whom he squandered his money; that he depended on his parents for aid and
assistance, and was never honest with his wife in regard to their finances, resulting in frequent quarrels
between them; that sometime in February 1986, Reynaldo was relieved of his job in Manila, and since
then Roridel had been the sole breadwinner of the family; that in October 1986 the couple had a very
intense quarrel, as a result of which their relationship was estranged; that in March 1987, Roridel
resigned from her job in Manila and went to live with her parents in Baguio City; that a few weeks later,
Reynaldo left Roridel and their child, and had since then abandoned them; that Reynaldo had thus shown
that he was psychologically incapable of complying with essential marital obligations and was a highly
immature and habitually quarrelsome individual who thought of himself as a king to be served; and that it
would be to the couples best interest to have their marriage declared null and void in order to free them
from what appeared to be an incompatible marriage from the start.
In his Answer filed on August 28, 1989, Reynaldo admitted that he and Roridel could no longer live
together as husband and wife, but contended that their misunderstandings and frequent quarrels were
due to (1) Roridels strange behavior of insisting on maintaining her group of friends even after their
marriage; (2) Roridels refusal to perform some of her marital duties such as cooking meals; and (3)
Roridels failure to run the household and handle their finances.
During the pre-trial on October 17, 1990, the following were stipulated:
1. That the parties herein were legally married on April 14, 1985 at the Church of St. Augustine, Manila;
2. That out of their marriage, a child named Albert Andre Olaviano Molina was born on July 29, 1986;
3.That the parties are separated-in-fact for more than three years;
4. That petitioner is not asking support for her and her child;
5. That the respondent is not asking for damages;
6. That the common child of the parties is in the custody of the petitioner wife.
Evidence for herein respondent wife consisted of her own testimony and that of her friends
Rosemarie Ventura and Maria Leonora Padilla as well as of Ruth G. Lalas, a social worker, and of Dr.
Teresita Hidalgo-Sison, a psychiatrist of the Baguio General Hospital and Medical Center. She also
submitted documents marked as Exhibits A to E-1. Reynaldo did not present any evidence as he
appeared only during the pre-trial conference.
On May 14, 1991, the trial court rendered judgment declaring the marriage void. The appeal of
petitioner was denied by the Court of Appeals which affirmed in toto the RTCs decision. Hence, the
present recourse.
The Issue
In his petition, the Solicitor General insists that the Court of Appeals made an erroneous and
incorrect interpretation of the phrase psychological incapacity (as provided under Art. 36 of the Family
Code) and made an incorrect application thereof to the facts of the case, adding that the appealed
Decision tended to establish in effect the most liberal divorce procedure in the world which is anathema
to our culture.
In denying the Solicitor Generals appeal, the respondent Court relied
[5]
heavily on the trial courts
findings that the marriage between the parties broke up because of their opposing and conflicting
personalities. Then, it added its own opinion that the Civil Code Revision Committee (hereinafter
referred to as the Committee) intended to liberalize the application of our civil laws on personal and family
rights x x x. It concluded that:
As a ground for annulment of marriage, We view psychological incapacity as a broad range of mental
and behavioral conduct on the part of one spouse indicative of how he or she regards the marital union,
his or her personal relationship with the other spouse, as well as his or her conduct in the long haul for
the attainment of the principal objectives of marriage. If said conduct, observed and considered as a
whole, tends to cause the union to self-destruct because it defeats the very objectives of marriage, then
there is enough reason to leave the spouses to their individual fates.
In the case at bar, We find that the trial judge committed no indiscretion in analyzing and deciding the
instant case, as it did, hence, We find no cogent reason to disturb the findings and conclusions thus
made.
Respondent, in her Memorandum, adopts these discussions of the Court of Appeals.
The petitioner, on the other hand, argues that opposing and conflicting personalities is not
equivalent to psychological incapacity, explaining that such ground is not simply the neglect by the
parties to the marriage of their responsibilities and duties, but a defect in their psychological nature which
renders them incapable of performing such marital responsibilities and duties.
The Courts Ruling
The petition is meritorious.
In Leouel Santos vs. Court of Appeals,
[6]
this Court, speaking thru Mr. Justice Jose C. Vitug, ruled
that psychological incapacity should refer to no less than a mental (not physical) incapacity x x x and that
(t)here is hardly any doubt that the intendment of the law has been to confine the meaning of
psychological incapacity to the most serious cases of personality disorders clearly demonstrative of an
utter insensitivity or inability to give meaning and significance to the marriage. This psychologic condition
must exist at the time the marriage is celebrated. Citing Dr. Gerardo Veloso, a former presiding judge of
the Metropolitan Marriage Tribunal of the Catholic Archdiocese of Manila,
[7]
Justice Vitug wrote that the
psychological incapacity must be characterized by (a) gravity, (b) juridical antecedence, and (c)
incurability.
On the other hand, in the present case, there is no clear showing to us that the psychological defect
spoken of is an incapacity. It appears to us to be more of a difficulty, if not outright refusal or neglect
in the performance of some marital obligations. Mere showing of irreconciliable differences and
conflicting personalities in no wise constitutes psychological incapacity. It is not enough to prove that
the parties failed to meet their responsibilities and duties as married persons; it is essential that they must
be shown to be incapable of doing so, due to some psychological (not physical) illness.
The evidence adduced by respondent merely showed that she and her husband could not get along
with each other. There had been no showing of the gravity of the problem; neither its juridical
antecedence nor its incurability. The expert testimony of Dr. Sison showed no incurable psychiatric
disorder but only incompatibility, not psychological incapacity. Dr. Sison testified:
[8]

COURT
Q It is therefore the recommendation of the psychiatrist based on your findings that it is better
for the Court to annul (sic) the marriage?
A Yes, Your Honor.
Q There is no hope for the marriage?
A There is no hope, the man is also living with another woman.
Q Is it also the stand of the psychiatrist that the parties are psychologically unfit for each other
but they are psychologically fit with other parties?
A Yes, Your Honor.
Q Neither are they psychologically unfit for their professions?
A Yes, Your Honor.
The Court has no more questions.
In the case of Reynaldo, there is no showing that his alleged personality traits were constitutive of
psychological incapacity existing at the time of marriage celebration. While some effort was made to
prove that there was a failure to fulfill pre-nuptial impressions of thoughtfulness and gentleness on
Reynaldos part and of being conservative, homely and intelligent on the part of Roridel, such failure of
expectation is not indicative of antecedent psychological incapacity. If at all, it merely shows loves
temporary blindness to the faults and blemishes of the beloved.
During its deliberations, the Court decided to go beyond merely ruling on the facts of this case vis-a-
vis existing law and jurisprudence. In view of the novelty of Art. 36 of the Family Code and the difficulty
experienced by many trial courts in interpreting and applying it, the Court decided to invite two amici
curiae, namely, the Most Reverend Oscar V. Cruz,
[9]
Vicar Judicial (Presiding Judge) of the National
Appellate Matrimonial Tribunal of the Catholic Church in the Philippines, and Justice Ricardo C.
Puno,
[10]
a member of the Family Code Revision Committee. The Court takes this occasion to thank
these friends of the Court for their informative and interesting discussions during the oral argument on
December 3, 1996, which they followed up with written memoranda.
From their submissions and the Courts own deliberations, the following guidelines in the
interpretation and application of Art. 36 of the Family Code are hereby handed down for the guidance of
the bench and the bar:
(1) The burden of proof to show the nullity of the marriage belongs to the plaintiff. Any doubt should be
resolved in favor of the existence and continuation of the marriage and against its dissolution and
nullity. This is rooted in the fact that both our Constitution and our laws cherish the validity of marriage
and unity of the family. Thus, our Constitution devotes an entire Article on the Family,
[11]
recognizing it
as the foundation of the nation. It decrees marriage as legally inviolable, thereby protecting it from
dissolution at the whim of the parties. Both the family and marriage are to be protected by the state.
The Family Code
[12]
echoes this constitutional edict on marriage and the family and emphasizes
their permanence, inviolability and solidarity.
(2) The root cause of the psychological incapacity must be (a) medically or clinically identified, (b) alleged
in the complaint, (c) sufficiently proven by experts and (d) clearly explained in the decision. Article 36 of
the Family Code requires that the incapacity must be psychological -- not physical, although its
manifestations and/or symptoms may be physical. The evidence must convince the court that the parties,
or one of them, was mentally or psychically ill to such an extent that the person could not have known the
obligations he was assuming, or knowing them, could not have given valid assumption thereof. Although
no example of such incapacity need be given here so as not to limit the application of the provision under
the principle of ejusdem generis,
[13]
nevertheless such root cause must be identified as a psychological
illness and its incapacitating nature fully explained. Expert evidence may be given by qualified
psychiatrists and clinical psychologists.
(3) The incapacity must be proven to be existing at the time of the celebration of the marriage. The
evidence must show that the illness was existing when the parties exchanged their I dos. The
manifestation of the illness need not be perceivable at such time, but the illness itself must have attached
at such moment, or prior thereto.
(4) Such incapacity must also be shown to be medically or clinically permanent or incurable. Such
incurability may be absolute or even relative only in regard to the other spouse, not necessarily absolutely
against everyone of the same sex. Furthermore, such incapacity must be relevant to the assumption of
marriage obligations, not necessarily to those not related to marriage, like the exercise of a profession or
employment in a job. Hence, a pediatrician may be effective in diagnosing illnesses of children and
prescribing medicine to cure them but may not be psychologically capacitated to procreate, bear and
raise his/her own children as an essential obligation of marriage.
(5) Such illness must be grave enough to bring about the disability of the party to assume the essential
obligations of marriage. Thus, mild characteriological peculiarities, mood changes, occasional emotional
outbursts cannot be accepted as root causes. The illness must be shown as downright incapacity or
inability, not a refusal, neglect or difficulty, much less ill will. In other words, there is a natal or
supervening disabling factor in the person, an adverse integral element in the personality structure that
effectively incapacitates the person from really accepting and thereby complying with the obligations
essential to marriage.
(6) The essential marital obligations must be those embraced by Articles 68 up to 71 of the Family Code
as regards the husband and wife as well as Articles 220, 221 and 225 of the same Code in regard to
parents and their children. Such non-complied marital obligation(s) must also be stated in the petition,
proven by evidence and included in the text of the decision.
(7) Interpretations given by the National Appellate Matrimonial Tribunal of the Catholic Church in the
Philippines, while not controlling or decisive, should be given great respect by our courts. It is clear that
Article 36 was taken by the Family Code Revision Committee from Canon 1095 of the New Code of
Canon Law, which became effective in 1983 and which provides:
The following are incapable of contracting marriage: Those who are unable to assume the essential
obligations of marriage due to causes of psychological nature.
[14]

Since the purpose of including such provision in our Family Code is to harmonize our civil laws with
the religious faith of our people, it stands to reason that to achieve such harmonization, great persuasive
weight should be given to decisions of such appellate tribunal. Ideally -- subject to our law on evidence --
what is decreed as canonically invalid should also be decreed civilly void.
This is one instance where, in view of the evident source and purpose of the Family Code provision,
contemporaneous religious interpretation is to be given persuasive effect. Here, the State and the
Church -- while remaining independent, separate and apart from each other -- shall walk together in
synodal cadence towards the same goal of protecting and cherishing marriage and the family as the
inviolable base of the nation.
(8) The trial court must order the prosecuting attorney or fiscal and the Solicitor General to appear as
counsel for the state. No decision shall be handed down unless the Solicitor General issues a
certification, which will be quoted in the decision, briefly stating therein his reasons for his agreement or
opposition, as the case may be, to the petition. The Solicitor General, along with the prosecuting
attorney, shall submit to the court such certification within fifteen (15) days from the date the case is
deemed submitted for resolution of the court. The Solicitor General shall discharge the equivalent
function of the defensor vinculi contemplated under Canon 1095.
In the instant case and applying Leouel Santos, we have already ruled to grant the petition. Such
ruling becomes even more cogent with the use of the foregoing guidelines.
WHEREFORE, the petition is GRANTED. The assailed Decision is REVERSED and SET
ASIDE. The marriage of Roridel Olaviano to Reynaldo Molina subsists and remains valid.
SO ORDERED.
Narvasa, C.J., Regalado, Davide, Jr., Bellosillo, Melo, Puno, Kapunan, Mendoza, Francisco,
Hermosisima, Jr., and Torres, Jr., JJ., concur
Padilla, J., See Separate Statement.
Romero, J., See Separate Statement.
Vitug, J., Please see Concurring opinion.




THIRD DIVISION

EDWARD KENNETH NGO TE,
Petitioner,


- versus -


ROWENA ONG GUTIERREZ YU-TE,
Respondent,

REPUBLIC OF THEPHILIPPINES,
Oppositor.

G.R. No. 161793

Present:

YNARES-SANTIAGO, J.,
Chairperson,
AUSTRIA-MARTINEZ,
CHICO-NAZARIO,
NACHURA, and
PERALTA, JJ.

Promulgated:

February 13, 2009

x------------------------------------------------------------------------------------x





D E C I S I O N

NACHURA, J .:



Far from novel is the issue involved in this petition. Psychological incapacity, since its incorporation
in our laws, has become a clichd subject of discussion in our jurisprudence. The Court treats this case,
however, with much ado, it having realized that current jurisprudential doctrine has unnecessarily
imposed a perspective by which psychological incapacity should be viewed, totally inconsistent with the
way the concept was formulatedfree in form and devoid of any definition.

For the resolution of the Court is a petition for review on certiorari under Rule 45 of the Rules of
Court assailing the August 5, 2003 Decision
[1]
of the Court of Appeals (CA) in CA-G.R. CV No.
71867. The petition further assails the January 19, 2004 Resolution
[2]
denying the motion for the
reconsideration of the challenged decision.

The relevant facts and proceedings follow.

Petitioner Edward Kenneth Ngo Te first got a glimpse of respondent Rowena Ong Gutierrez Yu-
Te in a gathering organized by the Filipino-Chinese association in their college. Edward was then initially
attracted to Rowenas close friend; but, as the latter already had a boyfriend, the young man decided to
court Rowena. That was in January 1996, when petitioner was a sophomore student and respondent, a
freshman.
[3]


Sharing similar angst towards their families, the two understood one another and developed a
certain degree of closeness towards each other. In March 1996, or around three months after their first
meeting, Rowena asked Edward that they elope. At first, he refused, bickering that he was young and
jobless. Her persistence, however, made him relent. Thus, they left Manila and sailed to Cebu that
month; he, providing their travel money and she, purchasing the boat ticket.
[4]


However, Edwards P80,000.00 lasted for only a month. Their pension house accommodation
and daily sustenance fast depleted it. And they could not find a job. In April 1996, they decided to go
back to Manila. Rowena proceeded to her uncles house and Edward to his parents home. As his family
was abroad, and Rowena kept on telephoning him, threatening him that she would commit suicide,
Edward agreed to stay with Rowena at her uncles place.
[5]


On April 23, 1996, Rowenas uncle brought the two to a court to get married. He was then 25
years old, and she, 20.
[6]
The two then continued to stay at her uncles place where Edward was treated
like a prisonerhe was not allowed to go out unaccompanied. Her uncle also showed Edward his guns
and warned the latter not to leave Rowena.
[7]
At one point, Edward was able to call home and talk to his
brother who suggested that they should stay at their parents home and live with them. Edward relayed
this to Rowena who, however, suggested that he should get his inheritance so that they could live on their
own. Edward talked to his father about this, but the patriarch got mad, told Edward that he would be
disinherited, and insisted that Edward must go home.
[8]


After a month, Edward escaped from the house of Rowenas uncle, and stayed with his
parents. His family then hid him from Rowena and her family whenever they telephoned to ask for him.
[9]


In June 1996, Edward was able to talk to Rowena. Unmoved by his persistence that they should
live with his parents, she said that it was better for them to live separate lives. They then parted ways.
[10]


After almost four years, or on January 18, 2000, Edward filed a petition before the Regional Trial
Court (RTC) of Quezon City, Branch 106, for the annulment of his marriage to Rowena on the basis of the
latters psychological incapacity. This was docketed as Civil Case No. Q-00-39720.
[11]


As Rowena did not file an answer, the trial court, on July 11, 2000, ordered the Office of the City
Prosecutor (OCP) of Quezon City to investigate whether there was collusion between the parties.
[12]
In the
meantime, on July 27, 2000, the Office of the Solicitor General (OSG) entered its appearance and
deputized the OCP to appear on its behalf and assist it in the scheduled hearings.
[13]


On August 23, 2000, the OCP submitted an investigation report stating that it could not determine
if there was collusion between the parties; thus, it recommended trial on the merits.
[14]


The clinical psychologist who examined petitioner found both parties psychologically
incapacitated, and made the following findings and conclusions:

BACKGROUND DATA & BRIEF MARITAL HISTORY:

EDWARD KENNETH NGO TE is a [29-year-old] Filipino male adult born and
baptized Born Again Christian at Manila. He finished two years in college
at AMA Computer College last 1994 and is currently unemployed. He is married to and
separated from ROWENA GUTIERREZ YU-TE. He presented himself at my office for a
psychological evaluation in relation to his petition for Nullification of Marriage against the
latter by the grounds of psychological incapacity. He is now residing at 181 P. Tuazon
Street, Quezon City.

Petitioner got himself three siblings who are now in business and one deceased
sister. Both his parents are also in the business world by whom he [considers] as
generous, hospitable, and patient. This said virtues are said to be handed to each of the
family member. He generally considers himself to be quiet and simple. He clearly
remembers himself to be afraid of meeting people. After 1994, he tried his luck in being a
Sales Executive of Mansfield International Incorporated. And because of job
incompetence, as well as being quiet and loner, he did not stay long in the job until
1996. His interest lie[s] on becoming a full servant of God by being a priest or a
pastor. He [is] said to isolate himself from his friends even during his childhood days as
he only loves to read the Bible and hear its message.

Respondent is said to come from a fine family despite having a lazy father and a
disobedient wife. She is said to have not finish[ed] her collegiate degree and shared
intimate sexual moments with her boyfriend prior to that with petitioner.

In January of 1996, respondent showed her kindness to petitioner and this
became the foundation of their intimate relationship. After a month of dating, petitioner
mentioned to respondent that he is having problems with his family. Respondent
surprisingly retorted that she also hates her family and that she actually wanted to get out
of their lives. From that [time on], respondent had insisted to petitioner that they should
elope and live together. Petitioner hesitated because he is not prepared as they are both
young and inexperienced, but she insisted that they would somehow manage because
petitioner is rich. In the last week of March 1996, respondent seriously brought the idea
of eloping and she already bought tickets for the boat going to Cebu. Petitioner
reluctantly agreed to the idea and so they eloped to Cebu. The parties are supposed to
stay at the house of a friend of respondent, but they were not able to locate her, so
petitioner was compelled to rent an apartment. The parties tried to look for a job but
could not find any so it was suggested by respondent that they should go back and seek
help from petitioners parents. When the parties arrived at the house of petitioner, all of
his whole family was all out of the country so respondent decided to go back to her home
for the meantime while petitioner stayed behind at their home. After a few days of
separation, respondent called petitioner by phone and said she wanted to talk to
him. Petitioner responded immediately and when he arrived at their house, respondent
confronted petitioner as to why he appeared to be cold, respondent acted irrationally and
even threatened to commit suicide. Petitioner got scared so he went home
again. Respondent would call by phone every now and then and became angry as
petitioner does not know what to do. Respondent went to the extent of threatening to file
a case against petitioner and scandalize his family in the newspaper. Petitioner asked
her how he would be able to make amends and at this point in time[,] respondent brought
the idea of marriage. Petitioner[,] out of frustration in life[,] agreed to her to pacify
her. And so on April 23, 1996, respondents uncle brought the parties to Valenzuela[,]
and on that very same day[,] petitioner was made to sign the Marriage Contract before
the Judge. Petitioner actually never applied for any Marriage License.

Respondent decided that they should stay first at their house until after arrival of
the parents of petitioner. But when the parents of petitioner arrived, respondent refused
to allow petitioner to go home. Petitioner was threatened in so many ways with her uncle
showing to him many guns. Respondent even threatened that if he should persist in
going home, they will commission their military friends to harm his family. Respondent
even made petitioner sign a declaration that if he should perish, the authorities should
look for him at his parents[] and relatives[] houses. Sometime in June of 1996, petitioner
was able to escape and he went home. He told his parents about his predicament and
they forgave him and supported him by giving him military escort. Petitioner, however,
did not inform them that he signed a marriage contract with respondent. When they knew
about it[,] petitioner was referred for counseling. Petitioner[,] after the counseling[,] tried
to contact respondent. Petitioner offered her to live instead to[sic] the home of
petitioners parents while they are still studying. Respondent refused the idea and
claimed that she would only live with him if they will have a separate home of their own
and be away from his parents. She also intimated to petitioner that he should already get
his share of whatever he would inherit from his parents so they can start a new
life. Respondent demanded these not knowing [that] the petitioner already settled his
differences with his own family. When respondent refused to live with petitioner where he
chose for them to stay, petitioner decided to tell her to stop harassing the home of his
parents. He told her already that he was disinherited and since he also does not have a
job, he would not be able to support her. After knowing that petitioner does not have any
money anymore, respondent stopped tormenting petitioner and informed petitioner that
they should live separate lives.

The said relationship between Edward and Rowena is said to be undoubtedly in
the wreck and weakly-founded. The break-up was caused by both parties[] unreadiness
to commitment and their young age. He was still in the state of finding his fate and
fighting boredom, while she was still egocentrically involved with herself.

TESTS ADMINISTERED:

Revised Beta Examination
Bender Visual Motor Gestalt Test
Draw A Person Test
Rorschach Psychodiagnostic Test
Sachs Sentence Completion Test
M M P I

TEST RESULTS & EVALUATION:

Both petitioner and respondent are dubbed to be emotionally immature and
recklessly impulsive upon swearing to their marital vows as each of them was motivated
by different notions on marriage.

Edward Kenneth Ngo Te, the petitioner in this case[,] is said to be still unsure and
unready so as to commit himself to marriage. He is still founded to be on the search of
what he wants in life. He is absconded as an introvert as he is not really sociable and
displays a lack of interest in social interactions and mingling with other individuals. He is
seen too akin to this kind of lifestyle that he finds it boring and uninteresting to commit
himself to a relationship especially to that of respondent, as aggravated by her
dangerously aggressive moves. As he is more of the reserved and timid type of person,
as he prefer to be religiously attached and spend a solemn time alone.

ROWENA GUTIERREZ YU-TE, the respondent, is said to be of the aggressive-
rebellious type of woman. She is seen to be somewhat exploitative in her [plight] for a life
of wealth and glamour. She is seen to take move on marriage as she thought that her
marriage with petitioner will bring her good fortune because he is part of a rich family. In
order to have her dreams realized, she used force and threats knowing that [her]
husband is somehow weak-willed. Upon the realization that there is really no chance for
wealth, she gladly finds her way out of the relationship.

REMARKS:

Before going to marriage, one should really get to know himself and marry
himself before submitting to marital vows. Marriage should not be taken out of intuition as
it is profoundly a serious institution solemnized by religious and law. In the case
presented by petitioner and respondent[,] (sic) it is evidently clear that both parties have
impulsively taken marriage for granted as they are still unaware of their own selves. He is
extremely introvert to the point of weakening their relationship by his weak behavioral
disposition. She, on the other hand[,] is extremely exploitative and aggressive so as to be
unlawful, insincere and undoubtedly uncaring in her strides toward convenience. It is
apparent that she is suffering the grave, severe, and incurable presence of Narcissistic
and Antisocial Personality Disorder that started since childhood and only manifested
during marriage. Both parties display psychological incapacities that made marriage a big
mistake for them to take.
[15]



The trial court, on July 30, 2001, rendered its Decision
[16]
declaring the marriage of the parties null
and void on the ground that both parties were psychologically incapacitated to comply with the essential
marital obligations.
[17]
The Republic, represented by the OSG, timely filed its notice of appeal.
[18]


On review, the appellate court, in the assailed August 5, 2003 Decision
[19]
in CA-G.R. CV No.
71867, reversed and set aside the trial courts ruling.
[20]
It ruled that petitioner failed to prove the
psychological incapacity of respondent. The clinical psychologist did not personally examine respondent,
and relied only on the information provided by petitioner. Further, the psychological incapacity was not
shown to be attended by gravity, juridical antecedence and incurability. In sum, the evidence adduced fell
short of the requirements stated in Republic v. Court of Appeals and Molina
[21]
needed for the declaration
of nullity of the marriage under Article 36 of the Family Code.
[22]
The CA faulted the lower court for
rendering the decision without the required certification of the OSG briefly stating therein the OSGs
reasons for its agreement with or opposition to, as the case may be, the petition.
[23]
The CA later denied
petitioners motion for reconsideration in the likewise assailed January 19, 2004 Resolution.
[24]


Dissatisfied, petitioner filed before this Court the instant petition for review on certiorari. On June
15, 2005, the Court gave due course to the petition and required the parties to submit their respective
memoranda.
[25]


In his memorandum,
[26]
petitioner argues that the CA erred in substituting its own judgment for
that of the trial court. He posits that the RTC declared the marriage void, not only because of
respondents psychological incapacity, but rather due to both parties psychological incapacity. Petitioner
also points out that there is no requirement for the psychologist to personally examine
respondent. Further, he avers that the OSG is bound by the actions of the OCP because the latter
represented it during the trial; and it had been furnished copies of all the pleadings, the trial court orders
and notices.
[27]


For its part, the OSG contends in its memorandum,
[28]
that the annulment petition filed before the
RTC contains no statement of the essential marital obligations that the parties failed to comply with. The
root cause of the psychological incapacity was likewise not alleged in the petition; neither was it medically
or clinically identified. The purported incapacity of both parties was not shown to be medically or clinically
permanent or incurable. And the clinical psychologist did not personally examine the respondent. Thus,
the OSG concludes that the requirements in Molina
[29]
were not satisfied.
[30]


The Court now resolves the singular issue of whether, based on Article 36 of the Family Code,
the marriage between the parties is null and void.
[31]

I.

We begin by examining the provision, tracing its origin and charting the development of
jurisprudence interpreting it.

Article 36 of the Family Code
[32]
provides:

Article 36. A marriage contracted by any party who, at the time of the celebration,
was psychologically incapacitated to comply with the essential marital obligations of
marriage, shall likewise be void even if such incapacity becomes manifest only after its
solemnization.


As borne out by the deliberations of the Civil Code Revision Committee that drafted the Family
Code, Article 36 was based on grounds available in the Canon Law. Thus, Justice Flerida Ruth P.
Romero elucidated in her separate opinion in Santos v. Court of Appeals:
[33]


However, as a member of both the Family Law Revision Committee of the
Integrated Bar of the Philippines and the Civil Code Revision Commission of the
UP Law Center, I wish to add some observations. The letter dated April 15, 1985 of then
Judge Alicia V. Sempio-Diy written in behalf of the Family Law and Civil Code Revision
Committee to then Assemblywoman Mercedes Cojuangco-Teodoro traced the
background of the inclusion of the present Article 36 in the Family Code.

During its early meetings, the Family Law Committee had
thought of including a chapter on absolute divorce in the draft of a new
Family Code (Book I of the Civil Code) that it had been tasked by the IBP
and the UP Law Center to prepare. In fact, some members of the
Committee were in favor of a no-fault divorce between the spouses after
a number of years of separation, legal or de facto. Justice J.B.L. Reyes
was then requested to prepare a proposal for an action for dissolution of
marriage and the effects thereof based on two grounds: (a) five
continuous years of separation between the spouses, with or without a
judicial decree of legal separation, and (b) whenever a married person
would have obtained a decree of absolute divorce in another country.
Actually, such a proposal is one for absolute divorce but called by
another name. Later, even the Civil Code Revision Committee took time
to discuss the proposal of Justice Reyes on this matter.

Subsequently, however, when the Civil Code Revision
Committee and Family Law Committee started holding joint meetings on
the preparation of the draft of the New Family Code, they agreed and
formulated the definition of marriage as

a special contract of permanent partnership
between a man and a woman entered into in accordance
with law for the establishment of conjugal and family life.
It is an inviolable social institution whose nature,
consequences, and incidents are governed by law and
not subject to stipulation, except that marriage
settlements may fix the property relations during the
marriage within the limits provided by law.

With the above definition, and considering the Christian
traditional concept of marriage of the Filipino people as a permanent,
inviolable, indissoluble social institution upon which the family and
society are founded, and also realizing the strong opposition that any
provision on absolute divorce would encounter from the Catholic Church
and the Catholic sector of our citizenry to whom the great majority of our
people belong, the two Committees in their joint meetings did not pursue
the idea of absolute divorce and, instead, opted for an action for judicial
declaration of invalidity of marriage based on grounds available in the
Canon Law. It was thought that such an action would not only be an
acceptable alternative to divorce but would also solve the nagging
problem of church annulments of marriages on grounds not recognized
by the civil law of the State. Justice Reyes was, thus, requested to again
prepare a draft of provisions on such action for celebration of invalidity of
marriage. Still later, to avoid the overlapping of provisions on void
marriages as found in the present Civil Code and those proposed by
Justice Reyes on judicial declaration of invalidity of marriage on grounds
similar to the Canon Law, the two Committees now working as a Joint
Committee in the preparation of a New Family Code decided to
consolidate the present provisions on void marriages with the proposals
of Justice Reyes. The result was the inclusion of an additional kind of
void marriage in the enumeration of void marriages in the present Civil
Code, to wit:

(7) those marriages contracted by any party
who, at the time of the celebration, was wanting in the
sufficient use of reason or judgment to understand the
essential nature of marriage or was psychologically or
mentally incapacitated to discharge the essential marital
obligations, even if such lack or incapacity is made
manifest after the celebration.

as well as the following implementing provisions:

Art. 32. The absolute nullity of a marriage may
be invoked or pleaded only on the basis of a final
judgment declaring the marriage void, without prejudice
to the provision of Article 34.

Art. 33. The action or defense for the
declaration of the absolute nullity of a marriage shall not
prescribe.

x x x x x x x x x

It is believed that many hopelessly broken marriages in our country today may already be
dissolved or annulled on the grounds proposed by the Joint Committee on declaration of
nullity as well as annulment of marriages, thus rendering an absolute divorce law
unnecessary. In fact, during a conference with Father Gerald Healy of the Ateneo
University, as well as another meeting with Archbishop Oscar Cruz of the Archdiocese of
Pampanga, the Joint Committee was informed that since Vatican II, the Catholic Church
has been declaring marriages null and void on the ground of lack of due discretion for
causes that, in other jurisdictions, would be clear grounds for divorce, like teen-age or
premature marriages; marriage to a man who, because of some personality disorder or
disturbance, cannot support a family; the foolish or ridiculous choice of a spouse by an
otherwise perfectly normal person; marriage to a woman who refuses to cohabit with her
husband or who refuses to have children. Bishop Cruz also informed the Committee that
they have found out in tribunal work that a lot of machismo among husbands are
manifestations of their sociopathic personality anomaly, like inflicting physical violence
upon their wives, constitutional indolence or laziness, drug dependence or addiction, and
psychosexual anomaly.
[34]



In her separate opinion in Molina,
[35]
she expounded:

At the Committee meeting of July 26, 1986, the draft provision read:

(7) Those marriages contracted by any party who, at the time of the
celebration, was wanting in the sufficient use of reason or judgment to understand the
essential nature of marriage or was psychologically or mentally incapacitated to
discharge the essential marital obligations, even if such lack of incapacity is made
manifest after the celebration.

The twists and turns which the ensuing discussion took finally produced the
following revised provision even before the session was over:

(7) That contracted by any party who, at the time of the celebration, was
psychologically incapacitated to discharge the essential marital obligations, even if such
lack or incapacity becomes manifest after the celebration.

Noticeably, the immediately preceding formulation above has dropped any
reference to wanting in the sufficient use of reason or judgment to understand the
essential nature of marriage and to mentally incapacitated. It was explained that these
phrases refer to defects in the mental faculties vitiating consent, which is not the idea . . .
but lack of appreciation of one's marital obligation. There being a defect in consent, it is
clear that it should be a ground for voidable marriage because there is the appearance of
consent and it is capable of convalidation for the simple reason that there are lucid
intervals and there are cases when the insanity is curable . . . Psychological incapacity
does not refer to mental faculties and has nothing to do with consent; it refers to
obligations attendant to marriage.

My own position as a member of the Committee then was that psychological
incapacity is, in a sense, insanity of a lesser degree.

As to the proposal of Justice Caguioa to use the term psychological or mental
impotence, Archbishop Oscar Cruz opined in the earlier February 9, 1984 session that
this term is an invention of some churchmen who are moralists but not canonists, that is
why it is considered a weak phrase. He said that the Code of Canon Law would rather
express it as psychological or mental incapacity to discharge . . . Justice Ricardo C.
Puno opined that sometimes a person may be psychologically impotent with one but not
with another.

One of the guidelines enumerated in the majority opinion for the interpretation
and application of Art. 36 is: Such incapacity must also be shown to be medically or
clinically permanent or incurable. Such incurability may be absolute or even relative only
in regard to the other spouse, not necessarily absolutely against everyone of the same
sex.

The Committee, through Prof. Araceli T. Barrera, considered the inclusion of the
phrase and is incurable but Prof. Esteban B. Bautista commented that this would give
rise to the question of how they will determine curability and Justice Caguioa agreed that
it would be more problematic. Yet, the possibility that one may be cured after the
psychological incapacity becomes manifest after the marriage was not ruled out by
Justice Puno and Justice Alice Sempio-Diy. Justice Caguioa suggested that the remedy
was to allow the afflicted spouse to remarry.

For clarity, the Committee classified the bases for determining void marriages,
viz.:

1. lack of one or more of the essential requisites of marriage as contract;
2. reasons of public policy;
3. special cases and special situations.

The ground of psychological incapacity was subsumed under special cases and special
situations, hence, its special treatment in Art. 36 in the Family Code as finally enacted.

Nowhere in the Civil Code provisions on Marriage is there a ground for avoiding
or annulling marriages that even comes close to being psychological in nature.

Where consent is vitiated due to circumstances existing at the time of the
marriage, such marriage which stands valid until annulled is capable of ratification or
convalidation.

On the other hand, for reasons of public policy or lack of essential requisites,
some marriages are void from the beginning.

With the revision of Book I of the Civil Code, particularly the provisions on
Marriage, the drafters, now open to fresh winds of change in keeping with the more
permissive mores and practices of the time, took a leaf from the relatively liberal
provisions of Canon Law.

Canon 1095 which states, inter alia, that the following persons are incapable of
contracting marriage: 3. (those) who, because of causes of a psychological nature, are
unable to assume the essential obligations of marriage provided the model for what is
now Art. 36 of the Family Code: A marriage contracted by any party who, at the time of
the celebration, was psychologically incapacitated to comply with the essential marital
obligations of marriage, shall likewise be void even if such incapacity becomes manifest
only after its solemnization.

It bears stressing that unlike in Civil Law, Canon Law recognizes only two types
of marriages with respect to their validity: valid and void. Civil Law, however, recognizes
an intermediate state, the voidable or annullable marriages. When the Ecclesiastical
Tribunal annuls a marriage, it actually declares the marriage null and void, i.e., it never
really existed in the first place, for a valid sacramental marriage can never be dissolved.
Hence, a properly performed and consummated marriage between two living Roman
Catholics can only be nullified by the formal annulment process which entails a full
tribunal procedure with a Court selection and a formal hearing.

Such so-called church annulments are not recognized by Civil Law as severing
the marriage ties as to capacitate the parties to enter lawfully into another marriage. The
grounds for nullifying civil marriage, not being congruent with those laid down by Canon
Law, the former being more strict, quite a number of married couples have found
themselves in limbofreed from the marriage bonds in the eyes of the Catholic Church
but yet unable to contract a valid civil marriage under state laws. Heedless of civil law
sanctions, some persons contract new marriages or enter into live-in relationships.

It was precisely to provide a satisfactory solution to such anomalous situations
that the Civil Law Revision Committee decided to engraft the Canon Law concept of
psychological incapacity into the Family Codeand classified the same as a ground for
declaring marriages void ab initio or totally inexistent from the beginning.

A brief historical note on the Old Canon Law (1917). This Old Code, while it did
not provide directly for psychological incapacity, in effect, recognized the same indirectly
from a combination of three old canons: Canon #1081 required persons to be capable
according to law in order to give valid consent; Canon #1082 required that persons be at
least not ignorant of the major elements required in marriage; and Canon #1087 (the
force and fear category) required that internal and external freedom be present in order
for consent to be valid. This line of interpretation produced two distinct but related
grounds for annulment called lack of due discretion and lack of due competence. Lack
of due discretion means that the person did not have the ability to give valid consent at
the time of the wedding and, therefore, the union is invalid. Lack of due competence
means that the person was incapable of carrying out the obligations of the promise he or
she made during the wedding ceremony.

Favorable annulment decisions by the Roman Rota in the 1950s and 1960s
involving sexual disorders such as homosexuality and nymphomania laid the foundation
for a broader approach to the kind of proof necessary for psychological grounds for
annulment. The Rota had reasoned for the first time in several cases that the capacity to
give valid consent at the time of marriage was probably not present in persons who had
displayed such problems shortly after the marriage. The nature of this change was
nothing short of revolutionary. Once the Rota itself had demonstrated a cautious
willingness to use this kind of hindsight, the way was paved for what came after
1970. Diocesan Tribunals began to accept proof of serious psychological problems that
manifested themselves shortly after the ceremony as proof of an inability to give valid
consent at the time of the ceremony.
[36]



Interestingly, the Committee did not give any examples of psychological incapacity for fear that by
so doing, it might limit the applicability of the provision under the principle of ejusdem generis. The
Committee desired that the courts should interpret the provision on a case-to-case basis; guided by
experience, the findings of experts and researchers in psychological disciplines, and by decisions of
church tribunals which, although not binding on the civil courts, may be given persuasive effect since the
provision itself was taken from the Canon Law.
[37]
The law is then so designed as to allow some
resiliency in its application.
[38]


Yet, as held in Santos,
[39]
the phrase psychological incapacity is not meant to comprehend all
possible cases of psychoses. It refers to no less than a mental (not physical) incapacity that causes a
party to be truly noncognitive of the basic marital covenants that concomitantly must be assumed and
discharged by the parties to the marriage which, as expressed by Article 68
[40]
of the Family Code, include
their mutual obligations to live together, observe love, respect and fidelity; and render help and
support. The intendment of the law has been to confine it to the most serious of cases of personality
disorders clearly demonstrative of an utter insensitivity or inability to give meaning and significance to the
marriage.
[41]
This interpretation is, in fact, consistent with that in Canon Law, thus:

3.5.3.1. The Meaning of Incapacity to Assume. A sharp conceptual distinction must
be made between the second and third paragraphs of C.1095, namely between the grave
lack of discretionary judgment and the incapacity to assume the essential
obligation. Mario Pompedda, a rotal judge, explains the difference by an ordinary, if
somewhat banal, example. Jose wishes to sell a house to Carmela, and on the
assumption that they are capable according to positive law to enter such contract, there
remains the object of the contract, viz, the house. The house is located in a different
locality, and prior to the conclusion of the contract, the house was gutted down by fire
unbeknown to both of them. This is the hypothesis contemplated by the third paragraph
of the canon. The third paragraph does not deal with the psychological process of giving
consent because it has been established a priori that both have such a capacity to give
consent, and they both know well the object of their consent [the house and its
particulars]. Rather, C.1095.3 deals with the object of the consent/contract which does
not exist. The contract is invalid because it lacks its formal object. The consent as a
psychological act is both valid and sufficient. The psychological act, however, is directed
towards an object which is not available. Urbano Navarrete summarizes this distinction:
the third paragraph deals not with the positing of consent but with positing the object of
consent. The person may be capable of positing a free act of consent, but he is not
capable of fulfilling the responsibilities he assumes as a result of the consent he elicits.

Since the address of Pius XII to the auditors of the Roman Rota in 1941 regarding
psychic incapacity with respect to marriage arising from pathological conditions, there
has been an increasing trend to understand as ground of nullity different from others, the
incapacity to assume the essential obligations of marriage, especially the incapacity
which arises from sexual anomalies. Nymphomania is a sample which ecclesiastical
jurisprudence has studied under this rubric.

The problem as treated can be summarized, thus: do sexual anomalies always and in
every case imply a grave psychopathological condition which affects the higher faculties
of intellect, discernment, and freedom; or are there sexual anomalies that are purely so
that is to say, they arise from certain physiological dysfunction of the hormonal system,
and they affect the sexual condition, leaving intact the higher faculties however, so that
these persons are still capable of free human acts. The evidence from the empirical
sciences is abundant that there are certain anomalies of a sexual nature which may impel
a person towards sexual activities which are not normal, either with respect to its
frequency [nymphomania, satyriasis] or to the nature of the activity itself [sadism,
masochism, homosexuality]. However, these anomalies notwithstanding, it is altogether
possible that the higher faculties remain intact such that a person so afflicted continues to
have an adequate understanding of what marriage is and of the gravity of its
responsibilities. In fact, he can choose marriage freely. The question though is whether
such a person can assume those responsibilities which he cannot fulfill, although he may
be able to understand them. In this latter hypothesis, the incapacity to assume the
essential obligations of marriage issues from the incapacity to posit the object of consent,
rather than the incapacity to posit consent itself.

Ecclesiastical jurisprudence has been hesitant, if not actually confused, in this
regard. The initial steps taken by church courts were not too clear whether this
incapacity is incapacity to posit consent or incapacity to posit the object of consent. A
case c. Pinna, for example, arrives at the conclusion that the intellect, under such an
irresistible impulse, is prevented from properly deliberating and its judgment lacks
freedom. This line of reasoning supposes that the intellect, at the moment of consent, is
under the influence of this irresistible compulsion, with the inevitable conclusion that such
a decision, made as it was under these circumstances, lacks the necessary freedom. It
would be incontrovertible that a decision made under duress, such as this irresistible
impulse, would not be a free act. But this is precisely the question: is it, as a matter of
fact, true that the intellect is always and continuously under such an irresistible
compulsion? It would seem entirely possible, and certainly more reasonable, to think that
there are certain cases in which one who is sexually hyperaesthetic can understand
perfectly and evaluate quite maturely what marriage is and what it implies; his consent
would be juridically ineffective for this one reason that he cannot posit the object of
consent, the exclusive jus in corpus to be exercised in a normal way and with usually
regularity. It would seem more correct to say that the consent may indeed be free, but is
juridically ineffective because the party is consenting to an object that he cannot
deliver. The house he is selling was gutted down by fire.

3.5.3.2. Incapacity as an Autonomous Ground. Sabattani seems to have seen his
way more clearly through this tangled mess, proposing as he did a clear conceptual
distinction between the inability to give consent on the one hand, and the inability to fulfill
the object of consent, on the other. It is his opinion that nymphomaniacs usually
understand the meaning of marriage, and they are usually able to evaluate its
implications. They would have no difficulty with positing a free and intelligent
consent. However, such persons, capable as they are of eliciting an intelligent and free
consent, experience difficulty in another sphere: delivering the object of the
consent. Anne, another rotal judge, had likewise treated the difference between the act
of consenting and the act of positing the object of consent from the point of view of a
person afflicted with nymphomania. According to him, such an affliction usually leaves
the process of knowing and understanding and evaluating intact. What it affects is the
object of consent: the delivering of the goods.

3.5.3.3 Incapacity as Incapacity to Posit the Object of Consent. From the selected
rotal jurisprudence cited, supra, it is possible to see a certain progress towards a
consensus doctrine that the incapacity to assume the essential obligations of marriage
(that is to say, the formal object of consent) can coexist in the same person with the
ability to make a free decision, an intelligent judgment, and a mature evaluation and
weighing of things. The decision coram Sabattani concerning a nymphomaniac affirmed
that such a spouse can have difficulty not only with regard to the moment of consent but
also, and especially, with regard to the matrimonium in facto esse. The decision
concludes that a person in such a condition is incapable of assuming the conjugal
obligation of fidelity, although she may have no difficulty in understanding what the
obligations of marriage are, nor in the weighing and evaluating of those same obligations.

Prior to the promulgation of the Code of Canon Law in 1983, it was not unusual to refer to
this ground as moral impotence or psychic impotence, or similar expressions to express a
specific incapacity rooted in some anomalies and disorders in the personality. These
anomalies leave intact the faculties of the will and the intellect. It is qualified as moral or
psychic, obviously to distinguish it from the impotence that constitutes the impediment
dealt with by C.1084. Nonetheless, the anomalies render the subject incapable of
binding himself in a valid matrimonial pact, to the extent that the anomaly renders that
person incapable of fulfilling the essential obligations. According to the principle affirmed
by the long tradition of moral theology: nemo ad impossibile tenetur.

x x x x

3.5.3.5 Indications of Incapacity. There is incapacity when either or both of the
contractants are not capable of initiating or maintaining this consortium. One immediately
thinks of those cases where one of the parties is so self-centered [e.g., a narcissistic
personality] that he does not even know how to begin a union with the other, let alone
how to maintain and sustain such a relationship. A second incapacity could be due to the
fact that the spouses are incapable of beginning or maintaining a heterosexual
consortium, which goes to the very substance of matrimony. Another incapacity could
arise when a spouse is unable to concretize the good of himself or of the other
party. The canon speaks, not of the bonum partium, but of the bonum conjugum. A
spouse who is capable only of realizing or contributing to the good of the other party qua
persona rather than qua conjunx would be deemed incapable of contracting
marriage. Such would be the case of a person who may be quite capable of procuring
the economic good and the financial security of the other, but not capable of realizing
the bonum conjugale of the other. These are general strokes and this is not the place for
detained and individual description.

A rotal decision c. Pinto resolved a petition where the concrete circumstances of the case
concerns a person diagnosed to be suffering from serious sociopathy. He concluded that
while the respondent may have understood, on the level of the intellect, the essential
obligations of marriage, he was not capable of assuming them because of his
constitutional immorality.

Stankiewicz clarifies that the maturity and capacity of the person as regards the fulfillment
of responsibilities is determined not only at the moment of decision but also and
especially during the moment of execution of decision. And when this is applied to
constitution of the marital consent, it means that the actual fulfillment of the essential
obligations of marriage is a pertinent consideration that must be factored into the
question of whether a person was in a position to assume the obligations of marriage in
the first place. When one speaks of the inability of the party to assume and fulfill the
obligations, one is not looking at matrimonium in fieri, but also and especially
at matrimonium in facto esse. In [the] decision of 19 Dec. 1985, Stankiewicz collocated
the incapacity of the respondent to assume the essential obligations of marriage in the
psychic constitution of the person, precisely on the basis of his irresponsibility as regards
money and his apathy as regards the rights of others that he had violated. Interpersonal
relationships are invariably disturbed in the presence of this personality disorder. A lack
of empathy (inability to recognize and experience how others feel) is common. A sense
of entitlement, unreasonable expectation, especially favorable treatment, is usually
present. Likewise common is interpersonal exploitativeness, in which others are taken
advantage of in order to achieve ones ends.

Authors have made listings of obligations considered as essential matrimonial
obligations. One of them is the right to the communio vitae. This and their corresponding
obligations are basically centered around the good of the spouses and of the
children. Serious psychic anomalies, which do not have to be necessarily incurable, may
give rise to the incapacity to assume any, or several, or even all of these rights. There
are some cases in which interpersonal relationship is impossible. Some characteristic
features of inability for interpersonal relationships in marriage include affective
immaturity, narcissism, and antisocial traits.

Marriage and Homosexuality. Until 1967, it was not very clear under what rubric
homosexuality was understood to be invalidating of marriage that is to say, is
homosexuality invalidating because of the inability to evaluate the responsibilities of
marriage, or because of the inability to fulfill its obligations. Progressively, however, rotal
jurisprudence began to understand it as incapacity to assume the obligations of marriage
so that by 1978, Parisella was able to consider, with charity, homosexuality as an
autonomous ground of nullity. This is to say that a person so afflicted is said to be unable
to assume the essential obligations of marriage. In this same rotal decision, the object of
matrimonial consent is understood to refer not only to the jus in corpus but also
the consortium totius vitae. The third paragraph of C.1095 [incapacity to assume the
essential obligations of marriage] certainly seems to be the more adequate juridical
structure to account for the complex phenomenon that homosexuality is. The
homosexual is not necessarily impotent because, except in very few exceptional cases,
such a person is usually capable of full sexual relations with the spouse. Neither is it a
mental infirmity, and a person so afflicted does not necessarily suffer from a grave lack of
due discretion because this sexual anomaly does not by itself affect the critical, volitive,
and intellectual faculties. Rather, the homosexual person is unable to assume the
responsibilities of marriage because he is unable to fulfill this object of the matrimonial
contract. In other words, the invalidity lies, not so much in the defect of consent, as in the
defect of the object of consent.

3.5.3.6 Causes of Incapacity. A last point that needs to be addressed is the source of
incapacity specified by the canon: causes of a psychological nature. Pompedda proffers
the opinion that the clause is a reference to the personality of the contractant. In other
words, there must be a reference to the psychic part of the person. It is only when there
is something in the psyche or in the psychic constitution of the person which impedes his
capacity that one can then affirm that the person is incapable according to the hypothesis
contemplated by C.1095.3. A person is judged incapable in this juridical sense only to
the extent that he is found to have something rooted in his psychic constitution which
impedes the assumption of these obligations. A bad habit deeply engrained in ones
consciousness would not seem to qualify to be a source of this invalidating
incapacity. The difference being that there seems to be some freedom, however remote,
in the development of the habit, while one accepts as given ones psychic constitution. It
would seem then that the law insists that the source of the incapacity must be one which
is not the fruit of some degree of freedom.
[42]



Conscious of the laws intention that it is the courts, on a case-to-case basis, that should
determine whether a party to a marriage is psychologically incapacitated, the Court, in sustaining the
lower courts judgment of annulment in Tuason v. Court of Appeals,
[43]
ruled that the findings of the trial
court are final and binding on the appellate courts.
[44]


Again, upholding the trial courts findings and declaring that its decision was not a judgment on
the pleadings, the Court, in Tsoi v. Court of Appeals,
[45]
explained that when private respondent testified
under oath before the lower court and was cross-examined by the adverse party, she thereby presented
evidence in the form of testimony. Importantly, the Court, aware of parallel decisions of Catholic marriage
tribunals, ruled that the senseless and protracted refusal of one of the parties to fulfill the marital
obligation of procreating children is equivalent to psychological incapacity.

The resiliency with which the concept should be applied and the case-to-case basis by which the
provision should be interpreted, as so intended by its framers, had, somehow, been rendered ineffectual
by the imposition of a set of strict standards in Molina,
[46]
thus:

From their submissions and the Court's own deliberations, the following
guidelines in the interpretation and application of Art. 36 of the Family Code are hereby
handed down for the guidance of the bench and the bar:

(1) The burden of proof to show the nullity of the marriage belongs to the
plaintiff. Any doubt should be resolved in favor of the existence and continuation of the
marriage and against its dissolution and nullity. This is rooted in the fact that both our
Constitution and our laws cherish the validity of marriage and unity of the family. Thus,
our Constitution devotes an entire Article on the Family, recognizing it as the foundation
of the nation. It decrees marriage as legally inviolable, thereby protecting it from
dissolution at the whim of the parties. Both the family and marriage are to be protected
by the state.

The Family Code echoes this constitutional edict on marriage and the family and
emphasizes their permanence, inviolability and solidarity.

(2) The root cause of the psychological incapacity must be (a) medically or
clinically identified, (b) alleged in the complaint, (c) sufficiently proven by experts and (d)
clearly explained in the decision. Article 36 of the Family Code requires that the
incapacity must be psychologicalnot physical, although its manifestations and/or
symptoms may be physical. The evidence must convince the court that the parties, or
one of them, was mentally or psychically ill to such an extent that the person could not
have known the obligations he was assuming, or knowing them, could not have given
valid assumption thereof. Although no example of such incapacity need be given here so
as not to limit the application of the provision under the principle of ejusdem generis,
nevertheless such root cause must be identified as a psychological illness and its
incapacitating nature fully explained. Expert evidence may be given by qualified
psychiatrists and clinical psychologists.

(3) The incapacity must be proven to be existing at the time of the
celebration of the marriage. The evidence must show that the illness was existing when
the parties exchanged their I do's. The manifestation of the illness need not be
perceivable at such time, but the illness itself must have attached at such moment, or
prior thereto.

(4) Such incapacity must also be shown to be medically or clinically
permanent or incurable. Such incurability may be absolute or even relative only in regard
to the other spouse, not necessarily absolutely against everyone of the same sex.
Furthermore, such incapacity must be relevant to the assumption of marriage obligations,
not necessarily to those not related to marriage, like the exercise of a profession or
employment in a job. Hence, a pediatrician may be effective in diagnosing illnesses of
children and prescribing medicine to cure them but may not be psychologically
capacitated to procreate, bear and raise his/her own children as an essential obligation of
marriage.

(5) Such illness must be grave enough to bring about the disability of the
party to assume the essential obligations of marriage. Thus, mild characterological
peculiarities, mood changes, occasional emotional outbursts cannot be accepted as root
causes. The illness must be shown as downright incapacity or inability, not a refusal,
neglect or difficulty, much less ill will. In other words, there is a natal or supervening
disabling factor in the person, an adverse integral element in the personality structure
that effectively incapacitates the person from really accepting and thereby complying with
the obligations essential to marriage.

(6) The essential marital obligations must be those embraced by Articles 68
up to 71 of the Family Code as regards the husband and wife as well as Articles 220, 221
and 225 of the same Code in regard to parents and their children. Such non-complied
marital obligation(s) must also be stated in the petition, proven by evidence and included
in the text of the decision.

(7) Interpretations given by the National Appellate Matrimonial Tribunal of
the Catholic Church in the Philippines, while not controlling or decisive, should be given
great respect by our courts. It is clear that Article 36 was taken by the Family Code
Revision Committee from Canon 1095 of the New Code of Canon Law, which became
effective in 1983 and which provides:

The following are incapable of contracting marriage: Those who are unable to
assume the essential obligations of marriage due to causes of psychological nature.

Since the purpose of including such provision in our Family Code is to harmonize
our civil laws with the religious faith of our people, it stands to reason that to achieve such
harmonization, great persuasive weight should be given to decisions of such appellate
tribunal. Ideally subject to our law on evidencewhat is decreed as canonically invalid
should also be decreed civilly void.

This is one instance where, in view of the evident source and purpose of the
Family Code provision, contemporaneous religious interpretation is to be given
persuasive effect. Here, the State and the Churchwhile remaining independent,
separate and apart from each othershall walk together in synodal cadence towards the
same goal of protecting and cherishing marriage and the family as the inviolable base of
the nation.

(8) The trial court must order the prosecuting attorney or fiscal and the
Solicitor General to appear as counsel for the state. No decision shall be handed down
unless the Solicitor General issues a certification, which will be quoted in the decision,
briefly stating therein his reasons for his agreement or opposition, as the case may be, to
the petition. The Solicitor General, along with the prosecuting attorney, shall submit to the
court such certification within fifteen (15) days from the date the case is deemed
submitted for resolution of the court. The Solicitor General shall discharge the equivalent
function of the defensor vinculi contemplated under Canon 1095.
[47]



Noteworthy is that in Molina, while the majority of the Courts membership concurred in
the ponencia of then Associate Justice (later Chief Justice) Artemio V. Panganiban, three justices
concurred in the result and another threeincluding, as aforesaid, Justice Romerotook pains to
compose their individual separate opinions. Then Justice Teodoro R. Padilla even emphasized that each
case must be judged, not on the basis of a priori assumptions, predelictions or generalizations, but
according to its own facts. In the field of psychological incapacity as a ground for annulment of marriage,
it is trite to say that no case is on all fours with another case. The trial judge must take pains in
examining the factual milieu and the appellate court must, as much as possible, avoid substituting its own
judgment for that of the trial court.
[48]


Predictably, however, in resolving subsequent cases,
[49]
the Court has applied the aforesaid
standards, without too much regard for the laws clear intention that each case is to be treated
differently, as courts should interpret the provision on a case-to-case basis; guided by experience, the
findings of experts and researchers in psychological disciplines, and by decisions of church tribunals.

In hindsight, it may have been inappropriate for the Court to impose a rigid set of rules, as the
one in Molina, in resolving all cases of psychological incapacity. Understandably, the Court was then
alarmed by the deluge of petitions for the dissolution of marital bonds, and was sensitive to the OSGs
exaggeration of Article 36 as the most liberal divorce procedure in the world.
[50]
The unintended
consequences of Molina, however, has taken its toll on people who have to live with deviant behavior,
moral insanity and sociopathic personality anomaly, which, like termites, consume little by little the very
foundation of their families, our basic social institutions. Far from what was intended by the
Court, Molina has become a strait-jacket, forcing all sizes to fit into and be bound by it. Wittingly or
unwittingly, the Court, in conveniently applying Molina, has allowed diagnosed sociopaths,
schizophrenics, nymphomaniacs, narcissists and the like, to continuously debase and pervert the sanctity
of marriage. Ironically, the Roman Rota has annulled marriages on account of the personality disorders
of the said individuals.
[51]


The Court need not worry about the possible abuse of the remedy provided by Article 36, for
there are ample safeguards against this contingency, among which is the intervention by the State,
through the public prosecutor, to guard against collusion between the parties and/or fabrication of
evidence.
[52]
The Court should rather be alarmed by the rising number of cases involving marital abuse,
child abuse, domestic violence and incestuous rape.

In dissolving marital bonds on account of either partys psychological incapacity, the Court is not
demolishing the foundation of families, but it is actually protecting the sanctity of marriage, because it
refuses to allow a person afflicted with a psychological disorder, who cannot comply with or assume the
essential marital obligations, from remaining in that sacred bond. It may be stressed that the infliction of
physical violence, constitutional indolence or laziness, drug dependence or addiction, and psychosexual
anomaly are manifestations of a sociopathic personality anomaly.
[53]
Let it be noted that in Article 36,
there is no marriage to speak of in the first place, as the same is void from the very beginning.
[54]
To
indulge in imagery, the declaration of nullity under Article 36 will simply provide a decent burial to a
stillborn marriage.

The prospect of a possible remarriage by the freed spouses should not pose too much of a
concern for the Court. First and foremost, because it is none of its business. And second, because the
judicial declaration of psychological incapacity operates as a warning or a lesson learned. On one hand,
the normal spouse would have become vigilant, and never again marry a person with a personality
disorder. On the other hand, a would-be spouse of the psychologically incapacitated runs the risk of the
latters disorder recurring in their marriage.

Lest it be misunderstood, we are not suggesting the abandonment of Molina in this case. We
simply declare that, as aptly stated by Justice Dante O. Tinga in Antonio v. Reyes,
[55]
there is need to
emphasize other perspectives as well which should govern the disposition of petitions for declaration of
nullity under Article 36. At the risk of being redundant, we reiterate once more the principle that each
case must be judged, not on the basis of a priori assumptions, predilections or generalizations but
according to its own facts. And, to repeat for emphasis, courts should interpret the provision on a case-to-
case basis; guided by experience, the findings of experts and researchers in psychological disciplines,
and by decisions of church tribunals.

II.

We now examine the instant case.

The parties whirlwind relationship lasted more or less six (6) months. They met in January 1996,
eloped in March, exchanged marital vows in May, and parted ways in June. The psychologist who
provided expert testimony found both parties psychologically incapacitated. Petitioners behavioral pattern
falls under the classification of dependent personality disorder, and respondents, that of the narcissistic
and antisocial personality disorder.
[56]


By the very nature of Article 36, courts, despite having the primary task and burden of decision-
making, must not discount but, instead, must consider as decisive evidence the expert opinion on
the psychological and mental temperaments of the parties.
[57]


Justice Romero explained this in Molina, as follows:

Furthermore, and equally significant, the professional opinion of a psychological
expert became increasingly important in such cases. Data about the person's entire life,
both before and after the ceremony, were presented to these experts and they were
asked to give professional opinions about a party's mental capacity at the time of the
wedding. These opinions were rarely challenged and tended to be accepted as decisive
evidence of lack of valid consent.

The Church took pains to point out that its new openness in this area did not
amount to the addition of new grounds for annulment, but rather was an accommodation
by the Church to the advances made in psychology during the past decades. There was
now the expertise to provide the all-important connecting link between a marriage
breakdown and premarital causes.

During the 1970s, the Church broadened its whole idea of marriage from that of a
legal contract to that of a covenant. The result of this was that it could no longer be
assumed in annulment casesthat a person who could intellectually understand the
concept of marriage could necessarily give valid consent to marry. The ability to both
grasp and assume the real obligations of a mature, lifelong commitment are now
considered a necessary prerequisite to valid matrimonial consent.

Rotal decisions continued applying the concept of incipient psychological
incapacity, not only to sexual anomalies but to all kinds of personality disorders that
incapacitate a spouse or both spouses from assuming or carrying out the essential
obligations of marriage. For marriage . . . is not merely cohabitation or the right of the
spouses to each other's body for heterosexual acts, but is, in its totality the right to the
community of the whole of life; i.e., the right to a developing lifelong relationship. Rotal
decisions since 1973 have refined the meaning of psychological or psychic capacity for
marriage as presupposing the development of an adult personality; as meaning the
capacity of the spouses to give themselves to each other and to accept the other as a
distinct person; that the spouses must be other oriented since the obligations of
marriage are rooted in a self-giving love; and that the spouses must have the capacity for
interpersonal relationship because marriage is more than just a physical reality but
involves a true intertwining of personalities. The fulfillment of the obligations of marriage
depends, according to Church decisions, on the strength of this interpersonal
relationship. A serious incapacity for interpersonal sharing and support is held to impair
the relationship and consequently, the ability to fulfill the essential marital obligations. The
marital capacity of one spouse is not considered in isolation but in reference to the
fundamental relationship to the other spouse.

Fr. Green, in an article in Catholic Mind, lists six elements necessary to the
mature marital relationship:

The courts consider the following elements crucial to the marital
commitment: (1) a permanent and faithful commitment to the marriage
partner; (2) openness to children and partner; (3) stability; (4) emotional
maturity; (5) financial responsibility; (6) an ability to cope with the
ordinary stresses and strains of marriage, etc.

Fr. Green goes on to speak about some of the psychological conditions that might lead to
the failure of a marriage:

At stake is a type of constitutional impairment precluding conjugal
communion even with the best intentions of the parties. Among the
psychic factors possibly giving rise to his or her inability to fulfill marital
obligations are the following: (1) antisocial personality with its
fundamental lack of loyalty to persons or sense of moral values; (2)
hyperesthesia, where the individual has no real freedom of sexual
choice; (3) the inadequate personality where personal responses
consistently fall short of reasonable expectations.

x x x x

The psychological grounds are the best approach for anyone
who doubts whether he or she has a case for an annulment on any other
terms. A situation that does not fit into any of the more traditional
categories often fits very easily into the psychological category.

As new as the psychological grounds are, experts are already
detecting a shift in their use. Whereas originally the emphasis was on the
parties' inability to exercise proper judgment at the time of the marriage
(lack of due discretion), recent cases seem to be concentrating on the
parties' incapacity to assume or carry out their responsibilities and
obligations as promised (lack of due competence). An advantage to
using the ground of lack of due competence is that at the time the
marriage was entered into civil divorce and breakup of the family almost
always is proof of someone's failure to carry out marital responsibilities
as promised at the time the marriage was entered into.
[58]



Hernandez v. Court of Appeals
[59]
emphasizes the importance of presenting expert testimony to
establish the precise cause of a partys psychological incapacity, and to show that it existed at the
inception of the marriage. And as Marcos v. Marcos
[60]
asserts, there is no requirement that the person to
be declared psychologically incapacitated be personally examined by a physician, if the totality of
evidence presented is enough to sustain a finding of psychological incapacity.
[61]
Verily, the evidence
must show a link, medical or the like, between the acts that manifest psychological incapacity and the
psychological disorder itself.

This is not to mention, but we mention nevertheless for emphasis, that the presentation of expert
proof presupposes a thorough and in-depth assessment of the parties by the psychologist or expert, for a
conclusive diagnosis of a grave, severe and incurable presence of psychological
incapacity.
[62]
Parenthetically, the Court, at this point, finds it fitting to suggest the inclusion in the Rule on
Declaration of Absolute Nullity of Void Marriages and Annulment of Voidable Marriages,
[63]
an option for
the trial judge to refer the case to a court-appointed psychologist/expert for an independent assessment
and evaluation of the psychological state of the parties. This will assist the courts, who are no experts in
the field of psychology, to arrive at an intelligent and judicious determination of the case. The rule,
however, does not dispense with the parties prerogative to present their own expert witnesses.

Going back, in the case at bench, the psychological assessment, which we consider as adequate,
produced the findings that both parties are afflicted with personality disordersto repeat, dependent
personality disorder for petitioner, and narcissistic and antisocial personality disorder for respondent. We
note that The Encyclopedia of Mental Health discusses personality disorders as follows

A group of disorders involving behaviors or traits that are characteristic of a persons
recent and long-term functioning. Patterns of perceiving and thinking are not usually
limited to isolated episodes but are deeply ingrained, inflexible, maladaptive and severe
enough to cause the individual mental stress or anxieties or to interfere with interpersonal
relationships and normal functioning. Personality disorders are often recognizable by
adolescence or earlier, continue through adulthood and become less obvious in middle or
old age. An individual may have more than one personality disorder at a time.

The common factor among individuals who have personality disorders, despite a
variety of character traits, is the way in which the disorder leads to pervasive problems in
social and occupational adjustment. Some individuals with personality disorders are
perceived by others as overdramatic, paranoid, obnoxious or even criminal, without an
awareness of their behaviors. Such qualities may lead to trouble getting along with other
people, as well as difficulties in other areas of life and often a tendency to blame others
for their problems. Other individuals with personality disorders are not unpleasant or
difficult to work with but tend to be lonely, isolated or dependent. Such traits can lead to
interpersonal difficulties, reduced self-esteem and dissatisfaction with life.

Causes of Personality Disorders Different mental health viewpoints propose a
variety of causes of personality disorders. These include Freudian, genetic factors,
neurobiologic theories and brain wave activity.

Freudian Sigmund Freud believed that fixation at certain stages of development
led to certain personality types. Thus, some disorders as described in the Diagnostic and
Statistical Manual of Mental Disorders (3d ed., rev.) are derived from his oral, anal and
phallic character types. Demanding and dependent behavior (dependent and passive-
aggressive) was thought to derive from fixation at the oral stage. Characteristics of
obsessionality, rigidity and emotional aloofness were thought to derive from fixation at the
anal stage; fixation at the phallic stage was thought to lead to shallowness and an
inability to engage in intimate relationships. However, later researchers have found little
evidence that early childhood events or fixation at certain stages of development lead to
specific personality patterns.

Genetic Factors Researchers have found that there may be a genetic factor
involved in the etiology of antisocial and borderline personality disorders; there is less
evidence of inheritance of other personality disorders. Some family, adoption and twin
studies suggest that schizotypal personality may be related to genetic factors.

Neurobiologic Theories In individuals who have borderline personality,
researchers have found that low cerebrospinal fluid 5-hydroxyindoleacetic acid (5-HIAA)
negatively correlated with measures of aggression and a past history of suicide
attempts. Schizotypal personality has been associated with low platelet monoamine
oxidase (MAO) activity and impaired smooth pursuit eye movement.

Brain Wave Activity Abnormalities in electroencephalograph (EEG) have been
reported in antisocial personality for many years; slow wave is the most widely reported
abnormality. A study of borderline patients reported that 38 percent had at least marginal
EEG abnormalities, compared with 19 percent in a control group.

Types of Disorders According to the American Psychiatric
Associations Diagnostic and Statistical Manual of Mental Disorders (3d ed., rev., 1987),
or DSM-III-R, personality disorders are categorized into three major clusters:

Cluster A: Paranoid, schizoid and schizotypal personality disorders. Individuals
who have these disorders often appear to have odd or eccentric habits and traits.

Cluster B: Antisocial, borderline, histrionic and narcissistic personality
disorders. Individuals who have these disorders often appear overly emotional, erratic
and dramatic.

Cluster C: Avoidant, dependent, obsessive-compulsive and passive-aggressive
personality disorders. Individuals who have these disorders often appear anxious or
fearful.

The DSM-III-R also lists another category, personality disorder not otherwise
specified, that can be used for other specific personality disorders or for mixed
conditions that do not qualify as any of the specific personality disorders.

Individuals with diagnosable personality disorders usually have long-term
concerns, and thus therapy may be long-term.
[64]



Dependent personality disorder is characterized in the following manner


A personality disorder characterized by a pattern of dependent and submissive
behavior. Such individuals usually lack self-esteem and frequently belittle their
capabilities; they fear criticism and are easily hurt by others comments. At times they
actually bring about dominance by others through a quest for overprotection.

Dependent personality disorder usually begins in early adulthood. Individuals
who have this disorder may be unable to make everyday decisions without advice or
reassurance from others, may allow others to make most of their important decisions
(such as where to live), tend to agree with people even when they believe they are
wrong, have difficulty starting projects or doing things on their own, volunteer to do things
that are demeaning in order to get approval from other people, feel uncomfortable or
helpless when alone and are often preoccupied with fears of being abandoned.
[65]



and antisocial personality disorder described, as follows


Characteristics include a consistent pattern of behavior that is intolerant of the
conventional behavioral limitations imposed by a society, an inability to sustain a job over
a period of years, disregard for the rights of others (either through exploitiveness or
criminal behavior), frequent physical fights and, quite commonly, child or spouse abuse
without remorse and a tendency to blame others. There is often a faade of charm and
even sophistication that masks disregard, lack of remorse for mistreatment of others and
the need to control others.

Although characteristics of this disorder describe criminals, they also may befit
some individuals who are prominent in business or politics whose habits of self-
centeredness and disregard for the rights of others may be hidden prior to a public
scandal.

During the 19
th
century, this type of personality disorder was referred to as moral
insanity. The term described immoral, guiltless behavior that was not accompanied by
impairments in reasoning.

According to the classification system used in the Diagnostic and Statistical
Manual of Mental Disorders (3d ed., rev. 1987), anti-social personality disorder is one of
the four dramatic personality disorders, the others being borderline, histrionic and
narcissistic.
[66]



The seriousness of the diagnosis and the gravity of the disorders considered, the Court, in this
case, finds as decisive the psychological evaluation made by the expert witness; and, thus, rules that the
marriage of the parties is null and void on ground of both parties psychological incapacity. We further
consider that the trial court, which had a first-hand view of the witnesses deportment, arrived at the same
conclusion.

Indeed, petitioner, who is afflicted with dependent personality disorder, cannot assume the
essential marital obligations of living together, observing love, respect and fidelity and rendering help and
support, for he is unable to make everyday decisions without advice from others, allows others to make
most of his important decisions (such as where to live), tends to agree with people even when he believes
they are wrong, has difficulty doing things on his own, volunteers to do things that are demeaning in order
to get approval from other people, feels uncomfortable or helpless when alone and is often preoccupied
with fears of being abandoned.
[67]
As clearly shown in this case, petitioner followed everything dictated to
him by the persons around him. He is insecure, weak and gullible, has no sense of his identity as a
person, has no cohesive self to speak of, and has no goals and clear direction in life.

Although on a different plane, the same may also be said of the respondent. Her being afflicted with
antisocial personality disorder makes her unable to assume the essential marital obligations. This finding
takes into account her disregard for the rights of others, her abuse, mistreatment and control of others
without remorse, her tendency to blame others, and her intolerance of the conventional behavioral
limitations imposed by society.
[68]
Moreover, as shown in this case, respondent is impulsive and
domineering; she had no qualms in manipulating petitioner with her threats of blackmail and of committing
suicide.

Both parties being afflicted with grave, severe and incurable psychological incapacity, the
precipitous marriage which they contracted on April 23, 1996 is thus, declared null and void.

WHEREFORE, premises considered, the petition for review on certiorari is GRANTED. The
August 5, 2003 Decision and the January 19, 2004 Resolution of the Court of Appeals in CA-G.R. CV No.
71867 are REVERSED and SET ASIDE, and the Decision, dated July 30, 2001, REINSTATED.

SO ORDERED.


ANTONIO EDUARDO B. NACHURA
Associate Justice
























THIRD DIVISION
[G.R. No. 136490. October 19, 2000]
BRENDA B. MARCOS, petitioner, vs. WILSON G. MARCOS, respondent.
D E C I S I O N
PANGANIBAN, J .:
Psychological incapacity, as a ground for declaring the nullity of a marriage, may be established by
the totality of evidence presented. There is no requirement, however, that the respondent should be
examined by a physician or a psychologist as a conditio sine qua non for such declaration.
The Case

Before us is a Petition for Review on Certiorari under Rule 45 of the Rules of Court, assailing the
July 24, 1998 Decision
[1]
of the Court of Appeals (CA) in CA-GR CV No. 55588, which disposed as
follows:
"WHEREFORE, the contested decision is set aside and the marriage between the parties is hereby
declared valid."
[2]

Also challenged by petitioner is the December 3, 1998 CA Resolution denying her Motion for
Reconsideration.
Earlier, the Regional Trial Court (RTC) had ruled thus:
"WHEREFORE, the marriage between petitioner Brenda B. Marcos and respondent Wilson G. Marcos,
solemnized on September 6, 1982 in Pasig City is declared null and void ab initio pursuant to Art. 36 of
the Family Code. The conjugal properties, if any, is dissolved [sic] in accordance with Articles 126 and
129 of the same Code in relation to Articles 50, 51 and 52 relative to the delivery of the legitime of [the]
parties' children. In the best interest and welfare of the minor children, their custody is granted to
petitioner subject to the visitation rights of respondent.
"Upon finality of this Decision, furnish copy each to the Office of the Civil Registrar of Pasig City where
the marriage was solemnized, the National Census and Statistics Office, Manila and the Register of
Deeds of Mandaluyong City for their appropriate action consistent with this Decision.
"SO ORDERED."
The Facts

The facts as found by the Court of Appeals are as follows:
"It was established during the trial that the parties were married twice: (1) on September 6, 1982 which
was solemnized by Judge Eriberto H. Espiritu at the Municipal Court of Pasig (Exh. A); and (2) on May 8,
1983 which was solemnized by Rev. Eduardo L. Eleazar, Command Chaplain, at the Presidential
Security Command Chapel in Malacaang Park, Manila (Exh. A-1). Out of their marriage, five (5) children
were born (Exhs. B, C, D, E and F).
"Appellant Wilson G. Marcos joined the Armed Forces of the Philippines in 1973. Later on, he was
transferred to the Presidential Security Command in Malacaang during the Marcos Regime. Appellee
Brenda B. Marcos, on the other hand, joined the Women's Auxilliary Corps under the Philippine Air Force
in 1978. After the Edsa Revolution, both of them sought a discharge from the military service.
"They first met sometime in 1980 when both of them were assigned at the Malacaang Palace, she as an
escort of Imee Marcos and he as a Presidential Guard of President Ferdinand Marcos. Through
telephone conversations, they became acquainted and eventually became sweethearts.
"After their marriage on September 6, 1982, they resided at No. 1702 Daisy Street, Hulo Bliss,
Mandaluyong, a housing unit which she acquired from the Bliss Development Corporation when she was
still single.
"After the downfall of President Marcos, he left the military service in 1987 and then engaged in different
business ventures that did not however prosper. As a wife, she always urged him to look for work so that
their children would see him, instead of her, as the head of the family and a good provider. Due to his
failure to engage in any gainful employment, they would often quarrel and as a consequence, he would
hit and beat her. He would even force her to have sex with him despite her weariness. He would also
inflict physical harm on their children for a slight mistake and was so severe in the way he chastised
them. Thus, for several times during their cohabitation, he would leave their house. In 1992, they were
already living separately.
"All the while, she was engrossed in the business of selling "magic uling" and chickens. While she was
still in the military, she would first make deliveries early in the morning before going to Malacaang. When
she was discharged from the military service, she concentrated on her business. Then, she became a
supplier in the Armed Forces of the Philippines until she was able to put up a trading and construction
company, NS Ness Trading and Construction Development Corporation.
"The 'straw that broke the camel's back' took place on October 16, 1994, when they had a bitter
quarrel. As they were already living separately, she did not want him to stay in their house anymore. On
that day, when she saw him in their house, she was so angry that she lambasted him. He then turned
violent, inflicting physical harm on her and even on her mother who came to her aid. The following day,
October 17, 1994, she and their children left the house and sought refuge in her sister's house.
"On October 19, 1994, she submitted herself [to] medical examination at the Mandaluyong Medical
Center where her injuries were diagnosed as contusions (Exh. G, Records, 153).
"Sometime in August 1995, she together with her two sisters and driver, went to him at the Bliss unit in
Mandaluyong to look for their missing child, Niko. Upon seeing them, he got mad. After knowing the
reason for their unexpected presence, he ran after them with a samurai and even [beat] her driver.
"At the time of the filing of this case, she and their children were renting a house in Camella, Paraaque,
while the appellant was residing at the Bliss unit in Mandaluyong.
"In the case study conducted by Social Worker Sonia C. Millan, the children described their father as
cruel and physically abusive to them (Exh. UU, Records, pp. 85-100).
"The appellee submitted herself to psychologist Natividad A. Dayan, Ph.D., for psychological evaluation
(Exh. YY, Records, pp. 207-216), while the appellant on the other hand, did not.
"The court a quo found the appellant to be psychologically incapacitated to perform his marital obligations
mainly because of his failure to find work to support his family and his violent attitude towards appellee
and their children, x x x."
[3]

Ruling of the Court of Appeals

Reversing the RTC, the CA held that psychological incapacity had not been established by the
totality of the evidence presented. It ratiocinated in this wise:
"Essential in a petition for annulment is the allegation of the root cause of the spouse's psychological
incapacity which should also be medically or clinically identified, sufficiently proven by experts and clearly
explained in the decision. The incapacity must be proven to be existing at the time of the celebration of
the marriage and shown to be medically or clinically permanent or incurable. It must also be grave
enough to bring about the disability of the parties to assume the essential obligations of marriage as set
forth in Articles 68 to 71 and Articles 220 to 225 of the Family Code and such non-complied marital
obligations must similarly be alleged in the petition, established by evidence and explained in the
decision.
"In the case before us, the appellant was not subjected to any psychological or psychiatric
evaluation. The psychological findings about the appellant by psychiatrist Natividad Dayan were based
only on the interviews conducted with the appellee. Expert evidence by qualified psychiatrists and clinical
psychologists is essential if only to prove that the parties were or any one of them was mentally or
psychically ill to be truly incognitive of the marital obligations he or she was assuming, or as would make
him or her x x x unable to assume them. In fact, he offered testimonial evidence to show that he [was] not
psychologically incapacitated. The root cause of his supposed incapacity was not alleged in the petition,
nor medically or clinically identified as a psychological illness or sufficiently proven by an expert. Similarly,
there is no evidence at all that would show that the appellant was suffering from an incapacity which [was]
psychological or mental - not physical to the extent that he could not have known the obligations he was
assuming: that the incapacity [was] grave, ha[d] preceded the marriage and [was] incurable."
[4]

Hence, this Petition.
[5]

Issues

In her Memorandum,
[6]
petitioner presents for this Court's consideration the following issues:
"I. Whether or not the Honorable Court of Appeals could set aside the findings by the Regional
Trial Court of psychological incapacity of a respondent in a Petition for declaration of nullity
of marriage simply because the respondent did not subject himself to psychological
evaluation.
II. Whether or not the totality of evidence presented and the demeanor of all the witnesses
should be the basis of the determination of the merits of the Petition."
[7]

The Court's Ruling

We agree with petitioner that the personal medical or psychological examination of respondent is not
a requirement for a declaration of psychological incapacity. Nevertheless, the totality of the evidence she
presented does not show such incapacity.
Preliminary Issue: Need for Personal Medical Examination

Petitioner contends that the testimonies and the results of various tests that were submitted to
determine respondent's psychological incapacity to perform the obligations of marriage should not have
been brushed aside by the Court of Appeals, simply because respondent had not taken those tests
himself. Petitioner adds that the CA should have realized that under the circumstances, she had no
choice but to rely on other sources of information in order to determine the psychological capacity of
respondent, who had refused to submit himself to such tests.
In Republic v. CA and Molina,
[8]
the guidelines governing the application and the interpretation
of psychological incapacity referred to in Article 36 of the Family Code
[9]
were laid down by this Court as
follows:
"1) The burden of proof to show the nullity of the marriage belongs to the plaintiff. Any doubt
should be resolved in favor of the existence and continuation of the marriage and against its
dissolution and nullity. This is rooted in the fact that both our Constitution and our laws
cherish the validity of marriage and unity of the family. Thus, our Constitution devotes an
entire Article on the Family, recognizing it 'as the foundation of the nation.' It decrees
marriage as legally 'inviolable,' thereby protecting it from dissolution at the whim of the
parties. Both the family and marriage are to be 'protected' by the state.
x x x x x x x x x
2) The root cause of the psychological incapacity must be: (a) medically or clinically identified,
(b) alleged in the complaint, (c) sufficiently proven by experts and (d) clearly explained in the
decision.Article 36 of the Family Code requires that the incapacity must be psychological -
not physical, although its manifestations and/or symptoms may be physical. The evidence
must convince the court that the parties, or one of them, was mentally or psychically ill to
such an extent that the person could not have known the obligations he was assuming, or
knowing them, could not have given valid assumption thereof. Although no example of such
incapacity need be given here so as not to limit the application of the provision under the
principle of ejusdem generis, nevertheless such root cause must be identified as a
psychological illness and its incapacitating nature fully explained. Expert evidence may be
given by qualified psychiatrists and clinical psychologists.
3) The incapacity must be proven to be existing at 'the time of the celebration' of the
marriage. The evidence must show that the illness was existing when the parties exchanged
their 'I do's.' The manifestation of the illness need not be perceivable at such time, but the
illness itself must have attached at such moment, or prior thereto.
4) Such incapacity must also be shown to be medically or clinically permanent or
incurable. Such incurability may be absolute or even relative only in regard to the other
spouse, not necessarily absolutely against everyone of the same sex. Furthermore, such
incapacity must be relevant to the assumption of marriage obligations, not necessarily to
those not related to marriage, like the exercise of a profession or employment in a
job. Hence, a pediatrician may be effective in diagnosing illnesses of children and
prescribing medicine to cure them but not be psychologically capacitated to procreate, bear
and raise his/her own children as an essential obligation of marriage.
5) Such illness must be grave enough to bring about the disability of the party to assume the
essential obligations of marriage. Thus, 'mild characteriological peculiarities, mood changes,
occasional emotional outbursts cannot be accepted as root causes. The illness must be
shown as downright incapacity or inability, not a refusal, neglect or difficulty, much less ill
will. In other words, there is a natal or supervening disabling factor in the person, an adverse
integral element in the personality structure that effectively incapacitates the person from
really accepting and thereby complying with the obligations essential to marriage.
6) The essential marital obligations must be those embraced by Articles 68 up to 71 of the
Family Code as regards the husband and wife as well as Articles 220, 221 and 225 of the
same Code in regard to parents and their children. Such non-complied marital obligation(s)
must also be stated in the petition, proven by evidence and included in the text of the
decision.
7) Interpretations given by the National Appellate Matrimonial Tribunal of the Catholic Church in
the Philippines, while not controlling or decisive, should be given great respect by our courts.
x x x x x x x x x
(8) The trial court must order the prosecuting attorney or fiscal and the Solicitor General to
appear as counsel for the state. No decision shall be handed down unless the Solicitor
General issues a certification, which will be quoted in the decision, briefly stating therein his
reasons for his agreement or opposition, as the case may be, to the petition. The Solicitor
General, along with the prosecuting attorney, shall submit to the court such certification
within fifteen (15) days from the date the case is deemed submitted for resolution of the
court. The Solicitor General shall discharge the equivalent function of the defensor
vinculi contemplated under Canon 1095."
[10]

The guidelines incorporate the three basic requirements earlier mandated by the Court in Santos v.
Court of Appeals:
[11]
"psychological incapacity must be characterized by (a) gravity (b) juridical
antecedence, and (c) incurability." The foregoing guidelines do not require that a physician examine the
person to be declared psychologically incapacitated. In fact, the root cause may be "medically or
clinically identified." What is important is the presence of evidence that can adequately establish the
party's psychological condition. For indeed, if the totality of evidence presented is enough to sustain a
finding of psychological incapacity, then actual medical examination of the person concerned need not be
resorted to.
Main Issue: Totality of Evidence Presented

The main question, then, is whether the totality of the evidence presented in the present case --
including the testimonies of petitioner, the common children, petitioner's sister and the social worker --
was enough to sustain a finding that respondent was psychologically incapacitated.
We rule in the negative. Although this Court is sufficiently convinced that respondent failed to provide
material support to the family and may have resorted to physical abuse and abandonment, the totality of
his acts does not lead to a conclusion of psychological incapacity on his part. There is absolutely no
showing that his "defects" were already present at the inception of the marriage or that they are incurable.
Verily, the behavior of respondent can be attributed to the fact that he had lost his job and was not
gainfully employed for a period of more than six years. It was during this period that he became
intermittently drunk, failed to give material and moral support, and even left the family home.
Thus, his alleged psychological illness was traced only to said period and not to the inception of the
marriage. Equally important, there is no evidence showing that his condition is incurable, especially now
that he is gainfully employed as a taxi driver.
Article 36 of the Family Code, we stress, is not to be confused with a divorce law that cuts the marital
bond at the time the causes therefor manifest themselves. It refers to a serious psychological illness
afflicting a party even before the celebration of the marriage. It is a malady so grave and so permanent as
to deprive one of awareness of the duties and responsibilities of the matrimonial bond one is about to
assume. These marital obligations are those provided under Articles 68 to 71, 220, 221 and 225 of the
Family Code.
Neither is Article 36 to be equated with legal separation, in which the grounds need not be rooted in
psychological incapacity but on physical violence, moral pressure, moral corruption, civil interdiction, drug
addiction, habitual alcoholism, sexual infidelity, abandonment and the like.
[12]
At best, the evidence
presented by petitioner refers only to grounds for legal separation, not for declaring a marriage void.
Because Article 36 has been abused as a convenient divorce law, this Court laid down the
procedural requirements for its invocation in Molina. Petitioner, however, has not faithfully observed them.
In sum, this Court cannot declare the dissolution of the marriage for failure of petitioner to show that
the alleged psychological incapacity is characterized by gravity, juridical antecedence and incurability;
and for her failure to observe the guidelines outlined in Molina.
WHEREFORE, the Petition is DENIED and assailed Decision AFFIRMED, except that portion
requiring personal medical examination as a conditio sine qua non to a finding of psychological
incapacity. No costs.
SO ORDERED.
Melo, (Chairman), Vitug, Purisima, and Gonzaga-Reyes, JJ., concur.




















SECOND DIVISION
[G.R. No. 119190. January 16, 1997]
CHI MING TSOI, petitioner, vs. COURT OF APPEALS and GINA LAO-TSOI, respondents.
D E C I S I O N
TORRES, JR., J .:
Man has not invented a reliable compass by which to steer a marriage in its journey over troubled
waters. Laws are seemingly inadequate. Over time, much reliance has been placed in the works of the
unseen hand of Him who created all things.
Who is to blame when a marriage fails?
This case was originally commenced by a distraught wife against her uncaring husband in the
Regional Trial Court of Quezon City (Branch 89) which decreed the annulment of the marriage on the
ground of psychological incapacity. Petitioner appealed the decision of the trial court to respondent Court
of Appeals (CA-G.R. CV No. 42758) which affirmed the Trial Court's decision on November 29, 1994 and
correspondingly denied the motion for reconsideration in a resolution dated February 14, 1995.
The statement of the case and of the facts made by the trial court and reproduced by the Court of
Appeals
[1]
in its decision are as follows:
"From the evidence adduced, the following facts were preponderantly established:
"Sometime on May 22, 1988, the plaintiff married the defendant at the Manila Cathedral, xxx Intramuros Manila, as
evidenced by their Marriage Contract. (Exh. "A")
"After the celebration of their marriage and wedding reception at the South Villa, Makati, they went and proceeded
to the house of defendant's mother.
"There, they slept together on the same bed in the same room for the first night of their married life.
"It is the version of the plaintiff, that contrary to her expectations, that as newlyweds they were supposed to enjoy
making love, or having sexual intercourse, with each other, the defendant just went to bed, slept on one side thereof,
then turned his back and went to sleep. There was no sexual intercourse between them during the first night. The
same thing happened on the second, third and fourth nights.
"In an effort to have their honeymoon in a private place where they can enjoy together during their first week as
husband and wife, they went to Baguio City. But, they did so together with her mother, an uncle, his mother and his
nephew. They were all invited by the defendant to join them. [T]hey stayed in Baguio City for four (4) days. But,
during this period, there was no sexual intercourse between them, since the defendant avoided her by taking a long
walk during siesta time or by just sleeping on a rocking chair located at the living room. They slept together in the
same room and on the same bed since May 22, 1988 until March 15, 1989. But during this period, there was no
attempt of sexual intercourse between them. [S]he claims, that she did not even see her husband's private parts nor
did he see hers.
"Because of this, they submitted themselves for medical examinations to Dr. Eufemio Macalalag, a urologist at the
Chinese General Hospital, on January 20, 1989.
The results of their physical examinations were that she is healthy, normal and still a virgin, while that of her
husbands examination was kept confidential up to this time. While no medicine was prescribed for her, the doctor
prescribed medications for her husband which was also kept confidential. No treatment was given to her. For her
husband, he was asked by the doctor to return but he never did.
"The plaintiff claims, that the defendant is impotent, a closet homosexual as he did not show his penis. She said, that
she had observed the defendant using an eyebrow pencil and sometimes the cleansing cream of his mother. And that,
according to her, the defendant married her, a Filipino citizen, to acquire or maintain his residency status here in the
country and to publicly maintain the appearance of a normal man.
"The plaintiff is not willing to reconcile with her husband.
"On the other hand, it is the claim of the defendant that if their marriage shall be annulled by reason of psychological
incapacity, the fault lies with his wife.
"But, he said that he does not want his marriage with his wife annulled for several reasons, viz: (1) that he loves her
very much; (2) that he has no defect on his part and he is physically and psychologically capable; and, (3) since the
relationship is still very young and if there is any differences between the two of them, it can still be reconciled and
that, according to him, if either one of them has some incapabilities, there is no certainty that this will not be cured.
He further claims, that if there is any defect, it can be cured by the intervention of medical technology or science
"The defendant admitted that since their marriage on May 22, 1988, until their separation on March 15, 1989, there
was no sexual contact between them. But, the reason for this, according to the defendant, was that everytime he
wants to have sexual intercourse with his wife, she always avoided him and whenever he caresses her private parts,
she always removed his hands. The defendant claims, that he forced his wife to have sex with him only once but he
did not continue because she was shaking and she did not like it. So he stopped.
"There are two (2) reasons, according to the defendant, why the plaintiff filed this case against him, and these are:
(1) that she is afraid that she will be forced to return the pieces of jewelry of his mother, and, (2) that her husband,
the defendant, will consummate their marriage.
"The defendant insisted that their marriage will remain valid because they are still very young and there is still a
chance to overcome their differences.
"The defendant submitted himself to a physical examination. His penis was examined by Dr. Sergio Alteza, Jr., for
the purpose of finding out whether he is impotent. As a result thereof, Dr. Alteza submitted his Doctor's Medical
Report. (Exh. "2"). It is stated there, that there is no evidence of impotency (Exh. "2-B"), and he is capable of
erection. (Exh. "2-C")
"The doctor said, that he asked the defendant to masturbate to find out whether or not he has an erection and he
found out that from the original size of two (2) inches, or five (5) centimeters, the penis of the defendant lengthened
by one (1) inch and one centimeter. Dr. Alteza said, that the defendant had only a soft erection which is why his
penis is not in its full length. But, still is capable of further erection, in that with his soft erection, the defendant is
capable of having sexual intercourse with a woman.
"In open Court, the Trial Prosecutor manifested that there is no collusion between the parties and that the evidence is
not fabricated.
[2]

After trial, the court rendered judgment, the dispositive portion of which reads:
"ACCORDINGLY, judgment is hereby rendered declaring as VOID the marriage entered into by the plaintiff with
the defendant on May 22, 1988 at the Manila Cathedral, Basilica of the Immaculate Conception, Intramuros, Manila,
before the Rt. Rev. Msgr. Melencio de Vera. Without costs. Let a copy of this decision be furnished the Local Civil
Registrar of Quezon City. Let another copy be furnished the Local Civil Registrar of Manila.
"SO ORDERED. "
On appeal, the Court of Appeals affirmed the trial court's decision.
Hence, the instant petition.
Petitioner alleges that the respondent Court of Appeals erred:
I
in affirming the conclusions of the lower court that there was no sexual intercourse between the parties without
making any findings of fact.
II
in holding that the refusal of private respondent to have sexual communion with petitioner is a psychological
incapacity inasmuch as proof thereof is totally absent.
III
in holding that the alleged refusal of both the petitioner and the private respondent to have sex with each other
constitutes psychological incapacity of both.
IV
in affirming the annulment of the marriage between the parties decreed by the lower court without fully satisfying
itself that there was no collusion between them.
We find the petition to be bereft of merit.
Petitioner contends that being the plaintiff in Civil Case No. Q-89-3141, private respondent has the
burden of proving the allegations in her complaint; that since there was no independent evidence to prove
the alleged non-coitus between the parties, there remains no other basis for the court's conclusion except
the admission of petitioner; that public policy should aid acts intended to validate marriage and should
retard acts intended to invalidate them; that the conclusion drawn by the trial court on the admissions and
confessions of the parties in their pleadings and in the course of the trial is misplaced since it could have
been a product of collusion; and that in actions for annulment of marriage, the material facts alleged in the
complaint shall always be proved.
[3]

Section 1, Rule 19 of the Rules of Court reads:
"Section 1. Judgment on the pleadings. - Where an answer fails to tender an issue, or otherwise admits the material
allegations of the adverse party's pleading, the court may, on motion of that party, direct judgment on such pleading.
But in actions for annulment of marriage or for legal separation the material facts alleged in the complaint shall
always be proved."
The foregoing provision pertains to a judgment on the pleadings. What said provision seeks to
prevent is annulment of marriage without trial. The assailed decision was not based on such a judgment
on the pleadings. When private respondent testified under oath before the trial court and was cross-
examined by oath before the trial court and was cross-examined by the adverse party, she thereby
presented evidence in the form of a testimony. After such evidence was presented, it became incumbent
upon petitioner to present his side. He admitted that since their marriage on May 22, 1988, until their
separation on March 15, 1989, there was no sexual intercourse between them.
To prevent collusion between the parties is the reason why, as stated by the petitioner, the Civil
Code provides that no judgment annulling a marriage shall be promulgated upon a stipulation of facts or
by confession of judgment (Arts. 88 and 101[par. 2]) and the Rules of Court prohibit such annulment
without trial (Sec. 1, Rule 19).
The case has reached this Court because petitioner does not want their marriage to be annulled.
This only shows that there is no collusion between the parties. When petitioner admitted that he and his
wife (private respondent) have never had sexual contact with each other, he must have been only telling
the truth. We are reproducing the relevant portion of the challenged resolution denying petitioner's Motion
for Reconsideration, penned with magisterial lucidity by Associate Justice Minerva Gonzaga-Reyes, viz:
"The judgment of the trial court which was affirmed by this Court is not based on a stipulation of facts. The issue of
whether or not the appellant is psychologically incapacitated to discharge a basic marital obligation was resolved
upon a review of both the documentary and testimonial evidence on record. Appellant admitted that he did not have
sexual relations with his wife after almost ten months of cohabitation, and it appears that he is not suffering from
any physical disability. Such abnormal reluctance or unwillingness to consummate his marriage is strongly
indicative of a serious personality disorder which to the mind of this Court clearly demonstrates an 'utter
insensitivity or inability to give meaning and significance to the marriage' within the meaning of Article 36 of the
Family Code (See Santos vs. Court of Appeals, G.R. No. 112019, January 4, 1995)."
[4]

Petitioner further contends that respondent court erred in holding that the alleged refusal of both the
petitioner and the private respondent to have sex with each other constitutes psychological incapacity of
both. He points out as error the failure of the trial court to make "a categorical finding about the alleged
psychological incapacity and an in-depth analysis of the reasons for such refusal which may not be
necessarily due to psychological disorders" because there might have been other reasons, - i.e., physical
disorders, such as aches, pains or other discomforts, - why private respondent would not want to have
sexual intercourse from May 22, 1988 to March 15, 1989, in a short span of 10 months.
First, it must be stated that neither the trial court nor the respondent court made a finding on who
between petitioner and private respondent refuses to have sexual contact with the other. The fact
remains, however, that there has never been coitus between them. At any rate, since the action to
declare the marriage void may be filed by either party, i.e., even the psychologically incapacitated, the
question of who refuses to have sex with the other becomes immaterial.
Petitioner claims that there is no independent evidence on record to show that any of the parties is
suffering from psychological incapacity. Petitioner also claims that he wanted to have sex with private
respondent; that the reason for private respondent's refusal may not be psychological but physical
disorder as stated above.
We do not agree. Assuming it to be so, petitioner could have discussed with private respondent or
asked her what is ailing her, and why she balks and avoids him everytime he wanted to have sexual
intercourse with her. He never did. At least, there is nothing in the record to show that he had tried to find
out or discover what the problem with his wife could be. What he presented in evidence is his doctor's
Medical Report that there is no evidence of his impotency and he is capable of erection.
[5]
Since it is
petitioner's claim that the reason is not psychological but perhaps physical disorder on the part of private
respondent, it became incumbent upon him to prove such a claim.
"If a spouse, although physically capable but simply refuses to perform his or her essential marriage obligations, and
the refusal is senseless and constant, Catholic marriage tribunals attribute the causes to psychological incapacity
than to stubborn refusal. Senseless and protracted refusal is equivalent to psychological incapacity. Thus, the
prolonged refusal of a spouse to have sexual intercourse with his or her spouse is considered a sign of psychological
incapacity."
[6]

Evidently, one of the essential marital obligations under the Family Code is "To procreate children
based on the universal principle that procreation of children through sexual cooperation is the basic end
of marriage." Constant non-fulfillment of this obligation will finally destroy the integrity or wholeness of the
marriage. In the case at bar, the senseless and protracted refusal of one of the parties to fulfill the above
marital obligation is equivalent to psychological incapacity.
As aptly stated by the respondent court,
"An examination of the evidence convinces Us that the husband's plea that the wife did not want carnal intercourse
with him does not inspire belief. Since he was not physically impotent, but he refrained from sexual intercourse
during the entire time (from May 22, 1988 to March 15, 1989) that he occupied the same bed with his wife, purely
out of sympathy for her feelings, he deserves to be doubted for not having asserted his rights even though she balked
(Tompkins vs. Tompkins, 111 Atl. 599, cited in I Paras, Civil Code, at p. 330). Besides, if it were true that it is the
wife who was suffering from incapacity, the fact that defendant did not go to court and seek the declaration of
nullity weakens his claim. This case was instituted by the wife whose normal expectations of her marriage were
frustrated by her husband's inadequacy. Considering the innate modesty of the Filipino woman, it is hard to believe
that she would expose her private life to public scrutiny and fabricate testimony against her husband if it were not
necessary to put her life in order and put to rest her marital status.
"We are not impressed by defendant's claim that what the evidence proved is the unwillingness or lack of intention
to perform the sexual act, which is not psychological incapacity, and which can be achieved "through proper
motivation." After almost ten months of cohabitation, the admission that the husband is reluctant or unwilling to
perform the sexual act with his wife whom he professes to love very dearly, and who has not posed any
insurmountable resistance to his alleged approaches, is indicative of a hopeless situation, and of a serious personality
disorder that constitutes psychological incapacity to discharge the basic marital covenants within the contemplation
of the Family Code.
[7]

While the law provides that the husband and the wife are obliged to live together, observe mutual
love, respect and fidelity (Art. 68, Family Code), the sanction therefor is actually the "spontaneous, mutual
affection between husband and wife and not any legal mandate or court order" (Cuaderno vs. Cuaderno,
120 Phil. 1298). Love is useless unless it is shared with another. Indeed, no man is an island, the cruelest
act of a partner in marriage is to say "I could not have cared less." This is so because an ungiven self is
an unfulfilled self. The egoist has nothing but himself. In the natural order, it is sexual intimacy which
brings spouses wholeness and oneness. Sexual intimacy is a gift and a participation in the mystery of
creation. It is a function which enlivens the hope of procreation and ensures the continuation of family
relations.
It appears that there is absence of empathy between petitioner and private respondent. That is - a
shared feeling which between husband and wife must be experienced not only by having spontaneous
sexual intimacy but a deep sense of spiritual communion. Marital union is a two-way process. An
expressive interest in each other's feelings at a time it is needed by the other can go a long way in
deepening the marital relationship. Marriage is definitely not for children but for two consenting adults who
view the relationship with love amor gignit amorem, respect, sacrifice and a continuing commitment to
compromise, conscious of its value as a sublime social institution.
This Court, finding the gravity of the failed relationship in which the parties found themselves trapped
in its mire of unfulfilled vows and unconsummated marital obligations, can do no less but sustain the
studied judgment of respondent appellate court.
IN VIEW OF THE FOREGOING PREMISES, the assailed decision of the Court of Appeals dated
November 29, 1994 is hereby AFFIRMED in all respects and the petition is hereby DENIED for lack of
merit.
SO ORDERED.
Regalado, (Chairman), Romero, Puno, and Mendoza, JJ., concur.

SECOND DIVISION
[G.R. No. 137567. June 20, 2000]
MEYNARDO L. BELTRAN, petitioner, vs. PEOPLE OF THE PHILIPPINES, and HON. JUDGE
FLORENTINO TUAZON, JR., being the Judge of the RTC, Branch 139, Makati City, respondents.
D E C I S I O N
BUENA, J .:
This petition for review, filed under Rule 45 of the 1997 Rules of Civil Procedure, seeks to review and set
aside the Order dated January 28, 1999 issued by Judge Florentino A. Tuazon, Jr. of the Regional Trial
Court of Makati City, Branch 139 in Special Civil Case No. 98-3056, entitled "Meynardo Beltran vs.
People of the Philippines and Hon. Judge Alden Cervantes of the Metropolitan Trial Court of Makati city,
Branch 61." The said Order denied petitioners prayer for the issuance of a writ of preliminary injunction to
enjoin Judge Cervantes from proceeding with the trial of Criminal Case No. 236176, a concubinage case
against petitioner on the ground that the pending petition for declaration of nullity of marriage filed by
petitioner against his wife constitutes a prejudicial question.
The antecedent facts of the case are undisputed:
Petitioner Meynardo Beltran and wife Charmaine E. Felix were married on June 16, 1973 at the
Immaculate Concepcion Parish Church in Cubao, Quezon City.
[1]

On February 7, 1997, after twenty-four years of marriage and four children,
[2]
petitioner filed a petition for
nullity of marriage on the ground of psychological incapacity under Article 36 of the Family Code before
Branch 87 of the Regional Trial Court of Quezon City. The case was docketed as Civil Case No. Q-97-
30192.
[3]

In her Answer to the said petition, petitioner's wife Charmaine Felix alleged that it was petitioner
who abandoned the conjugal home and lived with a certain woman named Milagros Salting.
[4]
Charmaine
subsequently filed a criminal complaint for concubinage
[5]
under Article 334 of the Revised Penal Code
against petitioner and his paramour before the City Prosecutor's Office of Makati who, in a Resolution
dated September 16, 1997, found probable cause and ordered the filing of an Information
[6]
against them.
The case, docketed as Criminal Case No. 236176, was filed before the Metropolitan Trial Court of Makati
City, Branch 61.
On March 20, 1998, petitioner, in order to forestall the issuance of a warrant for his arrest, filed a Motion
to Defer Proceedings Including the Issuance of the Warrant of Arrest in the criminal case. Petitioner
argued that the pendency of the civil case for declaration of nullity of his marriage posed a prejudicial
question to the determination of the criminal case. Judge Alden Vasquez Cervantes denied the foregoing
motion in the Order
[7]
dated August 31, 1998. Petitioner's motion for reconsideration of the said Order of
denial was likewise denied in an Order dated December 9, 1998.
In view of the denial of his motion to defer the proceedings in the concubinage case, petitioner went to the
Regional Trial Court of Makati City, Branch 139 on certiorari, questioning the Orders dated August 31,
1998 and December 9, 1998 issued by Judge Cervantes and praying for the issuance of a writ of
preliminary injunction.
[8]
In an Order
[9]
dated January 28, 1999, the Regional Trial Court of Makati denied
the petition for certiorari. Said Court subsequently issued another Order
[10]
dated February 23, 1999,
denying his motion for reconsideration of the dismissal of his petition.
Undaunted, petitioner filed the instant petition for review.
Petitioner contends that the pendency of the petition for declaration of nullity of his marriage based on
psychological incapacity under Article 36 of the Family Code is a prejudicial question that should merit the
suspension of the criminal case for concubinage filed against him by his wife.
Petitioner also contends that there is a possibility that two conflicting decisions might result from the civil
case for annulment of marriage and the criminal case for concubinage. In the civil case, the trial court
might declare the marriage as valid by dismissing petitioner's complaint but in the criminal case, the trial
court might acquit petitioner because the evidence shows that his marriage is void on ground of
psychological incapacity. Petitioner submits that the possible conflict of the courts' ruling regarding
petitioner's marriage can be avoided, if the criminal case will be suspended, until the court rules on the
validity of marriage; that if petitioner's marriage is declared void by reason of psychological incapacity
then by reason of the arguments submitted in the subject petition, his marriage has never existed; and
that, accordingly, petitioner could not be convicted in the criminal case because he was never before a
married man.
Petitioner's contentions are untenable.
The rationale behind the principle of prejudicial question is to avoid two conflicting decisions. It has two
essential elements: (a) the civil action involves an issue similar or intimately related to the issue raised in
the criminal action; and (b) the resolution of such issue determines whether or not the criminal action may
proceed.
[11]

The pendency of the case for declaration of nullity of petitioner's marriage is not a prejudicial question to
the concubinage case. For a civil case to be considered prejudicial to a criminal action as to cause the
suspension of the latter pending the final determination of the civil case, it must appear not only that the
said civil case involves the same facts upon which the criminal prosecution would be based, but also that
in the resolution of the issue or issues raised in the aforesaid civil action, the guilt or innocence of the
accused would necessarily be determined.
Article 40 of the Family Code provides:
"The absolute nullity of a previous marriage may be invoked for purposes of remarriage
on the basis solely of a final judgment declaring such previous marriage void."
In Domingo vs. Court of Appeals,
[12]
this Court ruled that the import of said provision is that for purposes
of remarriage, the only legally acceptable basis for declaring a previous marriage an absolute nullity is a
final judgment declaring such previous marriage void, whereas, for purposes of other than remarriage,
other evidence is acceptable. The pertinent portions of said Decision read:
"xxx Undoubtedly, one can conceive of other instances where a party might well invoke
the absolute nullity of a previous marriage for purposes other than remarriage, such as in
case of an action for liquidation, partition, distribution and separation of property between
the erstwhile spouses, as well as an action for the custody and support of their common
children and the delivery of the latters' presumptive legitimes. In such cases, evidence
needs must be adduced, testimonial or documentary, to prove the existence of grounds
rendering such a previous marriage an absolute nullity. These needs not be limited solely
to an earlier final judgment of a court declaring such previous marriage void."
So that in a case for concubinage, the accused, like the herein petitioner need not present a final
judgment declaring his marriage void for he can adduce evidence in the criminal case of the nullity of his
marriage other than proof of a final judgment declaring his marriage void.
With regard to petitioner's argument that he could be acquitted of the charge of concubinage should his
marriage be declared null and void, suffice it to state that even a subsequent pronouncement that his
marriage is void from the beginning is not a defense.
Analogous to this case is that of Landicho vs. Reloval
[13]
cited in Donato vs. Luna
[14]
where this Court
held that:
"xxx Assuming that the first marriage was null and void on the ground alleged by
petitioner, that fact would not be material to the outcome of the criminal case. Parties to
the marriage should not be permitted to judge for themselves its nullity, for the same
must be submitted to the judgment of the competent courts and only when the nullity of
the marriage is so declared can it be held as void, and so long as there is no such
declaration the presumption is that the marriage exists. Therefore, he who contracts a
second marriage before the judicial declaration of nullity of the first marriage assumes the
risk of being prosecuted for bigamy."
Thus, in the case at bar it must also be held that parties to the marriage should not be permitted to judge
for themselves its nullity, for the same must be submitted to judgment of the competent courts and only
when the nullity of the marriage is so declared can it be held as void, and so long as there is no such
declaration the presumption is that the marriage exists for all intents and purposes. Therefore, he who
cohabits with a woman not his wife before the judicial declaration of nullity of the marriage assumes the
risk of being prosecuted for concubinage. The lower court therefore, has not erred in affirming the Orders
of the judge of the Metropolitan Trial Court ruling that pendency of a civil action for nullity of marriage
does not pose a prejudicial question in a criminal case for concubinage.
WHEREFORE, for lack of merit, the instant petition is DISMISSED.
SO ORDERED.
Bellosillo, (Chairman), Mendoza, Quisumbing, and De Leon, Jr., JJ., concur.












Republic of the Philippines
SUPREME COURT
Manila
THIRD DIVISION

G.R. No. 94053 March 17, 1993
REPUBLIC OF THE PHILIPPINES, petitioner,
vs.
GREGORIO NOLASCO, respondent.
The Solicitor General for plaintiff-appellee.
Warloo G. Cardenal for respondent.
R E S O L U T I O N

FELICIANO, J .:
On 5 August 1988, respondent Gregorio Nolasco filed before the Regional Trial Court of Antique, Branch
10, a petition for the declaration of presumptive death of his wife Janet Monica Parker, invoking Article 41
of the Family Code. The petition prayed that respondent's wife be declared presumptively dead or, in the
alternative, that the marriage be declared null and void.
1

The Republic of the Philippines opposed the petition through the Provincial Prosecutor of Antique who
had been deputized to assist the Solicitor-General in the instant case. The Republic argued, first, that
Nolasco did not possess a "well-founded belief that the absent spouse was already dead,"
2
and second,
Nolasco's attempt to have his marriage annulled in the same proceeding was a "cunning attempt" to
circumvent the law on marriage.
3

During trial, respondent Nolasco testified that he was a seaman and that he had first met Janet Monica
Parker, a British subject, in a bar in England during one of his ship's port calls. From that chance meeting
onwards, Janet Monica Parker lived with respondent Nolasco on his ship for six (6) months until they
returned to respondent's hometown of San Jose, Antique on 19 November 1980 after his seaman's
contract expired. On 15 January 1982, respondent married Janet Monica Parker in San Jose, Antique, in
Catholic rites officiated by Fr. Henry van Tilborg in the Cathedral of San Jose.
Respondent Nolasco further testified that after the marriage celebration, he obtained another employment
contract as a seaman and left his wife with his parents in San Jose, Antique. Sometime in January 1983,
while working overseas, respondent received a letter from his mother informing him that Janet Monica
had given birth to his son. The same letter informed him that Janet Monica had left Antique. Respondent
claimed he then immediately asked permission to leave his ship to return home. He arrived in Antique in
November 1983.
Respondent further testified that his efforts to look for her himself whenever his ship docked in England
proved fruitless. He also stated that all the letters he had sent to his missing spouse at No. 38 Ravena
Road, Allerton, Liverpool, England, the address of the bar where he and Janet Monica first met, were all
returned to him. He also claimed that he inquired from among friends but they too had no news of Janet
Monica.
On cross-examination, respondent stated that he had lived with and later married Janet Monica Parker
despite his lack of knowledge as to her family background. He insisted that his wife continued to refuse to
give him such information even after they were married. He also testified that he did not report the matter
of Janet Monica's disappearance to the Philippine government authorities.
Respondent Nolasco presented his mother, Alicia Nolasco, as his witness. She testified that her
daughter-in-law Janet Monica had expressed a desire to return to England even before she had given
birth to Gerry Nolasco on 7 December 1982. When asked why her daughter-in-law might have wished to
leave Antique, respondent's mother replied that Janet Monica never got used to the rural way of life in
San Jose, Antique. Alicia Nolasco also said that she had tried to dissuade Janet Monica from leaving as
she had given birth to her son just fifteen days before, but when she (Alicia) failed to do so, she gave
Janet Monica P22,000.00 for her expenses before she left on 22 December 1982 for England. She further
claimed that she had no information as to the missing person's present whereabouts.
The trial court granted Nolasco's petition in a Judgment dated 12 October 1988 the dispositive portion of
which reads:
Wherefore, under Article 41, paragraph 2 of the Family Code of the Philippines
(Executive Order No. 209, July 6, 1987, as amended by Executive Order No. 227, July
17, 1987) this Court hereby declares as presumptively dead Janet Monica Parker
Nolasco, without prejudice to her reappearance.
4

The Republic appealed to the Court of Appeals contending that the trial court erred in declaring Janet
Monica Parker presumptively dead because respondent Nolasco had failed to show that there existed a
well founded belief for such declaration.
The Court of Appeals affirmed the trial court's decision, holding that respondent had sufficiently
established a basis to form a belief that his absent spouse had already died.
The Republic, through the Solicitor-General, is now before this Court on a Petition for Review where the
following allegations are made:
1. The Court of Appeals erred in affirming the trial court's finding that there existed a well-
founded belief on the part of Nolasco that Janet Monica Parker was already dead; and
2. The Court of Appeals erred in affirming the trial Court's declaration that the petition
was a proper case of the declaration of presumptive death under Article 41, Family
Code.
5

The issue before this Court, as formulated by petitioner is "[w]hether or not Nolasco has a well-founded
belief that his wife is already dead."
6

The present case was filed before the trial court pursuant to Article 41 of the Family Code which provides
that:
Art. 41. A marriage contracted by any person during the subsistence of a previous
marriage shall be null and void, unless before the celebration of the subsequent
marriage, the prior spouse had been absent for four consecutive years and the spouse
present had a well-founded belief that the absent spouse was already dead. In case of
disappearance where there is danger of death under the circumstances set forth in the
provision of Article 391 of the Civil Code, an absence of only two years shall be sufficient.
For the purpose of contracting the subsequent marriage under the preceding paragraph,
the spouse present must institute a summary proceeding as provided in this Code for the
declaration of presumptive death of the absentee, without prejudice to the effect of
reappearance of the absent spouse. (Emphasis supplied).
When Article 41 is compared with the old provision of the Civil Code, which it superseded,
7
the following
crucial differences emerge. Under Article 41, the time required for the presumption to arise has been
shortened to four (4) years; however, there is need for a judicial declaration of presumptive death to
enable the spouse present to remarry.
8
Also, Article 41 of the Family Code imposes a stricter standard
than the Civil Code: Article 83 of the Civil Code merely requires either that there be no news that such
absentee is still alive; or the absentee is generally considered to be dead and believed to be so by the
spouse present, or is presumed dead under Article 390 and 391 of the Civil Code.
9
The Family Code,
upon the other hand, prescribes as "well founded belief" that the absentee is already dead before a
petition for declaration of presumptive death can be granted.
As pointed out by the Solicitor-General, there are four (4) requisites for the declaration of presumptive
death under Article 41 of the Family Code:
1. That the absent spouse has been missing for four consecutive years, or two
consecutive years if the disappearance occurred where there is danger of death under
the circumstances laid down in Article 391, Civil Code;
2. That the present spouse wishes to remarry;
3. That the present spouse has a well-founded belief that the absentee is dead; and
4. That the present spouse files a summary proceeding for the declaration of presumptive
death of the absentee.
10

Respondent naturally asserts that he had complied with all these requirements.
11

Petitioner's argument, upon the other hand, boils down to this: that respondent failed to prove that he had
complied with the third requirement, i.e., the existence of a "well-founded belief" that the absent spouse is
already dead.
The Court believes that respondent Nolasco failed to conduct a search for his missing wife with such
diligence as to give rise to a "well-founded belief" that she is dead.
United States v. Biasbas,
12
is instructive as to degree of diligence required in searching for a missing
spouse. In that case, defendant Macario Biasbas was charged with the crime of bigamy. He set-up the
defense of a good faith belief that his first wife had already died. The Court held that defendant had not
exercised due diligence to ascertain the whereabouts of his first wife, noting that:
While the defendant testified that he had made inquiries concerning the whereabouts of
his wife, he fails to state of whom he made such inquiries. He did not even write to the
parents of his first wife, who lived in the Province of Pampanga, for the purpose of
securing information concerning her whereabouts. He admits that he had a suspicion
only that his first wife was dead. He admits that the only basis of his suspicion was the
fact that she had been absent. . . .
13

In the case at bar, the Court considers that the investigation allegedly conducted by respondent in his
attempt to ascertain Janet Monica Parker's whereabouts is too sketchy to form the basis of a reasonable
or well-founded belief that she was already dead. When he arrived in San Jose, Antique after learning of
Janet Monica's departure, instead of seeking the help of local authorities or of the British Embassy,
14
he
secured another seaman's contract and went to London, a vast city of many millions of inhabitants, to
look for her there.
Q After arriving here in San Jose, Antique, did you exert efforts to inquire
the whereabouts of your wife?
A Yes, Sir.
Court:
How did you do that?
A I secured another contract with the ship and we had a trip to London
and I went to London to look for her I could not find
her (sic).
15
(Emphasis supplied)
Respondent's testimony, however, showed that he confused London for Liverpool and this casts doubt on
his supposed efforts to locate his wife in England. The Court of Appeal's justification of the mistake, to wit:
. . . Well, while the cognoscente (sic) would readily know the geographical difference
between London and Liverpool, for a humble seaman like Gregorio the two places could
mean one place in England, the port where his ship docked and where he found
Janet. Our own provincial folks, every time they leave home to visit relatives in Pasay
City, Kalookan City, or Paraaque, would announce to friends and relatives, "We're going
to Manila." This apparent error in naming of places of destination does not appear to be
fatal.
16

is not well taken. There is no analogy between Manila and its neighboring cities, on one hand, and
London and Liverpool, on the other, which, as pointed out by the Solicitor-General, are around three
hundred fifty (350) kilometers apart. We do not consider that walking into a major city like Liverpool or
London with a simple hope of somehow bumping into one particular person there which is in effect
what Nolasco says he did can be regarded as a reasonably diligent search.
The Court also views respondent's claim that Janet Monica declined to give any information as to her
personal background even after she had married respondent
17
too convenient an excuse to justify his
failure to locate her. The same can be said of the loss of the alleged letters respondent had sent to his
wife which respondent claims were all returned to him. Respondent said he had lost these returned
letters, under unspecified circumstances.
Neither can this Court give much credence to respondent's bare assertion that he had inquired from their
friends of her whereabouts, considering that respondent did not identify those friends in his testimony.
The Court of Appeals ruled that since the prosecutor failed to rebut this evidence during trial, it is good
evidence. But this kind of evidence cannot, by its nature, be rebutted. In any case, admissibility is not
synonymous with credibility.
18
As noted before, there are serious doubts to respondent's credibility.
Moreover, even if admitted as evidence, said testimony merely tended to show that the missing spouse
had chosen not to communicate with their common acquaintances, and not that she was dead.
Respondent testified that immediately after receiving his mother's letter sometime in January 1983, he cut
short his employment contract to return to San Jose, Antique. However, he did not explain the delay of
nine (9) months from January 1983, when he allegedly asked leave from his captain, to November 1983
when be finally reached San Jose. Respondent, moreover, claimed he married Janet Monica Parker
without inquiring about her parents and their place of residence.
19
Also, respondent failed to explain why
he did not even try to get the help of the police or other authorities in London and Liverpool in his effort to
find his wife. The circumstances of Janet Monica's departure and respondent's subsequent behavior
make it very difficult to regard the claimed belief that Janet Monica was dead a well-founded one.
In Goitia v. Campos-Rueda,
20
the Court stressed that:
. . . Marriage is an institution, the maintenance of which in its purity the public is deeply
interested. It is a relationship for life and the parties cannot terminate it at any shorter
period by virtue of any contract they make. . . . .
21
(Emphasis supplied)
By the same token, the spouses should not be allowed, by the simple expedient of agreeing that one of
them leave the conjugal abode and never to return again, to circumvent the policy of the laws on
marriage. The Court notes that respondent even tried to have his marriage annulled before the trial court
in the same proceeding.
In In Re Szatraw,
22
the Court warned against such collusion between the parties when they find it
impossible to dissolve the marital bonds through existing legal means.
While the Court understands the need of respondent's young son, Gerry Nolasco, for maternal care, still
the requirements of the law must prevail. Since respondent failed to satisfy the clear requirements of the
law, his petition for a judicial declaration of presumptive death must be denied. The law does not view
marriage like an ordinary contract. Article 1 of the Family Code emphasizes that.
. . . Marriage is a special contract of permanent union between a man and a
woman entered into in accordance with law for the establishment of conjugal and family
life. It is the foundation of the familyand an inviolable social institution whose nature,
consequences, and incidents are governed by law and not subject to stipulation, except
that marriage settlements may fix the property relations during the marriage within the
limits provided by this Code. (Emphasis supplied)
In Arroyo, Jr. v. Court of Appeals,
23
the Court stressed strongly the need to protect.
. . . the basic social institutions of marriage and the family in the preservation of which the
State bas the strongest interest; the public policy here involved is of the most
fundamental kind. In Article II, Section 12 of the Constitution there is set forth the
following basic state policy:
The State recognizes the sanctity of family life and shall protect and
strengthen the family as a basic autonomous social institution. . . .
The same sentiment bas been expressed in the Family Code of the Philippines in Article
149:
The family, being the foundation of the nation, is a basic social institution
which public policy cherishes and protects. Consequently, family
relations are governed by law and no custom, practice or agreement
destructive of the family shall be recognized or given effect.
24

In fine, respondent failed to establish that he had the well-founded belief required by law that his absent
wife was already dead that would sustain the issuance of a court order declaring Janet Monica Parker
presumptively dead.
WHEREFORE, the Decision of the Court of Appeals dated 23 February 1990, affirming the trial court's
decision declaring Janet Monica Parker presumptively dead is hereby REVERSED and both Decisions
are hereby NULLIFIED and SET ASIDE. Costs against respondent.
Bidin, Davide, Jr., Romero and Melo, JJ., concur.
Gutierrez, Jr. J., is on leave.






















Republic of the Philippines
SUPREME COURT
Manila
EN BANC
G.R. No. L-15853 July 27, 1960
FERNANDO AQUINO, petitioner,
vs.
CONCHITA DELIZO, respondent.
GUTIERREZ DAVID, J .:
This is a petition for certiorari to review a decision of the Court of Appeals affirming that of the Court of
First Instance of Rizal which dismissed petitioner's complaint for annulment of his marriage with
respondent Conchita Delizo.
The dismissed complaint, which was filed on September 6, 1955, was based on the ground of fraud, it
being alleged, among other things, that defendant Conchita Delizo, herein respondent, at the date of her
marriage to plaintiff, herein petitioner Fernando Aquino, on December 27, 1954, concealed from the latter
that fact that she was pregnant by another man, and sometime in April, 1955, or about four months after
their marriage, gave birth to a child. In her answer, defendant claimed that the child was conceived out of
lawful wedlock between her and the plaintiff.
At the trial, the attorney's for both parties appeared and the court a quo ordered Assistant Provincial
Fiscal Jose Goco to represent the State in the proceedings to prevent collusion. Only the plaintiff
however, testified and the only documentary evidence presented was the marriage contract between the
parties. Defendant neither appeared nor presented any evidence despite the reservation made by her
counsel that he would present evidence on a later date.
On June 16, 1956, the trial court noting that no birth certificate was presented to show that the child
was born within 180 days after the marriage between the parties, and holding that concealment of
pregnancy as alleged by the plaintiff does not constitute such fraud sa would annul a marriage
dismissed the complaint. Through a verified "petition to reopen for reception of additional evidence",
plaintiff tried to present the certificates of birth and delivery of the child born of the defendant on April 26,
1955, which documents, according to him, he had failed to secure earlier and produce before the trial
court thru excusable negligence. The petition, however, was denied.
On appeal to the Court of Appeals, that court held that there has been excusable neglect in plaintiff's
inability to present the proof of the child's birth, through her birth certificate, and for that reason the court a
quo erred in denying the motion for reception of additional evidence. On the theory, however, that it was
not impossible for plaintiff and defendant to have had sexual intercourse during their engagement so that
the child could be their own, and finding unbelievable plaintiff's claim that he did not notice or even
suspect that defendant was pregnant when he married her, the appellate court, nevertheless, affirmed the
dismissal of the complaint.
On March 17, 1959, plaintiff filed a motion praying that the decision be reconsidered, or, if such
reconsideration be denied, that the case be remanded to the lower court for new trial. In support of the
motion, plaintiff attached as annexes thereof the following documents:
1. Affidavit of Cesar Aquino (Annex A) (defendant's brother-in-law and plaintiff's brother, with
whom defendant was living at the time plaintiff met, courted and married her, and with whom
defendant has begotten two more children, aside from her first born, in common-law relationship)
admitting that he is the father of defendant's first born, Catherine Bess Aquino, and that he and
defendant hid her pregnancy from plaintiff at the time of plaintiff's marriage to defendant;
2. Affidavit of defendant, Conchita Delizo (Annex "B") admitting her pregnancy by Cesar Aquino,
her brother-in-law and plaintiff's own brother, at the time of her marriage to plaintiff and her
having hidden this fact from plaintiff before and up to the time of their marriage;
3. Affidavit of Albert Powell (Annex "C") stating that he knew Cesar Aquino and defendant lived
together as husband and wife before December 27, 1954, the date of plaintiff's marriage to
defendant;
4. Birth Certificate of defendant's first born, Catherine Bess Aquino showing her date of birth to be
April 26, 1955;
5. Birth Certificate (Annex "D") of Carolle Ann Aquino, the second child of defendant with Cesar
Aquino, her brother-in-law;
6. Birth Certificate (Annex "E") of Chris Charibel Aquino, the third child of Cesar Aquino and
defendant; and
7. Pictures of defendant showing her natural plumpness as early as 1952 to as late as November,
1954, the November, 1954 photo itself does not show defendant's pregnancy which must have
been almost four months old at the time the picture was taken.
Acting upon the motion, the Court of Appeals ordered the defendant Conchita Delizo and Assistant
Provincial Fiscal of Rizal, who was representing the Government, to answer the motion for
reconsideration, and deferred action on the prayer for new trial until after the case is disposed of. As both
the defendant and the fiscal failed to file an answer, and stating that it "does not believe the veracity of the
contents of the motion and its annexes", the Court of Appeals, on August 6, 1959, denied the motion.
From that order, the plaintiff brought the case to this Court thru the present petition for certiorari.
After going over the record of the case, we find that the dismissal of plaintiff's complaint cannot be
sustained.
Under the new Civil Code, concealment by the wife of the fact that at the time of the marriage, she was
pregnant by a man other than her husband constitutes fraud and is ground for annulment of marriage.
(Art. 85, par. (4) in relation to Art. 86, par. (3). In the case of Buccat vs. Buccat (72 Phil., 19) cited in the
decision sought to be reviewed, which was also an action for the annulment of marriage on the ground of
fraud, plaintiff's claim that he did not even suspect the pregnancy of the defendant was held to be
unbelievable, it having been proven that the latter was already in an advanced stage of pregnancy (7th
month) at the time of their marriage. That pronouncement, however, cannot apply to the case at bar. Here
the defendant wife was alleged to be only more than four months pregnant at the time of her marriage to
plaintiff. At that stage, we are not prepared to say that her pregnancy was readily apparent, especially
since she was "naturally plump" or fat as alleged by plaintiff. According to medical authorities, even on the
5th month of pregnancy, the enlargement of a woman's abdomen is still below the umbilicus, that is to
say, the enlargement is limited to the lower part of the abdomen so that it is hardly noticeable and may, if
noticed, be attributed only to fat formation on the lower part of the abdomen. It is only on the 6th month of
pregnancy that the enlargement of the woman's abdomen reaches a height above the umbilicus, making
the roundness of the abdomen more general and apparent. (See Lull, Clinical Obstetrics, p. 122) If, as
claimed by plaintiff, defendant is "naturally plump", he could hardly be expected to know, merely by
looking, whether or not she was pregnant at the time of their marriage more so because she must have
attempted to conceal the true state of affairs. Even physicians and surgeons, with the aid of the woman
herself who shows and gives her subjective and objective symptoms, can only claim positive diagnosis of
pregnancy in 33% at five months. and 50% at six months. (XI Cyclopedia of Medicine, Surgery, etc.
Pregnancy, p. 10).
The appellate court also said that it was not impossible for plaintiff and defendant to have had sexual
intercourse before they got married and therefore the child could be their own. This statement, however,
is purely conjectural and finds no support or justification in the record.
Upon the other hand, the evidence sought to be introduced at the new trial, taken together with what has
already been adduced would, in our opinion, be sufficient to sustain the fraud alleged by plaintiff. The
Court of Appeals should, therefore, not have denied the motion praying for new trial simply because
defendant failed to file her answer thereto. Such failure of the defendant cannot be taken as evidence of
collusion, especially since a provincial fiscal has been ordered of represent the Government precisely to
prevent such collusion. As to the veracity of the contents of the motion and its annexes, the same can
best be determined only after hearing evidence. In the circumstance, we think that justice would be better
served if a new trial were ordered.
Wherefore, the decision complained of is set aside and the case remanded to the court a quo for new
trial. Without costs.
Paras, C.J., Bengzon, Montemayor, Labrador, Concepcion, and Reyes, J.B.L., JJ., concur.
Barrera, J., concurs in the result.
















Republic of the Philippines
SUPREME COURT
Manila
EN BANC
G.R. No. L-12790 August 31, 1960
JOEL JIMENEZ, plaintiff-appellee,
vs.
REMEDIOS CAIZARES, defendant.
Republic of the Philippines, intervenor-appellant.
Acting Solicitor General Guillermo E. Torres and Solicitor Pacifico P. de Castro for appellant.
Climaco, Ascarraga and Silang for appellee.
PADILLA, J .:
In a complaint filed on 7 June 1955 in the Court of First Instance of Zamboanga the plaintiff Joel Jimenez
prays for a decree annulling his marriage to the defendant Remedios Caizares contracted on 3 August
1950 before a judge of the municipal court of Zamboanga City, upon the ground that the office of her
genitals or vagina was to small to allow the penetration of a male organ or penis for copulation; that the
condition of her genitals as described above existed at the time of marriage and continues to exist; and
that for that reason he left the conjugal home two nights and one day after they had been married. On 14
June 1955 the wife was summoned and served a copy of the complaint. She did not file an answer. On
29 September 1956, pursuant to the provisions of article 88 of the Civil Code, the Court directed the city
attorney of Zamboanga to inquire whether there was a collusion, to intervene for the State to see that the
evidence for the plaintiff is not a frame-up, concocted or fabricated. On 17 December 1956 the Court
entered an order requiring the defendant to submit to a physical examination by a competent lady
physician to determine her physical capacity for copulation and to submit, within ten days from receipt of
the order, a medical certificate on the result thereof. On 14 March 1957 the defendant was granted
additional five days from notice to comply with the order of 17 December 1956 with warning that her
failure to undergo medical examination and submit the required doctor's certificate would be deemed lack
of interest on her part in the case and that judgment upon the evidence presented by her husband would
be rendered.
After hearing, at which the defendant was not present, on 11 April 1957 the Court entered a decree
annulling the marriage between the plaintiff and the defendant. On 26 April 1957 the city attorney filed a
motion for reconsideration of the decree thus entered, upon the ground, among others, that the
defendant's impotency has not been satisfactorily established as required by law; that she had not been
physically examined because she had refused to be examined; that instead of annulling the marriage the
Court should have punished her for contempt of court and compelled her to undergo a physical
examination and submit a medical certificate; and that the decree sought to be reconsidered would open
the door to married couples, who want to end their marriage to collude or connive with each other by just
alleging impotency of one of them. He prayed that the complaint be dismissed or that the wife be
subjected to a physical examination. Pending resolution of his motion, the city attorney timely appealed
from the decree. On 13 May 1957 the motion for reconsideration was denied.
The question to determine is whether the marriage in question may be annulled on the strength only of
the lone testimony of the husband who claimed and testified that his wife was and is impotent. The latter
did not answer the complaint, was absent during the hearing, and refused to submit to a medical
examination.
Marriage in this country is an institution in which the community is deeply interested. The state has
surrounded it with safeguards to maintain its purity, continuity and permanence. The security and stability
of the state are largely dependent upon it. It is the interest of each and every member of the community to
prevent the bringing about of a condition that would shake its foundation and ultimately lead to its
destruction. The incidents of the status are governed by law, not by will of the parties. The law specifically
enumerates the legal grounds, that must be proved to exist by indubitable evidence, to annul a marriage.
In the case at bar, the annulment of the marriage in question was decreed upon the sole testimony of the
husband who was expected to give testimony tending or aiming at securing the annulment of his marriage
he sought and seeks. Whether the wife is really impotent cannot be deemed to have been satisfactorily
established, becase from the commencement of the proceedings until the entry of the decree she had
abstained from taking part therein. Although her refusal to be examined or failure to appear in court show
indifference on her part, yet from such attitude the presumption arising out of the suppression of evidence
could not arise or be inferred because women of this country are by nature coy, bashful and shy and
would not submit to a physical examination unless compelled to by competent authority. This the Court
may do without doing violence to and infringing in this case is not self-incrimination. She is not charged
with any offense. She is not being compelled to be a witness against herself.
1
"Impotency being an
abnormal condition should not be presumed. The presumption is in favor of potency."
2
The lone testimony
of the husband that his wife is physically incapable of sexual intercourse is insufficient to tear asunder the
ties that have bound them together as husband and wife.
The decree appealed from is set aside and the case remanded to the lower court for further proceedings
in accordance with this decision, without pronouncement as to costs.
Paras, C.J., Bengzon, Bautista Angelo, Labrador, Concepcion, Reyes, J.B.L., Barrera, Gutierrez David,
and Dizon, JJ. concur.


Footnotes
1
Section 1, paragraph 18, Article III of the Constitution.
2
Marciano vs. San Jose, 89 Phil., 62.









Republic of the Philippines
SUPREME COURT
Manila
SECOND DIVISION
G.R. No. 169900 March 18, 2010
MARIO SIOCHI, Petitioner,
vs.
ALFREDO GOZON, WINIFRED GOZON, GIL TABIJE, INTER-DIMENSIONAL REALTY, INC., and
ELVIRA GOZON, Respondents.
x - - - - - - - - - - - - - - - - - - - - - - -x
G.R. No. 169977
INTER-DIMENSIONAL REALTY, INC., Petitioner,
vs.
MARIO SIOCHI, ELVIRA GOZON, ALFREDO GOZON, and WINIFRED GOZON, Respondents.
R E S O L U T I O N
CARPIO, J .:
This is a consolidation of two separate petitions for review,
1
assailing the 7 July 2005 Decision
2
and the
30 September 2005 Resolution
3
of the Court of Appeals in CA-G.R. CV No. 74447.
This case involves a 30,000 sq.m. parcel of land (property) covered by TCT No. 5357.
4
The property is
situated in Malabon, Metro Manila and is registered in the name of "Alfredo Gozon (Alfredo), married to
Elvira Gozon (Elvira)."
On 23 December 1991, Elvira filed with the Cavite City Regional Trial Court (Cavite RTC) a petition for
legal separation against her husband Alfredo. On 2 January 1992, Elvira filed a notice of lis pendens,
which was then annotated on TCT No. 5357.
On 31 August 1993, while the legal separation case was still pending, Alfredo and Mario Siochi (Mario)
entered into an Agreement to Buy and Sell
5
(Agreement) involving the property for the price of P18
million. Among the stipulations in the Agreement were that Alfredo would: (1) secure an Affidavit from
Elvira that the property is Alfredos exclusive property and to annotate the Agreement at the back of TCT
No. 5357; (2) secure the approval of the Cavite RTC to exclude the property from the legal separation
case; and (3) secure the removal of the notice of lis pendens pertaining to the said case and annotated
on TCT No. 5357. However, despite repeated demands from Mario, Alfredo failed to comply with these
stipulations. After paying the P5 million earnest money as partial payment of the purchase price, Mario
took possession of the property in September 1993. On 6 September 1993, the Agreement was
annotated on TCT No. 5357.
Meanwhile, on 29 June 1994, the Cavite RTC rendered a decision
6
in the legal separation case, the
dispositive portion of which reads:
WHEREFORE, judgment is hereby rendered decreeing the legal separation between petitioner and
respondent. Accordingly, petitioner Elvira Robles Gozon is entitled to live separately from respondent
Alfredo Gozon without dissolution of their marriage bond. The conjugal partnership of gains of the
spouses is hereby declared DISSOLVED and LIQUIDATED. Being the offending spouse, respondent is
deprived of his share in the net profits and the same is awarded to their child Winifred R. Gozon whose
custody is awarded to petitioner.
Furthermore, said parties are required to mutually support their child Winifred R. Gozon as her needs
arises.
SO ORDERED.
7

As regards the property, the Cavite RTC held that it is deemed conjugal property.
On 22 August 1994, Alfredo executed a Deed of Donation over the property in favor of their daughter,
Winifred Gozon (Winifred). The Register of Deeds of Malabon, Gil Tabije, cancelled TCT No. 5357 and
issued TCT No. M-10508
8
in the name of Winifred, without annotating the Agreement and the notice of lis
pendens on TCT No. M-10508.
On 26 October 1994, Alfredo, by virtue of a Special Power of Attorney
9
executed in his favor by Winifred,
sold the property to Inter-Dimensional Realty, Inc. (IDRI) for P18 million.
10
IDRI paid Alfredo P18 million,
representing full payment for the property.
11
Subsequently, the Register of Deeds of Malabon cancelled
TCT No. M-10508 and issued TCT No. M-10976
12
to IDRI.
Mario then filed with the Malabon Regional Trial Court (Malabon RTC) a complaint for Specific
Performance and Damages, Annulment of Donation and Sale, with Preliminary Mandatory and
Prohibitory Injunction and/or Temporary Restraining Order.
On 3 April 2001, the Malabon RTC rendered a decision,
13
the dispositive portion of which reads:
WHEREFORE, premises considered, judgment is hereby rendered as follows:
01. On the preliminary mandatory and prohibitory injunction:
1.1 The same is hereby made permanent by:
1.1.1 Enjoining defendants Alfredo Gozon, Winifred Gozon, Inter-Dimensional
Realty, Inc. and Gil Tabije, their agents, representatives and all persons acting in
their behalf from any attempt of commission or continuance of their wrongful acts
of further alienating or disposing of the subject property;
1.1.2. Enjoining defendant Inter-Dimensional Realty, Inc. from entering and
fencing the property;
1.1.3. Enjoining defendants Alfredo Gozon, Winifred Gozon, Inter-Dimensional
Realty, Inc. to respect plaintiffs possession of the property.
02. The Agreement to Buy and Sell dated 31 August 1993, between plaintiff and defendant
Alfredo Gozon is hereby approved, excluding the property and rights of defendant Elvira Robles-
Gozon to the undivided one-half share in the conjugal property subject of this case.
03. The Deed of Donation dated 22 August 1994, entered into by and between defendants
Alfredo Gozon and Winifred Gozon is hereby nullified and voided.
04. The Deed of Absolute Sale dated 26 October 1994, executed by defendant Winifred Gozon,
through defendant Alfredo Gozon, in favor of defendant Inter-Dimensional Realty, Inc. is hereby
nullified and voided.
05. Defendant Inter-Dimensional Realty, Inc. is hereby ordered to deliver its Transfer Certificate
of Title No. M-10976 to the Register of Deeds of Malabon, Metro Manila.
06. The Register of Deeds of Malabon, Metro Manila is hereby ordered to cancel Certificate of
Title Nos. 10508 "in the name of Winifred Gozon" and M-10976 "in the name of Inter-Dimensional
Realty, Inc.," and to restore Transfer Certificate of Title No. 5357 "in the name of Alfredo Gozon,
married to Elvira Robles" with the Agreement to Buy and Sell dated 31 August 1993 fully
annotated therein is hereby ordered.
07. Defendant Alfredo Gozon is hereby ordered to deliver a Deed of Absolute Sale in favor of
plaintiff over his one-half undivided share in the subject property and to comply with all the
requirements for registering such deed.
08. Ordering defendant Elvira Robles-Gozon to sit with plaintiff to agree on the selling price of her
undivided one-half share in the subject property, thereafter, to execute and deliver a Deed of
Absolute Sale over the same in favor of the plaintiff and to comply with all the requirements for
registering such deed, within fifteen (15) days from the receipt of this DECISION.
09. Thereafter, plaintiff is hereby ordered to pay defendant Alfredo Gozon the balance of Four
Million Pesos (P4,000,000.00) in his one-half undivided share in the property to be set off by the
award of damages in plaintiffs favor.
10. Plaintiff is hereby ordered to pay the defendant Elvira Robles-Gozon the price they had
agreed upon for the sale of her one-half undivided share in the subject property.
11. Defendants Alfredo Gozon, Winifred Gozon and Gil Tabije are hereby ordered to pay the
plaintiff, jointly and severally, the following:
11.1 Two Million Pesos (P2,000,000.00) as actual and compensatory damages;
11.2 One Million Pesos (P1,000,000.00) as moral damages;
11.3 Five Hundred Thousand Pesos (P500,000.00) as exemplary damages;
11.4 Four Hundred Thousand Pesos (P400,000.00) as attorneys fees; and
11.5 One Hundred Thousand Pesos (P100,000.00) as litigation expenses.
11.6 The above awards are subject to set off of plaintiffs obligation in paragraph 9
hereof.
12. Defendants Alfredo Gozon and Winifred Gozon are hereby ordered to pay Inter-Dimensional
Realty, Inc. jointly and severally the following:
12.1 Eighteen Million Pesos (P18,000,000.00) which constitute the amount the former
received from the latter pursuant to their Deed of Absolute Sale dated 26 October 1994,
with legal interest therefrom;
12.2 One Million Pesos (P1,000,000.00) as moral damages;
12.3 Five Hundred Thousand Pesos (P500,000.00) as exemplary damages; and
12.4 One Hundred Thousand Pesos (P100,000.00) as attorneys fees.
13. Defendants Alfredo Gozon and Winifred Gozon are hereby ordered to pay costs of suit.
SO ORDERED.
14

On appeal, the Court of Appeals affirmed the Malabon RTCs decision with modification. The dispositive
portion of the Court of Appeals Decision dated 7 July 2005 reads:
WHEREFORE, premises considered, the assailed decision dated April 3, 2001 of the RTC, Branch 74,
Malabon is hereby AFFIRMED with MODIFICATIONS, as follows:
1. The sale of the subject land by defendant Alfredo Gozon to plaintiff-appellant Siochi is declared
null and void for the following reasons:
a) The conveyance was done without the consent of defendant-appellee Elvira Gozon;
b) Defendant Alfredo Gozons one-half () undivided share has been forfeited in favor of
his daughter, defendant Winifred Gozon, by virtue of the decision in the legal separation
case rendered by the RTC, Branch 16, Cavite;
2. Defendant Alfredo Gozon shall return/deliver to plaintiff-appellant Siochi the amount of P5
Million which the latter paid as earnest money in consideration for the sale of the subject land;
3. Defendants Alfredo Gozon, Winifred Gozon and Gil Tabije are hereby ordered to pay plaintiff-
appellant Siochi jointly and severally, the following:
a) P100,000.00 as moral damages;
b) P100,000.00 as exemplary damages;
c) P50,000.00 as attorneys fees;
d) P20,000.00 as litigation expenses; and
e) The awards of actual and compensatory damages are hereby ordered deleted for lack
of basis.
4. Defendants Alfredo Gozon and Winifred Gozon are hereby ordered to pay defendant-appellant
IDRI jointly and severally the following:
a) P100,000.00 as moral damages;
b) P100,000.00 as exemplary damages; and
c) P50,000.00 as attorneys fees.
Defendant Winifred Gozon, whom the undivided one-half share of defendant Alfredo Gozon was
awarded, is hereby given the option whether or not to dispose of her undivided share in the subject land.
The rest of the decision not inconsistent with this ruling stands.
SO ORDERED.
15

Only Mario and IDRI appealed the decision of the Court of Appeals. In his petition, Mario alleges that the
Agreement should be treated as a continuing offer which may be perfected by the acceptance of the other
spouse before the offer is withdrawn. Since Elviras conduct signified her acquiescence to the sale, Mario
prays for the Court to direct Alfredo and Elvira to execute a Deed of Absolute Sale over the property upon
his payment of P9 million to Elvira.
On the other hand, IDRI alleges that it is a buyer in good faith and for value. Thus, IDRI prays that the
Court should uphold the validity of IDRIs TCT No. M-10976 over the property.
We find the petitions without merit.
This case involves the conjugal property of Alfredo and Elvira. Since the disposition of the property
occurred after the effectivity of the Family Code, the applicable law is the Family Code. Article 124 of the
Family Code provides:
Art. 124. The administration and enjoyment of the conjugal partnership property shall belong to both
spouses jointly. In case of disagreement, the husbands decision shall prevail, subject to the recourse to
the court by the wife for a proper remedy, which must be availed of within five years from the date of the
contract implementing such decision.
In the event that one spouse is incapacitated or otherwise unable to participate in the administration of
the conjugal properties, the other spouse may assume sole powers of administration. These powers do
not include the powers of disposition or encumbrance which must have the authority of the court or the
written consent of the other spouse. In the absence of such authority or consent, the disposition or
encumbrance shall be void. However, the transaction shall be construed as a continuing offer on the part
of the consenting spouse and the third person, and may be perfected as a binding contract upon the
acceptance by the other spouse or authorization by the court before the offer is withdrawn by either or
both offerors. (Emphasis supplied)
In this case, Alfredo was the sole administrator of the property because Elvira, with whom Alfredo was
separated in fact, was unable to participate in the administration of the conjugal property. However, as
sole administrator of the property, Alfredo still cannot sell the property without the written consent of Elvira
or the authority of the court. Without such consent or authority, the sale is void.
16
The absence of the
consent of one of the spouse renders the entire sale void, including the portion of the conjugal property
pertaining to the spouse who contracted the sale.
17
Even if the other spouse actively participated in
negotiating for the sale of the property, that other spouses written consent to the sale is still required by
law for its validity.
18
The Agreement entered into by Alfredo and Mario was without the written consent of
Elvira. Thus, the Agreement is entirely void. As regards Marios contention that the Agreement is a
continuing offer which may be perfected by Elviras acceptance before the offer is withdrawn, the fact that
the property was subsequently donated by Alfredo to Winifred and then sold to IDRI clearly indicates that
the offer was already withdrawn.
However, we disagree with the finding of the Court of Appeals that the one-half undivided share of Alfredo
in the property was already forfeited in favor of his daughter Winifred, based on the ruling of the Cavite
RTC in the legal separation case. The Court of Appeals misconstrued the ruling of the Cavite RTC that
Alfredo, being the offending spouse, is deprived of his share in the net profits and the same is awarded to
Winifred.
The Cavite RTC ruling finds support in the following provisions of the Family Code:
Art. 63. The decree of legal separation shall have the following effects:
(1) The spouses shall be entitled to live separately from each other, but the marriage bonds shall
not be severed;
(2) The absolute community or the conjugal partnership shall be dissolved and liquidated
but the offending spouse shall have no right to any share of the net profits earned by the
absolute community or the conjugal partnership, which shall be forfeited in accordance
with the provisions of Article 43(2);
(3) The custody of the minor children shall be awarded to the innocent spouse, subject to the
provisions of Article 213 of this Code; and
The offending spouse shall be disqualified from inheriting from the innocent spouse by intestate
succession. Moreover, provisions in favor of the offending spouse made in the will of the innocent spouse
shall be revoked by operation of law.
Art. 43. The termination of the subsequent marriage referred to in the preceding Article shall produce the
following effects:
x x x
(2) The absolute community of property or the conjugal partnership, as the case may be, shall be
dissolved and liquidated, but if either spouse contracted said marriage in bad faith, his or her share of the
net profits of the community property or conjugal partnership property shall be forfeited in favor of the
common children or, if there are none, the children of the guilty spouse by a previous marriage or, in
default of children, the innocent spouse; (Emphasis supplied)
Thus, among the effects of the decree of legal separation is that the conjugal partnership is dissolved and
liquidated and the offending spouse would have no right to any share of the net profits earned by the
conjugal partnership. It is only Alfredos share in the net profits which is forfeited in favor of Winifred.
Article 102(4) of the Family Code provides that "[f]or purposes of computing the net profits subject to
forfeiture in accordance with Article 43, No. (2) and 63, No. (2), the said profits shall be the increase in
value between the market value of the community property at the time of the celebration of the marriage
and the market value at the time of its dissolution." Clearly, what is forfeited in favor of Winifred is not
Alfredos share in the conjugal partnership property but merely in the net profits of the conjugal
partnership property.
With regard to IDRI, we agree with the Court of Appeals in holding that IDRI is not a buyer in good faith.
As found by the RTC Malabon and the Court of Appeals, IDRI had actual knowledge of facts and
circumstances which should impel a reasonably cautious person to make further inquiries about the
vendors title to the property. The representative of IDRI testified that he knew about the existence of the
notice of lis pendens on TCT No. 5357 and the legal separation case filed before the Cavite RTC. Thus,
IDRI could not feign ignorance of the Cavite RTC decision declaring the property as conjugal.
Furthermore, if IDRI made further inquiries, it would have known that the cancellation of the notice of lis
pendens was highly irregular. Under Section 77 of Presidential Decree No. 1529,
19
the notice of lis
pendens may be cancelled (a) upon order of the court, or (b) by the Register of Deeds upon verified
petition of the party who caused the registration of the lis pendens. In this case, the lis pendens was
cancelled by the Register of Deeds upon the request of Alfredo. There was no court order for the
cancellation of the lis pendens. Neither did Elvira, the party who caused the registration of the lis
pendens, file a verified petition for its cancellation.
Besides, had IDRI been more prudent before buying the property, it would have discovered that Alfredos
donation of the property to Winifred was without the consent of Elvira. Under Article 125
20
of the Family
Code, a conjugal property cannot be donated by one spouse without the consent of the other spouse.
Clearly, IDRI was not a buyer in good faith.1avvphi1
Nevertheless, we find it proper to reinstate the order of the Malabon RTC for the reimbursement of
the P18 million paid by IDRI for the property, which was inadvertently omitted in the dispositive portion of
the Court of Appeals decision.
WHEREFORE, we DENY the petitions. We AFFIRM the 7 July 2005 Decision of the Court of Appeals in
CA-G.R. CV No. 74447 with the following MODIFICATIONS:
(1) We DELETE the portions regarding the forfeiture of Alfredo Gozons one-half undivided share
in favor of Winifred Gozon and the grant of option to Winifred Gozon whether or not to dispose of
her undivided share in the property; and
(2) We ORDER Alfredo Gozon and Winifred Gozon to pay Inter-Dimensional Realty, Inc. jointly
and severally the Eighteen Million Pesos (P18,000,000) which was the amount paid by Inter-
Dimensional Realty, Inc. for the property, with legal interest computed from the finality of this
Decision.
SO ORDERED.
ANTONIO T. CARPIO
Associate Justice













Republic of the Philippines
SUPREME COURT
Manila
EN BANC
G.R. No. L-3047 May 16, 1951
THE PEOPLE OF THE PHILIPPINES, plaintiff-appellant,
vs.
GUADALUPE ZAPATA and DALMACIO BONDOC, defendants-appellees.
First Assistant Solicitor General Roberto A. Gianzon and Solicitor Jaime de los Angeles for
appellant.
Francisco M. Ramos and Moises Sevilla Ocampo for appellee Dalmacio Bondoc.
Hernandez and Laquian for appellee Guadalupe Zapata.
PADILLA, J .:
In the Court of First Instance of Pampanga a complaint for adultery was filed by Andres Bondoc
against Guadalupe Zapata, his wife, and Dalmacio Bondoc, her paramour, for cohabiting and having
repeated sexual intercourse during the period from the year 1946 14 March 1947, the date of the
filing of the complaint, Dalmacio Bondoc knowing his codefendant to be a married woman (criminal
case No. 426). The defendant wife entered the plea of guilty and was sentenced to suffer four
months of arresto mayor which penalty she served. In the same court, on 17 September 1948, the
offended husband filed another complaint for adulterous acts committed by his wife and her
paramour from 15 March 1947 to 17 September 1948, the date of the filing of the second complaint
(criminal case No. 735). On 21 February 1949, each of the defendants filed a motion to quash the
complaint of the ground that they would be twice put in jeopardy of punishment for the same offense.
The trial court upheld the contention of the defendants and quashed the second complaint. From the
other sustaining the motions to quash the prosecution has appealed.
The trial court held that the adulterous acts charged in the first and second complains must be
deemed one continuous offense, the defendants in both complaints being the same and identical
persons and the two sets of unlawful acts having taken place continuously during the years 1946,
1947 and part of 1948, and that the acts or two sets of acts that gave rise to the crimes of adultery
complained of in both cases constitute one and the same offense, within the scope and meaning of
the constitutional provision that "No person shall be twice put in jeopardy of punishment for the same
offense.".
Adultery is a crime of result and not of tendency, as the Supreme Court of Spain has held (S. 10
December 1945); it is a instantaneous crime which is consummated and exhausted or completed at
the moment of the carnal union. Each sexual intercourse constitutes a crime of adultery (Cuello
Calon, Derecho Penal, Vol. II, p. 569). True, two or more adulterous acts committed by the same
defendants are against the same person the offended husband, the same status the union of
the husband and wife by their marriage, and the same community represented by the State for its
interest in maintaining and preserving such status. But this identity of the offended party, status
society does not argue against the commission of the crime of adultery as many times as there were
carnal consummated, for as long as the status remain unchanged, the nexus undissolved and
unbroken, an encroachment or trespass upon that status constitutes a crime. There is no
constitutional or legal provision which bars the filing of as many complaints for adultery as there
were adulterous acts committed, each constituting one crime.
The notion or concept of a continuous crime has its origin in the juridical fiction favorable to the law
transgressors and in many a case against the interest of society (Cuello Calon, Derecho Penal, Vol.
II, p. 521). For it to exist there would be plurality of acts performed seperately during a period of time;
unity of penal provision infringed upon or violated; and unity of criminal intent or purpose, which
means that two or more violations of the same penal provision are united in one and the same intent
leading to the perpetration of the same criminal purpose or aim (Ibid. p. 520).In the instant case the
last unity does not exist, because as already stated the culprits perpetrate the crime in every sexual
intercourse and they need not to another or other adulterous acts to consummate it. After the last
acts of adultery had been committed as charged in the first complaint, the defendants again
committed adulterous acts not included in the first complaint and for which the second complaint was
filed. It was held by the Supreme Court of Spain that another crime of adultery was committed, if the
defendants, after their provincional release during the pendency of the case in which they were sent
to prison to serve the penalty imposed upon them(S. 28 February 1906; 76 Jur. Crim. pp. 208-210).
Another reason why a second complaint charging the commission of adulterous acts not included in
the first complaint does not constitute a violation of the double jeopardy clause of the constitution is
that, if the second places complaint the defendants twice in jeopardy of punishment for the same
offense, the adultery committed by the male defendant charged in the second complaint, should he
be absolved from, or acquitted of, the first charge upon the evidence that he did not know that his
codefendant was a married woman, would remain or go unpunished. The defense set up by him
against the first charge upon which he was acquitted would no longer be available, because at the
time of the commission of the crime charged in the second complaint, he already knew that this
defendant was a married woman and he continued to have carnal knowledge of her. Even if the
husband should pardon his adulterous wife, such pardon would not exempt the wife and her
paramour from criminal liability for adulterous acts committed after the pardon was granted because
the pardon refers to previous and not to subsequent adulterous acts(Viada [5th ed.] Vol. 5, p. 208;
Groizard [2nd ed.] Vol. 5, pp. 57-58).
The order appealed from, which quashed the second complaint for adultery, is hereby reversed and
set aside, and trial court directed to proceed with the trial of the defendants in accordance with law,
with costs against the appellees.
Feria, Pablo, Tuason and Jugo, JJ., concur.
Paras, C.J., Bengzon and Montemayor, JJ., concur in the result.
Paras, C.J., Mr. Justice Reyes voted for the reversal.









Republic of the Philippines
SUPREME COURT
Manila
EN BANC
G.R. No. L-11766 October 25, 1960
SOCORRO MATUBIS, plaintiff-appellant,
vs.
ZOILO PRAXEDES, defendant-appellee.
Luis N. de Leon for appellant.
Lucio La. Margallo for appellee.
PAREDES, J .:
Alleging abandonment and concubinage, plaintiff Socorro Matubis, filed with the Court of First
Instance of Camarines Sur, on April 24, 1956, a complaint for legal Separation and changed of
surname against her husband defendant Zoilo Praxedes.
The allegations of the complaint were denied by defendant spouse, who interposed the defense that
it was plaintiff who left the conjugal home.
During the trial, wherein the plaintiff alone introduced oral as well as documentary evidence, the
following facts were established:.
Plaintiff and defendant were legally married on January 10, 1943 at Iriga, Camarines Sur. For failure
to agree on how they should live as husband and wife, the couple, on May 30, 1944, agreed to live
separately from each other, which status remained unchanged until the present. On April 3, 1948,
plaintiff and defendant entered into an agreement (Exhibit B), the significant portions of which are
hereunder reproduced..
. . . (a) That both of us relinquish our right over the other as legal husband and wife.
(b) That both without any interference by any of us, nor either of us can prosecute the other
for adultery or concubinage or any other crime or suit arising from our separation.
(c) That I, the, wife, is no longer entitled for any support from my husband or any benefits he
may received thereafter, nor I the husband is not entitled for anything from my wife.
(d) That neither of us can claim anything from the other from the time we verbally separated,
that is from May 30, 1944 to the present when we made our verbal separation into writing.
In January, 1955, defendant began cohabiting with one Asuncion Rebulado and on September 1,
1955, said Asuncion gave birth to a child who was recorded as the child of said defendant (Exh.
C.).It was shown also that defendant and Asuncion deported themselves as husband and wife and
were generally reputed as such in the community.
After the trial, without the defendant adducing any evidence, the court a quo rendered judgment
holding that the acts of defendant constituted concubinage, a ground for legal separation. It
however, dismissed the complaint by stating:
While this legal ground exist, the suit must be dismissed for two reasons, viz:
Under Art. 102 of the new Civil Code, an action for legal separation cannot be filed except
within one year from and after the date on which the plaintiff became cognizant of the cause
and within five years from and after the date when the cause occurred. The plaintiff became
aware of the illegal cohabitation of her husband with Asuncion Rebulado in January, 1955.
The complaint was filed on April 24, 1956. The present action was, therefore, filed out of time
and for that reason action is barred.
Article 100 of the new Civil Code provides that the legal separation may be claimed only by
the innocent spouse, provided there has been no condonation of or consent to the adultery
or concubinage. As shown in Exhibit B, the plaintiff has consented to the commission of
concubinage by her husband. Her consent is clear from the following stipulations:
(b) That both of us is free to get any mate and live with as husband and wife without
any interference by any of us, nor either of us can prosecute the other for adultery or
concubinage or any other crime or suit arising from our separation. (Exh. B).
This stipulation is an unbridled license she gave her husband to commit concubinage.
Having consented to the concubinage, the plaintiff cannot claim legal separation.
The above decision is now before us for review, plaintiff- appellant claiming that it was error for the
lower court to have considered that the period to bring the action has already elapsed and that there
was consent on the part of the plaintiff to the concubinage. The proposition, therefore, calls for the
interpretation of the provisions of the law upon which the lower court based its judgment of
dismissal.
Article 102 of the new Civil Code provides:
An action for legal separation cannot be filed except within one year from and after the date
on which the plaintiff became cognizant of the cause and within five years from after the date
when cause occurred.
The complaint was filed outside the periods provided for by the above Article. By the very admission
of plaintiff, she came to know the ground (concubinage) for the legal separation in January, 1955.
She instituted the complaint only on April 24, 1956. It is to be noted that appellant did not even press
this matter in her brief.
The very wording of the agreement Exhibit B. gives no room for interpretation other than that given
by the trial judge. Counsel in his brief submits that the agreement is divided in two parts. The first
part having to do with the act of living separately which he claims to be legal, and the second part
that which becomes a license to commit the ground for legal separation which is admittedly illegal.
We do not share appellant's view. Condonation and consent on the part of plaintiff are necessarily
the import of paragraph 6(b) of the agreement. The condonation and consent here are not only
implied but expressed. The law (Art. 100 Civil Code), specifically provides that legal separation may
be claimed only by the innocent spouse, provided there has been no condonation of or consent to
the adultery or concubinage. Having condoned and/or consented in writing, the plaintiff is now
undeserving of the court's sympathy (People vs. Scheneckenburger, 73 Phil., 413). Plaintiff's
counsel even agrees that the complaint should be dismissed. He claims however, that the grounds
for the dismissal should not be those stated in the decision of the lower court, "but on the ground that
plaintiff and defendant have already been legally separated from each other, but without the marital
bond having been affected, long before the effectivity of the new Civil Code" (appellants brief, pp. 7-
8). Again, we cannot subscribed to counsel's contention, because it is contrary to the evidence.
Conformably with the foregoing, we find that the decision appealed from is in accordance with the
evidence and the law on the matter. The same is hereby affirmed, with costs.
Paras, C.J., Bengzon, Padilla, Bautista Angelo, Labrador, Reyes, J.B.L., Barrera, and Gutierrez
David, JJ., concur.





















Republic of the Philippines
SUPREME COURT
Manila
SECOND DIVISION
G.R. No. 79284 November 27, 1987
FROILAN C. GANDIONCO, petitioner,
vs.
HON. SENEN C. PEARANDA, as Presiding Judge of the Regional Trial Court of Misamis
Oriental, Branch 18, Cagayan de Oro City, and TERESITA S. GANDIONCO, respondents.

PADILLA, J .:
A special civil action for certiorari, with application for injunction, to annul (1) the Order of the
respondent Judge, dated 10 December 1986, ordering petitioner to pay support pendente lite to
private respondent (his wife) and their child, and (2) the Order of the same respondent Judge, dated
5 August 1987, denying petitioner's motion to suspend hearings in the action for legal separation
filed against him by private respondent as well as his motion to inhibit respondent Judge from further
hearing and trying the case.
On 29 May 1986, private respondent, the legal wife of the petitioner, filed with the Regional Trial
Court of Misamis Oriental, 10th Judicial District, Branch 18, in Cagayan de Oro City, presided over
by respondent Judge, a complaint against petitioner for legal separation, on the ground of
concubinage, with a petition for support and payment of damages. This case was docketed as Civil
Case No. 10636. On 13 October 1986, private respondent also filed with the Municipal Trial Court,
General Santos City, a complaint against petitioner for concubinage, which was docketed on 23
October 1986 as Criminal Case No. 15437111. On 14 November 1986, application for the
provisional remedy of support pendente lite, pending a decision in the action for legal separation,
was filed by private respondent in the civil case for legal separation. The respondent judge, as
already stated, on 10 December 1986, ordered The payment of support pendente lite.
In this recourse, petitioner contends that the civil action for legal separation and the incidents
consequent thereto, such as, application for support pendente lite, should be suspended in view of
the criminal case for concubinage filed against him the private respondent. In support of his
contention, petitioner cites Art. III. Sec. 3 of the 1985 Rules on Criminal Procedure, which states:
SEC. 3. Other Civil action arising from offenses. Whenever the offended party
shall have instituted the civil action to enforce the civil liability arising from the
offense. as contemplated in the first Section 1 hereof, the following rules shall be
observed:
(a) After a criminal action has been commenced the pending civil action arising from
the same offense shall be suspended, in whatever stage it may be found, until final
judgment in the criminal proceeding has been rendered. . . .
The civil action for legal separation, grounded as it is on concubinage, it is petitioner's position that
such civil action arises from, or is inextricably tied to the criminal action for concubinage, so that all
proceedings related to legal separation will have to be suspended to await conviction or acquittal for
concubinage in the criminal case. Authority for this position is this Court's decision in the case
of Jerusalem vs. Hon. Roberto Zurbano. 1
Petitioner's contention is not correct.
In Jerusalem, the Court's statement to the effect that suspension of an action for legal separation
would be proper if an allegation of concubinage is made therein, relied solely on Sec. 1 of Rule 107
of the then provisions of the Rules of Court on criminal procedure, to wit:
Sec. 1. Rules governing civil actions arising from offenses.-Except as otherwise
provided by law, the following rules shall he observed:
(a) When a criminal action is instituted, the civil action for recovery of civil liability
arising from the offense charged is impliedly instituted with the criminal action, unless
the offended party expressly waives the civil action or reserves his right to institute it
separately;
(b) Criminal and civil actions arising from the same offense may be instituted
separately, but after the criminal action has been commenced the civil action can not
be instituted until final judgment has been rendered in the criminal action;
(c) After a criminal action has been commenced, no civil action arising from the same
offense can be prosecuted and the same shall be suspended in whatever stage it
may be found until final judgment in the criminal proceeding has been rendered ...
(Emphasis supplied)
The provisions last quoted did not clearly state, as the 1985 Rules do, that the civil action to be
suspended, with or upon the filing of a criminal action, is one which is "to enforce the civil liability
arising from the offense". In other words, in view of the amendment under the 1985 Rules on
Criminal Procedure, a civil action for legal separation, based on concubinage, may proceed ahead
of, or simultaneously with, a criminal action for concubinage, because said civil action is not one "to
enforce the civil liability arising from the offense" even if both the civil and criminal actions arise from
or are related to the same offense. Such civil action is one intended to obtain the right to live
separately, with the legal consequences thereof, such as, the dissolution of the conjugal partnership
of gains, custody of offsprings, support, and disqualification from inheriting from the innocent
spouse, among others. As correctly pointed out by the respondent Judge in his Order dated 5
August 1987:
The unreported case of JERUSALEM vs. Hon. Roberto Zurbano, Judge of CFI of
Antique, et al., L-11935, April 24, 1959 (105 Phil. 1277) is not controlling. It applied
paragraph C of Sec. 1, of then Rule 107 of the Rules of Court, which reads:
After a criminal action has been commenced, no civil action arising
from the same offense can be prosecuted and the same shall be
suspended, in whatever stage it may be found, until final judgment in
the criminal proceeding has been rendered. (Emphasis supplied)
The governing rule is now Sec. 3, Rule 111, 1985 Rules on Criminal Procedure which refers to "civil
actions to enforce the civil liability arising from the offense" as contemplated in the first paragraph of
Section 1 of Rule 111-which is a civil action "for recovery of civil liability arising from the offense
charged." Sec. 1, Rule 111, (1985) is specific that it refers to civil action for the recovery of civil
liability arising from the offense charged. Whereas, the old Sec. 1 (c), Rule 107 simply referred to
"Civil action arising from the offense."
As earlier noted this action for legal separation is not to recover civil liability, in the main, but is
aimed at the conjugal rights of the spouses and their relations to each other, within the
contemplation of Articles 7 to 108, of the Civil Code."
2

Petitioner also argues that his conviction for concubinage will have to be first secured before the
action for legal separation can prosper or succeed, as the basis of the action for legal separation is
his alleged offense of concubinage.
Petitioner's assumption is erroneous.
A decree of legal separation, on the ground of concubinage, may be issued upon proof by
preponderance of evidence in the action for legal separation.
3
No criminal proceeding or conviction
is necessary. To this end, the doctrine in Francisco vs. Tayao
4
has been modified, as that case was
decided under Act. No. 2710, when absolute divorce was then allowed and had for its grounds the
same grounds for legal separation under the New Civil Code, with the requirement, under such
former law, that the guilt of defendant spouses had to be established by final judgment in a criminal
action. That requirement has not been reproduced or adopted by the framers of the present Civil
Code, and the omission has been uniformly accepted as a modification of the stringent rule
inFrancisco v. Tayao.
5

Petitioner's attempt to resist payment of support pendente lite to his wife must also fail, as we find no
proof of grave abuse of discretion on the part of the respondent Judge in ordering the same.
Support pendente lite, as a remedy, can be availed of in an action for legal separation, and granted
at the discretion of the judge.
6
If petitioner finds the amount of support pendente lite ordered as too
onerous, he can always file a motion to modify or reduce the same.
7

Petitioner lastly seeks to have the respondent Judge disqualified from hearing the case, as the grant
of supportpendente lite and the denial of the motion to suspend hearings in the case, are taken by
the petitioner as a disregard of applicable laws and existing doctrines, thereby showing the
respondent Judge's alleged manifest partiality to private respondent.
Petitioner's contention is without merit. Divergence of opinions between a judge hearing a case and
a party's counsel, as to applicable laws and jurisprudence, is not a sufficient ground to disqualify the
judge from hearing the case, on the ground of bias and manifest partiality. This is more so, in this
case, where we find the judge's disposition of petitioner's motions to be sound and well-taken.
WHEREFORE, the instant petition is hereby DISMISSED. Costs against petitioner.
SO ORDERED.
Yap (Chairman), Melencio-Herrera, Paras and Sarmiento, JJ., concur.
































Republic of the Philippines
SUPREME COURT
Manila
THIRD DIVISION
G.R. No. 156125 August 25, 2010
FRANCISCO MUOZ, JR., Petitioner,
vs.
ERLINDA RAMIREZ and ELISEO CARLOS, Respondents.
D E C I S I O N
BRION, J .:
We resolve the present petition for review on certiorari
1
filed by petitioner Francisco Muoz, Jr.
(petitioner) to challenge the decision
2
and the resolution
3
of the Court of Appeals (CA) in CA-G.R. CV
No. 57126.
4
The CA decision set aside the decision
5
of the Regional Trial Court (RTC), Branch 166,
Pasig City, in Civil Case No. 63665. The CA resolution denied the petitioners subsequent motion for
reconsideration.
FACTUAL BACKGROUND
The facts of the case, gathered from the records, are briefly summarized below.
Subject of the present case is a seventy-seven (77)-square meter residential house and lot located
at 170 A. Bonifacio Street, Mandaluyong City (subject property), covered by Transfer Certificate of
Title (TCT) No. 7650 of the Registry of Deeds of Mandaluyong City in the name of the petitioner.
6

The residential lot in the subject property was previously covered by TCT No. 1427, in the name of
Erlinda Ramirez, married to Eliseo Carlos (respondents).
7

On April 6, 1989, Eliseo, a Bureau of Internal Revenue employee, mortgaged TCT No. 1427, with
Erlindas consent, to the Government Service Insurance System (GSIS) to secure a P136,500.00
housing loan, payable within twenty (20) years, through monthly salary deductions
of P1,687.66.
8
The respondents then constructed a thirty-six (36)-square meter, two-story residential
house on the lot.
On July 14, 1993, the title to the subject property was transferred to the petitioner by virtue of a Deed
of Absolute Sale, dated April 30, 1992, executed by Erlinda, for herself and as attorney-in-fact of
Eliseo, for a stated consideration of P602,000.00.
9

On September 24, 1993, the respondents filed a complaint with the RTC for the nullification of the
deed of absolute sale, claiming that there was no sale but only a mortgage transaction, and the
documents transferring the title to the petitioners name were falsified.
The respondents alleged that in April 1992, the petitioner granted them a P600,000.00 loan, to be
secured by a first mortgage on TCT No. 1427; the petitioner gave Erlinda a P200,000.00
10
advance
to cancel the GSIS mortgage, and made her sign a document purporting to be the mortgage
contract; the petitioner promised to give the P402,000.00 balance when Erlinda surrenders TCT No.
1427 with the GSIS mortgage cancelled, and submits an affidavit signed by Eliseo stating that he
waives all his rights to the subject property; with the P200,000.00 advance, Erlinda paid
GSIS P176,445.27
11
to cancel the GSIS mortgage on TCT No. 1427;
12
in May 1992, Erlinda
surrendered to the petitioner the clean TCT No. 1427, but returned Eliseos affidavit, unsigned; since
Eliseos affidavit was unsigned, the petitioner refused to give the P402,000.00 balance and to cancel
the mortgage, and demanded that Erlinda return the P200,000.00 advance; since Erlinda could not
return theP200,000.00 advance because it had been used to pay the GSIS loan, the petitioner kept
the title; and in 1993, they discovered that TCT No. 7650 had been issued in the petitioners name,
cancelling TCT No.1427 in their name.
The petitioner countered that there was a valid contract of sale. He alleged that the respondents sold
the subject property to him after he refused their offer to mortgage the subject property because they
lacked paying capacity and were unwilling to pay the incidental charges; the sale was with the
implied promise to repurchase within one year,
13
during which period (from May 1, 1992 to April 30,
1993), the respondents would lease the subject property for a monthly rental of P500.00;
14
when the
respondents failed to repurchase the subject property within the one-year period despite notice, he
caused the transfer of title in his name on July 14, 1993;
15
when the respondents failed to pay the
monthly rentals despite demand, he filed an ejectment case
16
against them with the Metropolitan
Trial Court (MeTC), Branch 60, Mandaluyong City, on September 8, 1993, or sixteen days before the
filing of the RTC case for annulment of the deed of absolute sale.
During the pendency of the RTC case, or on March 29, 1995, the MeTC decided the ejectment case.
It ordered Erlinda and her family to vacate the subject property, to surrender its possession to the
petitioner, and to pay the overdue rentals.
17

In the RTC, the respondents presented the results of the scientific examination
18
conducted by the
National Bureau of Investigation of Eliseos purported signatures in the Special Power of
Attorney
19
dated April 29, 1992 and the Affidavit of waiver of rights dated April 29, 1992,
20
showing
that they were forgeries.
The petitioner, on the other hand, introduced evidence on the paraphernal nature of the subject
property since it was registered in Erlindas name; the residential lot was part of a large parcel of
land owned by Pedro Ramirez and Fructuosa Urcla, Erlindas parents; it was the subject of Civil
Case No. 50141, a complaint for annulment of sale, before the RTC, Branch 158, Pasig City, filed by
the surviving heirs of Pedro against another heir, Amado Ramirez, Erlindas brother; and, as a result
of a compromise agreement, Amado agreed to transfer to the other compulsory heirs of Pedro,
including Erlinda, their rightful shares of the land.
21

THE RTC RULING
In a Decision dated January 23, 1997, the RTC dismissed the complaint. It found that the subject
property was Erlindas exclusive paraphernal property that was inherited from her father. It also
upheld the sale to the petitioner, even without Eliseos consent as the deed of absolute sale bore the
genuine signatures of Erlinda and the petitioner as vendor and vendee, respectively. It concluded
that the NBI finding that Eliseos signatures in the special power of attorney and in the affidavit were
forgeries was immaterial because Eliseos consent to the sale was not necessary.
22

The respondents elevated the case to the CA via an ordinary appeal under Rule 41 of the Revised
Rules of Court.
THE CA RULING
The CA decided the appeal on June 25, 2002. Applying the second paragraph of Article 158
23
of the
Civil Code and Calimlim-Canullas v. Hon. Fortun,
24
the CA held that the subject property, originally
Erlindas exclusive paraphernal property, became conjugal property when it was used as collateral
for a housing loan that was paid through conjugal funds Eliseos monthly salary deductions; the
subject property, therefore, cannot be validly sold or mortgaged without Eliseos consent, pursuant to
Article 124
25
of the Family Code. Thus, the CA declared void the deed of absolute sale, and set
aside the RTC decision.
When the CA denied
26
the subsequent motion for reconsideration,
27
the petitioner filed the present
petition for review on certiorari under Rule 45 of the Revised Rules of Court.
THE PETITION
The petitioner argues that the CA misapplied the second paragraph of Article 158 of the Civil Code
and Calimlim-Canullas
28
because the respondents admitted in the complaint that it was the petitioner
who gave the money used to cancel the GSIS mortgage on TCT No. 1427; Article 120
29
of the
Family Code is the applicable rule, and since the value of the house is less than the value of the lot,
then Erlinda retained ownership of the subject property. He also argues that the contract between
the parties was a sale, not a mortgage, because (a) Erlinda did not deny her signature in the
document;
30
(b) Erlinda agreed to sign a contract of lease over the subject property;
31
and, (c) Erlinda
executed a letter, dated April 30, 1992, confirming the conversion of the loan application to a deed of
sale.
32

THE CASE FOR THE RESPONDENTS
The respondents submit that it is unnecessary to compare the respective values of the house and of
the lot to determine ownership of the subject property; it was acquired during their marriage and,
therefore, considered conjugal property. They also submit that the transaction between the parties
was not a sale, but an equitable mortgage because (a) they remained in possession of the subject
property even after the execution of the deed of absolute sale, (b) they paid the 1993 real property
taxes due on the subject property, and (c) they receivedP200,000.00 only of the total stated price
of P602,000.00.
THE ISSUE
The issues in the present case boil down to (1) whether the subject property is paraphernal or
conjugal; and, (2) whether the contract between the parties was a sale or an equitable mortgage.
OUR RULING
We deny the present Petition but for reasons other than those advanced by the CA.
This Court is not a trier of facts. However, if the inference, drawn by the CA, from the facts is
manifestly mistaken, as in the present case, we can review the evidence to allow us to arrive at the
correct factual conclusions based on the record.
33

First Issue:
Paraphernal or Conjugal?
As a general rule, all property acquired during the marriage, whether the acquisition appears to have
been made, contracted or registered in the name of one or both spouses, is presumed to be
conjugal unless the contrary is proved.
34

In the present case, clear evidence that Erlinda inherited the residential lot from her father has
sufficiently rebutted this presumption of conjugal ownership.
35
Pursuant to Articles 92
36
and 109
37
of
the Family Code, properties acquired by gratuitous title by either spouse, during the marriage, shall
be excluded from the community property and be the exclusive property of each spouse.
38
The
residential lot, therefore, is Erlindas exclusive paraphernal property.
The CA, however, held that the residential lot became conjugal when the house was built thereon
through conjugal funds, applying the second paragraph of Article 158 of the Civil Code and Calimlim-
Canullas.
39
Under the second paragraph of Article 158 of the Civil Code, a land that originally
belonged to one spouse becomes conjugal upon the construction of improvements thereon at the
expense of the partnership. We applied this provision in Calimlim-Canullas,
40
where we held that
when the conjugal house is constructed on land belonging exclusively to the husband, the land ipso
facto becomes conjugal, but the husband is entitled to reimbursement of the value of the land at the
liquidation of the conjugal partnership.
The CA misapplied Article 158 of the
Civil Code and Calimlim-Canullas
We cannot subscribe to the CAs misplaced reliance on Article 158 of the Civil Code and Calimlim-
Canullas.
As the respondents were married during the effectivity of the Civil Code, its provisions on conjugal
partnership of gains (Articles 142 to 189) should have governed their property relations. However,
with the enactment of the Family Code on August 3, 1989, the Civil Code provisions on conjugal
partnership of gains, including Article 158, have been superseded by those found in the Family Code
(Articles 105 to 133). Article 105 of the Family Code states:
x x x x
The provisions of this Chapter [on the Conjugal Partnership of Gains] shall also apply to conjugal
partnerships of gains already established between spouses before the effectivity of this Code,
without prejudice to vested rights already acquired in accordance with the Civil Code or other laws,
as provided in Article 256.
Thus, in determining the nature of the subject property, we refer to the provisions of the Family
Code, and not the Civil Code, except with respect to rights then already vested.
Article 120 of the Family Code, which supersedes Article 158 of the Civil Code, provides the solution
in determining the ownership of the improvements that are made on the separate property of the
spouses, at the expense of the partnership or through the acts or efforts of either or both spouses.
Under this provision, when the cost of the improvement and any resulting increase in value are more
than the value of the property at the time of the improvement, the entire property of one of the
spouses shall belong to the conjugal partnership, subject to reimbursement of the value of the
property of the owner-spouse at the time of the improvement; otherwise, said property shall be
retained in ownership by the owner-spouse, likewise subject to reimbursement of the cost of the
improvement.
41

In the present case, we find that Eliseo paid a portion only of the GSIS loan through monthly salary
deductions. From April 6, 1989
42
to April 30, 1992,
43
Eliseo paid about P60,755.76,
44
not the entire
amount of the GSIS housing loan plus interest, since the petitioner advanced the P176,445.27
45
paid
by Erlinda to cancel the mortgage in 1992. Considering the P136,500.00 amount of the GSIS
housing loan, it is fairly reasonable to assume that the value of the residential lot is considerably
more than the P60,755.76 amount paid by Eliseo through monthly salary deductions.
Thus, the subject property remained the exclusive paraphernal property of Erlinda at the time she
contracted with the petitioner; the written consent of Eliseo to the transacti on was not necessary.
The NBI finding that Eliseos signatures in the special power of attorney and affidavit were forgeries
was immaterial.
Nonetheless, the RTC and the CA apparently failed to consider the real nature of the contract
between the parties.
Second Issue:
Sale or Equitable Mortgage?
Jurisprudence has defined an equitable mortgage "as one which although lacking in some formality,
or form or words, or other requisites demanded by a statute, nevertheless reveals the intention of the
parties to charge real property as security for a debt, there being no impossibility nor anything
contrary to law in this intent."
46

Article 1602 of the Civil Code enumerates the instances when a contract, regardless of its
nomenclature, may be presumed to be an equitable mortgage: (a) when the price of a sale with right
to repurchase is unusually inadequate; (b) when the vendor remains in possession as lessee or
otherwise; (c) when upon or after the expiration of the right to repurchase another instrument
extending the period of redemption or granting a new period is executed; (d) when the purchaser
retains for himself a part of the purchase price; (e) when the vendor binds himself to pay the
taxes on the thing sold; and, (f) in any other case where it may be fairly inferred that the real
intention of the parties is that the transaction shall secure the payment of a debt or the
performance of any other obligation. These instances apply to a contract purporting to be an
absolute sale.
47

For the presumption of an equitable mortgage to arise under Article 1602 of the Civil Code, two (2)
requisites must concur: (a) that the parties entered into a contract denominated as a contract of sale;
and, (b) that their intention was to secure an existing debt by way of a mortgage. Any of the
circumstances laid out in Article 1602 of the Civil Code, not the concurrence nor an overwhelming
number of the enumerated circumstances, is sufficient to support the conclusion that a contract of
sale is in fact an equitable mortgage.
48

Contract is an equitable mortgage
In the present case, there are four (4) telling circumstances pointing to the existence of an equitable
mortgage.
First, the respondents remained in possession as lessees of the subject property; the parties, in fact,
executed a one-year contract of lease, effective May 1, 1992 to April 30, 1993.
49

Second, the petitioner retained part of the "purchase price," the petitioner gave a P200,000.00
advance to settle the GSIS housing loan, but refused to give the P402,000.00 balance when Erlinda
failed to submit Eliseos signed affidavit of waiver of rights.
Third, respondents paid the real property taxes on July 8, 1993, despite the alleged sale on April 30,
1992;
50
payment of real property taxes is a usual burden attaching to ownership and when, as here,
such payment is coupled with continuous possession of the property, it constitutes evidence of great
weight that the person under whose name the realty taxes were declared has a valid and rightful
claim over the land.
51

Fourth, Erlinda secured the payment of the principal debt owed to the petitioner with the subject
property. The records show that the petitioner, in fact, sent Erlinda a Statement of Account showing
that as of February 20, 1993, she owed P384,660.00, and the daily interest, starting February 21,
1993, was P641.10.
52
Thus, the parties clearly intended an equitable mortgage and not a contract of
sale.
That the petitioner advanced the sum of P200,000.00 to Erlinda is undisputed. This advance, in fact,
prompted the latter to transfer the subject property to the petitioner. Thus, before the respondents
can recover the subject property, they must first return the amount of P200,000.00 to the petitioner,
plus legal interest of 12% per annum, computed from April 30, 1992.
We cannot sustain the ballooned obligation of P384,660.00, claimed in the Statement of Account
sent by the petitioner,
53
sans any evidence of how this amount was arrived at. Additionally, a daily
interest of P641.10 orP19,233.00 per month for a P200,000.00 loan is patently unconscionable.
While parties are free to stipulate on the interest to be imposed on monetary obligations, we can
step in to temper the interest rates if they are unconscionable.
54

In Lustan v. CA,
55
where we established the reciprocal obligations of the parties under an equitable
mortgage, we ordered the reconveyance of the property to the rightful owner therein upon the
payment of the loan within ninety (90) days from the finality of the decision.
56

WHEREFORE, in light of all the foregoing, we hereby DENY the present petition. The assailed
decision and resolution of the Court of Appeals in CA-G.R. CV No. 57126 are AFFIRMED with the
following MODIFICATIONS:
1. The Deed of Absolute Sale dated April 30, 1992 is hereby declared an equitable
mortgage; and
2. The petitioner is obligated to RECONVEY to the respondents the property covered by
Transfer Certificate of Title No. 7650 of the Register of Deeds of Mandaluyong City, UPON
THE PAYMENT OF P200,000.00, with 12% legal interest from April 30, 1992, by
respondents within NINETY DAYS FROM THE FINALITY OF THIS DECISION.
Costs against the petitioner.
SO ORDERED.
ARTURO D. BRION
Associate Justice



Supreme Court
Manila

THIRD DIVISION


SPOUSES REX AND CONCEPCION
AGGABAO,
Petitioners,



-versus-



DIONISIO Z. PARULAN, JR.
and MA. ELENA PARULAN,
Respondents.
G.R. No. 165803

Present:

CARPIO MORALES, Chairperson
BERSAMIN,
DEL CASTILLO,
*

VILLARAMA, JR., and
SERENO, JJ.

Promulgated:

September 1, 2010
x-----------------------------------------------------------------------------------------x
D E C I S I O N
BERSAMIN, J :

On July 26, 2000, the Regional Trial Court (RTC), Branch 136, in Makati City annulled the deed of
absolute sale executed in favor of the petitioners covering two parcels of registered land the respondents owned for
want of the written consent of respondent husband Dionisio Parulan, Jr. On July 2, 2004, in C.A.-G.R. CV No.
69044,
[1]
the Court of Appeals (CA) affirmed the RTC decision.

Hence, the petitioners appeal by petition for review on certiorari, seeking to reverse the decision of the CA.
They present as the main issue whether the sale of conjugal property made by respondent wife by presenting a
special power of attorney to sell (SPA) purportedly executed by respondent husband in her favor was validly made
to the vendees, who allegedly acted in good faith and paid the full purchase price, despite the showing by the
husband that his signature on the SPA had been forged and that the SPA had been executed during his absence from
the country.

We resolve the main issue against the vendees and sustain the CAs finding that the vendees were not
buyers in good faith, because they did not exercise the necessary prudence to inquire into the wifes authority to sell.
We hold that the sale of conjugal property without the consent of the husband was not merely voidable but void;
hence, it could not be ratified.

Antecedents

Involved in this action are two parcels of land and their improvements (property) located at No. 49 Miguel
Cuaderno Street, Executive Village, BF Homes, Paraaque City and registered under Transfer Certificate of Title
(TCT) No. 63376
[2]
and TCT No. 63377
[3]
in the name of respondents Spouses Maria Elena A. Parulan (Ma.
Elena) and Dionisio Z. Parulan, Jr. (Dionisio), who have been estranged from one another.

In January 1991, real estate broker Marta K. Atanacio (Atanacio) offered the property to the petitioners, who
initially did not show interest due to the rundown condition of the improvements. But Atanacios persistence
prevailed upon them, so that on February 2, 1991, they and Atanacio met with Ma. Elena at the site of the property.
During their meeting, Ma. Elena showed to them the following documents, namely: (a) the owners original copy of
TCT No. 63376; (b) a certified true copy of TCT No. 63377; (c) three tax declarations; and (d) a copy of the special
power of attorney (SPA) dated January 7, 1991 executed by Dionisio authorizing Ma. Elena to sell the
property.
[4]
Before the meeting ended, they paid P20,000.00 as earnest money, for which Ma. Elena executed a
handwritten Receipt of Earnest Money, whereby the parties stipulated that: (a) they would pay an additional
payment of P130,000.00 on February 4, 1991; (b) they would pay the balance of the bank loan of the respondents
amounting to P650,000.00 on or before February 15, 1991; and (c) they would make the final payment
of P700,000.00 once Ma. Elena turned over the property on March 31, 1991.
[5]


On February 4, 1991, the petitioners went to the Office of the Register of Deeds and the Assessors Office of
Paraaque City to verify the TCTs shown by Ma. Elena in the company of Atanacio and her husband (also a
licensed broker).
[6]
There, they discovered that the lot under TCT No. 63376 had been encumbered to Banco Filipino
in 1983 or 1984, but that the encumbrance had already been cancelled due to the full payment of the
obligation.
[7]
They noticed that the Banco Filipino loan had been effected through an SPA executed by Dionisio in
favor of Ma. Elena.
[8]
They found on TCT No. 63377 the annotation of an existing mortgage in favor of the Los
Baos Rural Bank, also effected through an SPA executed by Dionisio in favor of Ma. Elena, coupled with a copy of
a court order authorizing Ma. Elena to mortgage the lot to secure a loan of P500,000.00.
[9]


The petitioners and Atanacio next inquired about the mortgage and the court order annotated on TCT No.
63377 at the Los Baos Rural Bank. There, they met with Atty. Noel Zarate, the banks legal counsel, who related
that the bank had asked for the court order because the lot involved was conjugal property.
[10]


Following their verification, the petitioners delivered P130,000.00 as additional down payment on February 4,
1991; and P650,000.00 to the Los Baos Rural Bank onFebruary 12, 1991, which then released the owners
duplicate copy of TCT No. 63377 to them.
[11]


On March 18, 1991, the petitioners delivered the final amount of P700,000.00 to Ma. Elena, who executed a
deed of absolute sale in their favor. However, Ma. Elena did not turn over the owners duplicate copy of TCT No.
63376, claiming that said copy was in the possession of a relative who was then in Hongkong.
[12]
She assured them
that the owners duplicate copy of TCT No. 63376 would be turned over after a week.

On March 19, 1991, TCT No. 63377 was cancelled and a new one was issued in the name of the
petitioners.

Ma. Elena did not turn over the duplicate owners copy of TCT No. 63376 as promised. In due time, the
petitioners learned that the duplicate owners copy of TCT No. 63376 had been all along in the custody of Atty.
Jeremy Z. Parulan, who appeared to hold an SPA executed by his brother Dionisio authorizing him to
sell both lots.
[13]


At Atanacios instance, the petitioners met on March 25, 1991 with Atty. Parulan at
the Manila Peninsula.
[14]
For that meeting, they were accompanied by one Atty. Olandesca.
[15]
They recalled that
Atty. Parulan smugly demanded P800,000.00 in exchange for the duplicate owners copy of TCT No. 63376,
because Atty. Parulan represented the current value of the property to be P1.5 million. As a counter-offer, however,
they tendered P250,000.00, which Atty. Parulan declined,
[16]
giving them only untilApril 5, 1991 to decide.

Hearing nothing more from the petitioners, Atty. Parulan decided to call them on April 5, 1991, but they
informed him that they had already fully paid to Ma. Elena.
[17]


Thus, on April 15, 1991, Dionisio, through Atty. Parulan, commenced an action (Civil Case No. 91-
1005 entitled Dionisio Z. Parulan, Jr., represented by Jeremy Z. Parulan, as attorney in fact, v. Ma. Elena Parulan,
Sps. Rex and Coney Aggabao), praying for the declaration of the nullity of the deed of absolute sale executed by Ma.
Elena, and the cancellation of the title issued to the petitioners by virtue thereof.

In turn, the petitioners filed on July 12, 1991 their own action for specific performance with damages
against the respondents.

Both cases were consolidated for trial and judgment in the RTC.
[18]


Ruling of the RTC

After trial, the RTC rendered judgment, as follows:

WHEREFORE, and in consideration of the foregoing, judgment is hereby rendered in favor
of plaintiff Dionisio A. Parulan, Jr. and against defendants Ma. Elena Parulan and the Sps. Rex
and Concepcion Aggabao, without prejudice to any action that may be filed by the Sps. Aggabao
against co-defendant Ma. Elena Parulan for the amounts they paid her for the purchase of the
subject lots, as follows:

1. The Deed of Absolute Sale dated March 18, 1991 covering the sale of the lot located
at No. 49 M. Cuaderno St., Executive Village, BF Homes, Paraaque, Metro Manila, and covered
by TCT Nos. 63376 and 63377 is declared null and void.

2. Defendant Mrs. Elena Parulan is directed to pay litigation expenses amounting
to P50,000.00 and the costs of the suit.

SO ORDERED.
[19]


The RTC declared that the SPA in the hands of Ma. Elena was a forgery, based on its finding that Dionisio
had been out of the country at the time of the execution of the SPA;
[20]
that NBI Sr. Document Examiner Rhoda B.
Flores had certified that the signature appearing on the SPA purporting to be that of Dionisio and the set of standard
sample signatures of Dionisio had not been written by one and the same person;
[21]
and that Record Officer III Eliseo
O. Terenco and Clerk of Court Jesus P. Maningas of the Manila RTC had issued a certification to the effect that
Atty. Alfred Datingaling, the Notary Public who had notarized the SPA, had not been included in the list of Notaries
Public in Manila for the year 1990-1991.
[22]


The RTC rejected the petitioners defense of being buyers in good faith because of their failure to exercise
ordinary prudence, including demanding from Ma. Elena a court order authorizing her to sell the properties similar
to the order that the Los Baos Rural Bank had required before accepting the mortgage of the property.
[23]
It
observed that they had appeared to be in a hurry to consummate the transaction despite Atanacios advice that they
first consult a lawyer before buying the property; that with ordinary prudence, they should first have obtained the
owners duplicate copies of the TCTs before paying the full amount of the consideration; and that the sale was void
pursuant to Article 124 of theFamily Code.
[24]


Ruling of the CA

As stated, the CA affirmed the RTC, opining that Article 124 of the Family Code applied because Dionisio
had not consented to the sale of the conjugal property by Ma. Elena; and that the RTC correctly found the SPA to be
a forgery.
The CA denied the petitioners motion for reconsideration.
[25]


Issues

The petitioners now make two arguments: (1) they were buyers in good faith; and (2) the CA erred in
affirming the RTCs finding that the sale between Mrs. Elena and the petitioners had been a nullity under Article
124 of the Family Code.

The petitioners impute error to the CA for not applying the ordinary prudent mans standard in
determining their status as buyers in good faith. They contend that the more appropriate law to apply was Article
173 of the Civil Code, not Article 124 of the Family Code; and that even if the SPA held by Ma. Elena was a
forgery, the ruling in Veloso v. Court of Appeals
[26]
warranted a judgment in their favor.

Restated, the issues for consideration and resolution are as follows:

1) Which between Article 173 of the Civil Code and Article 124 of the Family Code should apply
to the sale of the conjugal property executed without the consent of Dionisio?

2) Might the petitioners be considered in good faith at the time of their purchase of the property?

3) Might the ruling in Veloso v. Court of Appeals be applied in favor of the petitioners despite the
finding of forgery of the SPA?

Ruling

The petition has no merit. We sustain the CA.


1.
Article 124, Family Code, applies to sale of conjugal
properties made after the effectivity of the Family Code

The petitioners submit that Article 173 of the Civil Code, not Article 124 of the Family Code, governed the
property relations of the respondents because they had been married prior to the effectivity of the Family Code; and
that the second paragraph of Article 124 of the Family Code should not apply because the other spouse held the
administration over the conjugal property. They argue that notwithstanding his absence from the country Dionisio
still held the administration of the conjugal property by virtue of his execution of the SPA in favor of his brother;
and that even assuming that Article 124 of the Family Code properly applied, Dionisio ratified the sale through Atty.
Parulans counter-offer during the March 25, 1991 meeting.

We do not subscribe to the petitioners submissions.

To start with, Article 254
[27]
the Family Code has expressly repealed several titles under the Civil Code,
among them the entire Title VI in which the provisions on the property relations between husband and wife, Article
173 included, are found.

Secondly, the sale was made on March 18, 1991, or after August 3, 1988, the effectivity of the Family
Code. The proper law to apply is, therefore, Article 124 of the Family Code, for it is settled that any alienation or
encumbrance of conjugal property made during the effectivity of the Family Code is governed by Article 124 of
the Family Code.
[28]


Article 124 of the Family Code provides:

Article 124. The administration and enjoyment of the conjugal partnership property shall
belong to both spouses jointly. In case of disagreement, the husbands decision shall prevail,
subject to recourse to the court by the wife for proper remedy, which must be availed of within
five years from the date of the contract implementing such decision.

In the event that one spouse is incapacitated or otherwise unable to participate in the
administration of the conjugal properties, the other spouse may assume sole powers of
administration. These powers do not include disposition or encumbrance without authority
of the court or the written consent of the other spouse. In the absence of such authority or
consent, the disposition or encumbrance shall be void. However, the transaction shall be
construed as a continuing offer on the part of the consenting spouse and the third person, and may
be perfected as a binding contract upon the acceptance by the other spouse or authorization by the
court before the offer is withdrawn by either or both offerors.

Thirdly, according to Article 256
[29]
of the Family Code, the provisions of the Family Code may apply
retroactively provided no vested rights are impaired. In Tumlos v. Fernandez,
[30]
the Court rejected the petitioners
argument that the Family Code did not apply because the acquisition of the contested property had occurred prior to
the effectivity of the Family Code, and pointed out that Article 256 provided that the Family Code could apply
retroactively if the application would not prejudice vested or acquired rights existing before the effectivity of
the Family Code. Herein, however, the petitioners did not show any vested right in the property acquired prior
to August 3, 1988 that exempted their situation from the retroactive application of the Family Code.

Fourthly, the petitioners failed to substantiate their contention that Dionisio, while holding the
administration over the property, had delegated to his brother, Atty. Parulan, the administration of the property,
considering that they did not present in court the SPA granting to Atty. Parulan the authority for the administration.
Nonetheless, we stress that the power of administration does not include acts of disposition or
encumbrance, which are acts of strict ownership. As such, an authority to dispose cannot proceed from an authority
to administer, and vice versa, for the two powers may only be exercised by an agent by following the provisions on
agency of the Civil Code (from Article 1876 to Article 1878). Specifically, the apparent authority of Atty. Parulan,
being a special agency, was limited to the sale of the property in question, and did not include or extend to the power
to administer the property.
[31]


Lastly, the petitioners insistence that Atty. Parulans making of a counter-offer during the March 25,
1991 meeting ratified the sale merits no consideration. Under Article 124 of the Family Code, the transaction
executed sans the written consent of Dionisio or the proper court order was void; hence, ratification did not occur,
for a void contract could not be ratified.
[32]


On the other hand, we agree with Dionisio that the void sale was a continuing offer from the petitioners and
Ma. Elena that Dionisio had the option of accepting or rejecting before the offer was withdrawn by either or both
Ma. Elena and the petitioners. The last sentence of the second paragraph of Article 124 of the Family Code makes
this clear, stating that in the absence of the other spouses consent, the transaction should be construed as a
continuing offer on the part of the consenting spouse and the third person, and may be perfected as a binding
contract upon the acceptance by the other spouse or upon authorization by the court before the offer is withdrawn by
either or both offerors.



2.
Due diligence required in verifying not only vendors title,
but also agents authority to sell the property

A purchaser in good faith is one who buys the property of another, without notice that some other person
has a right to, or interest in, such property, and pays the full and fair price for it at the time of such purchase or
before he has notice of the claim or interest of some other persons in the property. He buys the property with the
belief that the person from whom he receives the thing was the owner and could convey title to the property. He
cannot close his eyes to facts that should put a reasonable man on his guard and still claim he acted in good
faith.
[33]
The status of a buyer in good faith is never presumed but must be proven by the person invoking it.
[34]


Here, the petitioners disagree with the CA for not applying the ordinary prudent mans standard in
determining their status as buyers in good faith. They insist that they exercised due diligence by verifying the status
of the TCTs, as well as by inquiring about the details surrounding the mortgage extended by the Los Baos Rural
Bank. They lament the holding of the CA that they should have been put on their guard when they learned that the
Los Baos Rural Bank had first required a court order before granting the loan to the respondents secured by their
mortgage of the property.

The petitioners miss the whole point.

Article 124 of the Family Code categorically requires the consent of both spouses before the conjugal
property may be disposed of by sale, mortgage, or other modes of disposition. In Bautista v. Silva,
[35]
the Court
erected a standard to determine the good faith of the buyers dealing with
a seller who had title to and possession of the land but whose capacity to sell was restricted, in that the consent of
the other spouse was required before the conveyance, declaring that in order to prove good faith in such a situation,
the buyers must show that they inquired not only into the title of the seller but also into the sellers capacity to
sell.
[36]
Thus, the buyers of conjugal property must observe two kinds of requisite diligence, namely: (a) the
diligence in verifying the validity of the title covering the property; and (b) the diligence in inquiring into the
authority of the transacting spouse to sell conjugal property in behalf of the other spouse.

It is true that a buyer of registered land needs only to show that he has relied on the face of the certificate of
title to the property, for he is not required to explore beyond what the certificate indicates on its face.
[37]
In this
respect, the petitioners sufficiently proved that they had checked on the authenticity of TCT No. 63376 and TCT No.
63377 with the Office of the Register of Deeds in Pasay City as the custodian of the land records; and that they had
also gone to the Los Baos Rural Bank to inquire about the mortgage annotated on TCT No. 63377. Thereby, the
petitioners observed the requisite diligence in examining the validity of the TCTs concerned.

Yet, it ought to be plain enough to the petitioners that the issue was whether or not they had diligently
inquired into the authority of Ma. Elena to convey the property, not whether or not the TCT had been valid and
authentic, as to which there was no doubt. Thus, we cannot side with them.

Firstly, the petitioners knew fully well that the law demanded the written consent of Dionisio to the sale,
but yet they did not present evidence to show that they had made inquiries into the circumstances behind the
execution of the SPA purportedly executed by Dionisio in favor of Ma. Elena. Had they made the appropriate
inquiries, and not simply accepted the SPA for what it represented on its face, they would have uncovered soon
enough that the respondents had been estranged from each other and were under de factoseparation, and that they
probably held conflicting interests that would negate the existence of an agency between them. To lift this doubt,
they must, of necessity, further inquire into the SPA of Ma. Elena. The omission to inquire indicated their not being
buyers in good faith, for, as fittingly observed in Domingo v. Reed:
[38]


What was required of them by the appellate court, which we affirm, was merely to
investigate as any prudent vendee should the authority of Lolita to sell the property and to bind
the partnership. They had knowledge of facts that should have led them to inquire and to
investigate, in order to acquaint themselves with possible defects in her title. The law requires
them to act with the diligence of a prudent person; in this case, their only prudent course of action
was to investigate whether respondent had indeed given his consent to the sale and authorized his
wife to sell the property.
[39]


Indeed, an unquestioning reliance by the petitioners on Ma. Elenas SPA without first taking precautions to
verify its authenticity was not a prudent buyers move.
[40]
They should have done everything within their means and
power to ascertain whether the SPA had been genuine and authentic. If they did not investigate on the relations of
the respondents vis--vis each other, they could have done other things towards the same end, like attempting to
locate the notary public who had notarized the SPA, or checked with the RTC in Manila to confirm the authority of
Notary Public Atty. Datingaling. It turned out that Atty. Datingaling was not authorized to act as a Notary Public
for Manila during the period 1990-1991, which was a fact that they could easily discover with a modicum of zeal.

Secondly, the final payment of P700,000.00 even without the owners duplicate copy of the TCT No.
63376 being handed to them by Ma. Elena indicated a revealing lack of precaution on the part of the petitioners. It is
true that she promised to produce and deliver the owners copy within a week because her relative having custody of
it had gone to Hongkong, but their passivity in such an essential matter was puzzling light of their earlier alacrity in
immediately and diligently validating the TCTs to the extent of inquiring at the Los Baos Rural Bank about the
annotated mortgage. Yet, they could have rightly withheld the final payment of the balance. That they did not do so
reflected their lack of due care in dealing with Ma. Elena.

Lastly, another reason rendered the petitioners good faith incredible. They did not take immediate action
against Ma. Elena upon discovering that the owners original copy of TCT No. 63376 was in the possession of Atty.
Parulan, contrary to Elenas representation. Human experience would have impelled them to exert every effort to
proceed against Ma. Elena, including demanding the return of the substantial amounts paid to her. But they seemed
not to mind her inability to produce the TCT, and, instead, they contented themselves with meeting with Atty.
Parulan to negotiate for the possible turnover of the TCT to them.

3.
Veloso v. Court of Appeals cannot help petitioners

The petitioners contend that the forgery of the SPA notwithstanding, the CA could still have decided in
their favor conformably with Veloso v. Court of Appeals,
[41]
a casewhere the petitioner husband claimed that his
signature and that of the notary public who had notarized the SPA the petitioner supposedly executed to authorize
his wife to sell the property had been forged. In denying relief, the Court upheld the right of the vendee as an
innocent purchaser for value.

Veloso is inapplicable, however, because the contested property therein was exclusively owned by the
petitioner and did not belong to the conjugal regime. Veloso being upon conjugal property, Article 124 of the Family
Code did not apply.

In contrast, the property involved herein pertained to the conjugal regime, and, consequently, the lack of
the written consent of the husband rendered the sale void pursuant to Article 124 of the Family Code. Moreover,
even assuming that the property involved in Veloso was conjugal, its sale was made on November 2, 1987, or prior
to the effectivity of the Family Code; hence, the sale was still properly covered by Article 173 of the Civil
Code, which provides that a sale effected without the consent of one of the spouses is only voidable, not void.
However, the sale herein was made already during the effectivity of the Family Code, rendering the application of
Article 124 of the Family Code clear and indubitable.

The fault of the petitioner in Veloso was that he did not adduce sufficient evidence to prove that his
signature and that of the notary public on the SPA had been forged. The Court pointed out that his mere allegation
that the signatures had been forged could not be sustained without clear and convincing proof to substantiate the
allegation. Herein, however, both the RTC and the CA found from the testimonies and evidence presented by
Dionisio that his signature had been definitely forged, as borne out by the entries in his passport showing that he was
out of the country at the time of the execution of the questioned SPA; and that the alleged notary public, Atty.
Datingaling, had no authority to act as a Notary Public for Manila during the period of 1990-1991.

WHEREFORE, we deny the petition for review on certiorari, and affirm the decision dated July 2,
2004 rendered by the Court of Appeals in C.A.-G.R. CV No. 69044entitled Dionisio Z. Parulan, Jr. vs. Ma. Elena
Parulan and Sps. Rex and Concepcion Aggabao and Sps. Rex and Concepcion Aggabao vs. Dionisio Z. Parulan,
Jr. and Ma. Elena Parulan.

Costs of suit to be paid by the petitioners.

SO ORDERED.







Republic of the Philippines
SUPREME COURT
Manila
EN BANC
G.R. No. L-12707 August 10, 1918
MRS. HENRY E. HARDING, and her husband, plaintiffs-appellees,
vs.
COMMERCIAL UNION ASSURANCE COMPANY, defendant-appellant.
Lawrence & Ross for appellant.
Gibbs, McDonough & Johnson for appellees.
FISHER, J .:
This was an action by plaintiffs to recover from defendant the sum of P3,000 and interest, alleged to
be due under the terms of a policy of insurance. The trial court gave plaintiffs judgment for the
amount demanded, with interest and costs, and from that decision the defendant appeals.
The court below stated the issues made by the pleadings in this case, and its finding of fact, as
follows:
It is alleged by plaintiffs and admitted by defendant that plaintiffs are husband and wife and
residents of the city of Manila; that the defendant is a foreign corporation organized and
existing under and by virtue of the laws of Great Britain and duly registered in the Philippine
Islands, and Smith, Bell & Co. (limited), a corporation organized and existing under the laws
of the Philippine Islands, with its principal domicile in the city of Manila, is the agent in the
Philippine Islands of said defendant.
The plaintiffs alleged that on February 16, 1916, the plaintiff Mrs. Henry E. Harding was the
owner of a Studebaker automobile, registered number 2063, in the city of Manila; that on
said date; in consideration of the payment to the defendant of the premium of P150, by said
plaintiff, Mrs. Henry E. Harding, with the consent of her husband, the defendant by its duly
authorized agent, Smith, Bell & Company (limited), made its policy of insurance in writing
upon said automobile was set forth in said policy to be P3,000 that the value of said
automobile was set forth in said policy (Exhibit A) to be P3,000; that on March 24, 1916, said
automobile was totally destroyed by fire; that the loss thereby to plaintiffs was the sum of
P3,000; that thereafter, within the period mentioned in the said policy of insurance, the
plaintiff, Mrs. Henry E. Harding, furnished the defendant the proofs of her said loss and
interest, and otherwise performed all the conditions of said policy on her part, and that the
defendant has not paid said loss nor any part thereof, although due demand was made upon
defendant therefor.
The defendant, by its answer, admitted the allegations of the residence and status of the
parties and denied all the other allegation of the said complaint, and for a separate and
affirmative defense alleged (1) that on February 17, 1916, at the city of Manila, P.I. the
defendant upon request of plaintiff, Mrs. Henry E. Harding, issued to the said plaintiff the
policy of insurance on an automobile alleged by the said plaintiff to be her property; that the
said request for the issuance of said policy of insurance was made by means of a proposal in
writing signed and delivered by said plaintiff to the defendant, guaranteeing the truth of the
statements contained therein which said proposal is referred to in the said policy of
insurance made a part thereof; (2) that certain of the statements and representations
contained in said proposal and warranted by said plaintiff to be true, to wit: (a) the price paid
by the proposer for the said automobile; (b) the value of said automobile at the time of the
execution and delivery of the said proposal and (c) the ownership of said automobile, were
false and known to be false by the said plaintiff at the time of signing and delivering the said
proposal and were made for the purpose of misleading and deceiving the defendant, and
inducing the defendant, relying upon the warranties, statements, and representations
contained in the said proposal and believing the same to be true, issued the said policy of
insurance.
The defendant prays that judgment be entered declaring the said policy of insurance to be
null and void, and that plaintiffs take nothing by this action; and for such further relief as to
the court may seem just and equitable.
The evidence in this case shows that some time in the year 1913 Levy Hermanos, the Manila
agents for the Studebaker automobile, sold the automobile No. 2063 to John Canson for
P3,200 (testimony of Mr. Diehl); that under date of October 14, 1914, John Canson sold the
said automobile to Henry Harding for the sum of P1,500 (Exhibit 2); that under date of
November 19, 1914, the said Henry Harding sold the said automobile No. 2063 to J.
Brannigan, of Los Baos, Province of Laguna, P.I., for the sum of P2,000 (Exhibit 3); that
under date of December 20, 1915, J. C. Graham of Los Baos, Province of Laguna, P.I.,
sold the said automobile No. 2063 to Henry Harding of the city of Manila for the sum of
P2,800 (Exhibit 4 and testimony of J. C. Graham); that on or about January 1, 1916, the said
Henry Harding gave the said automobile to his wife; Mrs. Henry E. Harding, one of the
plaintiffs, as a present; that said automobile was repaired and repainted at the Luneta
Garage at a cost of some P900 (testimony of Mr. Server); that while the said automobile was
at the Luneta Garage; the said Luneta Garage, acting as agent for Smith, Bell & Company,
(limited), solicited of the plaintiff Mrs. Harding the insurance of said automobile by the
defendant Company (testimony of Mrs. Henry Harding and Mr. Server); that a proposal was
filled out by the said agent and signed by the plaintiff Mrs. Henry E. Harding, and in said
proposal under the heading "Price paid by proposer," is the amount of "3,500" and under
another heading "Present value" is the amount of "3,000" (Exhibit 1).
The evidence tends to show that after the said proposal was made a representative of the
Manila agent of defendant went to the Luneta Garage and examined said automobile No.
2063 and Mr. Server, the General Manager of the Luneta Garage, an experienced
automobile mechanic, testified that at the time this automobile was insured it was worth
about P3,000, and the defendant, by and through its said agent Smith, Bell & Company
(limited), thereafter issued a policy of insurance upon proposal in which policy the said
automobile was described as of the "present value" of P3,000 and the said defendant
charged the said plaintiff Mrs. Henry E. Harding as premium on said policy the sum of P150,
or 5 per cent of the then estimated value of P3,000. (Exhibit A.)
The "Schedule" in said policy of insurance describes the automobile here in question, and
provides in part of follows:
"Now it is hereby agreed as follows:
"That during the period above set forth and during any period for which the company
may agree to renew this policy the company will subject to the exception and
conditions contained herein or endorsed hereon indemnify the insured against loss of
or damage to any motor car described in the schedule hereto (including accessories)
by whatever cause such loss or damage may be occasioned and will further
indemnify the insured up to the value of the car or P3,000 whichever is the greater
against any claim at common law made by any person (not being a person in the
said motor car nor in the insured's service) for loss of life or for accidental bodily
injury or damage to property caused by the said motor car including law costs
payable in connection with such claim when incurred with the consent of the
company."
The evidence further shows that on March 24, 1916, the said automobile was totally
destroyed by fire, and that the iron and steel portions of said automobile which did not burn
were taken into the possession of the defendant by and through its agent Smith, Bell &
Company (limited), and sold by it for a small sum, which had never been tendered to the
plaintiff prior to the trial of this case, but in open court during the trial the sum of P10 as the
proceeds of such sale was tendered to plaintiff and refused.
Upon the facts so found, which we hold are supported by the evidence, the trial judge decided that
there was no proof of fraud on the part of plaintiff in her statement of the value of the automobile, or
with respect to its ownership; that she had an insurable interest therein; and that defendant, having
agreed to the estimated value, P3,000, and having insured the automobile for that amount, upon the
basis of which the premium was paid, is bound by it and must pay the loss in accordance with the
stipulated insured value. The assignments of error made on behalf of appellant put in issue the
correctness of those conclusions of law, and some others of minor importance relating to the
exclusion of evidence. Disposing of the minor objections first, as we have reached the conclusion
that the trial court was right in holding that the defendant is bound by the estimated value of the
automobile upon which policy was issued, and that the plaintiff was not guilty of fraud in regard
thereto, the exclusion of the testimony of the witness Diehl is without importance. It merely tended to
show the alleged actual value of the automobile, and in the view we take of the case such evidence
was irrelevant.
Appellant contends that Mrs. Harding was not the owner of the automobile at the time of the
issuance of the policy, and, therefore, had no insurable interest in it. The court below found that the
automobile was given to plaintiff by her husband shortly after the issuance of the policy here in
question. Appellant does not dispute the correctness of this finding, but contends that the gift was
void, citing article 1334 of the Civil Code which provides that "All gifts between spouses during the
marriage shall be void. Moderate gifts which the spouses bestow on each other on festive days of
the family are not included in this rule."
We are of the opinion that this contention is without merit. In the case of Cook vs. McMicking 27 Phil.
Rep., 10), this court said:
It is claimed by the appellants that the so-called transfer from plaintiff's husband to her was
completely void under article 1458 of the Civil Code and that, therefore, the property still
remains the property of Edward Cook and subject to levy under execution against him.
In our opinion the position taken by appellants is untenable. They are not in a position to
challenge the validity of the transfer, if it may be called such. They bore absolutely no
relation to the parties to the transfer at the time it occurred and had no rights or interests
inchoate, present, remote, or otherwise, in the property in question at the time the transfer
occurred. Although certain transfers from husband to wife or from wife to husband are
prohibited in the article referred to, such prohibition can be taken advantage of only by
persons who bear such a relation to the parties making the transfer or to the property itself
that such transfer interferes with their rights or interests. Unless such a relationship appears
the transfer cannot be attacked.
Even assuming that defendant might have invoked article 1334 as a defense, the burden would be
upon it to show that the gift in question does not fall within the exception therein established. We
cannot say, as a matter of law, that the gift of an automobile by a husband to his wife is not a
moderate one. Whether it is or is not would depend upon the circumstances of the parties, as to
which nothing is disclosed by the record.
Defendant contends that the statement regarding the cost of the automobile was a warranty, that the
statement was false, and that, therefore, the policy never attached to the risk. We are of the opinion
that it has not been shown by the evidence that the statement was false on the contrary we
believe that it shows that the automobile had in fact cost more than the amount mentioned. The
court below found, and the evidence shows, that the automobile was bought by plaintiff's husband a
few weeks before the issuance of the policy in question for the sum of P2,800, and that between that
time and the issuance of the policy some P900 was spent upon it in repairs and repainting. The
witness Server, an expert automobile mechanic, testified that the automobile was practically as good
as new at the time the insurance was effected. The form of proposal upon which the policy was
issued does not call for a statement regarding the value of the automobile at the time of its
acquisition by the applicant for the insurance, but merely a statement of its cost. The amount stated
was less than the actual outlay which the automobile represented to Mr. Harding, including repairs,
when the insurance policy was issued. It is true that the printed form calls for a statement of the
"price paid by the proposer," but we are of the opinion that it would be unfair to hold the policy void
simply because the outlay represented by the automobile was made by the plaintiff's husband and
not by his wife, to whom he had given the automobile. It cannot be assumed that defendant should
not have issued the policy unless it were strictly true that the price representing the cost of the
machine had been paid by the insured and by no other person that it would no event insure an
automobile acquired by gift, inheritance, exchange, or any other title not requiring the owner to make
a specific cash outlay for its acquisition.
Furthermore, the court below found and the evidence shows, without dispute, that the proposal upon
which the policy in question was issued was made out by defendant's agent by whom the insurance
was solicited, and that appellee simply signed the same. It also appears that an examiner employed
by the defendant made an inspection of the automobile before the acceptance of the risk, and that
the sum after this examination. The trial court found that Mrs. Harding, in fixing the value of the
automobile at P3,000, acted upon information given her by her husband and by Mr. Server, the
manager of the Luneta Garage. The Luneta Garage, it will be remembered, was the agent of the
defendant corporation in the solicitation of the insurance. Mrs. Harding did not state of her own
knowledge that the automobile originally cost P3,000, or that its value at the time of the insurance
was P3,000. She merely repeated the information which had been given her by her husband, and at
the same time disclosed to defendant's agent the source of her information. There is no evidence to
sustain the contention that this communication was made in bad faith. It appears that the statements
in the proposal as to the price paid for the automobile and as to its value were written by Mr. Quimby
who solicited the insurance on behalf of defendant, in his capacity as an employee of the Luneta
Garage, and wrote out the proposal for Mrs. Harding to sign. Under these circumstances, we do not
think that the facts stated in the proposal can be held as a warranty of the insured, even if it should
have been shown that they were incorrect in the absence of proof of willful misstatement. Under
such circumstance, the proposal is to be regarded as the act of the insurer and not of the insured.
This question was considered in the case of the Union Insurance Company vs. Wilkinson (13 Wall.,
222; 20 L. ed., 617), in which the Supreme Court of the United States said:
This question has been decided differently by courts of the highest respectability in cases
precisely analogous to the present. It is not to be denied that the application logically
considered, is the work of the assured, and if left to himself or to such assistance as he might
select, the person so selected would be his agent, and he alone would be responsible. On
the other hand, it is well-known, so well that no court would be justified in shutting its eyes to
it, that insurance companies organized under the laws of one State, and having in that State
their principal business office, send these agents all over the land, with directions to solicit
and procure applications for policies furnishing them with printed arguments in favor of the
value and necessity of life insurance, and of the special advantages of the corporation which
the agent represents. They pay these agents large commissions on the premiums thus
obtained, and the policies are delivered at their hands to the assured. The agents are
stimulated by letters and instructions to activity in procuring contracts, and the party who is in
this manner induced to take out a policy, rarely sees or knows anything about the company
or its officers by whom it is issued, but looks to and relies upon the agent who has persuaded
him to effect insurance as the full and complete representative of the company, in all that is
said or done in making the contract. Has he not a right to so regard him? It is quite true that
the reports of judicial decisions are filled with the efforts of these companies, by their
counsel, to establish the doctrine for the acts of these agents to the simple receipt of the
premium and delivery of the policy, the argument being that, as to all other acts of the agent,
he is the agent of the assured. This proposition is not without support in some of the earlier
decision on the subject; and, at a time when insurance companies waited for parties to come
to them to seek assurance, or to forward applications on their own motion, the doctrine had a
reasonable foundation to rest upon. But to apply such a doctrine, in its full force, to the
system of selling policies through agents, which we have described, would be a snare and a
delusion, leading, as it has done in numerous instances, to the grossest frauds, of which the
insurance corporations receive the benefits, and the parties supposing themselves insured
are the victims. The tendency of the modern decisions in this country is steadily in the
opposite direction. The powers of the agent are, prima facie, co-extensive with the business
intrusted to his care, and will not be narrowed by limitations not communicated to the person
with whom he deals. (Bebee vs. Ins. Co., 25 Conn., 51; Lycoming Ins.
Co. vs. Schoolenberger, 44 Pa., 259; Bealvs. Ins. Co., 16 Wis., 241; Davenport vs. Ins. Co.,
17 Iowa, 276.) An insurance company, establishing a local agency, must be held responsible
to the parties with whom they transact business, for the acts and declarations of the agent,
within the scope of his employment, as if they proceeded from the principal. (Sav. Bk. vs. Ins.
Co., 31 Conn., 517; Hortwitz vs. Ins. Co., 40 Mo., 557; Ayres vs. Ins. Co., 17 Iowa, 176;
Howard Ins. Co. vs. Bruner, 23 Pa., 50.)
In the fifth edition of American Leading Cases, 917, after a full consideration of the
authorities, it is said:
"By the interested or officious zeal of the agents employed by the insurance
companies in the wish to outbid each other and procure customers, they not
unfrequently mislead the insured, by a false or erroneous statement of what the
application should contain; or, taking the preparation of it into their own hands,
procure his signature by an assurance that it is properly drawn, and will meet the
requirements of the policy. The better opinion seems to be that, when this course is
pursued, the description of the risk should, though nominally proceeding from the
insured, be regarded as the act of the insurers." (Rowley vs. Empire Ins. Co., 36
N.Y., 550.)
The modern decisions fully sustain this proposition, and they seem to us founded on reason
and justice, and meet our entire approval. This principle does not admit oral testimony to
vary or contradict that which is in writing, but it goes upon the idea that the writing offered in
evidence was not the instrument of the party whose name is signed to it; that it was procured
under such circumstances by the other side as estops that side from using it or relying on its
contents; not that it may be contradicted by oral testimony, but that it may be shown by such
testimony that it cannot be lawfully used against the party whose name is signed to it. (See
also Am. Life Ins. Co. vs. Mahone, 21 Wallace, 152.)
The defendant, upon the information given by plaintiff, and after an inspection of the automobile by
its examiner, having agreed that it was worth P3,000, is bound by this valuation in the absence of
fraud on the part of the insured. All statements of value are, of necessity, to a large extent matters of
opinion, and it would be outrageous to hold that the validity of all valued policies must depend upon
the absolute correctness of such estimated value. As was said by the Supreme Court of the United
States in the case of the First National Bank vs. Hartford Fire Insurance Co. (5 Otto, 673; 24 L. ed.,
563), at. p. 565 of the Lawyers Edition:
The ordinary test of the value of property is the price it will commend in the market if offered
for sale. But that test cannot, in the very nature of the case, be applied at the time application
is made for insurance. Men may honestly differ about the value of property, or as to what it
will bring in the market; and such differences are often very marked among those whose
special business it is to buy and sell property of all kinds. The assured could do no more than
estimate such value; and that, it seems, was all that he was required to do in this case. His
duty was to deal fairly with the Company in making such estimate. The special finding shows
that he discharged that duty and observed good faith. We shall not presume that the
Company, after requiring the assured in his application to give the "estimated value," and
then to covenant that he had stated all material facts in regard to such value, so far as known
to him, and after carrying that covenant, by express words, into the written contract, intended
to abandon the theory upon which it sought the contract, and make the absolute correctness
of such estimated value a condition precedent to any insurance whatever. The application,
with its covenant and stipulations, having been made a part of the policy, that presumption
cannot be indulged without imputing to the Company a purpose, by studied intricacy or an
ingenious framing of the policy, to entrap the assured into incurring obligations which,
perhaps, he had no thought of assuming.
Section 163 of the Insurance Law (Act No. 2427) provides that "the effect of a valuation in a policy of
fire insurance is the same as in a policy of marine insurance."
By the terms of section 149 of the Act cited, the valuation in a policy of marine insurance is
conclusive if the insured had an insurable interest and was not guilty of fraud.
We are, therefore, of the opinion and hold that plaintiff was the owner of the automobile in question
and had an insurable interest therein; that there was no fraud on her part in procuring the insurance;
that the valuation of the automobile, for the purposes of the insurance, is binding upon the defendant
corporation, and that the judgment of the court below is, therefore, correct and must be affirmed,
with interest, the costs of this appeal to be paid by the appellant. So ordered.
Arellano, C.J., Torres, Street, Malcolm and Avancea, JJ., concur.




Republic of the Philippines
SUPREME COURT
Manila
EN BANC
G.R. No. L-23002 July 31, 1967
CONCEPCION FELIX VDA. DE RODRIGUEZ, plaintiff-appellant,
vs.
GERONIMO RODRIGUEZ., ET AL., defendants-appellees.
Ozaeta, Gibbs and Ozaeta for plaintiff-appellant.
Sycip, Salazar, Luna and Associates and Carolina C. Grio-Aquino for defendants-appellees.
REYES, J.B.L., J .:
This is an appeal by Concepcion Felix Vda. de Rodriguez from the decision of the Court of First
Instance of Bulacan in Civil Case No. 2565, which she commenced on May 28, 1962, to secure
declaration, of nullity of two contracts executed on January 24, 1934 and for recovery of certain
properties.
The facts of this case may be briefly stated as follows:
Concepcion Felix, widow of the late Don Felipe Calderon and with whom she had one living child,
Concepcion Calderon, contracted a second marriage on June 20, 1929, with Domingo Rodriguez,
widower with four children by a previous marriage, named Geronimo, Esmeragdo, Jose and
Mauricio, all surnamed Rodriguez. There was no issue in this second marriage.
Prior to her marriage to Rodriguez, Concepcion Felix was the registered owner of 2 fishponds
located in the barrio of Babagad, municipality of Bulacan, Bulacan province. with a total area of
557,711 square meters covered by OCT Nos. 605 and 807. Under date of January 24, 1934,
Concepcion Felix appeared to have executed a deed of sale conveying ownership of the aforesaid
properties to her daughter, Concepcion Calderon, for the sum of P2,500.00, which the latter in turn
appeared to have transferred to her mother and stepfather by means of a document dated January
27, 1934. Both deeds, notarized by Notary Public Jose D. Mendoza, were registered in the office of
the Register of Deeds of Bulacan on January 29, 1934, as a consequence of which, the original titles
were cancelled and TCT Nos. 13815 and 13816 were issued in the names of the spouses Domingo
Rodriguez and Concepcion Felix.
On March 6, 1953, Domingo Rodriguez died intestate, survived by the widow, Concepcion Felix, his
children Geronimo Esmeragdo and Mauricio and grandchildren Oscar, Juan and Ana, surnamed
Rodriguez, children of a son, Jose, who had predeceased him.
On March 16, 1953, the above-named widow, children and grandchildren of the deceased entered
into an extra-judicial settlement of his (Domingo's) estate, consisting of one-half of the properties
allegedly belonging to the conjugal partnership. Among the properties listed as conjugal were the
two parcels of land in Bulacan, Bulacan, which, together with another piece of property, were divided
among the heirs in this manner:
WHEREAS, the parties have furthermore agreed that the fishpond covered by TCT Nos.
13815, 13816 and 24109 of the Office of the Register of Deeds of Bulacan, containing an
area of 557,971 sq. m., which is likewise the conjugal property of the deceased and his
surviving spouse; 1/2 of the same or 278,985.5 sq. m. belongs to said Concepcion Felix Vda.
de Rodriguez, as her share in the conjugal property; and 3/4 of the remaining half or
209,239.125 sq. m. are transferred in full ownership to Geronimo Rodriguez, Esmeragdo
Rodriguez and Mauricio Rodriguez, share and share alike, while the other 1/4 or 69,746.375
sq. m. of the said remaining half goes in equal shares to Oscar Rodriguez, Juan Rodriguez
and Ana Rodriguez.
As a result of this partition, TCT Nos. 13815 and 13816 were cancelled and TCT Nos. T-11431 and
T-14432 were issued in the names of the said heirs of the deceased.
On March 23, 1953, in a power of attorney executed by the children and grandchildren of Domingo
Rodriguez, Concepcion Felix Vda. de Rodriguez was named their attorney in-fact, authorized to
manage their shares in the fishponds (Exh. 4).
On July 2, 1954, the heirs ended their co-ownership by executing a deed of partition, dividing and
segregating their respective shares in the properties, pursuant to a consolidation and subdivision
plan (PCS-3702), in accordance with which, Concepcion Felix Vda. de Rodriguez obtained TCT No.
T-12910, for the portion pertaining to her (Exh. L), while TCT No. T-12911 was issued to the other
heirs, for their shares. This latter title was subsequently replaced by TCT No. 16660 (Exh. M).
On October 12, 1954, the Rodriguez children executed another document granting unto the widow
lifetime usufruct over one-third of the fishpond which they received as hereditary share in the estate
of Domingo Rodriguez, which grant was accepted by Concepcion Felix Vda. de Rodriguez.
Then, in a contract dated December 15, 1961, the widow appeared to have leased from the
Rodriguez children and grandchildren the fishpond (covered by TCT No. 16660) for a period of 5
years commencing August 16, 1962, for an annual rental of P7,161.37 (Exh. 5).1w ph1. t
At about this time, it seemed that the relationship between the widow and her stepchildren had
turned for the worse. Thus, when she failed to deliver to them the balance of the earnings of the
fishponds, in the amount of P3,000.00, her stepchildren endorsed the matter to their lawyer who, on
May 16, 1962, sent a letter of demand to the widow for payment thereof. On, May 28, 1962,
Concepcion Felix Vda. de Rodriguez filed the present action in the Court of First Instance of Manila
naming as defendants, Geronimo Rodriguez, Esmeragdo Rodriguez, Oscar Rodriguez, Concepcion
Bautista Vda. de Rodriguez, as guardian of the minors Juan and Ana Rodriguez, and Antonio Diaz
de Rivera and Renato Diaz de Rivera, as guardians of the minors Maria Ana, Mercedes, Margarita,
Mauricio, Jr. and Domingo (Children of Mauricio Rodriguez who had also died).
The action to declare null and void the deeds of transfer of plaintiff's properties to the conjugal
partnership was based on the alleged employment or exercise by plaintiff's deceased husband of
force and pressure on her; that the conveyances of the properties from plaintiff to her daughter
and then to the conjugal partnership of plaintiff and her husband are both without consideration;
that plaintiff participated in the extrajudicial settlement of estate (of the deceased Domingo
Rodriguez) and in other subsequent deeds or instruments involving the properties in dispute, on the
false assumption that the said properties had become conjugal by reason of the execution of the
deeds of transfer in 1934; that laboring under the same false assumption, plaintiff delivered to
defendants, as income of the properties from 1956 to 1961, the total amount of P56,976.58. As
alternative cause of action, she contended that she would claim for her share, as surviving widow, of
1/5 of the properties in controversy, should such properties be adjudged as belonging to the conjugal
partnership. Thus, plaintiff prayed that the deeds of transfer mentioned in the complaint be declared
fictitious and simulated; that the "Extrajudicial Settlement of Estate" be also declared null and void;
that TCT No. 16660 of the Registry of Deeds of Bulacan be cancelled and another one be issued in
the name of plaintiff, Concepcion Felix Vda. de Felix; that defendants be ordered to pay plaintiff the
sum of P56,976.58, with legal interest thereon from the date of the filing of the complaint, and for
appropriate relief in connection with her alternative cause of action.
In their separate answers, defendants not only denied the material allegations of the complaint, but
also set up as affirmative defenses lack of cause of action, prescription, estoppel and laches. As
counterclaim, they asked for payment by the plaintiff of the unpaid balance of the earnings of the
land up to August 15, 1962 in the sum of P3,000.00, for attorney's fees and expenses of litigation.
On October 5, 1963, judgment was rendered for the defendants. In upholding the validity of the
contracts, the court found that although the two documents, Exhibits A and B, were executed for the
purpose of converting plaintiff's separate properties into conjugal assets of the marriage with
Domingo Rodriguez, the consent of the parties thereto was voluntary, contrary to the allegations of
plaintiff and her witness. The court also ruled that having taken part in the questioned transactions,
plaintiff was not the proper party to plead lack of consideration to avoid the transfers; that contracts
without consideration are not inexistent, but are only voidable, following the ruling in the case
of Concepcion vs. Sta. Ana (87 Phil. 787); that there was ratification or confirmation by the plaintiff of
the transfer of her property, by her execution (with the other heirs) of the extrajudicial settlement of
estate; that being a voluntary party to the contracts, Exhibits A and B, plaintiff cannot recover the
properties she gave thereunder. Plaintiff's alternative cause of action was also rejected on the
ground that action for rescission of the deed of extrajudicial settlement should have been filed within
4 years from its execution (on March 16, 1953).
From the decision of the Court of First Instance, plaintiff duly appealed to this Court, insisting that the
conveyances in issue were obtained through duress, and were inexistent, being simulated and
without consideration.
We agree with the trial Court that the evidence is not convincing that the contracts of transfer from
Concepcion Felix to her daughter, and from the latter to her mother and stepfather were executed
through violence or intimidation. The charge is predicated solely upon the improbable and biased
testimony of appellant's daughter, Concepcion C. Martelino, whom the trial court, refused to believe,
considering that her version of violence and harassment was contradicted by Bartolome Gualberto
who had lived with the Rodriguez spouses from 1917 to 1953, and by the improbability of Rodriguez
threatening his stepdaughter in front of the Notary Public who ratified her signature. Furthermore, as
pointed out by the appealed decision, the charge of duress should be treated with caution
considering that Rodriguez had already died when the suit was brought, for duress, like fraud, is not
to be lightly paid at the door of men already dead. (Cf. Prevost vs. Gratz, 6 Wheat. [U.S.] 481, 498;
Sinco vs. Longa, 51 Phil. 507).
What is more decisive is that duress being merely a vice or defect of consent, an action based upon
it must be brought within four years after it has ceased;
1
and the present action was instituted only in
1962, twenty eight (28) years after the intimidation is claimed to have occurred, and no less than
nine (9) years after the supposed culprit died (1953). On top of it, appellant entered into a series of
subsequent transactions with appellees that confirmed the contracts that she now tries to set aside.
Therefore, this cause of action is clearly barred.
Appellant's main stand in attacking the conveyances in question is that they are simulated or
fictitious, and inexistent for lack of consideration. We shall examine each purported defect
separately.
The charge of simulation is untenable, for the characteristic of simulation is the fact that the apparent
contract is not really desired or intended to produce legal effects or in way alter the juridical situation
of the parties. Thus, where a person, in order to place his property beyond the reach of his creditors,
simulates a transfer of it to another, he does not really intend to divest himself of his title and control
of the property; hence, the deed of transfer is but a sham. But appellant contends that the sale by
her to her daughter, and the subsequent sale by the latter to appellant and her husband, the late
Domingo Rodriguez, were done for the purpose of converting the property from paraphernal to
conjugal, thereby vesting a half interest in Rodriguez, and evading the prohibition against donations
from one spouse to another during coverture (Civil Code of 1889, Art. 1334). If this is true, then the
appellant and her daughter must have intended the two conveyance to be real and effective; for
appellant could not intend to keep the ownership of the fishponds and at the same time vest half of
them in her husband. The two contracts of sale then could not have been simulated, but were real
and intended to be fully operative, being the means to achieve the result desired.
Nor does the intention of the parties to circumvent by these contracts the law against donations
between spouses make them simulated ones.
Ferrara, in his classic book, "La Simulacion de los Negocios Juridicos" (Sp. trans, 1926), pp. 95,
105, clearly explains the difference between simulated transactions and transactions in fraudem
legis:
Otra figura que debe distinguirse de la simulacion es el fraus legis. Tambien aqui se da una
gran confusion que persiste aun en la jurisprudencia, apegada tenazmente a antiguos
errores. Se debe a Bahr el haber defendido con vigor la antitesis teorica que existe entre
negocio fingido y negocio fraudulento y haber atacado la doctrina comun que hacia una
mescolanza con los dos conceptos.
Se confunde dice (2) , el negocio in fraudem legis con el negocio simulado; aunque la
naturaleza de ambos sea totalmente diversa. El negocio fraudulento no es, en absolute, un
negocio aparente. Es perfectamente serio: se quiere realmente. Es mas, se quiere tal como
se ha realizado, con todas las consecuencias que correspondent a la forma juridica elegida.
Muchas veces, estas consecuencias con incomodas para una u otra de las partes, aunque
serian mucho mas incomodas las consecuencias que lievaria consigo el acto prohibido.
x x x x x x x x x
El resultado de las precedentes investigaciones es el siguiente el negocio simulado quiere
producir una apariencia; el negocio fraudulente, una realidad; los negocios simulados son
ficticios, no queridos; los negocios in fraudem son serios, reales, y realizados en tal forma
por las partes para consequir un resultado prohibido: la simulacion nunca es un medio para
eludir la ley sino para ocultar su violation. La transgresion del contenido verbal e inmediato
de la norma se encubre bajo el manto de un negocio licito, lo cual no altera el caracter
del contra legem agere. Tan verdad es, que si se ha redactado una contra-escritura que
documentary y declara la verdadera naturaleza del negocio realizado, no queda mas que
aplicar pura y simplementela prohibicion.
Tambien el fraude quiere perjudicar la ley, pero emplea para ello medios diversos y sigue
distintos caminus. No oculta el acto exterior, sino que lo deja claro y visible, tratando de huir
sesgadamente de la aplicacion de la ley merced a una artistica y sabia combinacion de
varios medios juridicos no reprobados.
Appellant invokes our decision in Vasquez vs. Porta, 98 Phil. 490, but to no purpose. The mortgage
and foreclosure sale involved in that case were typical simulations merely apparent but not really
intended to produce legal effects, as approved by the Court's finding that the alleged creditor and
buyer at the foreclosure sale "Porta himself ostensibly acknowledged by his inertia in allowing the
doctor (alleged mortgagor debtor) to exercise dominical power thereon without any protest on his
part." (cas. cit., p. 495). Not only this, but the mortgagor's wife, when her husband died, "found
among his papers Porta's cancellation of the mortgage in his favor and the draft of the complaint for
foreclosure." Plainly, the precedent cited is here inapplicable.
Were the two conveyances from appellant to her daughter and from the latter to the spouses
Rodriguez void ab initio or inexistent for lack of consideration? We do not find them to be so. In the
first transaction, the price of P2,500.00 is recited in the deed itself (Exh. A); in the second (Exh. B),
the consideration set forth is P3,000.00. Now, Article 1274 of the Civil Code of 1889 (in force when
the deeds were executed) provided that
In onerous contracts the cause is understood to be, for each contracting party, the
prestation or promise of a thing or service by the other. (emphasis supplied.)
Since in each conveyance the buyer became obligated to pay a definite price in money, such
undertaking constituted in themselves actual causa or consideration for the conveyance of the
fishponds. That the prices were not paid (assuming ad arguendo that Concepcion Martelino's
testimony, to this effect is true) does not make the sales inexistent for want of causa. As ruled
in Enriquez de la Cavada vs. Diaz, 37 Phil. 982, "the consideration (causa) need not pass from one
(party) to the other at the time the contract is entered into x x x . The consideration need not be paid
at the time of the promise. The one promise is a consideration for the other."
What would invalidate the conveyances now under scrutiny is the fact that they were resorted to in
order to circumvent the legal prohibition against donations between spouses contained in Article
1334, paragraph 1, of the Civil Code of 1889, then prevailing. That illegal purpose tainted the
contracts, for as held by the Spanish Tribunal Supreme in its decision of 2 April 1941.
ha de ser reputado ineficaz, por exigencias includibles del caracter social y moral del
Derecho, todo contrato que persiga un fin ilicito o immoral, sea cualquiera el medio
empleado por los contratantes para lograr esa finalidad, no justificada por un interes digno
de ser socialmente protegido.
The illicit purpose then becomes illegal causa within the terms of the old Civil Code, for as declared
by the same Spanish Court in its decision of 14 December 1940
toda vez que lo que caracteriza fundamentalmente la ilicitud de la causa es la lesion de un
interos general juridica 6 moral.
a ruling reiterated in the decision of 2 April 1941 when the Court ruled:
El concepto de la causa ilicita, tal como la desenvuelve y aplica con gran amplitud y
flexibilidad la doctrina moderna, permite cobijar, no solo las convenciones ilicitas por razon
de su objeto o de su motivo ... sino tambien multiples convenciones que no encerrando en si
ningun elemento de directa antijuricidad son ilicitas por el matiz immoral que reviste la
operation en su conjunto x x x .
Unfortunately for herein appellant, in contracts invalidated by illegal subject matter or illegal causa,
Articles 1305 and 1306 of the Civil Code then in force apply rigorously the rule in pari delicto non
oritur action, denying all recovery to the guilty parties inter se. And appellant is clearly as guilty as
her husband in the attempt to evade the legal interdiction of Article 1334 of the Code, already cited.
Wherefore, her present action to reivindicate the, conveyed properties was correctly repulsed by the
Court below.
Art. 1306. If the act which constitutes the illicit consideration is neither a crime nor a
misdemeanor, the following rules shall be observed:
1. When both parties are guilty, neither of them can recover what he may have given by
virtue of the contract, or enforce the performance of the undertaking of the other party;
x x x x x x x x x
That Article 1306 applies to cases where the nullity arises from the illegality of the consideration or
the purpose of the contract was expressly recognized by this Supreme Court in Gustilo vs. Maravilla,
48 Phil. 449-450.
2

Finally, it cannot be denied that plaintiff-appellant had knowledge of the nullity of the contract for the
transfer of her properties in 1934, because she was even a party thereto. And yet, her present action
was filed only on May 28, 1962 and after the breaking up of friendly relations between her and
defendants-appellees. Appellant's inaction to enforce her right, for 28 years, cannot be justified by
the lame excuse that she assumed that the transfer was valid. Knowledge of the effect of that
transaction would have been obtained by the exercise of diligence. Ignorance which is the effect of
inexcusable negligence, it has been said, is no excuse for laches. (Go Chi Gun, etc., et al. vs. Co
Cho, et al., G.R. No. L-5208, Feb. 28, 1955). Even assuming for the sake of argument that appellant
held her peace, during the lifetime of her husband, out of legitimate fear for her life, there is no
justification for her future to bring the proper action after his death in 1953. Instead, she entered into
a series of agreements with herein appellees, the children of her husband by a prior marriage, of
partition, usufruct and lease of their share in the fishponds, transactions that necessarily assumed
that Rodriguez had acquired one-half of the litigated fishponds. In the circumstances, appellant's
cause has become a stale demand and her conduct placed her in estoppel to question the Validity of
the transfer of her properties. (Manila, et al. vs. Galvan, et al., G.R. No. L-23507, May 24, 1967;
Perez vs. Herranz, 7 Phil. 695-696).
In view of the foregoing, the decision appealed from is affirmed. Costs against appellant Concepcion
Felix Vda. de Rodriguez. So ordered.
Makalintal, Bengzon, J.P., Zaldivar, Sanchez, Castro, Angeles and Fernando, JJ., concur.
Concepcion, C.J. and Dizon, J., are on leave.
Footnotes
1
Article 1301 of the Civil Code of 1889, in force when the assailed contracts were executed
(1934).
2
See also Liguez vs. Court of Appeals, 102 Phil. 581582; Perez vs. Herranz, 7 Phil. 695.




SECOND DIVISION
[G.R. No. 118305. February 12, 1998]
AYALA INVESTMENT & DEVELOPMENT CORP. and ABELARDO MAGSAJO, petitioners,
vs. COURT OF APPEALS and SPOUSES ALFREDO & ENCARNACION
CHING, respondents.
D E C I S I O N
MARTINEZ, J .:
Under Article 161 of the Civil Code, what debts and obligations contracted by the husband alone are
considered for the benefit of the conjugal partnership which are chargeable against the conjugal
partnership? Is a surety agreement or an accommodation contract entered into by the husband in favor
of his employer within the contemplation of the said provision?
These are the issues which we will resolve in this petition for review.
The petitioner assails the decision dated April 14, 1994 of the respondent Court of Appeals in
Spouses Alfredo and Encarnacion Ching vs. Ayala Investment and Development Corporation,et. al.,
docketed as CA-G.R. CV No. 29632,
[1]
upholding the decision of the Regional Trial Court of Pasig,
Branch 168, which ruled that the conjugal partnership of gains of respondents-spouses Alfredo and
Encarnacion Ching is not liable for the payment of the debts secured by respondent-husband Alfredo
Ching.
A chronology of the essential antecedent facts is necessary for a clear understanding of the case at
bar.
Philippine Blooming Mills (hereinafter referred to as PBM) obtained a P50,300,000.00 loan from
petitioner Ayala Investment and Development Corporation (hereinafter referred to as AIDC). As added
security for the credit line extended to PBM, respondent Alfredo Ching, Executive Vice President of PBM,
executed security agreements on December 10, 1980 and on March 20, 1981 making himself jointly and
severally answerable with PBMs indebtedness to AIDC.
PBM failed to pay the loan. Thus, on July 30, 1981, AIDC filed a case for sum of money against
PBM and respondent-husband Alfredo Ching with the then Court of First Instance of Rizal (Pasig), Branch
VIII, entitled Ayala Investment and Development Corporation vs. Philippine Blooming Mills and Alfredo
Ching, docketed as Civil Case No. 42228.
After trial, the court rendered judgment ordering PBM and respondent-husband Alfredo Ching to
jointly and severally pay AIDC the principal amount of P50,300,000.00 with interests.
Pending appeal of the judgment in Civil Case No. 42228, upon motion of AIDC, the lower court
issued a writ of execution pending appeal. Upon AIDCs putting up of an P8,000,000.00 bond, a writ of
execution dated May 12, 1982 was issued. Thereafter, petitioner Abelardo Magsajo, Sr., Deputy Sheriff
of Rizal and appointed sheriff in Civil Case No. 42228, caused the issuance and service upon
respondents-spouses of a notice of sheriff sale dated May 20, 1982 on three (3) of their conjugal
properties. Petitioner Magsajo then scheduled the auction sale of the properties levied.
On June 9, 1982, private respondents filed a case of injunction against petitioners with the then
Court of First Instance of Rizal (Pasig), Branch XIII, to enjoin the auction sale alleging that petitioners
cannot enforce the judgment against the conjugal partnership levied on the ground that, among others,
the subject loan did not redound to the benefit of the said conjugal partnership.
[2]
Upon application of
private respondents, the lower court issued a temporary restraining order to prevent petitioner Magsajo
from proceeding with the enforcement of the writ of execution and with the sale of the said properties at
public auction.
AIDC filed a petition for certiorari before the Court of Appeals,
[3]
questioning the order of the lower
court enjoining the sale. Respondent Court of Appeals issued a Temporary Restraining Order on June
25, 1982, enjoining the lower court
[4]
from enforcing its Order of June 14, 1982, thus paving the way for
the scheduled auction sale of respondents-spouses conjugal properties.
On June 25, 1982, the auction sale took place. AIDC being the only bidder, was issued a Certificate
of Sale by petitioner Magsajo, which was registered on July 2, 1982. Upon expiration of the redemption
period, petitioner sheriff issued the final deed of sale on August 4, 1982 which was registered on August
9, 1983.
In the meantime, the respondent court, on August 4, 1982, decided CA-G.R. SP No. 14404, in this
manner:
WHEREFORE, the petition for certiorari in this case is granted and the challenged order of
the respondent Judge dated June 14, 1982 in Civil Case No. 46309 is hereby set aside and
nullified. The same petition insofar as it seeks to enjoin the respondent Judge from
proceeding with Civil Case No. 46309 is, however, denied. No pronouncement is here made
as to costs. x x x x.
[5]

On September 3, 1983, AIDC filed a motion to dismiss the petition for injunction filed before Branch
XIII of the CFI of Rizal (Pasig) on the ground that the same had become moot and academic with the
consummation of the sale. Respondents filed their opposition to the motion arguing, among others, that
where a third party who claims ownership of the property attached or levied upon, a different legal
situation is presented; and that in this case, two (2) of the real properties are actually in the name of
Encarnacion Ching, a non-party to Civil Case No. 42228.
The lower court denied the motion to dismiss. Hence, trial on the merits proceeded. Private
respondents presented several witnesses. On the other hand, petitioners did not present any evidence.
On September 18, 1991, the trial court promulgated its decision declaring the sale on execution null
and void. Petitioners appealed to the respondent court, which was docketed as CA-G.R. CV No. 29632.
On April 14, 1994, the respondent court promulgated the assailed decision, affirming the decision of
the regional trial court. It held that:
The loan procured from respondent-appellant AIDC was for the advancement and benefit of
Philippine Blooming Mills and not for the benefit of the conjugal partnership of petitioners-
appellees.
x x x x x x x x x
As to the applicable law, whether it is Article 161 of the New Civil Code or Article 1211 of the
Family Code-suffice it to say that the two provisions are substantially the same. Nevertheless,
We agree with the trial court that the Family Code is the applicable law on the matter x x x x x
x.
Article 121 of the Family Code provides that The conjugal partnership shall be liable for: x x x
(2) All debts and obligations contracted during the marriage by the designated Administrator-
Spouse for the benefit of the conjugal partnership of gains x x x. The burden of proof that the
debt was contracted for the benefit of the conjugal partnership of gains, lies with the creditor-
party litigant claiming as such. In the case at bar, respondent-appellant AIDC failed to prove
that the debt was contracted by appellee-husband, for the benefit of the conjugal partnership
of gains.
The dispositive portion of the decision reads:
WHEREFORE, in view of all the foregoing, judgment is hereby rendered DISMISSING the
appeal. The decision of the Regional Trial Court is AFFIRMED in toto.
[6]

Petitioner filed a Motion for Reconsideration which was denied by the respondent court in a
Resolution dated November 28, 1994.
[7]

Hence, this petition for review. Petitioner contends that the respondent court erred in ruling that the
conjugal partnership of private respondents is not liable for the obligation by the respondent-husband.
Specifically, the errors allegedly committed by the respondent court are as follows:
I. RESPONDENT COURT ERRED IN RULING THAT THE OBLIGATION INCURRED BY
RESPONDENT HUSBAND DID NOT REDOUND TO THE BENEFIT OF THE CONJUGAL
PARTNERSHIP OF THE PRIVATE RESPONDENT.
II RESPONDENT COURT ERRED IN RULING THAT THE ACT OF RESPONDENT
HUSBAND IN SECURING THE SUBJECT LOAN IS NOT PART OF HIS INDUSTRY,
BUSINESS OR CAREER FROM WHICH HE SUPPORTS HIS FAMILY.
Petitioners in their appeal point out that there is no need to prove that actual benefit redounded to
the benefit of the partnership; all that is necessary, they say, is that the transaction was entered into for
the benefit of the conjugal partnership. Thus, petitioners aver that:
The wordings of Article 161 of the Civil Code is very clear: for the partnership to be held
liable, the husband must have contracted the debt for the benefit of the partnership, thus:
Art. 161. The conjugal partnership shall be liable for:
1) all debts and obligations contracted by the husband for the benefit
of the conjugal partnership x x x.
There is a difference between the phrases: redounded to the benefit of or benefited from
(on the one hand) and for the benefit of (on the other). The former require that actual benefit
must have been realized; the latter requires only that the transaction should be one which
normally would produce benefit to the partnership, regardless of whether or not actual benefit
accrued.
[8]

We do not agree with petitioners that there is a difference between the terms redounded to the
benefit of or benefited from on the one hand; and for the benefit of on the other. They mean one and
the same thing. Article 161 (1) of the Civil Code and Article 121 (2) of the Family Code are similarly
worded, i.e., both use the term for the benefit of. On the other hand, Article 122 of the Family Code
provides that The payment of personal debts by the husband or the wife before or during the marriage
shall not be charged to the conjugal partnership except insofar as they redounded to the benefit of the
family. As can be seen, the terms are used interchangeably.
Petitioners further contend that the ruling of the respondent court runs counter to the pronouncement
of this Court in the case of Cobb-Perez vs. Lantin,
[9]
that the husband as head of the family and as
administrator of the conjugal partnership is presumed to have contracted obligations for the benefit of the
family or the conjugal partnership.
Contrary to the contention of the petitioners, the case of Cobb-Perez is not applicable in the case at
bar. This Court has, on several instances, interpreted the term for the benefit of the conjugal
partnership.
In the cases of Javier vs. Osmea,
[10]
Abella de Diaz vs. Erlanger & Galinger, Inc.,
[11]
Cobb-Perez vs.
Lantin
[12]
and G-Tractors, Inc. vs. Court of Appeals,
[13]
cited by the petitioners, we held that:
The debts contracted by the husband during the marriage relation, for and in the exercise of
the industry or profession by which he contributes toward the support of his family, are not his
personal and private debts, and the products or income from the wifes own property, which,
like those of her husbands, are liable for the payment of the marriage expenses, cannot be
excepted from the payment of such debts. (Javier)
The husband, as the manager of the partnership (Article 1412, Civil Code), has a right to
embark the partnership in an ordinary commercial enterprise for gain, and the fact that the
wife may not approve of a venture does not make it a private and personal one of the
husband. (Abella de Diaz)
Debts contracted by the husband for and in the exercise of the industry or profession by
which he contributes to the support of the family, cannot be deemed to be his exclusive and
private debts. (Cobb-Perez)
x x x if he incurs an indebtedness in the legitimate pursuit of his career or profession or
suffers losses in a legitimate business, the conjugal partnership must equally bear the
indebtedness and the losses, unless he deliberately acted to the prejudice of his family. (G-
Tractors)
However, in the cases of Ansaldo vs. Sheriff of Manila, Fidelity Insurance & Luzon Insurance
Co.,
[14]
Liberty Insurance Corporation vs. Banuelos,
[15]
and Luzon Surety Inc. vs. De Garcia,
[16]
cited by the
respondents, we ruled that:
The fruits of the paraphernal property which form part of the assets of the conjugal
partnership, are subject to the payment of the debts and expenses of the spouses, but not to
the payment of the personal obligations (guaranty agreements) of the husband, unless it be
proved that such obligations were productive of some benefit to the family. (Ansaldo;
parenthetical phrase ours.)
When there is no showing that the execution of an indemnity agreement by the husband
redounded to the benefit of his family, the undertaking is not a conjugal debt but an obligation
personal to him. (Liberty Insurance)
In the most categorical language, a conjugal partnership under Article 161 of the new Civil
Code is liable only for such debts and obligations contracted by the husband for the benefit of
the conjugal partnership. There must be the requisite showing then of some advantage
which clearly accrued to the welfare of the spouses. Certainly, to make a conjugal
partnership respond for a liability that should appertain to the husband alone is to defeat and
frustrate the avowed objective of the new Civil Code to show the utmost concern for the
solidarity and well-being of the family as a unit. The husband, therefore, is denied the power
to assume unnecessary and unwarranted risks to the financial stability of the conjugal
partnership. (Luzon Surety, Inc.)
From the foregoing jurisprudential rulings of this Court, we can derive the following conclusions:
(A) If the husband himself is the principal obligor in the contract, i.e., he directly received the money and
services to be used in or for his own business or his own profession, that contract falls within the term x x
x x obligations for the benefit of the conjugal partnership. Here, no actual benefit may be proved. It is
enough that the benefit to the family is apparent at the time of the signing of the contract. From the very
nature of the contract of loan or services, the family stands to benefit from the loan facility or services to
be rendered to the business or profession of the husband. It is immaterial, if in the end, his business or
profession fails or does not succeed. Simply stated, where the husband contracts obligations on behalf of
the family business, the law presumes, and rightly so, that such obligation will redound to the benefit of
the conjugal partnership.
(B) On the other hand, if the money or services are given to another person or entity, and the husband
acted only as a surety or guarantor, that contract cannot, by itself, alone be categorized as falling within
the context of obligations for the benefit of the conjugal partnership. The contract of loan or services is
clearly for the benefit of the principal debtor and not for the surety or his family. No presumption can be
inferred that, when a husband enters into a contract of surety or accommodation agreement, it is for the
benefit of the conjugal partnership. Proof must be presented to establish benefit redounding to the
conjugal partnership.
Thus, the distinction between the Cobb-Perez case, and we add, that of the three other companion
cases, on the one hand, and that of Ansaldo, Liberty Insurance and Luzon Surety, is that in the former,
the husband contracted the obligation for his own business; while in the latter, the husband merely acted
as a surety for the loan contracted by another for the latters business.
The evidence of petitioner indubitably show that co-respondent Alfredo Ching signed as surety for
the P50M loan contracted on behalf of PBM. Petitioner should have adduced evidence to prove that
Alfredo Chings acting as surety redounded to the benefit of the conjugal partnership. The reason for this
is as lucidly explained by the respondent court:
The loan procured from respondent-appellant AIDC was for the advancement and benefit of
Philippine Blooming Mills and not for the benefit of the conjugal partnership of petitioners-
appellees. Philippine Blooming Mills has a personality distinct and separate from the family
of petitioners-appellees - this despite the fact that the members of the said family happened
to be stockholders of said corporate entity.
x x x x x x x x x
x x x. The burden of proof that the debt was contracted for the benefit of the conjugal
partnership of gains, lies with the creditor-party litigant claiming as such. In the case at bar,
respondent-appellant AIDC failed to prove that the debt was contracted by appellee-husband,
for the benefit of the conjugal partnership of gains. What is apparent from the facts of the
case is that the judgment debt was contracted by or in the name of the Corporation Philippine
Blooming Mills and appellee-husband only signed as surety thereof. The debt is clearly a
corporate debt and respondent-appellants right of recourse against appellee-husband as
surety is only to the extent of his corporate stockholdings. It does not extend to the conjugal
partnership of gains of the family of petitioners-appellees. x x x x x x.
[17]

Petitioners contend that no actual benefit need accrue to the conjugal partnership. To support this
contention, they cite Justice J.B.L. Reyes authoritative opinion in the Luzon Surety Company case:
I concur in the result, but would like to make of record that, in my opinion, the words all debts
and obligations contracted by the husband for the benefit of the conjugal partnership used in
Article 161 of the Civil Code of the Philippines in describing the charges and obligations for
which the conjugal partnership is liable do not require that actual profit or benefit must accrue
to the conjugal partnership from the husbands transaction; but it suffices that the transaction
should be one that normally would produce such benefit for the partnership. This is the ratio
behind our ruling in Javier vs. Osmea, 34 Phil. 336, that obligations incurred by the husband
in the practice of his profession are collectible from the conjugal partnership.
The aforequoted concurring opinion agreed with the majority decision that the conjugal partnership
should not be made liable for the surety agreement which was clearly for the benefit of a third
party. Such opinion merely registered an exception to what may be construed as a sweeping statement
that in all cases actual profit or benefit must accrue to the conjugal partnership. The opinion merely made
it clear that no actual benefits to the family need be proved in some cases such as in the Javier
case. There, the husband was the principal obligor himself. Thus, said transaction was found to be one
that would normally produce x x x benefit for the partnership. In the later case of G-Tractors, Inc., the
husband was also the principal obligor - not merely the surety. This latter case, therefore, did not create
any precedent. It did not also supersede the Luzon Surety Company case, nor any of the previous
accommodation contract cases, where this Court ruled that they were for the benefit of third parties.
But it could be argued, as the petitioner suggests, that even in such kind of contract of
accommodation, a benefit for the family may also result, when the guarantee is in favor of the husbands
employer.
In the case at bar, petitioner claims that the benefits the respondent family would reasonably
anticipate were the following:
(a) The employment of co-respondent Alfredo Ching would be prolonged and he would be
entitled to his monthly salary of P20,000.00 for an extended length of time because of the
loan he guaranteed;
(b) The shares of stock of the members of his family would appreciate if the PBM could be
rehabilitated through the loan obtained;
(c) His prestige in the corporation would be enhanced and his career would be boosted
should PBM survive because of the loan.
However, these are not the benefits contemplated by Article 161 of the Civil Code. The benefits
must be one directly resulting from the loan. It cannot merely be a by-product or a spin-off of the loan
itself.
In all our decisions involving accommodation contracts of the husband,
[18]
we underscored the
requirement that: there must be the requisite showing x x x of some advantage which clearly accrued to
the welfare of the spouses or benefits to his family or that such obligations are productive of some
benefit to the family. Unfortunately, the petition did not present any proof to show: (a) Whether or not the
corporate existence of PBM was prolonged and for how many months or years; and/or (b) Whether or not
the PBM was saved by the loan and its shares of stock appreciated, if so, how much and how substantial
was the holdings of the Ching family.
Such benefits (prospects of longer employment and probable increase in the value of stocks) might
have been already apparent or could be anticipated at the time the accommodation agreement was
entered into. But would those benefits qualify the transaction as one of the obligations x x x for the
benefit of the conjugal partnership? Are indirect and remote probable benefits, the ones referred to in
Article 161 of the Civil Code? The Court of Appeals in denying the motion for reconsideration, disposed
of these questions in the following manner:
No matter how one looks at it, the debt/credit extended by respondents-appellants is purely a
corporate debt granted to PBM, with petitioner-appellee-husband merely signing as
surety. While such petitioner-appellee-husband, as such surety, is solidarily liable with the
principal debtor AIDC, such liability under the Civil Code provisions is specifically restricted by
Article 122 (par. 1) of the Family Code, so that debts for which the husband is liable may not
be charged against conjugal partnership properties. Article 122 of the Family Code is explicit
The payment of personal debts contracted by the husband or the wife before or during the
marriage shall not be charged to the conjugal partnership except insofar as they redounded to
the benefit of the family.
Respondents-appellants insist that the corporate debt in question falls under the exception
laid down in said Article 122 (par. one). We do not agree. The loan procured from
respondent-appellant AIDC was for the sole advancement and benefit of Philippine Blooming
Mills and not for the benefit of the conjugal partnership of petitioners-appellees.
x x x appellee-husband derives salaries, dividends benefits from Philippine Blooming Mills
(the debtor corporation), only because said husband is an employee of said PBM. These
salaries and benefits, are not the benefits contemplated by Articles 121 and 122 of the
Family Code. The benefits contemplated by the exception in Article 122 (Family Code) is
that benefit derived directly from the use of the loan. In the case at bar, the loan is a
corporate loan extended to PBM and used by PBM itself, not by petitioner-appellee-husband
or his family. The alleged benefit, if any, continuously harped by respondents-appellants, are
not only incidental but also speculative.
[19]

We agree with the respondent court. Indeed, considering the odds involved in guaranteeing a large
amount (P50,000,000.00) of loan, the probable prolongation of employment in PBM and increase in value
of its stocks, would be too small to qualify the transaction as one for the benefit of the suretys
family. Verily, no one could say, with a degree of certainty, that the said contract is even productive of
some benefits to the conjugal partnership.
We likewise agree with the respondent court (and this view is not contested by the petitioners) that
the provisions of the Family Code is applicable in this case. These provisions highlight the underlying
concern of the law for the conservation of the conjugal partnership; for the husbands duty to protect and
safeguard, if not augment, not to dissipate it.
This is the underlying reason why the Family Code clarifies that the obligations entered into by one
of the spouses must be those that redounded to the benefit of the family and that the measure of the
partnerships liability is to the extent that the family is benefited.
[20]

These are all in keeping with the spirit and intent of the other provisions of the Civil Code which
prohibits any of the spouses to donate or convey gratuitously any part of the conjugal property.
[21]
Thus,
when co-respondent Alfredo Ching entered into a surety agreement he, from then on, definitely put in
peril the conjugal property (in this case, including the family home) and placed it in danger of being taken
gratuitously as in cases of donation.
In the second assignment of error, the petitioner advances the view that acting as surety is part of
the business or profession of the respondent-husband.
This theory is new as it is novel.
The respondent court correctly observed that:
Signing as a surety is certainly not an exercise of an industry or profession, hence the cited
cases of Cobb-Perez vs. Lantin; Abella de Diaz vs. Erlanger & Galinger; G-Tractors, Inc. vs.
CA do not apply in the instant case. Signing as a surety is not embarking in a business.
[22]

We are likewise of the view that no matter how often an executive acted or was persuaded to act, as a
surety for his own employer, this should not be taken to mean that he had thereby embarked in the
business of suretyship or guaranty.
This is not to say, however, that we are unaware that executives are often asked to stand as surety
for their companys loan obligations. This is especially true if the corporate officials have sufficient
property of their own; otherwise, their spouses signatures are required in order to bind the conjugal
partnerships.
The fact that on several occasions the lending institutions did not require the signature of the wife
and the husband signed alone does not mean that being a surety became part of his profession. Neither
could he be presumed to have acted for the conjugal partnership.
Article 121, paragraph 3, of the Family Code is emphatic that the payment of personal debts
contracted by the husband or the wife before or during the marriage shall not be charged to the conjugal
partnership except to the extent that they redounded to the benefit of the family.
Here, the property in dispute also involves the family home. The loan is a corporate loan not a
personal one. Signing as a surety is certainly not an exercise of an industry or profession nor an act of
administration for the benefit of the family.
On the basis of the facts, the rules, the law and equity, the assailed decision should be upheld as we
now uphold it. This is, of course, without prejudice to petitioners right to enforce the obligation in its favor
against the PBM receiver in accordance with the rehabilitation program and payment schedule approved
or to be approved by the Securities & Exchange Commission.
WHEREFORE, the petition for review should be, as it is hereby, DENIED for lack of merit.
SO ORDERED.
Regalado, (Chairman), Melo, Puno, and Mendoza, JJ., concur.

Republic of the Philippines
SUPREME COURT
Manila
FIRST DIVISION
G.R. No. L-55322 February 16, 1989
MOISES JOCSON, petitioner,
vs.
HON. COURT OF APPEALS, AGUSTINA JOCSON-VASQUEZ, ERNESTO
VASQUEZ, respondents.
Dolorfino and Dominguez Law Officers for petitioner.
Gabriel G. Mascardo for private respondents.
MEDIALDEA, J .:
This is a petition for review on certiorari under Rule 45 of the Rules of Court of the decision of the
Court of Appeals in CA- G.R. No. 63474, promulgated on April 30, 1980, entitled "MOISES
JOCSON, plaintiff-appellee, versus AGUSTINA JOCSON-VASQUEZ and ERNESTO VASQUEZ,
defendant-appellants," upholding the validity of three (3) documents questioned by Moises Jocson,
in total reversal of the decision of the then Court of First Instance of Cavite, Branch I, which declared
them as null and void; and of its resolution, dated September 30, 1980, denying therein appellee's
motion for reconsideration.
Petitioner Moises Jocson and respondent Agustina Jocson-Vasquez are the only surviving offsprings
of the spouses Emilio Jocson and Alejandra Poblete, while respondent Ernesto Vasquez is the
husband of Agustina. Alejandra Poblete predeceased her husband without her intestate estate being
settled. Subsequently, Emilio Jocson also died intestate on April 1, 1972.
As adverted to above, the present controversy concerns the validity of three (3) documents executed
by Emilio Jocson during his lifetime. These documents purportedly conveyed, by sale, to Agustina
Jocson-Vasquez what apparently covers almost all of his properties, including his one-third (1/3)
share in the estate of his wife. Petitioner Moises Jocson assails these documents and prays that
they be declared null and void and the properties subject matter therein be partitioned between him
and Agustina as the only heirs of their deceased parents.
The documents, which were presented as evidence not by Moises Jocson, as the party assailing its
validity, but rather by herein respondents, are the following:
1) "Kasulatan ng Bilihan ng Lupa," marked as Exhibit 3 (pp. 12-13, Records) for the
defendant in the court a quo, dated July 27, 1968. By this document Emilio Jocson
sold to Agustina Jocson-Vasquez six (6) parcels of land, all located at Naic, Cavite,
for the sum of ten thousand P10,000.00 pesos. On the same document Emilio
Jocson acknowledged receipt of the purchase price, thus:
Na ngayon, alang-alang sa halagang SAMPUNG LIBONG PISO (P10,000) salaping
Pilipino na aking tinanggap ng buong kasiyahan loob at ang pagkakatanggap ay
aking hayagang inaamin sa pamamagitan ng kasulatang ito, sa aking anak na si
Agustina Jocson, na may sapat na gulang, mamamayang Pilipino, asawa ni Ernesto
Vasquez, at naninirahan sa Poblacion, Naic, Cavite, ay aking ipinagbile ng lubusan
at kagyat at walang ano mang pasubali ang nabanggit na anim na pirasong lupa na
nasa unang dahon ng dokumentong ito, sa nabanggit na Agustina Jocson, at sa
kaniyang tagapagmana o makakahalili at gayon din nais kong banggitin na kahit na
may kamurahan ang ginawa kong pagbibile ay dahilan sa ang nakabile ay aking
anak na sa akin at mapaglingkod, madamayin at ma-alalahanin, na tulad din ng isa
ko pang anak na lalaki. Ang kuartang tinanggap ko na P10,000.00, ay gagamitin ko
sa aking katandaan at mga huling araw at sa aking mga ibang mahahalagang
pangangailangan. [Emphasis supplied]
Na nais ko ring banggitin na ang ginawa kong ito ay hindi labag sa ano mang batas o
kautusan, sapagkat ang aking pinagbile ay akin at nasa aking pangalan. Ang mga
lupang nasa pangalan ng aking nasirang asawa ay hindi ko ginagalaw ni
pinakikialaman at iyon ay dapat na hatiin ng dalawa kong anak alinsunod sa umiiral
na batas (p. 13, Records.)
2) "Kasulatan ng Ganap na Bilihan,"dated July 27,1968, marked as Exhibit 4 (p. 14,
Records). On the face of this document, Emilio Jocson purportedly sold to Agustina
Jocson-Vasquez, for the sum of FIVE THOUSAND (P5,000.00) PESOS, two rice
mills and a camarin (camalig) located at Naic, Cavite. As in the first document,
Moises Jocson acknowledged receipt of the purchase price:
'Na alang-alang sa halagang LIMANG LIBONG PISO (P5,000.00) salaping Pilipino
na aking tinanggap ng buong kasiyahan loob sa aking anak na Agustina Jocson ....
Na ang halagang ibinayad sa akin ay may kamurahan ng kaunti ngunit dahil sa
malaking pagtingin ko sa kaniya ... kaya at pinagbile ko sa kaniya ang mga
nabanggit na pagaari kahit na hindi malaking halaga ... (p. 14, Records).
3) Lastly, the "Deed of Extrajudicial Partition and Adjudication with Sale, "dated
March 9, 1969, marked as Exhibit 2 (p. 10-11, Records), whereby Emilio Jocson and
Agustina Jocson-Vasquez, without the participation and intervention of Moises
Jocson, extrajudicially partitioned the unsettled estate of Alejandra Poblete, dividing
the same into three parts, one-third (1/3) each for the heirs of Alejandra Poblete,
namely: Emilio Jocson, Agustina Jocson-Vasquez and Moises Jocson. By the same
instrument, Emilio sold his one- third (1/3) share to Agustin for the sum of EIGHT
THOUSAND (P8,000.00) PESOS. As in the preceding documents, Emilio Jocson
acknowledged receipt of the purchase price:
Now for and in consideration of the sum of only eight thousand (P8,000.00) pesos,
which I, the herein Emilio Jocson had received from my daughter Agustina Jocson,
do hereby sell, cede, convey and transfer, unto the said Agustina Jocson, her heirs
and assigns, administrators and successors in interests, in the nature of absolute
and irrevocable sale, all my rights, interest, shares and participation, which is
equivalent to one third (1/3) share in the properties herein mentioned and described
the one third being adjudicated unto Agustina Jocson and the other third (1/3) portion
being the share of Moises Jocson. (p. 11, Records).
These documents were executed before a notary public. Exhibits 3 and 4 were registered with the
Office of the Register of Deeds of Cavite on July 29, 1968 and the transfer certificates of title
covering the properties therein in the name of Emilio Jocson, married to Alejandra Poblete," were
cancelled and new certificates of title were issued in the name of Agustina Jocson-Vasquez. Exhibit
2 was not registered with the Office of the Register of Deeds.
Herein petitioner filed his original complaint (Record on Appeal, p. 27, Rollo) on June 20,1973 with
the then Court of First Instance of Naic, Cavite (docketed as Civil Case No. TM- 531), and which
was twice amended. In his Second Amended Complaint (pp. 47-58, Record on Appeal), herein
petitioner assailed the above documents, as aforementioned, for being null and void.
It is necessary to partly quote the allegation of petitioner in his complaint for the reason that the
nature of his causes of action is at issue, thus:
8. [With regard the first document, that] the defendants, through fraud, deceit, undue
pressure and influence and other illegal machinations, were able to induce, led, and
procured their father ... to sign [the] contract of sale ..., for the simulated price of
P10,000.00, which is a consideration that is shocking to the conscience of ordinary
man and despite the fact that said defendants have no work or livelihood of their own
...; that the sale is null and void, also, because it is fictitious, simulated and fabricated
contract x x x (pp. 52-53, Record on Appeal). [Emphasis supplied]
xxx xxx xxx
12. [With regards the second and third document, that they] are null and void
because the consent of the father, Emilio Jocson, was obtained with fraud, deceit,
undue pressure, misrepresentation and unlawful machinations and trickeries
committed by the defendant on him; and that the said contracts are simulated,
fabricated and fictitious, having been made deliberately to exclude the plaintiff from
participating and with the dishonest and selfish motive on the part of the defendants
to defraud him of his legitimate share on said properties [subject matter thereof]; and
that without any other business or employment or any other source of income,
defendants who were just employed in the management and administration of the
business of their parents, would not have the sufficient and ample means to
purchase the said properties except by getting the earnings of the business or by
simulated consideration ... (pp. 54-55, Record on Appeal). [Emphasis supplied]
Petitioner explained that there could be no real sale between a father and daughter who are living
under the same roof, especially so when the father has no need of money as the properties
supposedly sold were all income-producing. Further, petitioner claimed that the properties
mentioned in Exhibits 3 and 4 are the unliquidated conjugal properties of Emilio Jocson and
Alejandra Poblete which the former, therefore, cannot validly sell (pp. 53, 57, Record on Appeal). As
far as Exhibit 2 is concerned, petitioner questions not the extrajudicial partition but only the sale by
his father to Agustina of the former's 1/3 share (p. 13, Rollo).
The trial court sustained the foregoing contentions of petitioner (pp. 59-81, Record on Appeal). It
declared that the considerations mentioned in the documents were merely simulated and fictitious
because: 1) there was no showing that Agustina Jocson-Vasquez paid for the properties; 2) the
prices were grossly inadequate which is tantamount to lack of consideration at all; and 3) the
improbability of the sale between Emilio Jocson and Agustina Jocson-Vasquez, taking into
consideration the circumstances obtaining between the parties; and that the real intention of the
parties were donations designed to exclude Moises Jocson from participating in the estate of his
parents. It further declared the properties mentioned in Exhibits 3 and 4 as conjugal properties of
Emilio Jocson and Alejandra Poblete, because they were registered in the name of "Emilio Jocson,
married to Alejandra Poblete" and ordered that the properties subject matter of all the documents be
registered in the name of herein petitioners and private respondents.
On appeal, the Court of Appeals in CA-G.R. No. 63474-R rendered a decision (pp. 29-42, Rollo) and
reversed that of the trial court's and ruled that:
1. That insofar as Exhibits 3 and 4 are concerned the appellee's complaint for
annulment, which is indisputably based on fraud, and undue influence, is now barred
by prescription, pursuant to the settled rule that an action for annulment of a contract
based on fraud must be filed within four (4) years, from the discovery of the fraud, ...
which in legal contemplation is deemed to be the date of the registration of said
document with the Register of Deeds ... and the records admittedly show that both
Exhibits 3 and 4, were all registered on July 29, 1968, while on the other hand, the
appellee's complaint was filed on June 20, 1973, clearly beyond the aforesaid four-
year prescriptive period provided by law;
2. That the aforesaid contracts, Exhibits 2, 3, and 4, are decisively not simulated or
fictitious contracts, since Emilio Jocson actually and really intended them to be
effective and binding against him, as to divest him of the full dominion and ownership
over the properties subject of said assailed contracts, as in fact all his titles over the
same were all cancelled and new ones issued to appellant Agustina Jocson-Vasquez
...;
3. That in regard to Exhibit 2, the same is valid and subsisting, and the partition with
sale therein made by and between Emilio Jocson and Agustina Jocson-Vasquez,
affecting the 2/3 portion of the subject properties described therein have all been
made in accordance with Article 996 of the New Civil Code on intestate succession,
and the appellee's (herein petitioner) remaining 1/3 has not been prejudiced (pp. 41-
42, Rollo).
In this petition for review, Moises Jocson raised the following assignments of errors:
1. HAS THE RESPONDENT COURT OF APPEALS ERRED IN CONCLUDING
THAT THE SUIT FOR THE ANNULMENT OF CONTRACTS FILED BY
PETITIONERS WITH THE TRIAL COURT IS "BASED ON FRAUD" AND NOT ON
ITS INEXISTENCE AND NULLITY BECAUSE OF IT'S BEING SIMULATED OR
FICTITIOUS OR WHOSE CAUSE IS CONTRARY TO LAW, MORALS AND GOOD
CUSTOMS?
II. HAS THE RESPONDENT COURT OF APPEALS ERRED IN CONCLUDING
THAT THE COMPLAINT FILED BY PETITIONER IN THE TRIAL COURT IS
BARRED BY PRESCRIPTION?
III. HAS THE RESPONDENT COURT OF APPEALS ERRED IN NOT DECLARING
AS INEXISTENT AND NULL AND VOID THE CONTRACTS IN QUESTION AND IN
REVERSING THE DECLARING DECISION OF THE TRIAL COURT? (p. 2, Rollo)
I.
The first and second assignments of errors are related and shall be jointly discussed.
According to the Court of Appeals, herein petitioner's causes of action were based on fraud. Under
Article 1330 of the Civil Code, a contract tainted by vitiated consent, as when consent was obtained
through fraud, is voidable; and the action for annulment must be brought within four years from the
time of the discovery of the fraud (Article 1391, par. 4, Civil Code), otherwise the contract may no
longer be contested. Under present jurisprudence, discovery of fraud is deemed to have taken place
at the time the convenant was registered with the Register of Deeds (Gerona vs. De Guzman, No. L-
19060, May 29,1964, 11 SCRA 153). Since Exhibits 3 and 4 were registered on July 29, 1968 but
Moises Jocson filed his complaint only on June 20, 1973, the Court of Appeals ruled that insofar as
these documents were concerned, petitioner's "annulment suit" had prescribed.
If fraud were the only ground relied upon by Moises Jocson in assailing the questioned documents,
We would have sustained the above pronouncement. But it is not so. As pointed out by petitioner, he
further assailed the deeds of conveyance on the ground that they were without consideration since
the amounts appearing thereon as paid were in fact merely simulated.
According to Article 1352 of the Civil Code, contracts without cause produce no effect whatsoever. A
contract of sale with a simulated price is void (Article 1471; also Article 1409 [3]]), and an action for
the declaration of its nullity does not prescribe (Article 1410, Civil Code; See also, Castillo v. Galvan,
No. L-27841, October 20, l978, 85 SCRA 526). Moises Jocsons saction, therefore, being for the
judicial declaration of nullity of Exhibits 3 and 4 on the ground of simulated price, is imprescriptible.
II.
For petitioner, however, the above discussion may be purely academic. The burden of proof in
showing that contracts lack consideration rests on he who alleged it. The degree of proof becomes
more stringent where the documents themselves show that the vendor acknowledged receipt of the
price, and more so where the documents were notarized, as in the case at bar. Upon consideration
of the records of this case, We are of the opinion that petitioner has not sufficiently proven that the
questioned documents are without consideration.
Firstly, Moises Jocson's claim that Agustina Jocson-Vasquez had no other source of income other
than what she derives from helping in the management of the family business (ricefields and
ricemills), and which was insufficient to pay for the purchase price, was contradicted by his own
witness, Isaac Bagnas, who testified that Agustina and her husband were engaged in the buy and
sell of palay and rice (p. 10, t.s.n., January 14, 1975). Amazingly, petitioner himself and his wife
testified that they did not know whether or not Agustina was involved in some other business (p. 40,
t.s.n., July 30, 1974; p. 36, t.s.n., May 24, 1974).
On the other hand, Agustina testified that she was engaged in the business of buying and selling
palay and rice even before her marriage to Ernesto Vasquez sometime in 1948 and continued doing
so thereafter (p. 4, t.s.n., March 15, 1976). Considering the foregoing and the presumption that a
contract is with a consideration (Article 1354, Civil Code), it is clear that petitioner miserably failed to
prove his allegation.
Secondly, neither may the contract be declared void because of alleged inadequacy of price. To
begin with, there was no showing that the prices were grossly inadequate. In fact, the total purchase
price paid by Agustina Jocson-Vasquez is above the total assessed value of the properties alleged
by petitioner. In his Second Amended Complaint, petitioner alleged that the total assessed value of
the properties mentioned in Exhibit 3 was P8,920; Exhibit 4, P3,500; and Exhibit 2, P 24,840, while
the purchase price paid was P10,000, P5,000, and P8,000, respectively, the latter for the 1/3 share
of Emilio Jocson from the paraphernal properties of his wife, Alejandra Poblete. And any difference
between the market value and the purchase price, which as admitted by Emilio Jocson was only
slight, may not be so shocking considering that the sales were effected by a father to her daughter in
which case filial love must be taken into consideration (Alsua-Betts vs. Court of Appeals, No. L-
46430-31, April 30, 1979, 92 SCRA 332).
Further, gross inadequacy of price alone does not affect a contract of sale, except that it may
indicate a defect in the consent, or that the parties really intended a donation or some other act or
contract (Article 1470, Civil Code) and there is nothing in the records at all to indicate any defect in
Emilio Jocson's consent.
Thirdly, any discussion as to the improbability of a sale between a father and his daughter is purely
speculative which has no relevance to a contract where all the essential requisites of consent, object
and cause are clearly present.
There is another ground relied upon by petitioner in assailing Exhibits 3 and 4, that the properties
subject matter therein are conjugal properties of Emilio Jocson and Alejandra Poblete. It is the
position of petitioner that since the properties sold to Agustina Jocson-Vasquez under Exhibit 3 were
registered in the name of "Emilio Jocson, married to Alejandra Poblete," the certificates of title he
presented as evidence (Exhibits "E', to "J', pp. 4-9, Records) were enough proof to show that the
properties covered therein were acquired during the marriage of their parents, and, therefore, under
Article 160 of the Civil Code, presumed to be conjugal properties.
Article 160 of the Civil Code provides that:
All property of the marriage is presumed to belong to the conjugal partnership, unless
it be proved that it pertains exclusively to the husband or to the wife.
In Cobb-Perez vs. Hon. Gregorio Lantin, No. L-22320, May 22, 1968, 23 SCRA 637, 644, We held
that:
Anent their claim that the shares in question are conjugal assets, the spouses Perez
adduced not a modicum of evidence, although they repeatedly invoked article 160 of
the New Civil Code which provides that ... . As interpreted by this Court, the party
who invokes this presumption must first prove that the property in controversy was
acquired during the marriage. In other words, proof of acquisition during the
coverture is a condition sine qua non for the operation of the presumption in favor of
conjugal ownership. Thus in Camia de Reyes vs. Reyes de Ilano [62 Phil. 629, 639],
it was held that "according to law and jurisprudence, it is sufficient to prove that the
Property was acquired during the marriage in order that the same may be deemed
conjugal property." In the recent case of Maramba vs. Lozano, et. al. [L-21533, June
29, 1967, 20 SCRA 474], this Court, thru Mr. Justice Makalintal, reiterated that "the
presumption under Article 160 of the Civil Code refers to property acquired during the
marriage," and then concluded that since "there is no showing as to when the
property in question was acquired...the fact that the title is in the wife's name alone is
determinative." Similarly, in the case at bar, since there is no evidence as to when
the shares of stock were acquired, the fact that they are registered in the name of the
husband alone is an indication that the shares belong exclusively to said spouse.'
This pronouncement was reiterated in the case of Ponce de Leon vs. Rehabilitation Finance
Corporation, No. L-24571, December 18, 1970, 36 SCRA 289, and later in Torela vs. Torela, No.
1,27843, October 11, 1979, 93 SCRA 391.
It is thus clear that before Moises Jocson may validly invoke the presumption under Article 160 he
must first present proof that the disputed properties were acquired during the marriage of Emilio
Jocson and Alejandra Poblete. The certificates of title, however, upon which petitioner rests his claim
is insufficient. The fact that the properties were registered in the name of "Emilio Jocson, married to
Alejandra Poblete" is no proof that the properties were acquired during the spouses' coverture.
Acquisition of title and registration thereof are two different acts. It is well settled that registration
does not confer title but merely confirms one already existing (See Torela vs. Torela, supra). It may
be that the properties under dispute were acquired by Emilio Jocson when he was still a bachelor
but were registered only after his marriage to Alejandra Poblete, which explains why he was
described in the certificates of title as married to the latter.
Contrary to petitioner's position, the certificates of title show, on their face, that the properties were
exclusively Emilio Jocson's, the registered owner. This is so because the words "married to'
preceding "Alejandra Poblete' are merely descriptive of the civil status of Emilio Jocson Litam v.
Rivera, 100 Phil. 354; Stuart v. Yatco, No. L-16467, April 27, 1962, 4 SCRA 1143; Magallon v.
Montejo, G.R. No. L-73733, December 16, 1986, 146 SCRA 282). In other words, the import from
the certificates of title is that Emilio Jocson is the owner of the properties, the same having been
registered in his name alone, and that he is married to Alejandra Poblete.
We are not unmindful that in numerous cases We consistently held that registration of the property in
the name of only one spouse does not negate the possibility of it being conjugal (See Bucoy vs.
Paulino, No. L-25775, April 26, 1968, 23 SCRA 248). But this ruling is not inconsistent with the
above pronouncement for in those cases there was proof that the properties, though registered in
the name of only one spouse, were indeed conjugal properties, or that they have been acquired
during the marriage of the spouses, and therefore, presumed conjugal, without the adverse party
having presented proof to rebut the presumption (See Mendoza vs- Reyes, No. L-31618, August 17,
1983, 124 SCRA 154).
In the instant case, had petitioner, Moises Jocson, presented sufficient proof to show that the
disputed properties were acquired during his parents' coverture. We would have ruled that the
properties, though registered in the name of Emilio Jocson alone, are conjugal properties in view of
the presumption under Article 160. There being no such proof, the condition sine qua non for the
application of the presumption does not exist. Necessarily, We rule that the properties under Exhibit
3 are the exclusive properties of Emilio Jocson.
There being no showing also that the camarin and the two ricemills, which are the subject of Exhibit
4, were conjugal properties of the spouses Emilio Jocson and Alejandra Poblete, they should be
considered, likewise, as the exclusive properties of Emilio Jocson, the burden of proof being on
petitioner.
ACCORDINGLY, the petition is DISMISSED and the decision of the Court of Appeals is AFFIRMED.
SO ORDERED.
Narvasa, Cruz, Gancayco and Grio-Aquino, JJ., concur.



Republic of the Philippines
SUPREME COURT
Manila
EN BANC
G.R. No. L-26001 October 29, 1968
PHILIPPINE NATIONAL BANK, petitioner,
vs.
THE COURT OF APPEALS and PHILIPPINE COMMERCIAL AND INDUSTRIAL
BANK, respondents.
Tomas Besa, Jose B. Galang and Juan C. Jimenez for petitioner.
San Juan, Africa & Benedicto for respondents.
CONCEPCION, C.J .:
The Philippine National Bank hereinafter referred to as the PNB seeks the review
by certiorari of a decision of the Court of Appeals, which affirmed that of the Court of First Instance
of Manila, dismissing plaintiff's complaint against the Philippine Commercial and Industrial Bank
hereinafter referred to as the PCIB for the recovery of P57,415.00.
A partial stipulation of facts entered into by the parties and the decision of the Court of Appeals show
that, on about January 15, 1962, one Augusto Lim deposited in his current account with the PCIB
branch at Padre Faura, Manila, GSIS Check No. 645915- B, in the sum of P57,415.00, drawn
against the PNB; that, following an established banking practice in the Philippines, the check was,
on the same date, forwarded, for clearing, through the Central Bank, to the PNB, which did not
return said check the next day, or at any other time, but retained it and paid its amount to the PCIB,
as well as debited it against the account of the GSIS in the PNB; that, subsequently, or on January
31, 1962, upon demand from the GSIS, said sum of P57,415.00 was re-credited to the latter's
account, for the reason that the signatures of its officers on the check were forged; and that,
thereupon, or on February 2, 1962, the PNB demanded from the PCIB the refund of said sum, which
the PCIB refused to do. Hence, the present action against the PCIB, which was dismissed by the
Court of First Instance of Manila, whose decision was, in turn, affirmed by the Court of Appeals.
It is not disputed that the signatures of the General Manager and the Auditor of the GSIS on the
check, as drawer thereof, are forged; that the person named in the check as its payee was one
Mariano D. Pulido, who purportedly indorsed it to one Manuel Go; that the check purports to have
been indorsed by Manuel Go to Augusto Lim, who, in turn, deposited it with the PCIB, on January
15, 1962; that, thereupon, the PCIB stamped the following on the back of the check: "All prior
indorsements and/or Lack of Endorsement Guaranteed, Philippine Commercial and Industrial Bank,"
Padre Faura Branch, Manila; that, on the same date, the PCIB sent the check to the PNB, for
clearance, through the Central Bank; and that, over two (2) months before, or on November 13,
1961, the GSIS had notified the PNB, which acknowledged receipt of the notice, that said check had
been lost, and, accordingly, requested that its payment be stopped.
In its brief, the PNB maintains that the lower court erred: (1) in not finding the PCIB guilty of
negligence; (2) in not finding that the indorsements at the back of the check are forged; (3) in not
finding the PCIB liable to the PNB by virtue of the former's warranty on the back of the check; (4) in
not holding that "clearing" is not "acceptance", in contemplation of the Negotiable Instruments law;
(5) in not finding that, since the check had not been accepted by the PNB, the latter is entitled to
reimbursement therefor; and (6) in denying the PNB's right to recover from the PCIB.
The first assignment of error will be discussed later, together with the last,with which it is interrelated.
As regards the second assignment of error, the PNB argues that, since the signatures of the drawer
are forged, so must the signatures of the supposed indorsers be; but this conclusion does not
necessarily follow from said premise. Besides, there is absolutely no evidence, and the PNB has not
even tried to prove that the aforementioned indorsements are spurious. Again, the PNB refunded the
amount of the check to the GSIS, on account of the forgery in the signatures, not of the indorsers or
supposed indorsers, but of the officers of the GSISas drawer of the instrument. In other words, the
question whether or not the indorsements have been falsified is immaterial to the PNB's liability as a
drawee, or to its right to recover from the PCIB,
1
for, as against the drawee, the indorsement of an
intermediate bank does not guarantee the signature of the drawer,
2
since the forgery of the
indorsement is not the cause of the loss.
3

With respect to the warranty on the back of the check, to which the third assignment of error refers, it
should be noted that the PCIB thereby guaranteed "all prior indorsements," not the authenticity of the
signatures of the officers of the GSIS who signed on its behalf, because the GSIS is not an indorser
of the check, but its drawer.
4
Said warranty is irrelevant, therefore, to the PNB's alleged right to
recover from the PCIB. It could have been availed of by a subsequent indorsee
5
or a holder in due
course
6
subsequent to the PCIB, but, the PNB is neither.
7
Indeed, upon payment by the PNB, as
drawee, the check ceased to be a negotiable instrument, and became a mere voucher or proof of
payment.
8

Referring to the fourth and fifth assignments of error, we must bear in mind that, in general,
"acceptance", in the sense in which this term is used in the Negotiable Instruments Law
9
is not
required for checks, for the same are payable on demand.
10
Indeed, "acceptance" and "payment"
are, within the purview of said Law, essentially different things, for the former is "a promise to
perform an act," whereas the latter is the "actual performance" thereof.
11
In the words of the
Law,
12
"the acceptance of a bill is the signification by the drawee of his assent to the order of the
drawer," which, in the case of checks, is the payment, on demand, of a given sum of money. Upon
the other hand, actual payment of the amount of a check implies not only an assent to said order of
the drawer and a recognition of the drawer's obligation to pay the aforementioned sum, but, also,
a compliance with such obligation.
Let us now consider the first and the last assignments of error. The PNB maintains that the lower
court erred in not finding that the PCIB had been guilty of negligence in not discovering that the
check was forged. Assuming that there had been such negligence on the part of the PCIB, it is
undeniable, however, that the PNB has, also, been negligent, with the particularity that the PNB had
been guilty of a greater degree of negligence, because it had a previous and formal notice from the
GSIS that the check had been lost, with the request that payment thereof be stopped. Just as
important, if not more important and decisive, is the fact that the PNB's negligence was the main or
proximate cause for the corresponding loss.
In this connection, it will be recalled that the PCIB did not cash the check upon its presentation by
Augusto Lim; that the latter had merely deposited it in his current account with the PCIB; that, on the
same day, the PCIB sent it, through the Central Bank, to the PNB, for clearing; that the PNB
did not return the check to the PCIB the next day or at any other time; that said failure to return the
check to the PCIB implied, under the current banking practice, that the PNB considered the check
good and would honor it; that, in fact, the PNB honored the check and paid its amount to the PCIB;
and that only then did the PCIB allow Augusto Lim to draw said amount from his aforementioned
current account.
Thus, by not returning the check to the PCIB, by thereby indicating that the PNB had found nothing
wrong with the check and would honor the same, and by actually paying its amount to the PCIB, the
PNB induced the latter, not only to believe that the check was genuine and good in every respect,
but, also, to pay its amount to Augusto Lim. In other words, the PNB was the primary or proximate
cause of the loss, and, hence, may not recover from the PCIB.
13

It is a well-settled maxim of law and equity that when one of two (2) innocent persons must suffer by
the wrongful act of a third person, the loss must be borne by the one whose negligence was the
proximate cause of the loss or who put it into the power of the third person to perpetrate the wrong.
14

Then, again, it has, likewise, been held that, where the collecting (PCIB) and the drawee (PNB)
banks are equally at fault, the court will leave the parties where it finds them.
15

Lastly, Section 62 of Act No. 2031 provides:
The acceptor by accepting the instrument engages that he will pay it according to the tenor
of his acceptance; and admits:
(a) The existence of the drawer, the genuineness of his signature, and his capacity and
authority to draw the instrument; and
(b) The existence of the payee and his then capacity to indorse.
The prevailing view is that the same rule applies in the case of a drawee who pays a bill without
having previously accepted it.
16

WHEREFORE, the decision appealed from is hereby affirmed, with costs against the Philippine
National Bank. It is so ordered.
Reyes, J.B.L., Dizon, Makalintal, Sanchez, Castro, Angeles, Fernando and Capistrano, JJ., concur.
Zaldivar, J., took no part.









Republic of the Philippines
SUPREME COURT
Manila
EN BANC
G.R. No. L-21533 June 29, 1967
HERMOGENES MARAMBA, plaintiff-appellant,
vs.
NIEVES DE LOZANO, ET AL., defendants-appellees.
N. Tanopo, Jr. and Millora for plaintiff-appellant.
Manuel Ancheta and Bausa, Ampil and Suarez for defendants-appellees.
MAKALINTAL., J .:
Appeal from an order of the Court of First Instance of Dagupan City in its Civil Case No. 10485,
dated June 28, 1961. This case was originally brought to the Court of Appeals, but subsequently
certified to Us on the ground that the issues raised are purely legal.
It appears that on November 3, 1948, the plaintiff filed an action against the defendant Nieves de
Lozano and her husband Pascual Lozano for the collection of a sum of money. After trial, the court a
quo on June 23, 1959 rendered its decision, the dispositive part of which is as follows:
WHEREFORE, the court hereby renders judgment, sentencing the defendants herein,
Nieves de Lozano and Pascual Lozano, to pay unto the herein plaintiff, Hermogenes
Maramba, the total sum of Three Thousand Five Hundred Pesos and Seven Centavos
(P3,500.07), with legal interest thereon from date of the filing of the instant complaint until
fully paid.
With costs against the said defendants.
Not satisfied with the judgment, the defendants interposed an appeal to the Court of Appeals but the
appeal was dismissed on March 30, 1960 for failure of the defendants to file their brief on time. After
the record the case was remanded to the court a quo, a writ of execution was issued, and on August
18, 1960 levy was made upon a parcel of land covered by transfer certificate title No. 8192 of
Pangasinan in the name of Nieves de Lozano. The notice of sale at public auction was published in
accordance with law and scheduled for September 16, 1960.
On that date, however, defendant Nieves de Lozano made a partial satisfaction of the judgment in
the amount P2,000.00, and requested for an adjournment of the sale to October 26, 1960. On
October 17, 1960, she filed amended motion, dated October 14, alleging that on November 11,
1952, during the pendency of the case, defendant Pascual Lozano died and that the property levied
upon was her paraphernal property, and praying that her liability be fixed at one-half () of the
amount awarded in the judgment and that pending the resolution of the issue an order be issued
restraining the Sheriff from carrying out the auction sale scheduled on October 26, 1960.
On that date the sale proceeded anyway, and the property of Nieves de Lozano which has been
levied upon was sold to the judgment creditor, as the highest bidder, for the amount of P4,175.12,
the balance of the judgment debt.1wp h1. t
On October 27, 1960, plaintiff filed an opposition to the defendant's amended motion dated October
14, 1960. And on June 28, 1961, the trial court issued the questioned order, the dispositive part of
which is as follows:
WHEREFORE, the court hereby grants the motion of counsel for defendant Nieves de
Lozano, dated October 5, 1960, which was amended on October 14, 1960, and holds that
the liability of the said defendant under the judgment of June 23, 1959, is only joint, or
P1,750.04, which is one-half () of the judgment debt of P3,500.07 awarded to the plaintiff
and that the writ of execution be accordingly modified in the sense that the liability of
defendant Nieves de Lozano be only P1,750.04 with legal interest from the date of the filing
of the complaint on November 5, 1948 until fully paid, plus the amount of P21.28 which is
also one-half () of the costs taxed by the Clerk of Court against the defendant spouses. Let
the auction sale of the above-mentioned property of defendant Nieves de Lozano proceed to
satisfy her liability of P1,750.04 with legal interest as above stated and the further sum of
P21.28 representing the costs, unless she voluntarily pays the same to the judgment creditor
(herein plaintiff).
Plaintiff interposed an appeal from the above-quoted order and assigned several errors, which
present three major issues, to wit:
(a) whether or not the decision of the lower court dated June 23, 1959 could still be
questioned;
(b) whether or not the judgment was joint or solidary; and
(c) whether or not the judgment debt could be satisfied from the proceeds of the properties
sold at public auction.
Plaintiff-appellant submits that a "nunc pro tunc" order should have been issued by the trial court
dismissing, as of November 11, 1952, the case against the late Pascual Lozano by reason of his
death, and that the lower court should have corrected its decision of June 23, 1959, by striking out
the letter "s" in the word "defendants" and deleting the words "and Pascual Lozano."
We do not think that the action suggested would be legally justified. It would entail a substantial
amendment of the decision of June 23, 1959, which has long become final and in fact partially
executed. A decision which has become final and executory can no longer be amended or corrected
by the court except for clerical errors or mistakes,
1
and however erroneous it may be, cannot be
disobeyed;
2
otherwise litigations would be endless and no questions could be considered finally
settled.
3
The amendment sought by appellee involves not merely clerical errors but the very
substance of the controversy. And it cannot be accomplished by the issuance of a "nunc pro tunc"
order such as that sought in this case. The purpose of an "nunc pro tunc" is to make a present
record of an which the court made at a previous term, but which not then recorded. It can only be
made when the ordered has previously been made, but by inadvertence not been entered. In the
instant case there was no order previously made by the court and therefore there is no now to be
recorded.
Now then, it is clear that the decision of June 23, 1959 does not specify the extent of the liability of
each defendant. The rule is that when the judgment does not order the defendants to pay jointly and
severally their liability is merely joint, and none of them may be compelled to satisfy the judgment in
full. This is in harmony with Articles 1137 and 1138 of the Civil Code.
Plaintiff-appellant contends that in any event the entire judgment debt can be satisfied from the
proceeds the property sold at public auction in view of the presumption that it is conjugal in character
although in the of only one of the spouses. The contention is incorrect. The presumption under
Article 160 of the Civil Code to property acquired during the marriage. But in the instant case there is
no showing as to when the property in question was acquired and hence the fact that the title is in
the wife's name alone is determinative. Furthermore, appellant himself admits in his brief (p. 17) that
the property in question is paraphernal.
Appellant next points out that even if the land levied upon were originally paraphernal, it became
conjugal property by virtue of the construction of a house thereon at the expense of the common
fund, pursuant to Article 158 paragraph 2 of the Civil Code. However, it has been by this Court that
the construction of a house at conjugal expense on the exclusive property of one of the spouses doe
not automatically make it conjugal. It is true that meantime the conjugal partnership may use both in
the land and the building, but it does so not as owner but in the exercise of the right of usufruct. The
ownership of the land remains the same until the value thereof is paid, and this payment can only be
demanded in the liquidation of the partnership (Coingco vs. Flores, 82 Phil. 284; Paterno vs. Bibby
Vda. de Padilla, 74 Phil. 377; Testate Estate of Narciso Padilla, G.R.No. L-8748, Dec. 26, 1961).
The record does not show that there has already been a liquidation of the conjugal partnership
between the late Pascual Lozano and Nieves de Lozano. Consequently, the property levied upon,
being the separate property of defendant Nieves de Lozano, cannot be made to answer for the
liability of the other defendant.
On May 18, 1967 counsel for defendants-appellees filed with Us a petition alleging, inter alia; that
prior to the expiration of the redemption period and pursuant to an order of the lower court
defendants filed a surety bond in the amount of P3,175.12 as the redemption price, which bond was
duly approved by the lower court; that sometime last September 1966, defendants filed a petition
before the lower court praying that the sheriff of Pangasinan be ordered to execute the
corresponding deed of redemption in favor of defendant Nieves de Lozano represented by her
judicial administrator or that, in the alternative, the Register of Deeds of Dagupan City be directed to
cancel Entries Nos. 19234 and 20042 at the back of TCT No. 8192; and that said petition was
denied by the lower court. The same prayer made below is reiterated in the said petition of May 18,
1967.
The foregoing petition of May 18, 1967 alleges facts which occurred after the perfection of the
present appeal and which should therefore be submitted to and passed upon by the trial court in
connection with the implementation of the order appealed from, which is hereby affirmed, with costs.
Concepcion, C.J., Reyes, J.B.L., Dizon, Bengzon, J.P., Zaldivar, Sanchez and Castro, JJ., concur.







Republic of the Philippines
SUPREME COURT
Manila
EN BANC

G.R. No. L-28394 November 26, 1970
PEDRO GAYON, plaintiff-appellant,
vs.
SILVESTRE GAYON and GENOVEVA DE GAYON, defendants-appellees.
German M. Lopez for plaintiff-appellant.
Pedro R. Davila for defendants-appellees.

CONCEPCION, C.J .:
Appeal, taken by plaintiff Pedro Gayon, from an order of the Court of First Instance of Iloilo
dismissing his complaint in Civil Case No. 7334 thereof.
The records show that on July 31, 1967, Pedro Gayon filed said complaint against the spouses
Silvestre Gayon and Genoveva de Gayon, alleging substantially that, on October 1, 1952, said
spouses executed a deed copy of which was attached to the complaint, as Annex "A" whereby
they sold to Pedro Gelera, for the sum of P500.00, a parcel of unregistered land therein described,
and located in the barrio of Cabubugan, municipality of Guimbal, province of Iloilo, including the
improvements thereon, subject to redemption within five (5) years or not later than October 1, 1957;
that said right of redemption had not been exercised by Silvestre Gayon, Genoveva de Gayon, or
any of their heirs or successors, despite the expiration of the period therefor; that said Pedro Gelera
and his wife Estelita Damaso had, by virtue of a deed of sale copy of which was attached to the
complaint, as Annex "B" dated March 21, 1961, sold the aforementioned land to plaintiff Pedro
Gayon for the sum of P614.00; that plaintiff had, since 1961, introduced thereon improvements worth
P1,000; that he had, moreover, fully paid the taxes on said property up to 1967; and that Articles
1606 and 1616 of our Civil Code require a judicial decree for the consolidation of the title in and to a
land acquired through a conditional sale, and, accordingly, praying that an order be issued in
plaintiff's favor for the consolidation of ownership in and to the aforementioned property.
In her answer to the complaint, Mrs. Gayon alleged that her husband, Silvestre Gayon, died on
January 6, 1954, long before the institution of this case; that Annex "A" to the complaint is fictitious,
for the signature thereon purporting to be her signature is not hers; that neither she nor her
deceased husband had ever executed "any document of whatever nature in plaintiff's favor"; that the
complaint is malicious and had embarrassed her and her children; that the heirs of Silvestre Gayon
had to "employ the services of counsel for a fee of P500.00 and incurred expenses of at least
P200.00"; and that being a brother of the deceased Silvestre Gayon, plaintiff "did not exert efforts for
the amicable settlement of the case" before filing his complaint. She prayed, therefore, that the same
be dismissed and that plaintiff be sentenced to pay damages.
Soon later, she filed a motion to dismiss, reproducing substantially the averments made in her
answer and stressing that, in view of the death of Silvestre Gayon, there is a "necessity of amending
the complaint to suit the genuine facts on record." Presently, or on September 16, 1967, the lower
court issued the order appealed from, reading:
Considering the motion to dismiss and it appearing from Exhibit "A" annexed to the
complaint that Silvestre Gayon is the absolute owner of the land in question, and
considering the fact that Silvestre Gayon is now dead and his wife Genoveva de Gayon
has nothing to do with the land subject of plaintiff's complaint, as prayed for, this case is
hereby dismissed, without pronouncement as to costs.
1

A reconsideration of this order having been denied, plaintiff interposed the present appeal, which is
well taken.
Said order is manifestly erroneous and must be set aside. To begin with, it is not true that Mrs.
Gayon "has nothing to do with the land subject of plaintiff's complaint." As the widow of Silvestre
Gayon, she is one of his compulsory heirs
2
and has, accordingly, an interest in the property in
question. Moreover, her own motion to dismiss indicated merely "a necessity of amending the
complaint," to the end that the other successors in interest of Silvestre Gayon, instead of the latter,
be made parties in this case. In her opposition to the aforesaid motion for reconsideration of the
plaintiff, Mrs. Gayon alleged, inter alia, that the "heirs cannot represent the dead defendant, unless
there is a declaration of heirship." Inasmuch, however, as succession takes place, by operation of
law, "from the moment of the death of the decedent"
3
and "(t)he inheritance includes all the property,
rights and obligations of a person which are not extinguished by his death,"
4
it follows that if his heirs
were included as defendants in this case, they would be sued, not as "representatives" of the
decedent, but as owners of an aliquot interest in the property in question, even if the precise extent
of their interest may still be undetermined and they have derived it from the decent. Hence, they may
be sued without a previous declaration of heirship, provided there is no pending special proceeding
for the settlement of the estate of the decedent.
5

As regards plaintiff's failure to seek a compromise, as an alleged obstacle to the present case, Art.
222 of our Civil Code provides:
No suit shall be filed or maintained between members of the same family unless it
should appear that earnest efforts toward a compromise have been made, but that
the same have failed, subject to the limitations in article 2035.
It is noteworthy that the impediment arising from this provision applies to suits "filed or maintained
between members of the same family." This phrase, "members of the same family," should,
however, be construed in the light of Art. 217 of the same Code, pursuant to which:
Family relations shall include those:
(1) Between husband and wife;
(2) Between parent and child;
(3) Among other ascendants and their descendants;
(4) Among brothers and sisters.
Mrs. Gayon is plaintiff's sister-in-law, whereas her children are his nephews and/or nieces. Inasmuch
as none of them is included in the enumeration contained in said Art. 217 which should be
construed strictly, it being an exception to the general rule and Silvestre Gayon must necessarily
be excluded as party in the case at bar, it follows that the same does not come within the purview of
Art. 222, and plaintiff's failure to seek a compromise before filing the complaint does not bar the
same.
WHEREFORE, the order appealed from is hereby set aside and the case remanded to the lower
court for the inclusion, as defendant or defendants therein, of the administrator or executor of the
estate of Silvestre Gayon, if any, in lieu of the decedent, or, in the absence of such administrator or
executor, of the heirs of the deceased Silvestre Gayon, and for further proceedings, not inconsistent
with this decision, with the costs of this instance against defendant-appellee, Genoveva de Gayon. It
is so ordered.
Reyes, J.B.L., Makalintal, Zaldivar, Castro, Fernando, Teehankee, Barredo and Villamor, JJ.,
concur.
Dizon and Makasiar, JJ., are on leave.


















Republic of the Philippines
SUPREME COURT
Manila
FIRST DIVISION
G.R. No. 181258 March 18, 2010
BEN-HUR NEPOMUCENO, Petitioner,
vs.
ARHBENCEL ANN LOPEZ, represented by her mother ARACELI LOPEZ, Respondent.
D E C I S I O N
CARPIO MORALES, J .:
Respondent Arhbencel Ann Lopez (Arhbencel), represented by her mother Araceli Lopez (Araceli),
filed a Complaint
1
with the Regional Trial Court (RTC) of Caloocan City for recognition and support
against Ben-Hur Nepomuceno (petitioner).
Born on June 8, 1999, Arhbencel claimed to have been begotten out of an extramarital affair of
petitioner with Araceli; that petitioner refused to affix his signature on her Certificate of Birth; and
that, by a handwritten note dated August 7, 1999, petitioner nevertheless obligated himself to give
her financial support in the amount ofP1,500 on the 15th and 30th days of each month beginning
August 15, 1999.
Arguing that her filiation to petitioner was established by the handwritten note, Arhbencel prayed that
petitioner be ordered to: (1) recognize her as his child, (2) give her support pendente lite in the
increased amount of P8,000 a month, and (3) give her adequate monthly financial support until she
reaches the age of majority.
Petitioner countered that Araceli had not proven that he was the father of Arhbencel; and that he
was only forced to execute the handwritten note on account of threats coming from the National
Peoples Army.
2

By Order of July 4, 2001,
3
Branch 130 of the Caloocan RTC, on the basis of petitioners handwritten
note which it treated as "contractual support" since the issue of Arhbencels filiation had yet to be
determined during the hearing on the merits, granted Arhbencels prayer for support pendente lite in
the amount of P3,000 a month.
After Arhbencel rested her case, petitioner filed a demurrer to evidence which the trial court granted
by Order dated June 7, 2006,
4
whereupon the case was dismissed for insufficiency of evidence.
The trial court held that, among other things, Arhbencels Certificate of Birth was not prima facie
evidence of her filiation to petitioner as it did not bear petitioners signature; that petitioners
handwritten undertaking to provide support did not contain a categorical acknowledgment that
Arhbencel is his child; and that there was no showing that petitioner performed any overt act of
acknowledgment of Arhbencel as his illegitimate child after the execution of the note.
On appeal by Arhbencel, the Court of Appeals, by Decision of July 20, 2007,
5
reversed the trial
courts decision, declared Arhbencel to be petitioners illegitimate daughter and accordingly ordered
petitioner to give Arhbencel financial support in the increased amount of P4,000 every 15th and 30th
days of the month, or a total of P8,000 a month.
The appellate court found that from petitioners payment of Aracelis hospital bills when she gave
birth to Arhbencel and his subsequent commitment to provide monthly financial support, the only
logical conclusion to be drawn was that he was Arhbencels father; that petitioner merely acted in
bad faith in omitting a statement of paternity in his handwritten undertaking to provide financial
support; and that the amount of P8,000 a month was reasonable for Arhbencels subsistence and
not burdensome for petitioner in view of his income.
His Motion for Reconsideration having been denied by Resolution dated January 3, 2008,
6
petitioner
comes before this Court through the present Petition for Review on Certiorari.
7

Petitioner contends that nowhere in the documentary evidence presented by Araceli is an explicit
statement made by him that he is the father of Arhbencel; that absent recognition or
acknowledgment, illegitimate children are not entitled to support from the putative parent; that the
supposed payment made by him of Aracelis hospital bills was neither alleged in the complaint nor
proven during the trial; and that Arhbencels claim of paternity and filiation was not established by
clear and convincing evidence.
Arhbencel avers in her Comment that petitioner raises questions of fact which the appellate court
had already addressed, along with the issues raised in the present petition.
8

The petition is impressed with merit.
The relevant provisions of the Family Code
9
that treat of the right to support are Articles 194 to 196,
thus:
Article 194. Support compromises everything indispensable for sustenance, dwelling, clothing,
medical attendance, education and transportation, in keeping with the financial capacity of the
family.1awp h!1
The education of the person entitled to be supported referred to in the preceding paragraph shall
include his schooling or training for some profession, trade or vocation, even beyond the age of
majority. Transportation shall include expenses in going to and from school, or to and from place of
work.
Article 195. Subject to the provisions of the succeeding articles, the following are obliged to support
each other to the whole extent set forth in the preceding article:
1. The spouses;
2. Legitimate ascendants and descendants;
3. Parents and their legitimate children and the legitimate and illegitimate children of the
latter;
4. Parents and their illegitimate children and the legitimate and illegitimate children of the
latter; and
5. Legitimate brothers and sisters, whether of the full or half-blood.
Article 196. Brothers and sisters not legitimately related, whether of the full or half-blood, are likewise
bound to support each other to the full extent set forth in Article 194, except only when the need for
support of the brother or sister, being of age, is due to a cause imputable to the claimant's fault or
negligence. (emphasis and underscoring supplied)
Arhbencels demand for support, being based on her claim of filiation to petitioner as his illegitimate
daughter, falls under Article 195(4). As such, her entitlement to support from petitioner is dependent
on the determination of her filiation.
Herrera v. Alba
10
summarizes the laws, rules, and jurisprudence on establishing filiation, discoursing
in relevant part as follows:
Laws, Rules, and Jurisprudence
Establishing Filiation
The relevant provisions of the Family Code provide as follows:
ART. 175. Illegitimate children may establish their illegitimate filiation in the same way and on the
same evidence as legitimate children.
x x x x
ART. 172. The filiation of legitimate children is established by any of the following:
(1) The record of birth appearing in the civil register or a final judgment; or
(2) An admission of legitimate filiation in a public document or a private handwritten
instrument and signed by the parent concerned.
In the absence of the foregoing evidence, the legitimate filiation shall be proved by:
(1) The open and continuous possession of the status of a legitimate child; or
(2) Any other means allowed by the Rules of Court and special laws.
The Rules on Evidence include provisions on pedigree. The relevant sections of Rule 130 provide:
SEC. 39. Act or declaration about pedigree. The act or declaration of a person deceased, or
unable to testify, in respect to the pedigree of another person related to him by birth or marriage,
may be received in evidence where it occurred before the controversy, and the relationship between
the two persons is shown by evidence other than such act or declaration. The word "pedigree"
includes relationship, family genealogy, birth, marriage, death, the dates when and the places where
these facts occurred, and the names of the relatives. It embraces also facts of family history
intimately connected with pedigree.
SEC. 40. Family reputation or tradition regarding pedigree. The reputation or tradition existing in a
family previous to the controversy, in respect to the pedigree of any one of its members, may be
received in evidence if the witness testifying thereon be also a member of the family, either by
consanguinity or affinity. Entries in family bibles or other family books or charts, engraving on rings,
family portraits and the like, may be received as evidence of pedigree.
This Court's rulings further specify what incriminating acts are acceptable as evidence to establish
filiation. In Pe Lim v. CA, a case petitioner often cites, we stated that the issue of paternity still has to
be resolved by suchconventional evidence as the relevant incriminating verbal and written acts by
the putative father. Under Article 278 of the New Civil Code, voluntary recognition by a parent shall
be made in the record of birth, a will, a statement before a court of record, or in any authentic writing.
To be effective, the claim of filiation must be made by the putative father himself and the writing must
be the writing of the putative father. A notarial agreement to support a child whose filiation is
admitted by the putative father was considered acceptable evidence. Letters to the mother vowing to
be a good father to the child and pictures of the putative father cuddling the child on various
occasions, together with the certificate of live birth, proved filiation. However, a student permanent
record, a written consent to a father's operation, or a marriage contract where the putative father
gave consent, cannot be taken as authentic writing. Standing alone, neither a certificate of baptism
nor family pictures are sufficient to establish filiation. (emphasis and underscoring supplied)
In the present case, Arhbencel relies, in the main, on the handwritten note executed by petitioner
which reads:
Manila, Aug. 7, 1999
I, Ben-Hur C. Nepomuceno, hereby undertake to give and provide financial support in the amount
of P1,500.00 every fifteen and thirtieth day of each month for a total of P3,000.00 a month starting
Aug. 15, 1999, to Ahrbencel Ann Lopez, presently in the custody of her mother Araceli Lopez without
the necessity of demand, subject to adjustment later depending on the needs of the child and my
income.
The abovequoted note does not contain any statement whatsoever about Arhbencels filiation to
petitioner. It is, therefore, not within the ambit of Article 172(2) vis--vis Article 175 of the Family
Code which admits as competent evidence of illegitimate filiation an admission of filiation in a private
handwritten instrument signed by the parent concerned.
The note cannot also be accorded the same weight as the notarial agreement to support the child
referred to in Herrera. For it is not even notarized. And Herrera instructs that the notarial agreement
must be accompanied by the putative fathers admission of filiation to be an acceptable evidence of
filiation. Here, however, not only has petitioner not admitted filiation through contemporaneous
actions. He has consistently denied it.
The only other documentary evidence submitted by Arhbencel, a copy of her Certificate of
Birth,
11
has no probative value to establish filiation to petitioner, the latter not having signed the
same.
At bottom, all that Arhbencel really has is petitioners handwritten undertaking to provide financial
support to her which, without more, fails to establish her claim of filiation. The Court is mindful that
the best interests of the child in cases involving paternity and filiation should be advanced. It is,
however, just as mindful of the disturbance that unfounded paternity suits cause to the privacy and
peace of the putative fathers legitimate family.
WHEREFORE, the petition is GRANTED. The Court of Appeals Decision of July 20, 2007 is SET
ASIDE. The Order dated June 7, 2006 of Branch 130 of the Caloocan City RTC dismissing the
complaint for insufficiency of evidence is REINSTATED.
SO ORDERED.CONCHITA CARPIO MORALES
Associate Justice

FIRST DIVISION
EUGENIO R. REYES, joined by TIMOTHY JOSEPH M. REYES, MA. GRACIA
S. REYES, ROMAN GABRIEL M. REYES, and MA. ANGELA S.
REYES,
Petitioners,



-versus-


LIBRADA F. MAURICIO (deceased) and LEONIDA F. MAURICIO,
Respondents.
G.R. No. 175080

Present:
CORONA, C.J.,
Chairperso
n
VELASCO, JR.,
LEONARDO-DE
CASTRO,
PERALTA,* and
PEREZ, JJ.


Promulgated:

November 24,
2010
x---------------------------------------------------------------------------------------- x

D E C I S I O N

PEREZ, J .:

Subject of this petition is the Decision
[1]
of the Court of Appeals dated 10 August 2006 in CA-G.R.
SP No. 87148, affirming the Decision dated 7 July 1998 and Resolution dated 28 September 2004 of the
Department of Agrarian Reform Adjudication Board (DARAB).
Eugenio Reyes (Eugenio) was the registered owner of a parcel of land located at Turo, Bocaue,
Bulacan, with an area of four thousand five hundred twenty-seven (4,527) square meters, more or less,
and covered by Transfer Certificate of Title (TCT) No. 109456(M). Said title came from and cancelled
TCT No. T-62290 registered in the name of Eufracia and Susana Reyes, siblings of Eugenio. The subject
property was adjudicated to Eugenio by virtue of an extrajudicial settlement among the heirs following the
death of his parents.

The controversy stemmed from a complaint filed before the DARAB of Malolos, Bulacan by
respondents Librada F. Mauricio (Librada), now deceased, and her alleged daughter Leonida F. Mauricio
(Leonida) for annulment of contract denominated as Kasunduan and between Librada and Eugenio as
parties. Respondents also prayed for maintenance of their peaceful possession with damages.

Respondents alleged that they are the legal heirs of the late Godofredo Mauricio (Godofredo),
who was the lawful and registered tenant of Eugenio through his predecessors-in-interest to the subject
land; that from 1936 until his death in May 1994, Godofredo had been working on the subject land and
introduced improvements consisting of fruit-bearing trees, seasonal crops, a residential house and other
permanent improvements; that through fraud, deceit, strategy and other unlawful means, Eugenio caused
the preparation of a document denominated as Kasunduan dated 28 September 1994 to eject
respondents from the subject property, and had the same notarized by Notary Public Ma. Sarah G.
Nicolas in Pasig, Metro Manila; that Librada never appeared before the Notary Public; that Librada was
illiterate and the contents of the Kasunduan were not read nor explained to her; that Eugenio took undue
advantage of the weakness, age, illiteracy, ignorance, indigence and other handicaps of Librada in the
execution of the Kasunduanrendering it void for lack of consent; and that Eugenio had been employing all
illegal means to eject respondents from the subject property. Respondents prayed for the declaration of
nullity of the Kasunduan and for an order for Eugenio to maintain and place them in peaceful possession
and cultivation of the subject property. Respondents likewise demanded payment of damages.
[2]
During
trial, respondents presented a leasehold contract executed between Susana and Godofredo to reaffirm
the existing tenancy agreement.
[3]


Eugenio averred that no tenancy relationship existed between him and respondents. He clarified
that Godofredos occupation of the subject premises was based on the formers mere tolerance and
accommodation. Eugenio denied signing a tenancy agreement, nor authorizing any person to sign such
an agreement. He maintained that Librada, accompanied by a relative, voluntarily affixed her signature to
the Kasunduan and that she was fully aware of the contents of the document. Moreover, Librada
receivedP50,000.00 from Eugenio on the same day of the execution of the Kasunduan. Eugenio also
questioned the jurisdiction of the DARAB since the principal relief sought by respondents is the
annulment of the contract, over which jurisdiction is vested on the regular courts. Eugenio also asserted
that Leonida had no legal personality to file the present suit.
[4]


Based on the evidence submitted by both parties, the Provincial
Adjudicator
[5]
concluded that Godofredo was the tenant of Eugenio, and Librada, being the surviving
spouse, should be maintained in peaceful possession of the subject land. The dispositive portion of the
decision reads:

WHEREFORE, in view of the foregoing, judgment is hereby rendered in favor of
plaintiff Librada Mauricio and against defendant Eugenio R. Reyes and order is hereby
issued:

1. Declaring the kasunduan null and void;
2. Ordering defendant to respect the peaceful possession of herein plaintiff
Librada Mauricio over the subject landholding;
3. Ordering plaintiff to return the amount of P50,000.00 to herein defendant;
4. No pronouncement as to costs.
[6]


On appeal, two issues were presented to and taken up by the DARAB, namely: (1) Whether or
not there is tenancy relation between the parties; and (2) whether or not theKasunduan dated 28
September 1994 is valid and enforceable. The DARAB held that the Mauricios are former tenants of
Spouses Reyes. It found that when Spouses Reyes died, siblings Eufracia, Susana and Eugenio, among
others inherited the subject property. Under the law, they were subrogated to the rights and substituted
to the obligations of their late parents as the agricultural lessors over the farmholding tenanted by
respondents. Moreover, the DARAB banked on the Kasunduang Buwisan sa Sakahan or the leasehold
contract executed by Susana in favor of Godofredo to support the tenancy relationship. Furthermore, the
DARAB declared the other Kasunduan as void by relying on the evaluation of the Provincial Adjudicator
as to the legal incapacity of Librada to enter into such a contract.
[7]


Eugenio filed a motion for reconsideration which was denied by the DARAB on 28 September
2004.
[8]


Aggrieved by the DARAB ruling, Eugenio filed a petition for review with the Court of
Appeals. On 10 July 2006, the Court of Appeals issued a resolution regarding the status of Leonida as a
legal heir and allowed her to substitute Librada, who died during the pendency of the case.
[9]
On 10
August 2006, the Court of Appeals affirmed the decision and resolution of the DARAB. It sustained the
factual findings of the DARAB with respect to the tenancy relation between Godofredo and Spouses
Reyes and the nullity of the Kasunduan.
[10]


Undaunted, Eugenio filed the instant petition. Eugenio submits that no tenancy relationship exists
between him and respondents. He insists that the Kasunduang Buwisan sa Sakahan allegedly executed
between Godofredo and Susana in 1993 giving the former the right to occupy and cultivate the subject
property is unenforceable against Eugenio, having been entered into without his knowledge and
consent. Eugenio further asserts that per records of the Department of Agrarian Reform (DAR), no
leasehold contract was entered into by Godofredo and Eugenio with respect to the disputed
property. Eugenio attributes error on the part of the Court of Appeals in concluding that a tenancy
relationship existed between the parties despite the absence of some of the essential requisites of a
tenancy relationship such as personal cultivation and the subject land being agricultural. Finally, Eugenio
defends the validity of the Kasunduan entered into between him and Librada wherein the latter agreed to
vacate the subject property, in that it was voluntarily entered into and the contents thereof were mutually
understood by the parties.
[11]


In a Resolution dated 7 February 2007, this Court denied the petition for failure to show that the
Court of Appeals committed reversible error in its challenged decision and resolution. The Court also
dismissed the issues raised as factual. However, upon filing of a motion for reconsideration by Eugenio,
this Court reinstated the petition and required respondent Leonida to comment on the petition.
[12]


In her comment, respondent prayed for the denial of the petition because the jurisdiction of this
Court is limited to review of errors of law and not of facts.
[13]


In the main, Eugenio insists that no tenancy relationship existed between him and Godofredo. This
is a question of fact beyond the province of this Court in a petition for review under Rule 45 of the Rules
of Court in which only questions of law may be raised.
[14]
Absent any of the obtaining exceptions
[15]
to this
rule, the findings of facts of the Provincial Adjudicator, as affirmed by DARAB and especially by the Court
of Appeals, are binding on this Court.

The DARAB ruling outlined how the tenancy relationship between Godofredo and the Mauricios
came about, thus:

This Board, after a thorough evaluation of the evidences, is convinced that the
Mauricios are former tenants of the parents of the herein Defendant-Appeallant. A
perusal of Exhibit H which is the Tax Declaration of the property in controversy proves
that upon the death of the parents of Defendant-Appellant, the property was the subject
matter of their extra-judicial partition/settlement and this property was initially under the
ownership of the appellants sisters, Eufracia and Susana Reyes until the same property
was finally acquired/transferred in the name of Respondent-Appellant. Obviously, in
order to re-affirm the fact that the Mauricios are really the tenants, Susana Reyes had
voluntarily executed the Leasehold Contract with Godofredo Librada being the tenant on
the property and to prove that she (Susana Reyes) was the predecessor-in-interest of
Respondent-Appeallant Eugenio Reyes. x x x. The Kasunduang Buwisan sa Sakahan
alleging that their tenancy relationship began in the year 1973 and their agreement as to
the rental shall remain until further revised.
[16]


This is a contest of Kasunduans. Respondents rely on a Kasunduan of tenancy. Petitioners
swear by a Kasunduan of termination of tenancy.

Librada claims that her late husband had been working on the land since 1936 until his death in
1994. She presented the Kasunduang Buwisan sa Sakahan dated 26 May 1993 and executed by
Godofredo and Susana which reaffirmed the leasehold tenancy over the subject land. On the other hand,
Eugenio disputes the claims of Librada and presented another Kasunduan executed between him and
Librada on 28 September 1994 which effectively terminates the leasehold tenancy when the latter
allegedly agreed to vacate the subject premises in exchange of monetary considerations.

This second Kasunduan is the subject of the instant complaint. In its disquisition, the DARAB
nullified the second Kasunduan, to wit:

x x x Insofar as this Kasunduan is concerned, and after reading the transcript of the
testimony of the old woman Librada Mauricio, this Board is convinced that indeed the
purpose of the document was to eject her from the farmholding but that Librada Mauricio
wanted to return the money she received because the contents of the document was
never explained to her being illiterate who cannot even read or write. This Board is even
further convinced after reading the transcript of the testimonies that while the document
was allegedly signed by the parties in Turo, Bocaue, Bulacan, the same document was
notarized in Pasig, Metro Manila, thus, the Notary Public was not in a position to explain
much less ascertain the veracity of the contents of the alleged Kasunduan as to
whether or not Plaintiff-Appellee Librada Mauricio had really understood the contents
thereof. This Board further adheres to the principle that it cannot substitute its own
evaluation of the testimony of the witnesses with that of the personal evaluation of the
Adjudicator a quo who, in the case at bar, had the best opportunity to observe the
demeanor of the witness Librada Mauricio while testifying on the circumstances relevant
to the execution of the alleged Kasunduan. Furthermore, this Board adheres to the
principle that in all contractual, property or other relations, when one of the parties is at a
disadvantage on account of his moral dependence, ignorance, mental weakness or other
handicap, the courts (and in the case at bar, this Board) must be vigilant for his protection
(Art. 24, New Civil Code). In the case at bar, Plaintiff-Appellee is already eighty-one (81)
years old who can neither read nor write, thus, she just simply signs her name with her
thumbmark.
[17]


Applying the principle that only questions of law may be entertained by this Court, we defer to the
factual ruling of the Provincial Adjudicator, as affirmed by DARAB and the Court of Appeals, which clearly
had the opportunity to closely examine the witnesses and their demeanor on the witness stand.

Assuming that the leasehold contract between Susana and Godofredo is void, our conclusion
remains. We agree with the Court of Appeals that a tenancy relationship cannot be extinguished by mere
expiration of term or period in a leasehold contract; or by the sale, alienation or the transfer of legal
possession of the landholding. Section 9 ofRepublic Act No. 1199 or the Agricultural Tenancy Act
provides:

SECTION 9. Severance of Relationship. The tenancy relationship is
extinguished by the voluntary surrender of the land by, or the death or incapacity of, the
tenant, but his heirs or the members of his immediate farm household may continue to
work the land until the close of the agricultural year. The expiration of the period of the
contract as fixed by the parties, and the sale or alienation of the land does not of
themselves extinguish the relationship. In the latter case, the purchaser
or transferee shall assume the rights and obligations of the former landholder in
relation to the tenant. In case of death of the landholder, his heir or heirs shall likewise
assume his rights and obligations. (Emphasis supplied)

Moreover, Section 10 of Republic Act No. 3844 (Code of Agrarian Reforms of the Philippines)
likewise provides:

SEC. 10. Agricultural Leasehold Relation Not Extinguished by Expiration of
Period, etc. The agricultural leasehold relation under this Code shall not be
extinguished by mere expiration of the term or period in a leasehold contract nor by the
sale, alienation or transfer of the legal possession of the landholding. In case the
agricultural lessor sells, alienates or transfers the legal possession of the
landholding, the purchaser or transferee thereof shall be subrogated to the rights
and substituted to the obligations of the agricultural lessor. (Emphasis supplied)

As an incidental issue, Leonidas legal standing as a party was also assailed by
Eugenio. Eugenio submitted that the complaint was rendered moot with the death of Librada,
Godofredos sole compulsory heir. Eugenio contended that Leonida is a mere ward of Godofredo and
Librada, thus, not a legal heir.
[18]


We are in full accord with the Court of Appeals when it ruled that Eugenio cannot collaterally
attack the status of Leonida in the instant petition.
[19]


It is settled law that filiation cannot be collaterally attacked.
[20]
Well-known civilista Dr. Arturo M.
Tolentino, in his book Civil Code of the Philippines, Commentaries and Jurisprudence, noted that the
aforecited doctrine is rooted from the provisions of the Civil Code of the Philippines. He explained thus:
The legitimacy of the child cannot be contested by way of defense or as a collateral
issue in another action for a different purpose. The necessity of an independent action
directly impugning the legitimacy is more clearly expressed in the Mexican code (article
335) which provides: The contest of the legitimacy of a child by the husband or his heirs
must be made by proper complaint before the competent court; any contest made in any
other way is void. This principle applies under our Family Code. Articles 170 and 171 of
the code confirm this view, because they refer to the action to impugn the legitimacy.
This action can be brought only by the husband or his heirs and within the periods fixed
in the present articles.
[21]

In Braza v. City Civil Registrar of Himamaylan City, Negros Occidental,
[22]
the Court stated that
legitimacy and filiation can be questioned only in a direct action seasonably filed by the proper party, and
not through collateral attack.
[23]

The same rule is applied to adoption such that it cannot also be made subject to a collateral
attack. In Reyes v. Sotero,
[24]
this Court reiterated that adoption cannot be assailed collaterally in a
proceeding for the settlement of a decedents estate.
[25]
Furthermore, in Austria v. Reyes,
[26]
the Court
declared that the legality of the adoption by the testatrix can be assailed only in a separate action brought
for that purpose and cannot be subject to collateral attack.
[27]

Against these jurisprudential backdrop, we have to leave out the status of Leonida from the case
for annulment of the Kasunduan that supposedly favors petitioners cause.
WHEREFORE, based on the foregoing premises, the instant petition for review
on certiorari is DENIED and the Decision dated 10 August 2006 of the Court of Appeals in CA-G.R. SP
No. 87148 is AFFIRMED.


SO ORDERED.


JOSE PORTUGAL PEREZ
Associate Justice
Republic of the Philippines
SUPREME COURT
Manila
SECOND DIVISION

G.R. No. 121027 July 31, 1997
CORAZON DEZOLLER TISON and RENE R. DEZOLLER, petitioners,
vs.
COURT OF APPEALS and TEODORA DOMINGO, respondents.

REGALADO, J .:
The present appeal by certiorari seeks the reversal of the judgment rendered by respondent Court of
Appeals on June 30, 1995
1
which affirmed the Order of December 3, 1992 issued by the Regional
Trial Court of Quezon City, Branch 98, granting herein private respondent's Demurrer to Plaintiff's
Evidence filed in Civil Case No. Q-88-1054 pending therein.
The present appellate review involves an action for reconveyance filed by herein petitioners against
herein private respondent before the Regional Trial Court of Quezon City, Branch 98, docketed as
the aforesaid Civil Case No. Q-88-1054, over a parcel of land with a house and apartment thereon
located at San Francisco del Monte, Quezon City and which was originally owned by the spouses
Martin Guerrero and Teodora Dezoller Guerrero. It appears that petitioners Corazon Tison and Rene
Dezoller are the niece and nephew, respectively, of the deceased Teodora Dezoller Guerrero who is
the sister of petitioner's father, Hermogenes Dezoller. Teodora Dezoller Guerrero died on March 5,
1983 without any ascendant or descendant, and was survived only by her husband, Martin Guerrero,
and herein petitioners. Petitioners' father, Hermogenes, died on October 3, 1973, hence they seek to
inherit from Teodora Dezoller Guerrero by right of representation.
The records reveal that upon the death of Teodora Dezoller Guerrero, her surviving spouse, Martin,
executed on September 15, 1986 an Affidavit of Extrajudicial Settlement
2
adjudicating unto himself,
allegedly as sole heir, the land in dispute which is covered by Transfer Certificate of Title No. 66886,
as a consequence of which Transfer Certificate of Title No. 358074 was issued in the name of Martin
Guerrero. On January 2, 1988, Martin Guerrero sold the lot to herein private respondent Teodora
Domingo and thereafter, Transfer Certificate of Title No. 374012 was issued in the latter's name.
Martin Guerrero died on October 25, 1988. Subsequently, herein petitioners filed an action for
reconveyance on November 2, 1988, claiming that they are entitled to inherit one-half of the property
in question by right of representation.
At the pre-trial conference, the following issues were presented by both parties for resolution:
(1) whether or not the plaintiffs (herein petitioners) are the nephew and niece of the
late Teodora Dezoller;
(2) whether or not the plaintiffs are entitled to inherit by right of representation from
the estate of the late Teodora Dezoller;
(3) whether or not defendant (herein private respondent) must reconvey the reserved
participation of the plaintiffs to the estate of the late Teodora Dezoller under Section
4, Rule 74 of the Rules of Court which was duly annotated on the title of the
defendant;
(4) whether or not the plaintiffs are entitled to damages, moral and exemplary, plus
attorney's fees for the willful and malicious refusal of defendant to reconvey the
participation of plaintiffs in the estate of Teodora Dezoller, despite demands and
knowing fully well that plaintiffs are the niece and nephew of said deceased; and
(5) whether or not the subject property now in litigation can be considered as conjugal
property of the spouses Martin Guerrero and Teodora Dezoller Guerrero.
3

During the hearing, petitioner Corazon Dezoller Tison was presented as the lone witness, with the
following documentary evidence offered to prove petitioners' filiation to their father and their aunt, to
wit: a family picture; baptismal certificates of Teodora and Hermogenes Dezoller; certificates of
destroyed records of birth of Teodora Dezoller and Hermogenes Dezoller; death certificates of
Hermogenes Dezoller and Teodora Dezoller Guerrero; certification of destroyed records of live birth
of Corazon and Rene Dezoller; joint affidavits of Pablo Verzosa and Meliton Sitjar attesting to the
parents, date and place of birth of Corazon and Rene Dezoller; joint affidavit of Juliana Cariaga and
Manuela Cariaga attesting to the fact of marriage between Martin Guerrero and Teodora Dezoller;
and the marriage certificate of Martin and Teodora Guerrero.
4
Petitioners thereafter rested their case
and submitted a written offer of these exhibits to which a Comment
5
was filed by herein private
respondent.
Subsequently, private respondent filed a Demurrer to Plaintiff's Evidence on the ground that
petitioners failed to prove their legitimate filiation with the deceased Teodora Guerrero in accordance
with Article 172 of the Family Code. It is further averred that the testimony of petitioner Corazon
Dezoller Tison regarding her relationship with her alleged father and aunt is self-serving,
uncorroborated and incompetent, and that it falls short of the quantum of proof required under Article
172 of the Family Code to establish filiation. Also, the certification issued by the Office of the Local
Civil Registrar of Himamaylan, Negros Occidental is merely proof of the alleged destruction of the
records referred to therein, and the joint affidavit executed by Pablo Verzosa and Meliton Sitjar
certifying to the date, place of birth and parentage of herein petitioners is inadmissible for being
hearsay since the affiants were never presented for cross-examination.
6

On December 3, 1992, the trial court issued an order granting the demurrer to evidence and
dismissing the complaint for reconveyance.
7

In upholding the dismissal, respondent Court of Appeals declared that the documentary evidence
presented by herein petitioners, such as the baptismal certificates, family picture, and joint affidavits
are all inadmissible and insufficient to prove and establish filiation. Hence, this appeal.
We find for petitioners.
The bone of contention in private respondent's demurrer to evidence is whether or not herein
petitioners failed to meet the quantum of proof required by Article 172 of the Family Code to
establish legitimacy and filiation. There are two points for consideration before us: first is the issue
on petitioner's legitimacy, and second is the question regarding their filiation with Teodora Dezoller
Guerrero.
I. It is not debatable that the documentary evidence adduced by petitioners, taken separately and
independently of each other, are not per se sufficient proof of legitimacy nor even of pedigree. It is
important to note, however, that the rulings of both lower courts in the case are basically premised
on the erroneous assumption that, in the first place, the issue of legitimacy may be validly
controverted in an action for reconveyance, and, in the second place, that herein petitioners have
the onus probandi to prove their legitimacy and, corollarily, their filiation. We disagree on both
counts.
It seems that both the court a quo and respondent appellate court have regrettably overlooked the
universally recognized presumption on legitimacy. There is no presumption of the law more firmly
established and founded on sounder morality and more convincing reason than the presumption that
children born in wedlock are legitimate.
8
And well settled is the rule that the issue of legitimacy
cannot be attacked collaterally.
The rationale for these rules has been explained in this wise:
The presumption of legitimacy in the Family Code . . . actually fixes a civil status for
the child born in wedlock, and that civil status cannot be attacked collaterally. The
legitimacy of the child can be impugned only in a direct action brought for that
purpose, by the proper parties, and within the period limited by law.
The legitimacy of the child cannot be contested by way of defense or as a collateral
issue in another action for a different purpose. The necessity of an independent
action directly impugning the legitimacy is more clearly expressed in the Mexican
Code (Article 335) which provides: "The contest of the legitimacy of a child by the
husband or his heirs must be made by proper complaint before the competent court;
any contest made in any other way is void." This principle applies under our Family
Code. Articles 170 and 171 of the code confirm this view, because they refer to "the
action to impugn the legitimacy." This action can be brought only by the husband or
his heirs and within the periods fixed in the present articles.
Upon the expiration of the periods provided in Article 170, the action to impugn the
legitimacy of a child can no longer be brought. The status conferred by the
presumption, therefore, becomes fixed, and can no longer be questioned. The
obvious intention of the law is to prevent the status of a child born in wedlock from
being in a state of uncertainty for a long time. It also aims to force early action to
settle any doubt as to the paternity of such child, so that the evidence material to the
matter, which must necessarily be facts occurring during the period of the conception
of the child, may still be easily available.
xxx xxx xxx
Only the husband can contest the legitimacy of a child born to his wife. He is the one
directly confronted with the scandal and ridicule which the infidelity of his wife produces;
and he should decide whether to conceal that infidelity or expose it, in view of the moral
and economic interest involved. It is only in exceptional cases that his heir are allowed to
contest such legitimacy. Outside of these cases, none even his heirs can impugn
legitimacy; that would amount to an insult to his memory.
9

The issue, therefore, as to whether petitioners are the legitimate children of Hermogenes Dezoller
cannot be properly controverted in the present action for reconveyance. This is aside, of course,
from the further consideration that private respondent is not the proper party to impugn the
legitimacy of herein petitioners. The presumption consequently continues to operate in favor of
petitioners unless and until it is rebutted.
Even assuming that the issue is allowed to be resolved in this case, the burden of proof rests not on
herein petitioners who have the benefit of the presumption in their favor, but on private respondent
who is disputing the same. This fact alone should have been sufficient cause for the trial court to
exercise appropriate caution before acting, as it did, on the demurrer to evidence. It would have
delimited the issues for resolution, as well as the time and effort necessitated thereby.
Ordinarily, when a fact is presumed, it implies that the party in whose favor the presumption exists
does not have to introduce evidence to establish that fact, and in any litigation where that fact is put
in issue, the party denying it must bear the burden of proof to overthrow the presumption.
10
The
presumption of legitimacy is so strong that it is clear that its effect is to shift the burden of persuasion
to the party claiming illegitimacy.
11
And in order to destroy the presumption, the party against whom
it operates must adduce substantial and credible evidence to the contrary.
12

Where there is an entire lack of competent evidence to the contrary,
13
and unless or until it is
rebutted, it has been held that a presumption may stand in lieu of evidence and support a finding or
decision.
14
Perforce, a presumption must be followed if it is uncontroverted. This is based on the
theory that a presumption is prima facieproof of the fact presumed, and unless the fact thus
established prima facie by the legal presumption of its truth is disproved, it must stand as proved.
15

Indubitably, when private respondent opted not to present countervailing evidence to overcome the
presumption, by merely filing a demurrer to evidence instead, she in effect impliedly admitted the
truth of such fact. Indeed, she overlooked or disregarded the evidential rule that presumptions like
judicial notice and admissions, relieve the proponent from presenting evidence on the facts he
alleged and such facts are thereby considered as duly proved.
II. The weight and sufficiency of the evidence regarding petitioner's relationship with Teodora
Dezoller Guerrero, whose estate is the subject of the present controversy, requires a more intensive
and extensive examination.
Petitioners' evidence, as earlier explained, consists mainly of the testimony of Corazon Dezoller
Tison, the baptismal, death and marriage certificates, the various certifications from the civil
registrar, a family picture, and several joint affidavits executed by third persons all of which she
identified and explained in the course and as part of her testimony.
The primary proof to be considered in ascertaining the relationship between the parties concerned is
the testimony of Corazon Dezoller Tison to the effect that Teodora Dezoller Guerrero in her lifetime,
or sometime in 1946, categorically declared that the former is Teodora's niece.
16
Such a statement
is considered a declaration about pedigree which is admissible, as an exception to the hearsay rule,
under Section 39, Rule 130 of the Rules of Court, subject to the following conditions: (1) that the
declarant is dead or unable to testify; (2) that the declarant be related to the person whose pedigree
is the subject of inquiry; (3) that such relationship be shown by evidence other than the declaration;
and (4) that the declaration was made ante litem motam, that is, not only before the commencement
of the suit involving the subject matter of the declaration, but before any controversy has arisen
thereon.
There is no dispute with respect to the first, second and fourth elements. What remains for analysis
is the third element, that is, whether or not the other documents offered in evidence sufficiently
corroborated the declaration made by Teodora Dezoller Guerrero in her lifetime regarding the
pedigree of petitioner Corazon Dezoller Tison or, if at all, it is necessary to present evidence other
than such declaration.
American jurisdiction has it that a distinction must be made as to when the relationship of the
declarant may be proved by the very declaration itself, or by other declarations of said declarant, and
when it must be supported by evidence aliunde. The rule is stated thus:
One situation to be noted is that where one seeks to set up a claim through, but not from,
the declarant and to establish the admissibility of a declaration regarding claimant's
pedigree, he may not do by declarant's own statements as to declarant's relationship to
the particular family. The reason is that declarant's declaration of his own relationship is
of a self-serving nature. Accordingly there must be precedent proof from other sources
that declarant is what he claimed to be, namely, a member of the particular family;
otherwise the requirement to admissibility that declarant's relationship to the common
family must appear is not met. But when the party claiming seeks to establish relationship
in order to claim directly from the declarant or the declarant's estate, the situation and the
policy of the law applicable are quite different. In such case the declaration of the
decedent, whose estate is in controversy, that he was related to the one who claims his
estate, is admissible without other proof of the fact of relationship. While the nature of the
declaration is then disserving, that is not the real ground for its admission. Such
declarations do not derive their evidential value from that consideration, although it is a
useful, if not an artificial, aid in determining the class to which the declarations belong.
The distinction we have note is sufficiently apparent; in the one case the declarations are
self-serving, in the otherthey are competent from reasons of necessity.
17
(Emphasis
ours.)
The general rule, therefore, is that where the party claiming seeks recovery against a relative
common to both claimant and declarant, but not from the declarant himself or the declarant's estate,
the relationship of the declarant to the common relative may not be proved by the declaration itself.
There must be some independent proof of this fact.
18
As an exception, the requirement that there be
other proof than the declarations of the declarant as to the relationship, does not apply where it is
sought to reach the estate of the declarant himself and not merely to establish a right through his
declarations to the property of some other member of the family.
19

We are sufficiently convinced, and so hold, that the present case is one instance where the general
requirement on evidence aliunde may be relaxed. Petitioners are claiming a right to part of the estate
of the declarant herself. Conformably, the declaration made by Teodora Dezoller Guerrero that
petitioner Corazon is her niece, is admissible and constitutes sufficient proof of such relationship,
notwithstanding the fact that there was no other preliminary evidence thereof, the reason being such
declaration is rendered competent by virtue of the necessity of receiving such evidence to avoid a
failure of justice.
20
More importantly, there is in the present case an absolute failure by all and
sundry to refute that declaration made by the decedent.
From the foregoing disquisitions, it may thus be safely concluded, on the sole basis of the
decedent's declaration and without need for further proof thereof, that petitioners are the niece and
nephew of Teodora Dezoller Guerrero. As held in one case,
21
where the subject of the declaration is
the declarant's own relationship to another person, it seems absurb to require, as a foundation for
the admission of the declaration, proof of the very fact which the declaration is offered to establish.
The preliminary proof would render the main evidence unnecessary.
Applying the general rule in the present case would nonetheless produce the same result. For while
the documentary evidence submitted by petitioners do not strictly conform to the rules on their
admissibility, we are however of the considered opinion that the same may be admitted by reason of
private respondent's failure to interpose any timely objection thereto at the time they were being
offered in evidence.
22
It is elementary that an objection shall
be made at the time when an alleged inadmissible document is offered in evidence,
23
otherwise, the
objection shall be treated as waived,
24
since the right to object is merely a privilege which the party
may waive.
25

As explained in Abrenica vs. Gonda, et al.,
26
it has been repeatedly laid down as a rule of evidence
that a protest or objection against the admission of any evidence must be made at the proper time,
otherwise it will be deemed to have been waived. The proper time is when from the question
addressed to the witness, or from the answer thereto, or from the presentation of the proof, the
inadmissibility of the evidence is, or may be inferred.
Thus, a failure to except to the evidence because it does not conform with the statute is a waiver if
the provisions of the law. That objection to a question put to a witness must be made at the time the
question is asked. An objection to the admission of evidence on the ground of incompetency, taken
after the testimony has been given, is too late.
27
Thus, for instance, failure to object to parol
evidence given on the stand, where the party is in a position to object, is a waiver of any objections
thereto.
28

The situation is aggravated by the fact that counsel for private respondent unreservedly cross-
examined petitioners, as the lone witness, on the documentary evidence that were offered. At no
time was the issue of the supposed inadmissibility thereof, or the possible basis for objection thereto,
ever raised. Instead, private respondent's counsel elicited answers from the witness on the
circumstances and regularity of her obtention of said documents: The observations later made by
private respondent in her comment to petitioners' offer of exhibits, although the grounds therefor
were already apparent at the time these documents were being adduced in evidence during the
testimony of Corazon Dezoller Tison but which objections were not timely raised therein, may no
longer serve to rectify the legal consequences which resulted therefrom. Hence, even assuming ex
gratia argumenti that these documents are inadmissible for being hearsay, but on account of herein
private respondent's failure to object thereto, the same may be admitted and considered as sufficient
to prove the facts therein asserted.
29

Accordingly, the Certificate of Marriage (Exhibit S) wherein it is indicated that the parents of Teodora
Dezoller are Isabelo Dezoller and Cecilia Calpo, as well as the Certificates of Baptism of Teodora
Dezoller
30
(Exhibit H) and Hermogenes Dezoller (Exhibit J) which both reflect the names of their
parents as Isabelo Dezoller and Cecilia Calpo, to show that Hermogenes Dezoller is the brother of
Teodora Dezoller Guerrero; and the Death Certificate of Hermogenes Dezoller (Exhibit K) the entries
wherein were made by petitioner Corazon Dezoller Tison as his daughter, together with the Joint
Affidavits of Pablo Verzosa and Meliton Sitjar (Exhibits N and P), to prove that herein petitioners are
the children of Hermogenes Dezoller these can be deemed to have sufficiently established the
relationship between the declarant and herein petitioners. This is in consonance with the rule that
a prima facie showing is sufficient and that only slight proof of the relationship is required.
31
Finally,
it may not be amiss to consider as in the nature of circumstantial evidence the fact that both the
declarant and the claimants, who are the subject of the declaration, bear the surname Dezoller.
32

III. The following provisions of the Civil Code provide for the manner by which the estate of the
decedent shall be divided in this case, to wit:
Art. 975. When children of one or more brothers or sisters of the deceased survive,
they shall inherit from the latter by representation, if they survive with their uncles or
aunts. But if they alone survive, they shall inherit in equal portions.
Art. 995. In the absence of legitimate descendants and ascendants, and illegitimate
children and their descendants, whether legitimate or illegitimate, the surviving
spouse shall inherit the entire estate, without prejudice to the rights of brothers and
sisters, nephews and nieces, should there be any, under Article 1001.
Art. 1001. Should brothers and sisters or their children survive with the widow or
widower, the latter shall be entitled to one-half of the inheritance and the brothers
and sisters or theirs children to the other half.
Upon the death of Teodora Dezoller Guerrero, one-half of the subject property was automatically
reserved to the surviving spouse, Martin Guerrero, as his share in the conjugal partnership. Applying
the aforequoted statutory provisions, the remaining half shall be equally divided between the
widower and herein petitioners who are entitled to jointly inherit in their own right. Hence, Martin
Guerrero could only validly alienate his total undivided three-fourths (3/4) share in the entire property
to herein private respondent. Resultantly, petitioners and private respondent are deemed co-owners
of the property covered by Transfer Certificate of Title No. 374012 in the proportion of an undivided
one-fourth (1/4) and three-fourths (3/4) share thereof, respectively.
All told, on the basis of the foregoing considerations, the demurrer to plaintiff's evidence should have
been, as it is hereby, denied. Nonetheless, private respondent may no longer be allowed to present
evidence by reason of the mandate under Section 1 of revised Rule 3 of the Rules of Court which
provides that "if the motion is granted but on appeal the order of dismissal is reversed he shall be
deemed to have waived the right to present evidence."
33

WHEREFORE, the questioned judgment of respondent Court of Appeals is hereby REVERSED and
SET ASIDE, and herein petitioners and private respondent are declared co-owners of the subject
property with an undivided one-fourth (1/4) and three-fourths (3/4) share therein, respectively.
SO ORDERED.
Romero, Puno and Mendoza, JJ., concur.
Torres, Jr., J., is on leave.







Republic of the Philippines
SUPREME COURT
Manila
THIRD DIVISION

G.R. No. 57227 May 14, 1992
AMELITA CONSTANTINO and MICHAEL CONSTANTINO, the latter represented herein by the
former, his mother and natural guardian, petitioners,
vs.
IVAN MENDEZ and the HONORABLE COURT OF APPEALS, respondents.
Roberto M. Sarenas for petitioners.
Bienvinido D. Cariaga for private respondent.

BIDIN, J .:
This is a petition for review on certiorari questioning the decision
1
dated April 30, 1981 of the Court of
Appeals in CA-G.R. No. 61552-R which dismissed petitioner's complaint and set aside the
resolution
2
dated October 21, 1976 of the then Court of First Instance of Davao, 16th Judicial District,
amending the dispositive portion of its decision dated June 21, 1976 and ordering private respondent Ivan
Mendez: (1) to acknowledge the minor Michael Constantino as his illegitimate child; (2) to give a monthly
support of P300.00 to the minor child; (3) to pay complainant Amelita Constantino the sum of P8,200.00
as actual and moral damages; and (4) to pay attorney's fees in the sum of P5,000 plus costs.
It appears on record that on June 5, 1975, petitioner Amelita Constantino filed an action for
acknowledgment, support and damages against private respondent Ivan Mendez. The case was filed with
the then CFI of Davao, 10th Judicial District and docketed as Civil Case No. 8881. In her complaint,
Amelita Constantino alleges, among others, that sometime in the month of August, 1974, she met Ivan
Mendez at Tony's Restaurant located at Sta. Cruz, Manila, where she worked as a waitress; that the day
following their first meeting, Ivan invited Amelita to dine with him at Hotel Enrico where he was billeted;
that while dining, Ivan professed his love and courted Amelita; that Amelita asked for time to think about
Ivan's proposal; that at about 11:00 o'clock in the evening, Amelita asked Ivan to bring her home to which
the latter agreed, that on the pretext of getting something, Ivan brought Amelita inside his hotel room and
through a promise of marriage succeeded in having sexual intercourse with the latter; that after the sexual
contact, Ivan confessed to Amelita that he is a married man; that they repeated their sexual contact in the
months of September and November, 1974, whenever Ivan is in Manila, as a result of which Amelita got
pregnant; that her pleas for help and support fell on deaf ears; that Amelita had no sexual relations with
any other man except Ivan who is the father of the child yet to be born at the time of the filing of the
complaint; that because of her pregnancy, Amelita was forced to leave her work as a waitress; that Ivan is
a prosperous businessman of Davao City with a monthly income of P5,000 to P8,000. As relief, Amelita
prayed for the recognition of the unborn child, the payment of actual, moral and exemplary damages,
attorney's fees plus costs.
In his answer dated August 5, 1975, Ivan admitted that he met Amelita at Tony's Cocktail Lounge but
denied having sexual knowledge or illicit relations with her. He prayed for the dismissal of the complaint
for lack of cause of action. By way of counterclaim, he further prayed for the payment of exemplary
damages and litigation expense including attorney's fees for the filing of the malicious complaint.
On September 1, 1975, Amelita Constantino filed a motion for leave to amend the complaint impleading
as co-plaintiff her son Michael Constantino who was born on August 3, 1975. In its order dated
September 4, 1975, the trial court admitted the amended complaint.
On September 11, 1975, Ivan Mendez filed his answer to the amended complaint reiterating his previous
answer denying that Michael Constantino is his illegitimate son.
After hearing, the trial court rendered a decision dated June 21, 1976, the dispositive portion of which
reads, viz:
WHEREFORE, in view of the foregoing, judgment is hereby rendered in favor of plaintiff
Amelita Constantino and against defendant Ivan Mendez, ordering the latter to pay
Amelita Constantino the sum of P8,000.00 by way of actual and moral damages; and, the
sum of P3,000.00, as and by way of attorney's fees. The defendant shall pay the costs of
this suit.
SO ORDERED.
From the above decision, both parties filed their separate motion for reconsideration. Ivan Mendez
anchored his motion on the ground that the award of damages was not supported by evidence. Amelita
Constantino, on the other hand, sought the recognition and support of her son Michael Constantino as the
illegitimate son of Ivan Mendez.
In its resolution dated October 21, 1976, the trial court granted Amelita Constantino's motion for
reconsideration, and amended the dispositive portion of its decision dated June 21, 1976 to read as
follows, viz:
WHEREFORE, in view of the foregoing, judgment is hereby rendered in favor of plaintiff
Amelita Constantino and plaintiff-minor Michael Constantino, and against defendant Ivan
Mendez ordering the latter to pay Amelita Constantino the sum of P8,000.00 by way of
actual and moral damages and the sum of P200.00 as and by way of payment of the
hospital and medical bills incurred during the delivery of plaintiff-minor Michael
Constantino; to recognize as his own illegitimate child the plaintiff-minor Michael
Constantino who shall be entitled to all the rights, privileges and benefits appertaining to
a child of such status; to give a permanent monthly support in favor of plaintiff Michael
Constantino the amount of P300.00; and the sum of P5,000.00 as and by way of
attorney's fees. The defendant shall pay the costs of this suit.
Let this Order form part of the decision dated June 21, 1976.
SO ORDERED.
On appeal to the Court of Appeals, the above amended decision was set aside and the complaint was
dismissed. Hence, this petition for review.
Basically, the issue to be resolved in the case at bar is whether or not the Court of Appeals committed a
reversible error in setting aside the decision of the trial court and in dismissing the complaint.
Petitioners contend that the Court of Appeals erred in reversing the factual findings of the trial and in not
affirming the decision of the trial court. They also pointed out that the appellate court committed a
misapprehension of facts when it concluded that Ivan did not have sexual access with Amelita during the
first or second week of November, 1976 (should be 1974), the time of the conception of the child.
It must be stressed at the outset that factual findings of the trial court have only a persuasive and not a
conclusive effect on the Court of Appeals. In the exercise of its appellate jurisdiction, it is the duty of the
Court of Appeals to review the factual findings of the trial court and rectify the errors it committed as may
have been properly assigned and as could be established by a re-examination of the evidence on record.
It is the factual findings of the Court of Appeals, not those of the trial court, that as a rule are considered
final and conclusive even on this Court (Hermo v. Hon. Court of Appeals, et al., 155 SCRA 24 [1987]).
This being a petition for certiorari under Rule 45 of the Rules of Court, this Court will review only errors of
law committed by the Court of Appeals. It is not the function of this Court to re-examine all over again the
oral and documentary evidence submitted by the parties unless the findings of facts of the Court of
Appeals is not supported by the evidence on record or the judgment is based on misapprehension of
facts (Remalante v. Tibe, et al., 158 SCRA 138 [1988]; Hernandez v. Court of Appeals, et al., 149 SCRA
97 [1987]).
It is the conclusion of the Court of Appeals, based on the evidence on record, that Amelita Constantino
has not proved by clear and convincing evidence her claim that Ivan Mendez is the father of her son
Michael Constantino. Such conclusion based on the evaluation of the evidence on record is controlling on
this Court as the same is supported by the evidence on record. Even the trial court initially entertained
such posture. It ordered the recognition of Michael as the illegitimate son of Ivan only when acting on the
motions for reconsideration, it reconsidered, on October 21, 1976, its earlier decision dated June 21,
1976. Amelita's testimony on cross-examination that she had sexual contact with Ivan in Manila in the first
or second week of November, 1974 (TSN, December 8, 1975, p. 108) is inconsistent with her response
that she could not remember the date of their last sexual intercourse in November, 1974 (Ibid, p. 106).
Sexual contact of Ivan and Amelita in the first or second week of November, 1974 is the crucial point that
was not even established on direct examination as she merely testified that she had sexual intercourse
with Ivan in the months of September, October and November, 1974.
Michael Constantino is a full-term baby born on August 3, 1975 (Exhibit 6) so that as correctly pointed out
by private respondent's counsel, citing medical science (Williams Obstetrics, Tenth Ed., p. 198) to the
effect that "the mean duration of actual pregnancy, counting from the day of conception must be close to
267 days", the conception of the child (Michael) must have taken place about 267 days before August 3,
1975 or sometime in the second week of November, 1974. While Amelita testified that she had sexual
contact with Ivan in November, 1974, nevertheless said testimony is contradicted by her own evidence
(Exh. F), the letter dated February 11, 1975, addressed to Ivan Mendez requesting for a conference,
prepared by her own counsel Atty. Roberto Sarenas to whom she must have confided the attendant
circumstances of her pregnancy while still fresh in her memory, informing Ivan that Amelita is four (4)
months pregnant so that applying the period of the duration of actual pregnancy, the child was conceived
on or about October 11, 1974.
Petitioner's assertion that Ivan is her first and only boyfriend (TSN, December 8, 1975, p. 65) is belied by
Exhibit 2, her own letter addressed to Mrs. Mendez where she revealed the reason for her attachment to
Ivan who possessed certain traits not possessed by her boyfriend. She also confided that she had a
quarrel with her boyfriend because of gossips so she left her work. An order for recognition and support
may create an unwholesome atmosphere or may be an irritant in the family or lives of the parties so that it
must be issued only if paternity or filiation is established by clear and convincing evidence. The burden of
proof is on Amelita to establish her affirmative allegations that Ivan is the father of her son. Consequently,
in the absence of clear and convincing evidence establishing paternity or filiation, the complaint must be
dismissed.
As regards Amelita's claim for damages which is based on Articles 19
3
& 21
4
of the Civil Code on the
theory that through Ivan's promise of marriage, she surrendered her virginity, we cannot but agree with
the Court of Appeals that more sexual intercourse is not by itself a basis for recovery. Damages could
only be awarded if sexual intercourse is not a product of voluntariness and mutual desire. At the time she
met Ivan at Tony's Restaurant, Amelita was already 28 years old and she admitted that she was attracted
to Ivan (TSN, December 3, 1975, p. 83). Her attraction to Ivan is the reason why she surrendered her
womanhood. Had she been induced or deceived because of a promise of marriage, she could have
immediately severed her relation with Ivan when she was informed after their first sexual contact
sometime in August, 1974, that he was a married man. Her declaration that in the months of September,
October and November, 1974, they repeated their sexual intercourse only indicates that passion and not
the alleged promise of marriage was the moving force that made her submit herself to Ivan.
WHEREFORE, the instant petition is Dismissed for lack of merit.
SO ORDERED.
Gutierrez, Jr., Feliciano, Davide, Jr. and Romero, JJ., concur.






















FIRST DIVISION
[G.R. No. 124853. February 24, 1998]
FRANCISCO L. JISON, petitioner, vs. COURT OF APPEALS and MONINA JISON, respondent.
D E C I S I O N
DAVIDE, JR., J .:
This is a petition for review under Rule 45 of the Rules of Court of the 27 April 1995 decision of the
Court of Appeals (CA) in CA-G.R. CV No. 32860
[1]
which reversed the decision of Branch 24 of the
Regional Trial Court (RTC) of Iloilo City in Civil Case No. 16373.
[2]
The latter dismissed the complaint of
private respondent Monina Jison (hereafter MONINA) for recognition as an illegitimate child of petitioner
Francisco Jison (hereafter FRANCISCO).
In issue is whether or not public respondent Court of Appeals committed reversible error, which, in
this instance, necessitates an inquiry into the facts. While as a general rule, factual issues are not within
the province of this Court, nevertheless, in light of the conflicting findings of facts of the trial court and the
Court of Appeals, this case falls under an exception to this rule.
[3]

In her complaint
[4]
filed with the RTC on 13 March 1985, MONINA alleged that FRANCISCO had
been married to a certain Lilia Lopez Jison since 1940. At the end of 1945 or the start of 1946, however,
FRANCISCO impregnated Esperanza F. Amolar (who was then employed as the nanny of FRANCISCO's
daughter, Lourdes). As a result, MONINA was born on 6 August 1946, in Dingle, Iloilo, and since
childhood, had enjoyed the continuous, implied recognition as an illegitimate child of FRANCISCO by his
acts and that of his family. MONINA further alleged that FRANCISCO gave her support and spent for her
education, such that she obtained a Master's degree, became a certified public accountant (CPA) and
eventually, a Central Bank examiner. In view of FRANCISCO's refusal to expressly recognize her,
MONINA prayed for a judicial declaration of her illegitimate status and that FRANCISCO support and treat
her as such.
In his answer,
[5]
FRANCISCO alleged that he could not have had sexual relations with Esperanza
Amolar during the period specified in the complaint as she had ceased to be in his employ as early as
1944, and did not know of her whereabouts since then; further, he never recognized MONINA, expressly
or impliedly, as his illegitimate child. As affirmative and special defenses, FRANCISCO contended that
MONINA had no right or cause of action against him and that her action was barred by estoppel, laches
and/or prescription. He thus prayed for dismissal of the complaint and an award of damages due to the
malicious filing of the complaint.
After MONINA filed her reply,
[6]
pre-trial was conducted where the parties stipulated on the following
issues:
1. Did Francisco Jison have any sexual relation[s] with Esperanza Am[o]lar about the end of
1945 or the start of 1946?
2. Is Monina Jison the recognized illegitimate daughter of Francisco Jison by the latters own
acts and those of his family?
3. Is Monina Jison barred from instituting or prosecuting the present action by estoppel, laches
and/or prescription?
4. Damages.
[7]

At trial on the merits, MONINA presented a total of eleven (11) witnesses, namely: herself, Ruben
Castellanes, Sr., Adela Casabuena, Arsenio Duatin, Zafiro Ledesma, Danthea Lopez, Romeo Bilbao,
Rudy Tingson, Alfredo Baylosis, Dominador Zavariz and Lope Amolar.
Ruben Castellanes, Sr., a 63-year old resident of Iloilo City, testified that he had worked for
FRANCISCO for a total of six (6) years at Nelly Garden, FRANCISCO's Iloilo residence. Towards the end
of the Japanese occupation, FRANCISCOs wife suffered a miscarriage or abortion, thereby depriving
FRANCISCO of consortium; thereafter, FRANCISCOs wife managed a nightclub on the ground floor of
Nelly Garden which operated daily from 6:00 p.m. till 3:00 a.m. of the following day, thereby allowing
FRANCISCO free access to MONINAs mother, Esperanza Amolar, who was nicknamed Pansay.
Adela Casabuena, a 61-year old farmer, testified that she served as the yaya (nanny) of Lourdes
from July 1946 up to February 1947. Although Pansay had left Nelly Garden two (2) weeks before Adela
started working for the Jisons, Pansay returned sometime in September 1946, or about one month after
she gave birth to MONINA, to ask FRANCISCO for support. As a result,Pansay and Lilia Jison,
FRANCISCO's wife, quarreled in the living room, and in the course thereof, Pansay claimed that
FRANCISCO was the father of her baby. To which, Lilia replied: I did not tell you to make that baby so it
is your fault. During the quarrel which lasted from 10:30 till 11:00 a.m., FRANCISCO was supposedly
inside the house listening.
Arsenio Duatin, a 77-year old retired laborer, testified that from 1947 until 1977, he worked as
FRANCISCOs houseboy at the latters house on 12th Street, Capitol Subdivision, Bacolod City. Arsenio
met MONINA in 1967, when Felipe Lagarto, the bookkeeper at Nelly Garden, informed Arsenio that
MONINA, FRANCISCOs daughter, would arrive at Bacolod City with a letter of introduction from Lagarto.
Initially, Arsenio identified seven (7) black-and-white photographs (Exhs. X-5 to X-11) of
MONINA,
[8]
and as he paid for the telephone bills, he likewise identified six (6) telephone cards (Exhs. G
to L). Arsenio then declared that when MONINA arrived in Bacolod City, she introduced herself to him as
FRANCISCOs daughter. She stayed at FRANCISCOs house, but when the latter and his wife would
come over, Arsenio would conceal the presence of MONINA because Mrs. Jison did not like to see her
face. Once, Arsenio hid MONINA in the house of FRANCISCOs sister, Mrs. Luisa Jison Alano, in Silay
City; another time, at the residence of FRANCISCOs cousin, Mrs. Concha Lopez Cuaycong. Finally,
Arsenio declared that the last time he saw MONINA was when she left for Manila, after having fi nished
her schooling at La Salle College in Bacolod City.
On re-direct and upon questions by the court, Arsenio disclosed that it was FRANCISCO who
instructed that MONINA be hidden whenever FRANCISCO and his wife were around; that although
FRANCISCO and MONINA saw each other at the Bacolod house only once, they called each other
through long distance; and that MONINA addressed FRANCISCO as Daddy during their lone meeting
at the Bacolod house and were affectionate to each other. Arsenio likewise declared that MONINA
stayed at FRANCISCO's Bacolod house twice: first for a month, then for about a week the second
time. On both occasions, however, FRANCISCO and his wife were abroad. Finally, Arsenio recalled that
FRANCISCO likewise bade Arsenio to treat MONINA like his (FRANCISCOs) other daughters.
The testimony of Zafiro Ledesma, a 74-year old banker and former mayor of Iloilo City, initially
touched on how he and his wife were related to FRANCISCO, FRANCISCO's wife and MONINA. Zafiro
first identified Exhibit R, a diagram of the family trees of the Jison and Lopez families, which showed that
former Vice-President Fernando Lopez was the first cousin of FRANCISCOs wife, then told the court that
the family of Vice-President Lopez treated MONINA very well because she is considered a relative xxx
by reputation, by actual perception. Zafiro likewise identified Exhibits X-13 to X-18, photographs taken at
the 14 April 1985 birthday celebration of Mrs. Fernando Lopez, which showed MONINA with the former
Vice-President and other members of the Lopez family.
Zafiro further testified that while MONINA lived with Mrs. Cuaycong, the latter paid for some of
MONINAs school needs and even asked MONINA to work in a hospital owned by Mrs. Cuaycong; and
that another first cousin of FRANCISCOs wife, a certain Remedios Lopez Franco, likewise helped
MONINA with her studies and problems, and even attended MONINAs graduation in 1978 when she
obtained a masteral degree in Business Administration, as evidenced by another photograph (Exh. X-
12). Moreover, upon Remedios recommendation, MONINA was employed as a secretary at Merchant
Financing Company, which was managed by a certain Danthea Lopez, the wife of another first cousin of
FRANCISCOs wife, and among whose directors were Zafiro himself, his wife and Dantheas husband. In
closing, Zafiro identified MONINAs Social Security Record (Exh. W), which was signed by Danthea as
employer and where MONINA designated Remedios as the beneficiary.
Danthea Lopez, a 58-year old housekeeper, declared that FRANCISCO was the first cousin of her
husband, Eusebio D. Lopez; and that she came to know MONINA in the latter part of 1965 when
Remedios Franco recommended MONINA for employment at Merchant Financing Co., which Danthea
managed at that time. Remedios introduced MONINA to Danthea as being reputedly the daughter of Mr.
Frank Jison; and on several occasions thereafter, Remedios made Danthea and the latters husband
understand that MONINA was reputedly the daughter of [FRANCISCO]. While MONINA worked at
Merchant Financing, Danthea knew that MONINA lived with Remedios; however, in the latter part of
1966, as Remedios left for Manila and MONINA was still studying at San Agustin University, Danthea and
her husband invited MONINA to live with them. During MONINAs 6-month stay with them, she was not
charged for board and lodging and was treated as a relative, not a mere employee, all owing to what
Remedios had said regarding MONINAs filiation. As Danthea understood, MONINA resigned from
Merchant Financing as she was called by Mrs. Cuaycong, a first cousin of Dantheas husband who lived
in Bacolod City.
Romeo Bilbao, a 43-year old seaman, testified that he had worked for FRANCISCO from 1969 up to
1980 at Nelly Garden in various capacities: as a procurement officer, hacienda overseer and, later,
as hacienda administrator. Sometime in May, 1971, Romeo saw and heard MONINA ask her Daddy
(meaning FRANCISCO) for the money he promised to give her, but FRANCISCO answered that he did
not have the money to give, then told MONINA to go see Mr. Jose Cruz in Bacolod City. Then in the
middle of September that year, FRANCISCO told Romeo to pick up Mr. Cruz at the Iloilo pier and bring
him to the office of Atty. Benjamin Tirol. At said office, Atty. Tirol, Mr. Cruz and MONINA entered a room
while Romeo waited outside. When they came out, Atty. Tirol had papers for MONINA to sign, but she
refused. Atty. Tirol said that a check would be released to MONINA if she signed the papers, so MONINA
acceded, although Atty. Tirol intended not to give MONINA a copy of the document she
signed. Thereafter, Mr. Cruz gave MONINA a check (Exh. Q), then MONINA grabbed a copy of the
document she signed and ran outside. Romeo then brought Mr. Cruz to Nelly Garden. As to his motive
for testifying, Romeo stated that he wanted to help MONINA be recognized as FRANCISCOS daughter.
Rudy Tingson, a 45-year old antique dealer, testified that in 1963-1964, he was employed by
FRANCISCOs wife at the Baguio Military Institute in Baguio City; then in 1965, Rudy worked at
FRANCISCOs office at Nelly Garden recording hacienda expenses, typing vouchers and office papers,
and, at times, acting as paymaster for the haciendas. From the nature of his work, Rudy knew the
persons receiving money from FRANCISCOs office, and clearly remembered that in 1965, as part of his
job, Rudy gave MONINA her allowance from FRANCISCO four (4) times, upon instructions of a certain
Mr. Lagarto to give MONINA P15.00 a month. Rudy likewise recalled that he first met MONINA in 1965,
and that she would go to Nelly Garden whenever FRANCISCOs wife was not around. On some of these
occasions, MONINA would speak with and address FRANCISCO as Daddy, without objection from
FRANCISCO. In fact, in 1965, Rudy saw FRANCISCO give MONINA money thrice. Rudy further
declared that in April 1965, FRANCISCOs office paid P250.00 to Funeraria Bernal for the funeral
expenses of MONINAs mother. Finally, as to Rudy's motives for testifying, he told the court that he
simply wanted to help bring out the truth and nothing but the truth, and that MONINAs filiation was
common knowledge among the people in the office at Nelly Garden.
On re-direct, Rudy declared that the moneys given by FRANCISCOs office to MONINA were not
reflected in the books of the office, but were kept in a separate book, as Mr. Lagarto explained that
FRANCISCOs wife and children should not know [of] this. Rudy further revealed that as to the garden
meetings between FRANCISCO and MONINA, Rudy saw MONINA kiss FRANCISCO on the cheek both
upon arriving and before leaving, and FRANCISCOs reaction upon seeing her was to smile and say in
the Visayan dialect: Kamusta ka iha? (How are you, daughter?); and that MONINA was free to go
inside the house as the household staff knew of her filiation, and that, sometimes, MONINA would join
them for lunch.
Alfredo Baylosis, a 62-year old retired accountant, testified that he worked for FRANCISCO at
Central Santos-Lopez in Iloilo from 1951 up to 1961, then at Nelly Garden from 1961 until 1972. Alfredo
first served FRANCISCO as a bookkeeper, then when Mr. Lagarto died in 1967 or 1969, Alfredo replaced
Mr. Lagarto as office manager.
Alfredo knew MONINA since 1961 as she used to go to Nelly Garden to claim her P15.00 monthly
allowance given upon FRANCISCOs standing order. Alfredo further declared that MONINAs filiation
was pretty well-known in the office; that he had seen MONINA and FRANCISCO go from the main
building to the office, with FRANCISCOs arm on MONINAs shoulder; and that the office paid for the
burial expenses of Pansay, but this was not recorded in the books in order to hide it from FRANCISCOs
wife. Alfredo also disclosed that the disbursements for MONINAs allowance started in 1961 and were
recorded in a separate cash book. In 1967, the allowances ceased when MONINA stopped schooling
and was employed in Bacolod City with Miller, Cruz & Co., which served as FRANCISCOs accountant-
auditor. Once, when Alfredo went to the offices of Miller, Cruz & Co. to see the manager, Mr. Atienza,
and arrange for the preparation of FRANCISCOs income tax return, Alfredo chanced upon
MONINA. When Alfredo asked her how she came to work there, she answered that her Daddy,
FRANCISCO, recommended her, a fact confirmed by Mr. Atienza. Alfredo then claimed that Mr. Jose
Cruz, a partner at Miller, Cruz & Co., was the most trusted man of FRANCISCO.
Dominador Savariz, a 55-year old caretaker, testified that he worked as FRANCISCOs houseboy at
Nelly Garden from November 1953 up to 1965. One morning in April 1954, MONINA and her
mother Pansay went to Nelly Garden and spoke with FRANCISCO for about an hour, during which time,
Dominador was vacuuming the carpet about six (6) to seven (7) meters away. Due to the noise of the
vacuum cleaner, FRANCISCO and MONINA spoke in loud voices, thus Dominador overheard their
conversation. As FRANCISCO asked Pansay why they came, Pansayanswered that they came to ask for
the sustenance of his child MONINA. FRANCISCO then touched MONINA's head and asked: How are
you Hija?, to which MONINA answered: Good morning, Daddy. After FRANCISCO told Pansay and
MONINA to wait, he pulled something from his wallet and said to Pansay: I am giving this for the child.
In May 1954, Dominador saw MONINA at Mr. Lagartos office where Dominador was to get the
days expenses, while MONINA was claiming her allowance from Mr. Diasnes. The next month,
Dominador saw MONINA at Nelly Garden and heard in the office that MONINA was there to get her
allowance from her Daddy. In December 1960, Dominador saw MONINA at Nelly Garden, in the room
of Don Vicente (father of FRANCISCOs wife), where she asked for a Christmas gift and she was calling
Don Vicente, Lolo (grandfather). At that time, FRANCISCO and his wife were not around. Then
sometime in 1961, when Dominador went to Mr. Lagartos office to get the marketing expenses,
Dominador saw MONINA once more claiming her allowance.
Dominador further testified that in February 1966, after he had stopped working for FRANCISCO,
Dominador was at Mrs. Francos residence as she recommended him for employment with her sister,
Mrs. Concha Cuaycong. There, he saw MONINA, who was then about 15 years old, together with Mrs.
Francos daughter and son. Mrs. Franco pointed at MONINA and asked Dominador if he knew who
MONINA was. Dominador answered that MONINA was FRANCISCOs daughter with Pansay, and then
Mrs. Franco remarked that MONINA was staying with her (Mrs. Franco) and that she was sending
MONINA to school at the University of San Agustin.
Lope Amolar, a 50-year old resident of Dingle, Iloilo, and the younger brother of Esperanza Amolar
(Pansay), testified that he worked for FRANCISCO as a houseboy from March to November 1945 at Nelly
Garden. Thereafter, FRANCISCO sent Lope to work at Elena Apartments in Manila. By November
1945, Pansay was also working at Elena Apartments, where she revealed to Lope that FRANCISCO
impregnated her. Lope then confronted FRANCISCO, who told Lope dont get hurt and dont cause any
trouble, because I am willing to support your Inday Pansay and my child. Three (3) days after this
confrontation, Lope asked for and received permission from FRANCISCO to resign because he (Lope)
was hurt.
On 21 October 1986, MONINA herself took the witness stand. At that time, she was 40 years old
and a Central Bank Examiner. She affirmed that as evidenced by certifications from the Office of the
Local Civil Registrar (Exhs. E and F) and baptismal certificates (Exhs. C and D), she was born on 6
August 1946 in Barangay Tabugon, Dingle, Iloilo, to Esperanza Amolar (who passed away on 20 April
1965) and FRANCISCO.
[9]
MONINA first studied at Sagrado where she stayed as a boarder. While at
Sagrado from 1952 until 1955 (up to Grade 4), her father, FRANCISCO, paid for her tuition fees and other
school expenses. She either received the money from FRANCISCO or from Mr. Lagarto, or saw
FRANCISCO give money to her mother, or Mr. Lagarto would pay Sagrado directly. After Sagrado,
MONINA studied in different schools,
[10]
but FRANCISCO continuously answered for her schooling.
For her college education, MONINA enrolled at the University of Iloilo, but she later dropped due to
an accident which required a week's hospitalization. Although FRANCISCO paid for part of the
hospitalization expenses, her mother shouldered most of them. In 1963, she enrolled at the University of
San Agustin, where she stayed with Mrs. Franco who paid for MONINA's tuition fees. However, expenses
for books, school supplies, uniforms and the like were shouldered by FRANCISCO. At the start of each
semester, MONINA would show FRANCISCO that she was enrolled, then he would ask her to canvass
prices, then give her the money she needed. After finishing two (2) semesters at University of San
Agustin, as evidenced by her transcript of records (Exh. Z showing that FRANCISCO was listed as
Parent/Guardian [Exh. Z-1]), she transferred to De Paul College, just in front of Mrs. Francos house,
and studied there for a year. Thereafter, MONINA enrolled at Western Institute of Technology (WIT),
where she obtained a bachelors degree in Commerce in April 1967. During her senior year, she stayed
with Eusebio and Danthea Lopez at Hotel Kahirup, owned by said couple. She passed the CPA board
exams in 1974, and took up an M.B.A. at De La Salle University as evidenced by her transcript (Exh. AA),
wherein FRANCISCO was likewise listed as Guardian (Exhs. AA-1 and AA-2).
MONINA enumerated the different members of the household staff at Nelly Garden, to wit: Luz, the
household cook; the houseboys Silvestre and Doming; the housemaid Natang; the yaya of the adopted
triplets, Deling; the yaya of Lolo Vicente, Adelina; and others. MONINA likewise enumerated the
members of the office staff (Messrs. Baylosis, Lagarto, Tingson, Diasnes, Jalandoni, Supertisioso, Doroy,
and others), and identified them from a photograph marked as Exhibit X-2. She then corroborated the
prior testimony regarding her employment at Merchant Financing Co., and her having lived at Hotel
Kahirup and at Mrs. Cuaycongs residence in Bacolod City, while working at the hospital owned by Mrs.
Cuaycong.
MONINA further testified that in March 1968, she went to Manila and met FRANCISCO at Elena
Apartments at the corner of Romero and Salas Streets, Ermita. She told FRANCISCO that she was
going for a vacation in Baguio City with Mrs. Francos mother, with whom she stayed up to June
1968. Upon her return from Baguio City, MONINA told FRANCISCO that she wanted to work, so the
latter arranged for her employment at Miller & Cruz in Bacolod City. MONINA went to Bacolod City, was
interviewed by Mr. Jose Cruz, a partner at Miller & Cruz, who told her she would start working first week
of September, sans examination. She resigned from Miller & Cruz in 1971 and lived with Mrs. Cuaycong
at her Forbes Park residence in Makati. MONINA went to see FRANCISCO, told him that she resigned
and asked him for money to go to Spain, but FRANCISCO refused as she could not speak Spanish and
would not be able find a job. The two quarreled and FRANCISCO ordered a helper to send MONINA out
of the house. In the process, MONINA broke many glasses at the pantry and cut her hand, after which,
FRANCISCO hugged her, gave her medicine, calmed her down, asked her to return to Bacolod City and
promised that he would give her the money.
MONINA returned to Bacolod City by plane, using a Filipinas Orient Airways plane ticket (Exh. M)
which FRANCISCO gave. She called Mr. Cruz, then Atty. Tirol, as instructed by Mr. Cruz. These calls
were evidenced by PLDT long distance toll cards (Exhs. G to L), with annotations at the back reading:
charged and paid under the name of Frank L. Jison and were signed by Arsenio Duatin (Exhs. G-1 to L-
1). PLDT issued a certification as to the veracity of the contents of the toll cards (Exh. BB). Likewise
introduced in evidence was a letter of introduction prepared by Mr. Cruz addressed to Atty. Tirol, on
MONINA's behalf (Exh. N).
MONINA also declared that Atty. Tirol then told her that she would have to go to Iloilo and sign a
certain affidavit, before Mr. Cruz would turn over the money promised by FRANCISCO. She went to Atty.
Tirols office in Iloilo, but after going over the draft of the affidavit, refused to sign it as it stated that she
was not FRANCISCOs daughter. She explained that all she had agreed with FRANCISCO was that he
would pay for her fare to go abroad, and that since she was a little girl, she knew about her
illegitimacy. She started crying, begged Atty. Tirol to change the affidavit, to which Atty. Tirol responded
that he was also a father and did not want this to happen to his children as they could not be blamed for
being brought into the world. She then wrote a letter (Exh. O) to FRANCISCO and sent it to the latters
Forbes Park residence (Bauhinia Place) by JRS courier service (Exhs. O-5 to O-7). MONINA
subsequently met FRANCISCO in Bacolod City where they discussed the affidavit which she refused to
sign. FRANCISCO told her that the affidavit was for his wife, that in case she heard about MONINA going
abroad, the affidavit would keep her peace.
MONINA then narrated that the first time she went to Atty. Tirols office, she was accompanied by
one Atty. Fernando Divinagracia, who advised her that the affidavit (Exh. P)
[11]
would boomerang
against FRANCISCO as it is contrary to law. MONINA returned to Bacolod City, then met with Atty. Tirol
once more to reiterate her plea, but Atty. Tirol did not relent. Thus, on the morning of 20 or 21 September
1971, she signed the affidavit as she was jobless and needed the money to support herself and finish her
studies. In exchange for signing the document, MONINA received a Bank of Asia check for P15,000.00
(Exh. Q), which was less than the P25,000.00 which FRANCISCO allegedly promised to give. As Atty.
Tirol seemed hesitant to give her a copy of the affidavit after notarizing it, MONINA merely grabbed a
copy and immediately left.
MONINA then prepared to travel abroad, for which purpose, she procured letters of introduction
(Exhs. S and T) from a cousin, Mike Alano (son of FRANCISCOs elder sister Luisa); and an uncle,
Emilio Jison (FRANCISCOs elder brother), addressed to another cousin, Beth Jison (Emilios daughter),
for Beth to assist MONINA. Exhibit S contained a statement (Exh. S-1) expressly recognizing that
MONINA was FRANCISCOs daughter. Ultimately though, MONINA decided not to go abroad, opting
instead to spend the proceeds of the P15,000.00 check for her CPA review, board exam and graduate
studies. After finishing her graduate studies, she again planned to travel abroad, for which reason, she
obtained a letter of introduction from former Vice President Fernando Lopez addressed to then United
States Consul Vernon McAnnich (Exh. V).
As to other acts tending to show her filiation, MONINA related that on one occasion, as
FRANCISCOs wife was going to arrive at the latters Bacolod City residence, FRANCISCO called
Arsenio Duatin and instructed Arsenio to hide MONINA. Thus, MONINA stayed with Mrs. Luisa Jison for
the duration of the stay of FRANCISCOs wife. MONINA also claimed that she knew Vice President
Fernando Lopez and his wife, Mariquit, even before starting to go to school. Thus, MONINA asked for a
recommendation letter (Exh. U) from Mrs. Mariquit Lopez for possible employment with Mrs. Rosario
Lopez Cooper, another second cousin of FRANCISCO. In Exhibit U, Mrs. Lopez expressly recognized
MONINA as FRANCISCOs daughter. As additional proof of her close relationship with the family of Vice
President Lopez, MONINA identified photographs taken at a birthday celebration on 14 April 1985.
MONINA finally claimed that she knew the three (3) children of FRANCISCO by wife, namely,
Lourdes, Francisco, Jr. (Junior) and Elena, but MONINA had met only Lourdes and Junior. MONINA's
testimony dealt lengthily on her dealings with Junior and the two (2) occasions when she met with
Lourdes. The last time MONINA saw FRANCISCO was in March 1979, when she sought his blessings to
get married.
In his defense, FRANCISCO offered his deposition taken before then Judge Romeo Callejo of the
Regional Trial Court of Manila, Branch 48. As additional witnesses, FRANCISCO presented Nonito
Jalandoni, Teodoro Zulla, Iigo Supertisioso, Lourdes Ledesma, Jose Cruz and Dolores Argenal.
FRANCISCO declared that Pansays employment ceased as of October, 1944, and that while
employed by him, Pansay would sleep with the other female helpers on the first floor of his residence,
while he, his wife and daughter slept in a room on the second floor. At that time, his household staff was
composed of three (3) female workers and two (2) male workers. AfterPansay left in October 1944, she
never communicated with him again, neither did he know of her whereabouts. FRANCISCO staunchly
denied having had sexual relations with Pansay and disavowed any knowledge about MONINAs birth. In
the same vein, he denied having paid for MONINAs tuition fees, in person or otherwise, and asserted
that he never knew that Mr. Lagarto paid for these fees. Moreover, FRANCISCO could not believe that
Lagarto would pay for these fees despite absence of instructions or approval from FRANCISCO. He
likewise categorically denied that he told anyone, be it Danthea Lopez, Zafiro Ledesma, Concha
Cuaycong or Remedios Franco, that MONINA was his daughter.
FRANCISCO also disclosed that upon his return from the United States in 1971, he fired Alfredo
Baylosis upon discovering that Alfredo had taken advantage of his position during the formers
absence. FRANCISCO likewise fired Rudy Tingson and Romeo Bilbao, but did not give the reasons
therefor.
Finally, FRANCISCO denied knowledge of MONINAs long distance calls from his Bacolod
residence; nevertheless, when he subsequently discovered this, he fired certain people in his office for
their failure to report this anomaly. As regards the caretaker of his Bacolod residence, FRANCISCO
explained that since MONINA lived at Mrs. Cuaycongs residence, the caretaker thought that he could
allow people who lived at the Cuaycong residence to use the facilities at his (FRANCISCOs) house.
Nonito Jalandoni, bookkeeper and paymaster at Nellys Garden from 1963 up to 1974, then from
1980 up to 1986, the assistant overseer of Hacienda Lopez, testified that he did not know MONINA; that
he learned of her only in June 1988, when he was informed by FRANCISCO that MONINA had sued him;
and that he never saw MONINA at Nellys Garden, neither did he know of any instructions for anyone at
Nellys Garden to give money to MONINA.
Teodoro Zulla, FRANCISCOs bookkeeper and paymaster from 1951 up to 1986, testified that
FRANCISCO dismissed Alfredo Baylosis due to certain unspecified discrepancies; and that he never
saw MONINA receive funds from either Mr. Lagarto or Mr. Baylosis. Upon questions from the trial court,
however, Teodoro admitted that he prepared vouchers for only one of FRANCISCOs haciendas, and not
vouchers pertaining to the latters personal expenses.
Iigo Supertisioso testified that he worked for FRANCISCO at Nellys Garden from 1964 up to 1984
as a field inspector, paymaster, cashier and, eventually, officer-in-charge (OIC). He confirmed Alfredo
Baylosis dismissal due to these unspecified irregularities, then denied that FRANCISCO ever ordered
that MONINA be given her allowance. Likewise, Iigo never heard FRANCISCO mention that MONINA
was his (FRANCISCOs) daughter.
Lourdes Ledesma, FRANCISCOs daughter, testified that she saw (but did not know) MONINA at the
Our Lady of Mercy Hospital, on the occasion of the birth of Lourdes first son, Mark. Over lunch one day,
Lourdes aunt casually introduced Lourdes and MONINA to each other, but they were referred to only by
their first names. Then sometime in 1983 or 1984, MONINA allegedly went to Lourdes house in Sta.
Clara Subdivision requesting for a letter of introduction or referral as MONINA was then job-
hunting. However, Lourdes did not comply with the request.
Jose Cruz, a partner at Miller, Cruz & Co., testified that MONINA worked at Miller & Cruz from 1968
up to 1971, however, he did not personally interview her before she was accepted for
employment. Moreover, MONINA underwent the usual screening procedure before being hired. Jose
recalled that one of the accountants, a certain Mr. Atienza, reported that MONINA claimed to be
FRANCISCOs daughter. Jose then told Mr. Atienza to speak with MONINA and see if he (Mr. Atienza)
could stop her from spreading this rumor. Mr. Atienza reported that he spoke with MONINA, who told him
that she planned to leave for the United States and needed P20,000.00 for that purpose, and in
exchange, she would sign a document disclaiming filiation with FRANCISCO. Thus, Jose instructed Mr.
Atienza to request that MONINA meet with Jose, and at that meeting, MONINA confirmed Mr. Atienzas
report. Jose then informed Atty. Tirol, FRANCISCOs personal lawyer, about the matter.
Atty. Tirol told Jose to send MONINA and her lawyer to his (Atty. Tirols) office in Iloilo. Jose then
wrote out a letter of introduction for MONINA addressed to Atty. Tirol. Jose relayed Atty. Tirols message
to MONINA through Mr. Atienza, then later, Atty. Tirol told Jose to go to Iloilo with a check
for P15,000.00. Jose complied, and at Atty. Tirols office, Jose saw MONINA, Atty. Tirol and his secretary
reading some documents. MONINA then expressed her willingness to sign the
document, sans revisions. Jose alleged that he drew the P15,000.00 from his personal funds, subject to
reimbursement from and due to an understanding with FRANCISCO.
Dolores Argenal, a househelper at Nelly Garden from May 1944 up to May 1946, testified that she
knew that Pansay was Lourdes nanny; that Lourdes slept in her parents room; that she had not seen
FRANCISCO give special treatment to Pansay; that there was no unusual relationship between
FRANCISCO and Pansay, and if there was any, Dolores would have easily detected it since she slept in
the same room as Pansay. Dolores further declared that whenever FRANCISCOs wife was out of
town, Pansay would bring Lourdes downstairs at nighttime, and that Pansay would not sleep in the room
where FRANCISCO slept. Finally, Dolores declared that Pansay stopped working for FRANCISCO and
his wife in October, 1944.
The reception of evidence having been concluded, the parties filed their respective memoranda.
It need be recalled that Judge Catalino Castaeda, Jr. presided over trial up to 21 October 1986,
thereby hearing only the testimonies of MONINAs witnesses and about half of MONINAs testimony on
direct examination. Judge Norberto E. Devera, Jr. heard the rest of MONINA's testimony and those of
FRANCISCOs witnesses.
In its decision of 12 November 1990
[12]
the trial court, through Judge Devera, dismissed the
complaint with costs against MONINA. In the opening paragraph thereof, it observed:
This is a complaint for recognition of an illegitimate child instituted by plaintiff Monina Jison against
defendant Francisco Jison. This complaint was filed on March 13, 1985 at the time when plaintiff,
reckoned from her death of birth, was already thirty-nine years old. Noteworthy also is the fact that it was
instituted twenty years after the death of plaintiffs mother, Esperanza Amolar. For the years between
plaintiffs birth and Esperanzas death, no action of any kind was instituted against defendant either by
plaintiff, her mother Esperanza or the latters parents. Neither had plaintiff brought such an action against
defendant immediately upon her mothers death on April 20, 1965, considering that she was then already
nineteen years old or, within a reasonable time thereafter. Twenty years more had to supervene before
this complaint was eventually instituted.
The trial court then proceeded to discuss the four issues stipulated at pre-trial, without, however,
summarizing the testimonies of the witnesses nor referring to the testimonies of the witnesses other than
those mentioned in the discussion of the issues.
The trial court resolved the first issue in the negative, holding that it was improbable for witness Lope
Amolar to have noticed that Pansay was pregnant upon seeing her at the Elena Apartments in November
1945, since Pansay was then only in her first month of pregnancy; that there was no positive assertion
that copulation did indeed take place between Francisco and Esperanza; and that MONINAs attempt to
show opportunity on the part of FRANCISCO failed to consider that there was also the opportunity for
copulation between Esperanza and one of the several domestic helpers admittedly also residing at Nellys
Garden at that time. The RTC also ruled that the probative value of the birth and baptismal certificates of
MONINA paled in light of jurisprudence, especially when the misspellings therein were considered.
The trial court likewise resolved the second issue in the negative, finding that MONINAs evidence
thereon may either be one of three categories, namely: hearsay evidence, incredulous evidence, or self-
serving evidence." To the first category belonged the testimonies of Adela Casabuena and Alfredo
Baylosis, whose knowledge of MONINAs filiation was based, as to the former, on utterances of
defendants wife Lilia and Esperanza allegedly during the heat of their quarrel, while as to the latter,
Alfredo's conclusion was based from the rumors going [around] that plaintiff is defendants daughter,
from his personal observation of plaintiffs facial appearance which he compared with that of defendants
and from the way the two (plaintiff and defendant) acted and treated each other on one occasion that he
had then opportunity to closely observe them together. To the second category belonged that of
Dominador Savariz, as:
At each precise time that Esperanza allegedly visited Nellys Garden and allegedly on those occasions
when defendants wife, Lilia was in Manila, this witness was there and allegedly heard pieces of
conversation between defendant and Esperanza related to the paternity of the latters child. xxx
The RTC then placed MONINAs testimony regarding the acts of recognition accorded her by
FRANCISCOs relatives under the third category, since the latter were never presented as witnesses, for
which reason the trial court excluded the letters from FRANCISCOs relatives (Exhs. S to V).
As to the third issue, the trial court held that MONINA was not barred by prescription for it was of the
perception that the benefits of Article 268 accorded to legitimate children may be availed of or
extended to illegitimate children in the same manner as the Family Code has so provided; or by laches,
which is [a] creation of equity applied only to bring equitable results, and addressed to the sound
discretion of the court [and] the circumstances [here] would show that whether plaintiff filed this case
immediately upon the death of her mother Esperanza in 1965 or twenty years thereafter in 1985, xxx
there seems to be no inequitable result to defendant as related to the situation of plaintiff.
The RTC ruled, however, that MONINA was barred by estoppel by deed because of the affidavit
(Exh. P/Exh. 2) which she signed when she was already twenty-five years, a professional and under
the able guidance of counsel.
Finally, the RTC denied FRANCISCOs claim for damages, finding that MONINA did not file the
complaint with malice, she having been propelled by an honest belief, founded on probable cause.
MONINA seasonably appealed to the Court of Appeals (CA-G.R. CV No. 32860) and sought reversal
of the trial courts decision on the grounds that:
I
THE TRIAL COURT WAS ERRONEOUSLY PREDISPOSED TO ADJUDGE THIS CASE
AGAINST APPELLANT DUE TO ITS MISPERCEPTION THAT APPELLANTS DELAY IN
FILING HER COMPLAINT WAS FATAL TO HER CASE.
II
THE TRIAL COURT ERRED IN ITS REJECTION OF THE TESTIMONIES OF
APPELLANTS WITNESSES AS TAILOR-MADE, INADEQUATE AND INCREDIBLE.
III
THE TRIAL COURT ERRED IN ITS REJECTION OF THE ADMISSIBILITY OF THE
CERTIFIED COPIES OF PUBLIC DOCUMENTS PRESENTED BY APPELLANT AS PART
OF HER EVIDENCE.
IV
THE TRIAL COURT ERRED IN ITS REQUIREMENT THAT A WITNESS TO THE ACTUAL
ACT OF COPULATION BETWEEN THE APPELLEE AND APPELLANTS MOTHER
SHOULD HAVE POSITIVELY TESTIFIED TO SAID EFFECT.
V
THE TRIAL COURT ERRED IN REJECTING THE ADMISSIBILITY OF THE DULY
IDENTIFIED NOTES AND LETTER OF THE RELATIVES OF THE APPELLEE AS
HEARSAY.
VI
THE TRIAL COURT ERRED IN CONCLUDING THAT APPELLANTS AFFIDAVIT (EXH. P)
SERVED AS A BAR AGAINST HER CLAIM FOR RECOGNITION INSTEAD OF
REINFORCING SAID CLAIM.
[13]

Expectedly, FRANCISCO refuted these alleged errors in his Appellees Brief.
[14]

In its decision of 27 April 1995,
[15]
the Court of Appeals initially declared that as no vested or
acquired rights were affected, the instant case was governed by Article 175, in relation to Articles 172 and
173, of the Family Code.
[16]
While the Court of Appeals rejected the certifications issued by the Local Civil
Registrar of Dingle, Iloilo (Exhs. E and F) as FRANCISCO did not sign them, said court focused its
discussion on the other means by which illegitimate filiation could be proved, i.e., the open and
continuous possession of the status of an illegitimate child or, by any other means allowed by the Rules
of Court and special laws, such as the baptismal certificate of the child, a judicial admission, a family
bible wherein the name of the child is entered, common reputation respecting pedigree, admission by
silence, testimonies of witnesses xxx.
[17]
To the Court of Appeals, the bottom line issue was whether or
not MONINA established her filiation as FRANCISCOs illegitimate daughter by preponderance of
evidence, as to which issue said court found:
[N]ot just preponderant but overwhelming evidence on record to prove that [MONINA] is the illegitimate
daughter of [FRANCISCO] and that she had continuously enjoyed such status by direct acts of
[FRANCISCO] and/or his relatives.
In so ruling, the Court of Appeals observed that the testimonies of Lope Amolar, Adela Casabuena
and Dominador Savariz were already sufficient to establish MONINAs filiation:
As adverted to earlier, the trial court discredited Lope Amolars testimony by saying that Lope could not
have detected Esperanzas pregnant state in November, 1945 since at that point in time [sic] she was still
in the initial stage of pregnancy. Apparently, the trial court paid more emphasis on the date mentioned by
Lope Amolar than on the tenor and import of his testimony. As xxx Lope xxx was asked about an incident
that transpired more than 41 years back, [u]nder the circumstances, it is unreasonable to expect that
Lope could still be dead right on the specific month in 1945 that [he] met and confronted his sister. At any
rate, what is important is not the month that they met but the essence of his testimony that his sister
pointed to their employer [FRANCISCO] as the one responsible for her pregnancy, and that upon being
confronted, [FRANCISCO] assured him of support for Esperanza and their child. It would appear then
that in an attempt to find fault with Lopes testimony, the trial court has fallen oblivious to the fact that
even [FRANCISCO], in his deposition, did not deny that he was confronted by Lope about what he had
done to Esperanza, during which he unequivocally acknowledged paternity by assuring Lope of support
for both Esperanza and their child.
The Court of Appelas further noted that Casabuena and Savariz testified on something that they
personally observed or witnessed, which matters FRANCISCO did not deny or refute. Finally, said court
aptly held:
Taking into account all the foregoing uncontroverted testimonies xxx let alone such circumstantial
evidence as [MONINAs] Birth Certificates xxx and Baptismal Certificates which invariably bear the name
of [FRANCISCO] as her father, We cannot go along with the trial courts theory that [MONINAs]
illegitimate filiation has not been satisfactorily established.
xxx
Significantly, [MONINAs] testimony finds ample corroboration from [FRANCISCOs] former employees,
Arsenio Duatin, Rudy Tingson and Alfredo Baylosis. xxx
xxx
Carefully evaluating appellants evidence on her enjoyment of the status of an illegitimate daughter of
[FRANCISCO] vis-a-vis [FRANCISCOs] controversion thereof, We find more weight in the former. The
positive testimonies of [MONINA] and [her] witnesses xxx all bearing on [FRANCISCOs] acts and/or
conduct indubitably showing that he had continuously acknowledged [MONINA] as his illegitimate
daughter have not been succeessfully [sic] refuted. In fact, [FRANCISCO] himself, in his deposition, only
casually dismissed [MONINAs] exhaustive and detailed testimony as untrue, and with respect to those
given by [MONINAs] witnesses, he merely explained that he had fired [them] from their
employment. Needless to state, [FRANCISCOs] vague denial is grossly inadequate to overcome the
probative weight of [MONINAs] testimonial evidence.
Even the affidavit (Exh 2) which [FRANCISCO] had foisted on the trial court xxx does not hold sway in the
face of [MONINAs] logical explanation that she at first did agree to sign the affidavit which contained
untruthful statements. In fact, she promptly complained to [FRANCISCO] who, however explained to her
that the affidavit was only for the consumption of his spouse xxx. Further, the testimony of Jose Cruz
concerning the events that led to the execution of the affidavit xxx could not have been true, for as
pointed out by [MONINA], she signed the affidavit xxx almost five months after she had resigned from the
Miller, Cruz & Co. xxx
At any rate, if [MONINA] were not his illegitimate daughter, it would have been uncalled for, if not absurd,
for [FRANCISCO] or his lawyer to have secured [MONINAs] sworn statement xxx On the contrary, in
asking [MONINA] to sign the said affidavit at the cost of P15,000, [FRANCISCO] clearly betrayed his
intention to conceal or suppress his paternity of [MONINA]. xxx
In fine, We hold that [MONINAs] filiation as [FRANCISCOs] illegitimate daughter has been conclusively
established by the uncontroverted testimonies of Lope Amolar, Adela Casabuena and Dominador Savariz
to the effect that appellee himself had admitted his paternity of the appellee, and also by the testimonies
of appellant, Arsenio Duatin, Romeo Bilbao, Rudy Tingson and Alfredo Baylosis unerringly demonstrating
that by his own conduct or overt acts like sending appellant to school, paying for her tuition fees, school
uniforms, books, board and lodging at the Colegio del Sagrado Corazon de Jesus, defraying appellants
hospitalization expenses, providing her with [a] monthly allowance, paying for the funeral expenses of
appellants mother, acknowledging appellants paternal greetings and calling appellant his Hija or child,
instructing his office personnel to give appellants monthly allowance, recommending appellant for
employment at the Miller, Cruz & Co., allowing appellant to use his house in Bacolod and paying for her
long distance telephone calls, having appellant spend her vacation in his apartment in Manila and also at
his Forbes residence, allowing appellant to use his surname in her scholastic and other records (Exhs Z,
AA, AA-1 to AA-5, W & W-5), appellee had continuously recognized appellant as his illegitimate
daughter. Added to these are the acts of [FRANCISCOs] relatives acknowledging or treating [MONINA]
as [FRANCISCOs] daughter (Exh U) or as their relative (Exhs T & V). On this point, witness Zafiro
Ledesma, former Mayor of Iloilo City, whose spouse belongs to the Lopez clan just like [FRANCISCO],
testified that [MONINA] has been considered by the Lopezes as a relative. He identified pictures of the
appellee in the company of the Lopezes (Exhs X-16 & X-17). Another witness, Danthea H. Lopez, whose
husband Eusebio Lopez is appellees first cousin, testified that appellant was introduced to her by
appellees cousin, Remedios Lopez Franco, as the daughter of appellee Francisco Jison, for which
reason, she took her in as [a] secretary in the Merchants Financing Corporation of which she was the
manager, and further allowed her to stay with her family free of board and lodging. Still on this aspect,
Dominador Savariz declared that sometime in February, 1966 appellees relative, Ms. Remedios Lopez
Franco pointed to appellant as the daughter of appellee Francisco Jison.
Finally, the Certifications of the Local Civil Registrar of Dingle (Exhs E and F) as well as [MONINAs]
Baptismal Certificates (Exhs C & D) which the trial ocurt admitted in evidence as part of [MONINAs]
testimony, may serve as circumstantial evidence to further reinforce [MONINAs] claim that she is
[FRANCISCOs] illegitimate daughter by Esperanza Amolar.
True it is that a trial judges assessment of the credibility of witnesses is accorded great respect on
appeal. But the rule admits of certain exceptions. One such exception is where the judge who rendered
the judgment was not the one who heard the witnesses testify. [citations omitted] The other is where the
trial court had overlooked, misunderstood or misappreciated some facts or circumstances of weight and
substance which, if properly considered, might affect the result of the case. [citations omitted] In the
present case, both exceptions obtain. All of [MONINAs] witnesses xxx whose testimonies were not given
credence did not testify before the judge who rendered the disputed judgment. xxx
The Court of Appeals then decreed:
WHEREFORE, premises considered, the judgment of the trial court is SET ASIDE and another one is
hereby entered for appellant Monina Jison, declaring her as the illegitimate daughter of appellee
Francisco Jison, and entitled to all rights and privileges granted by law.
Costs against appellee.
SO ORDERED.
His motion for reconsideration having been denied by the Court of Appeals in its resolution of 29
March 1996,
[18]
FRANCISCO filed the instant petition. He urges us to reverse the judgment of the Court
of Appeals, alleging that said court committed errors of law:
I.
IN REVERSING THE DECISION OF THE TRIAL COURT AND DECLARING PRIVATE
RESPONDENT AS THE ILLEGITIMATE CHILD OF PETITIONER, CONSIDERING [THE]
IMPOSSIBILITY OF SEXUAL CONTACT BETWEEN THE PETITIONER AND THE
PRIVATE RESPONDENT'S MOTHER AT THE TIME CONCEPTION WAS SUPPOSED TO
HAVE OCCURRED.
II.
IN REVERSING THE TRIAL COURTS FINDING CONSIDERING THAT PRIVATE
RESPONDENT'S TESTIMONIAL EVIDENCE OF PATERNITY AND FILIATION IS NOT
CLEAR AND CONVINCING.
III.
IN GIVING CREDENCE TO DOCUMENTARY EVIDENCE PRESENTED BY THE
PRIVATE RESPONDENT AS EVIDENCE OF FILIATION CONSIDERING THAT THE SAME
ARE HEARSAY, SELF-SERVING AND CANNOT BIND THE PETITIONER UNDER THE
BASIC RULES OF EVIDENCE.
IV.
IN INTERPRETING THE PRIVATE RESPONDENT'S SWORN STATEMENT (EXH.
P/EXH. 2) IN A MANNER NOT IN CONSONANCE WITH THE RULINGS OF THE
HONORABLE SUPREME COURT.
V.
IN NOT CONSIDERING THE LONG AND UNEXPLAINED DELAY IN THE FILING OF
THE PRESENT PATERNITY SUIT AS EQUIVALENT TO LACHES.
As regards the first error, FRANCISCO insists that taking into account the second paragraph of
MONINAs complaint wherein she claimed that he and Pansay had sexual relations by about the end of
1945 or the start of 1946, it was physically impossible for him and Pansay to have had sexual contact
which resulted in MONINAs birth, considering that:
The normal period of human pregnancy is nine (9) months. If as claimed by private respondent in her
complaint that her mother was impregnated by FRANCISCO at the end of 1945 or the start of 1946, she
would have been born sometime in late September or early October and not August 6, 1946 xxx. The
instant case finds factual and legal parallels in Constantino vs. Mendez,
[19]
thus: xxx
FRANCISCO further claims that his testimony that Pansay was no longer employed by him at the
time in question was unrebutted, moreover, other men had access to Pansay during the time of or even
after her employment by him.
As to the second error, FRANCISCO submits that MONINAs testimonial evidence is shaky,
contradictory and unreliable, and proceeds to attack the credibility of her witnesses by claiming, in the
main, that: (a) Lope Amolar could not have detected Pansays pregnancy in November 1945 when they
met since she would have been only one (1) month pregnant then; (b) Dominador Savariz did not in fact
witness the meeting between FRANCISCO, Pansay and MONINA; (c) Zafiro Ledesma had an ulterior
motive in testifying for MONINA as he owned a bank in Iloilo which was then under Central Bank
supervision and MONINA was the Bank Examiner assigned to Iloilo; and (d) Danthea Lopez was not
related to him by blood and whatever favorable treatment MONINA received from Danthea was due to the
formers employment at Merchants Financing Company and additional services rendered at Kahirup
Hotel; besides, Danthea admitted that she had no personal knowledge as to the issue of paternity and
filiation of the contending parties, hence Sections 39 and 40
[20]
of Rule 130 of the Rules of Court did not
come into play. FRANCISCO likewise re-echoes the view of the trial court as regards the testimonies of
Adela Casabuena and Alfredo Baylosis.
FRANCISCO further asserts that MONINAs testimony that he answered for her schooling was self-
serving and uncorroborated by any receipt or other documentary evidence; and assuming he did, such
should be interpreted as a manifestation of kindness shown towards the family of a former household
helper.
Anent the treatment given by his relatives to MONINA as his daughter, FRANCISCO points to the
fact that Pansay was the former laundrywoman of Mrs. Franco; MONINA resided with the families of
Eusebio Lopez and Concha Cuaycong because she was in their employ at Kahirup Hotel and Our Lady of
Mercy Hospital, respectively; MONINA failed to present Mrs. Franco, Eusebio Lopez and Mrs. Cuaycong;
and MONINAs employment at the accounting firm of Miller, Cruz & Co. was attributable to her
educational attainment, there being absolutely no evidence to prove that FRANCISCO ever facilitated her
employment thereat. Hence, in light of Baluyot v. Baluyot,
[21]
the quantum of evidence to prove paternity
by clear and convincing evidence, not merely a preponderance thereof, was not met.
With respect to the third assigned error, FRANCISCO argues that the Court of Appeals reliance on
the certifications of the Local Civil Registrar (Exhs. E and F) and Baptismal Certificates (Exhs. C and D)
as circumstantial evidence is misplaced. First, their genuineness could not be ascertained as the persons
who issued them did not testify. Second, in light of Reyes v. Court of Appeals,
[22]
the contents of the
baptismal certificates were hearsay, as the data was based only on what was told to the priest who
solemnized the baptism, who likewise was not presented as a witness. Additionally, the name of the
father appearing therein was Franque Jison, which was not FRANCISCOs name. Third, in both
Exhibits E and F, the names of the childs parents were listed as Frank Heson and Esperanza Amador
(not Amolar). FRANCISCO further points out that in Exhibit F, the status of the child is listed as
legitimate, while the fathers occupation as laborer. Most importantly, there was no showing that
FRANCISCO signed Exhibits E and F or that he was the one who reported the childs birth to the Office of
the Local Civil Registrar. As to MONINAs educational records, FRANCISCO invokes Baas v.
Baas
[23]
which recognized that school records are prepared by school authorities, not by putative
parents, thus incompetent to prove paternity. And, as to the photographs presented by MONINA,
FRANCISCO cites Colorado v. Court of Appeals,
[24]
and further asserts that MONINA did not present any
of the persons with whom she is seen in the pictures to testify thereon; besides these persons were, at
best, mere second cousins of FRANCISCO. He likewise assails the various notes and letters written by
his relatives (Exhs. S to V) as they were not identified by the authors. Finally, he stresses that MONINA
did not testify as to the telephone cards (Exhs. G to L) nor did these reveal the circumstances
surrounding the calls she made from his residence.
Anent the fourth assigned error, FRANCISCO contends that the Court of Appeals interpretation of
MONINAs affidavit of 21 September 1971 ran counter to Dequito v. Llamas,
[25]
and overlooked that at the
time of execution, MONINA was more than 25 years old and assisted by counsel.
As to the last assigned error, FRANCISCO bewails the Court of Appeals failure to consider the long
and unexplained delay in the filing of the case.
In her comment, MONINA forcefully refuted FRANCISCOs arguments, leading FRANCISCO to file
his reply thereto.
On 20 November 1996, we gave due course to this petition and required the parties to submit their
respective memoranda, which they subsequently did.
A painstaking review of the evidence and arguments fails to support petitioner.
Before addressing the merits of the controversy, we first dispose of preliminary matters relating to
the applicable law and the guiding principles in paternity suits. As to the former, plainly, the Family Code
of the Philippines (Executive Order No. 209) governs the present controversy. As correctly cited by the
Court of Appeals, Uyguangco
[26]
served as a judicial confirmation of Article 256 of the Family
Code
[27]
regarding its retroactive effect unless there be impairment of vested rights, which does not hold
true here, it appearing that neither the putative parent nor the child has passed away and the former
having actually resisted the latters claim below.
Under Article 175 of the Family Code, illegitimate filiation, such as MONINA's, may be established in
the same way and on the same evidence as that of legitimate children. Article 172 thereof provides the
various forms of evidence by which legitimate filiation is established, thus:
ART. 172. The filiation of legitimate children is established by any of the following:
(1) The record of birth appearing in the civil register or a final judgment; or
(2) An admission of legitimate filiation in a public document or a private handwritten
instrument signed by the parent concerned.
In the absence of the foregoing evidence, the legitimate filiation shall be proved by:
(1) The open and continuous possession of the status of a legitimate child; or
(2) Any other means allowed by the Rules of Court and special laws.
This Article reproduces, with amendments, Articles 265, 266 and 267 of the Civil Code.
For the success of an action to establish illegitimate filiation under the second paragraph, which
MONINA relies upon given that she has none of the evidence mentioned in the first paragraph, a high
standard of proof
[28]
is required. Specifically, to prove open and continuous possession of the status of
an illegitimate child, there must be evidence of the manifestation of the permanent intention of the
supposed father to consider the child as his, by continuous and clear manifestations of parental affection
and care, which cannot be attributed to pure charity. Such acts must be of such a nature that they reveal
not only the conviction of paternity, but also the apparent desire to have and treat the child as such in all
relations in society and in life, not accidentally, but continuously.
[29]

By continuous is meant uninterrupted and consistent, but does not require any particular length of
time.
[30]

The foregoing standard of proof required to establish ones filiation is founded on the principle that
an order for recognition and support may create an unwholesome atmosphere or may be an irritant in the
family or lives of the parties, so that it must be issued only if paternity or filiation is established by clear
and convincing evidence.
[31]

The foregoing discussion, however, must be situated within the general rules on evidence, in light of
the burden of proof in civil cases, i.e., preponderance of evidence, and the shifting of the burden of
evidence in such cases. Simply put, he who alleges the affirmative of the issue has the burden of proof,
and upon the plaintiff in a civil case, the burden of proof never parts. However, in the course of trial in a
civil case, once plaintiff makes out a prima facie case in his favor, the duty or the burden of evidence
shifts to defendant to controvert plaintiffs prima facie case, otherwise, a verdict must be returned in favor
of plaintiff. Moreover, in civil cases, the party having the burden of proof must produce a preponderance
of evidence thereon, with plaintiff having to rely on the strength of his own evidence and not upon the
weakness of the defendants. The concept of preponderance of evidence refers to evidence which is of
greater weight, or more convincing, that which is offered in opposition to it; at bottom, it means probability
of truth.
[32]

With these in mind, we now proceed to resolve the merits of the instant controversy.
FRANCISCOs arguments in support of his first assigned error deserve scant consideration. While it
has been observed that unlawful intercourse will not be presumed merely from proof of an opportunity for
such indulgence,
[33]
this does not favor FRANCISCO. Akin to the crime of rape where, in most instances,
the only witnesses to the felony are the participants in the sexual act themselves, in deciding paternity
suits, the issue of whether sexual intercourse actually occurred inevitably redounds to the victims or
mothers word, as against the accuseds or putative fathers protestations. In the instant case, MONINAs
mother could no longer testify as to the fact of intercourse, as she had, unfortunately, passed away long
before the institution of the complaint for recognition. But this did not mean that MONINA could no longer
prove her filiation. The fact of her birth and her parentage may be established by evidence other than the
testimony of her mother. The paramount question then is whether MONINAs evidence is coherent,
logical and natural.
[34]

The complaint stated that FRANCISCO had carnal knowledge of Pansay by about the end of
1945. We agree with MONINA that this was broad enough to cover the fourth quarter of said year,
hence her birth on 6 August 1946 could still be attributed to sexual relations between FRANCISCO and
MONINAs mother. In any event, since it was established that her mother was still in the employ of
FRANCISCO at the time MONINA was conceived as determined by the date of her birth, sexual contact
between FRANCISCO and MONINAs mother was not at all impossible, especially in light of the
overwhelming evidence, as hereafter shown, that FRANCISCO fathered MONINA, has recognized her as
his daughter and that MONINA has been enjoying the open and continuous possession of the status as
FRANCISCOs illegitimate daughter.
We readily conclude that the testimonial evidence offered by MONINA, woven by her narration of
circumstances and events that occurred through the years, concerning her relationship with FRANCISCO,
coupled with the testimonies of her witnesses, overwhelmingly established the following facts:
1) FRANCISCO is MONINAs father and she was conceived at the time when her mother was in the
employ of the former;
2) FRANCISCO recognized MONINA as his child through his overt acts and conduct which the
Court of Appeals took pains to enumerate, thus:
[L]ike sending appellant to school, paying for her tuition fees, school uniforms, books, board and
lodging at the Colegio del Sagrado de Jesus, defraying appellants hospitalization expenses,
providing her with [a] monthly allowance, paying for the funeral expenses of appellants mother,
acknowledging appellants paternal greetings and calling appellant his Hija or child, instructing
his office personnel to give appellants monthly allowance, recommending appellant for
employment at the Miller, Cruz & Co., allowing appellant to use his house in Bacolod and
paying for her long distance telephone calls, having appellant spend her vacation in his
apartment in Manila and also at his Forbes residence, allowing appellant to use his surname in
her scholastic and other records (Exhs Z, AA, AA-1 to AA-5, W & W-5)
3) Such recognition has been consistently shown and manifested throughout the years
publicly,
[35]
spontaneously, continuously and in an uninterrupted manner.
[36]

Accordingly, in light of the totality of the evidence on record, the second assigned error must fail.
There is some merit, however, in the third assigned error against the probative value of some of
MONINAs documentary evidence.
MONINAs reliance on the certification issued by the Local Civil Registrar concerning her birth (Exhs.
E and F) is clearly misplaced. It is settled that a certificate of live birth purportedly identifying the putative
father is not competent evidence as to the issue of paternity, when there is no showing that the putative
father had a hand in the preparation of said certificates, and the Local Civil Registrar is devoid of authority
to record the paternity of an illegitimate child upon the information of a third person.
[37]
Simply put, if the
alleged father did not intervene in the birth certificate, e.g., supplying the information himself, the
inscription of his name by the mother or doctor or registrar is null and void; the mere certificate by the
registrar without the signature of the father is not proof of voluntary acknowledgment on the latters
part.
[38]
In like manner, FRANCISCOs lack of participation in the preparation of the baptismal certificates
(Exhs. C and D) and school records (Exhs. Z and AA) renders these documents incompetent to prove
paternity, the former being competent merely to prove the administration of the sacrament of baptism on
the date so specified.
[39]
However, despite the inadmissibility of the school records per se to prove
paternity, they may be admitted as part of MONINAs testimony to corroborate her claim that
FRANCISCO spent for her education.
We likewise disagree with the ruling of the Court of Appeals that the certificates issued by the Local
Civil Registrar and the baptismal certificates may be taken as circumstantial evidence to prove MONINAs
filiation. Since they are per se inadmissible in evidence as proof of such filiation, they cannot be admitted
indirectly as circumstantial evidence to prove the same.
As to Exhibits S, T, U and V, the various notes and letters written by FRANCISCOs relatives,
namely Mike Alano, Emilio Jison, Mariquit Lopez and Fernando Lopez, respectively, allegedly attesting to
MONINAs filiation, while their due execution and authenticity are not in issue,
[40]
as MONINA witnessed
the authors signing the documents, nevertheless, under Rule 130, Section 39, the contents of these
documents may not be admitted, there being no showing that the declarants-authors were dead or unable
to testify, neither was the relationship between the declarants and MONINA shown by evidence other
than the documents in question.
[41]
As to the admissibility of these documents under Rule 130, Section
40, however, this requires further elaboration.
Rule 130, Section 40, provides:
Section 40. Family reputation or tradition regarding pedigree. -- The reputation or tradition existing in a
family previous to the controversy, in respect to the pedigree of any one of its members, may be received
in evidence if the witness testifying thereon be also a member of the family, either by consanguinity or
affinity. Entries in family bibles or other family books or charts, engravings on rings, family portraits and
the like, may be received as evidence of pedigree. (underscoring supplied)
It is evident that this provision may be divided into two (2) parts: the portion containing the first
underscored clause which pertains to testimonial evidence, under which the documents in question may
not be admitted as the authors thereof did not take the witness stand; and the section containing the
second underscored phrase. What must then be ascertained is whether Exhibits S to V, as private
documents, fall within the scope of the clause and the like as qualified by the preceding phrase [e]ntries
in family bibles or other family books or charts, engravings on rights [and] family portraits.
We hold that the scope of the enumeration contained in the second portion of this provision, in light
of the rule of ejusdem generis, is limited to objects which are commonly known as family possessions,
or those articles which represent, in effect, a familys joint statement of its belief as to the pedigree of a
person.
[42]
These have been described as objects openly exhibited and well known to the family,
[43]
or
those which, if preserved in a family, may be regarded as giving a family tradition.
[44]
Other examples of
these objects which are regarded as reflective of a familys reputation or tradition regarding pedigree are
inscriptions on tombstones,
[45]
monuments or coffin plates.
[46]

Plainly then, Exhibits S to V, as private documents not constituting "family possessions" as
discussed above, may not be admitted on the basis of Rule 130, Section 40. Neither may these exhibits
be admitted on the basis of Rule 130, Section 41 regarding common reputation,
[47]
it having been
observed that:
[T]he weight of authority appears to be in favor of the theory that it is the general repute, the common
reputation in the family, and not the common reputation in community, that is a material element of
evidence going to establish pedigree. xxx [Thus] matters of pedigree may be proved by reputation in the
family, and not by reputation in the neighborhood or vicinity, except where the pedigree in question is
marriage which may be proved by common reputation in the community.
[48]

Their inadmissibility notwithstanding, Exhibits S to V, inclusive, may, in like manner as MONINA's
school records, properly be admitted as part of her testimony to strengthen her claim that, indeed,
relatives of FRANCISCO recognized her as his daughter.
We now direct our attention to MONINAs 21 September 1971 affidavit (Exh. P/Exh. 2), subject of the
fourth assigned error, where she attests that FRANCISCO is not her father. MONINA contends that she
signed it under duress, i.e., she was jobless, had no savings and needed the money to support herself
and finish her studies. Moreover, she signed Exhibit P upon the advice of Atty. Divinagracia that filiation
could not be waived and that FRANCISCOs ploy would boomerang upon him. On the other hand,
FRANCISCO asserts that full credence should be afforded Exhibit P as MONINA was already 25 years
old at the time of its execution and was advised by counsel; further, being a notarized document, its
genuineness and due execution could not be questioned. He relies on the testimony of Jose Cruz, a
partner at the accounting firm of Miller & Cruz, who declared that he intervened in the matter as MONINA
was spreading rumors about her filiation within the firm, which might have had deleterious effects upon
the relationship between the firm and FRANCISCO.
On this issue, we find for MONINA and agree with the following observations of the Court of Appeals:
Even the affidavit (Exh 2) which [FRANCISCO] had foisted on the trial court xxx does not hold sway in the
face of [MONINAs] logical explanation that she at first did agree to sign the affidavit which contained
untruthful statements. In fact, she promptly complained to [FRANCISCO] who, however explained to her
that the affidavit was only for the consumption of his spouse xxx.
At any rate, if [MONINA] were not his illegitimate daughter, it would have been uncalled for, if not absurd,
for [FRANCISCO] or his lawyer to have secured [MONINAs] sworn statement xxx On the contrary, in
asking [MONINA] to sign the said affidavit at the cost of P15,000, [FRANCISCO] clearly betrayed his
intention to conceal or suppress his paternity of [MONINA]. xxx
Indeed, if MONINA were truly not FRANCISCOs illegitimate daughter, it would have been
unnecessary for him to have gone to such great lengths in order that MONINA denounce her filiation. For
as clearly established before the trial court and properly appreciated by the Court of Appeals, MONINA
had resigned from Miller & Cruz five (5) months prior to the execution of the sworn statement in question,
hence negating FRANCISCOs theory of the need to quash rumors circulating within Miller & Cruz
regarding the identity of MONINAs father. Hence, coupled with the assessment of the credibility of the
testimonial evidence of the parties discussed above, it is evident that the standard to contradict a notarial
document, i.e., clear and convincing evidence and more than merely preponderant,
[49]
has been met by
MONINA.
Plainly then, the burden of evidence fully shifted to FRANCISCO.
Two (2) glaring points in FRANCISCOs defense beg to be addressed: First, that his testimony was
comprised of mere denials, rife with bare, unsubstantiated responses such as That is not true, I do not
believe that, or None that I know. In declining then to lend credence to FRANCISCOs testimony, we
resort to a guiding principle in adjudging the credibility of a witness and the truthfulness of his statements,
laid down as early as 1921:
The experience of courts and the general observation of humanity teach us that the natural limitations of
our inventive faculties are such that if a witness undertakes to fabricate and deliver in court a false
narrative containing numerous details, he is almost certain to fall into fatal inconsistencies, to make
statements which can be readily refuted, or to expose in his demeanor the falsity of his message.
For this reason it will be found that perjurers usually confine themselves to the incidents immediately
related to the principal fact about which they testify, and when asked about collateral facts by which their
truthfulness could be tested, their answers not infrequently take the stereotyped form of such expressions
as I dont know or I dont remember. xxx
[50]

Second, the reasons for the dismissals of Tingson, Baylosis and Savariz were unspecified or
likewise unsubstantiated, hence FRANCISCOs attempt to prove ill-motive on their part to falsely testify in
MONINAs favor may not succeed. As may be gleaned, the only detail which FRANCISCO could furnish
as to the circumstances surrounding the dismissals of his former employees was that Baylosis allegedly
took advantage of his position while FRANCISCO was in the United States. But aside from this bare
claim, FRANCISCOs account is barren, hence unable to provide the basis for a finding of bias against
FRANCISCO on the part of his former employees.
As to FRANCISCOs other witnesses, nothing substantial could be obtained either. Nonito Jalandoni
avowed that he only came to know of MONINA in June 1988;
[51]
that during his employment at Nelly
Garden from 1963 up to 1974, he did not recall ever having seen MONINA there, neither did he know of
any instructions from FRANCISCO nor Mr. Lagarto (FRANCISCOs office manager before passing away)
regarding the disbursement of MONINAs allowance.
[52]
Teodoro Zulla corroborated Jalandonis testimony
regarding not having seen MONINA at Nelly Garden and MONINAs allowance; declared that Alfredo
Baylosis was dismissed due to discrepancies discovered after an audit, without any further elaboration,
however; but admitted that he never prepared the vouchers pertaining to FRANCISCOs personal
expenses, merely those intended for one of FRANCISCOs haciendas.
[53]
Then, Iigo Superticioso
confirmed that according to the report of a certain Mr. Atienza, Baylosis was dismissed by Mr. Jison for
irregularities, while Superticioso was informed by FRANCISCO that Tingson was dismissed for loss of
confidence. Superticioso likewise denied that MONINA received money from FRANCISCOs office,
neither was there a standing order from FRANCISCO to release funds to her.
[54]

It is at once obvious that the testimonies of these witnesses for FRANCISCO are likewise insufficient
to overcome MONINAs evidence. The former merely consist of denials as regards the latters having
gone to Nelly Garden or having received her allowance from FRANCISCOs office, which, being in the
form of negative testimony, necessarily stand infirm as against positive testimony;
[55]
bare assertions as
regards the dismissal of Baylosis; ignorance of FRANCISCOs personal expenses incapable of evincing
that FRANCISCO did not provide MONINA with an allowance; or hearsay evidence as regards the cause
for the dismissals of Baylosis and Tingson. But what then serves as the coup de grce is that despite
Superticiosos claim that he did not know MONINA,
[56]
when confronted with Exhibit H, a telephone toll
ticket indicating that on 18 May 1971, MONINA called a certain Eing at FRANCISCOs office,
Superticioso admitted that his nickname was Iing and that there was no other person named Iing in
FRANCISCOs office.
[57]

All told, MONINAs evidence hurdled the high standard of proof required for the success of an
action to establish ones illegitimate filiation when relying upon the provisions regarding open and
continuous possession or any other means allowed by the Rules of Court and special laws; moreover,
MONINA proved her filiation by more than mere preponderance of evidence.
The last assigned error concerning laches likewise fails to convince. The essential elements of
laches are: (1) conduct on the part of the defendant, or of one under whom he claims, giving rise to the
situation of which the complaint seeks a remedy; (2) delay in asserting the complainants rights, the
complainant having had knowledge or notice of the defendants conduct as having been afforded an
opportunity to institute a suit; (3) lack of knowledge or notice on the part of the defendant that the
complaint would assert the right in which he bases his suit; and (4) injury or prejudice to the defendant in
the event relief is accorded to the complainant, or the suit is not held barred.
[58]
The last element is the
origin of the doctrine that stale demands apply only where by reason of the lapse of time it would be
inequitable to allow a party to enforce his legal rights.
[59]

As FRANCISCO set up laches as an affirmative defense, it was incumbent upon him to prove the
existence of its elements. However, he only succeeded in showing MONINAs delay in asserting her
claim, but miserably failed to prove the last element. In any event, it must be stressed that laches is
based upon grounds of public policy which requires, for the peace of society, the discouragement of stale
claims, and is principally a question of the inequity or unfairness of permitting a right or claim to be
enforced or asserted. There is no absolute rule as to what constitutes laches; each case is to be
determined according to its particular circumstances. The question of laches is addressed to the sound
discretion of the court, and since it is an equitable doctrine, its application is controlled by equitable
considerations. It cannot be worked to defeat justice or to perpetuate fraud and injustice.
[60]
Since the
instant case involves paternity and filiation, even if illegitimate, MONINA filed her action well within the
period granted her by a positive provision of law. A denial then of her action on ground of laches would
clearly be inequitable and unjust.
WHEREFORE, IN VIEW OF THE FOREGOING, the petition is hereby DENIED and the challenged
decision of the Court of Appeals of 27 April 1995 in CA-G.R. CV No. 32860 is AFFIRMED.
Costs against petitioner.
SO ORDERED.
Bellosillo, Vitug, Panganiban, and Quisumbing, JJ., concur.
















































Republic of the Philippines
SUPREME COURT
Manila
THIRD DIVISION

G.R. No. L-57062 January 24, 1992
MARIA DEL ROSARIO MARIATEGUI, ET AL., petitioners,
vs.
HON. COURT OF APPEALS, JACINTO MARIATEGUI, JULIAN MARIATEGUI and PAULINA
MARIATEGUI,respondents.
Montesa, Albon & Associates for petitioners.
Parmenio B. Patacsil, Patacsil Twins Law Office for the heirs of the late Maria del Rosario
Mariategui.
Tinga, Fuentes & Tagle Firm for private respondents.

BIDIN, J .:
This is a petition for review on certiorari of the decision * of the Court of Appeals dated December
24, 1980 in CA-G.R. No. 61841, entitled "Jacinto Mariategui, et al. v. Maria del Rosario Mariategui,
et al.," reversing the judgment of the then Court of First Instance of Rizal, Branch VIII ** at Pasig,
Metro Manila.
The undisputed facts are as follows:
Lupo Mariategui died without a will on June 26, 1953 (Brief for respondents, Rollo, pp. 116; 8).
During his lifetime, Lupo Mariategui contracted three (3) marriages. With his first wife, Eusebia
Montellano, who died on November 8, 1904, he begot four (4) children, namely: Baldomera, Maria
del Rosario, Urbana and Ireneo. Baldomera died and was survived by her children named Antero,
Rufina, Catalino, Maria, Gerardo, Virginia and Federico, all surnamed Espina. Ireneo also died and
left a son named Ruperto. With his second wife, Flaviana Montellano, he begot a daughter named
Cresenciana who was born on May 8, 1910 (Rollo, Annex "A", p. 36).
Lupo Mariategui and Felipa Velasco (Lupo's third wife) got married sometime in 1930. They had
three children, namely: Jacinto, born on July 3, 1929, Julian, born on February 16, 1931 and
Paulina, born on April 19, 1938. Felipa Velasco Mariategui died in 1941 (Rollo, Ibid).
At the time of his death, Lupo Mariategui left certain properties which he acquired when he was still
unmarried (Brief for respondents, Rollo, pp. 116; 4). These properties are described in the complaint
as Lots Nos. 163, 66, 1346 and 156 of the Muntinglupa Estate (Rollo, Annex "A", p. 39).
On December 2, 1967, Lupo's descendants by his first and second marriages, namely, Maria del
Rosario, Urbana, Ruperto, Cresencia, all surnamed Mariategui and Antero, Rufina, Catalino, Maria,
Gerardo, Virginia and Federico, all surnamed Espina, executed a deed of extrajudicial partition
whereby they adjudicated unto themselves Lot No. 163 of the Muntinglupa Estate. Thereafter, Lot
No. 163 was the subject of a voluntary registration proceedings filed by the adjudicatees under Act
No. 496, and the land registration court issued a decree ordering the registration of the lot. Thus, on
April 1, 1971, OCT No. 8828 was issued in the name of the above-mentioned heirs. Subsequently,
the registered owners caused the subdivision of the said lot into Lots Nos. 163-A to 163-H, for which
separate transfer certificates of title were issued to the respective parties (Rollo, ibid).
On April 23, 1973, Lupo's children by his third marriage with Felipa Velasco (Jacinto, Julian and
Paulina) filed with the lower court an amended complaint claiming that Lot No. 163 together with
Lots Nos. 669, 1346 and 154 were owned by their common father, Lupo Mariategui, and that, with
the adjudication of Lot No. 163 to their co-heirs, they (children of the third marriage) were deprived of
their respective shares in the lots. Plaintiffs pray for partition of the estate of their deceased father
and annulment of the deed of extrajudicial partition dated December 2, 1967 (Petition, Rollo, p. 10).
Cresencia Mariategui Abas, Flaviana Mariategui Cabrera and Isabel Santos were impleaded in the
complaint as unwilling defendants as they would not like to join the suit as plaintiffs although they
acknowledged the status and rights of the plaintiffs and agreed to the partition of the parcels of land
as well as the accounting of their fruits (Ibid., Rollo, p. 8; Record on Appeal, p. 4).
The defendants (now petitioners) filed an answer with counterclaim (Amended Record on Appeal, p.
13). Thereafter, they filed a motion to dismiss on the grounds of lack of cause of action and
prescription. They specifically contended that the complaint was one for recognition of natural
children. On August 14, 1974, the motion to dismiss was denied by the trial court, in an order the
dispositive portion of which reads:
It is therefore the opinion of the Court that Articles 278 and 285 of the Civil Code
cited by counsel for the defendants are of erroneous application to this case. The
motion to dismiss is therefore denied for lack of merit.
SO ORDERED. (Ibid, p. 37).
However, on February 16, 1977, the complaint as well as petitioners' counterclaim were dismissed
by the trial court, in its decision stating thus:
The plaintiffs' right to inherit depends upon the acknowledgment or recognition of
their continuous enjoyment and possession of status of children of their supposed
father. The evidence fails to sustain either premise, and it is clear that this action
cannot be sustained. (Ibid, Rollo, pp. 67-68)
The plaintiffs elevated the case to the Court of Appeals on the ground that the trial court committed
an error ". . . in not finding that the parents of the appellants, Lupo Mariategui and Felipa Velasco
(were) lawfully married, and in holding (that) they (appellants) are not legitimate children of their said
parents, thereby divesting them of their inheritance . . . " (Rollo, pp. 14-15).
On December 24, 1980, the Court of Appeals rendered a decision declaring all the children and
descendants of Lupo Mariategui, including appellants Jacinto, Julian and Paulina (children of the
third marriage) as entitled to equal shares in the estate of Lupo Mariategui; directing the
adjudicatees in the extrajudicial partition of real properties who eventually acquired transfer
certificates of title thereto, to execute deeds of reconveyance in favor, and for the shares, of Jacinto,
Julian and Paulina provided rights of innocent third persons are not prejudiced otherwise the said
adjudicatees shall reimburse the said heirs the fair market value of their shares; and directing all the
parties to submit to the lower court a project of partition in the net estate of Lupo Mariategui after
payment of taxes, other government charges and outstanding legal obligations.
The defendants-appellees filed a motion for reconsideration of said decision but it was denied for
lack of merit. Hence, this petition which was given due course by the court on December 7, 1981.
The petitioners submit to the Court the following issues: (a) whether or not prescription barred
private respondents' right to demand the partition of the estate of Lupo Mariategui, and (b) whether
or not the private respondents, who belatedly filed the action for recognition, were able to prove their
successional rights over said estate. The resolution of these issues hinges, however, on the
resolution of the preliminary matter, i.e., the nature of the complaint filed by the private respondents.
The complaint alleged, among other things, that "plaintiffs are the children of the deceased spouses
Lupo Mariategui . . . and Felipa Velasco"; that "during his lifetime, Lupo Mariategui had repeatedly
acknowledged and confirmed plaintiffs as his children and the latter, in turn, have continuously
enjoyed such status since their birth"; and "on the basis of their relationship to the deceased Lupo
Mariategui and in accordance with the law on intestate succession, plaintiffs are entitled to inherit
shares in the foregoing estate (Record on Appeal, pp. 5 & 6). It prayed, among others, that plaintiffs
be declared as children and heirs of Lupo Mariategui and adjudication in favor of plaintiffs their lawful
shares in the estate of the decedent (Ibid, p. 10).
A perusal of the entire allegations of the complaint, however, shows that the action is principally one
of partition. The allegation with respect to the status of the private respondents was raised only
collaterally to assert their rights in the estate of the deceased. Hence, the Court of Appeals correctly
adopted the settled rule that the nature of an action filed in court is determined by the facts alleged in
the complaint constituting the cause of action (Republic vs. Estenzo, 158 SCRA 282 [1988]).
It has been held that, if the relief demanded is not the proper one which may be granted under the
law, it does not characterize or determine the nature of plaintiffs' action, and the relief to which
plaintiff is entitled based on the facts alleged by him in his complaint, although it is not the relief
demanded, is what determines the nature of the action (1 Moran, p. 127, 1979 ed., citing Baguioro
vs. Barrios, et al., 77 Phil. 120).
With respect to the legal basis of private respondents' demand for partition of the estate of Lupo
Mariategui, the Court of Appeals aptly held that the private respondents are legitimate children of the
deceased.
Lupo Mariategui and Felipa Velasco were alleged to have been lawfully married in or about 1930.
This fact is based on the declaration communicated by Lupo Mariategui to Jacinto who testified that
"when (his) father was still living, he was able to mention to (him) that he and (his) mother were able
to get married before a Justice of the Peace of Taguig, Rizal." The spouses deported themselves as
husband and wife, and were known in the community to be such. Although no marriage certificate
was introduced to this effect, no evidence was likewise offered to controvert these facts. Moreover,
the mere fact that no record of the marriage exists does not invalidate the marriage, provided all
requisites for its validity are present (People vs. Borromeo, 133 SCRA 106 [1984]).
Under these circumstances, a marriage may be presumed to have taken place between Lupo and
Felipa. The laws presume that a man and a woman, deporting themselves as husband and wife,
have entered into a lawful contract of marriage; that a child born in lawful wedlock, there being no
divorce, absolute or from bed and board is legitimate; and that things have happened according to
the ordinary course of nature and the ordinary habits of life (Section 5 (z), (bb), (cc), Rule 131, Rules
of Court; Corpus v. Corpus, 85 SCRA 567 [1978]; Saurnaba v. Workmen's Compensation, 85 SCRA
502 [1978]; Alavado v. City Gov't. of Tacloban, 139 SCRA 230 [1985]; Reyes v. Court of Appeals,
135 SCRA 439 [1985]).
Courts look upon the presumption of marriage with great favor as it is founded on the following
rationale:
The basis of human society throughout the civilized world is that of marriage.
Marriage in this jurisdiction is not only a civil contract, but it is a new relation, an
institution in the maintenance of which the public is deeply interested. Consequentl y,
every intendment of the law leans toward legalizing matrimony. Persons dwelling
together in apparent matrimony are presumed, in the absence of any
counterpresumption or evidence special to that case, to be in fact married. The
reason is that such is the common order of society and if the parties were not what
they thus hold themselves out as being, they would be living in the constant violation
of decency and of
law . . . (Adong vs. Cheong Seng Gee, 43 Phil. 43, 56 [1922] quoted in Alavado vs.
City Government of Tacloban, 139 SCRA 230 [1985]).
So much so that once a man and a woman have lived as husband and wife and such relationship is
not denied nor contradicted, the presumption of their being married must be admitted as a fact
(Alavado v. City Gov't. of Tacloban, supra).
The Civil Code provides for the manner under which legitimate filiation may be proven. However,
considering the effectivity of the Family Code of the Philippines, the case at bar must be decided
under a new if not entirely dissimilar set of rules because the parties have been overtaken by events,
to use the popular phrase (Uyguangco vs. Court of Appeals, G.R. No. 76873, October 26, 1989).
Thus, under Title VI of the Family Code, there are only two classes of children legitimate and
illegitimate. The fine distinctions among various types of illegitimate children have been eliminated
(Castro vs. Court of Appeals, 173 SCRA 656 [1989]).
Article 172 of the said Code provides that the filiation of legitimate children may be established by
the record of birth appearing in the civil register or a final judgment or by the open and continuous
possession of the status of a legitimate child.
Evidence on record proves the legitimate filiation of the private respondents. Jacinto's birth certificate
is a record of birth referred to in the said article. Again, no evidence which tends to disprove facts
contained therein was adduced before the lower court. In the case of the two other private
respondents, Julian and Paulina, they may not have presented in evidence any of the documents
required by Article 172 but they continuously enjoyed the status of children of Lupo Mariategui in the
same manner as their brother Jacinto.
While the trial court found Jacinto's testimonies to be inconsequential and lacking in substance as to
certain dates and names of relatives with whom their family resided, these are but minor details. The
nagging fact is that for a considerable length of time and despite the death of Felipa in 1941, the
private respondents and Lupo lived together until Lupo's death in 1953. It should be noted that even
the trial court mentioned in its decision the admission made in the affidavit of Cresenciana
Mariategui Abas, one of the petitioners herein, that " . . . Jacinto, Julian and Paulina Mariategui ay
pawang mga kapatid ko sa
ama . . ." (Exh. M, Record on Appeal, pp. 65-66).
In view of the foregoing, there can be no other conclusion than that private respondents are
legitimate children and heirs of Lupo Mariategui and therefore, the time limitation prescribed in
Article 285 for filing an action for recognition is inapplicable to this case. Corollarily, prescription does
not run against private respondents with respect to the filing of the action for partition so long as the
heirs for whose benefit prescription is invoked, have not expressly or impliedly repudiated the co-
ownership. In other words, prescription of an action for partition does not lie except when the co-
ownership is properly repudiated by the co-owner (Del Banco vs. Intermediate Appellate Court, 156
SCRA 55 [1987] citing Jardin vs. Hollasco, 117 SCRA 532 [1982]).
Otherwise stated, a co-owner cannot acquire by prescription the share of the other co-owners
absent a clear repudiation of co-ownership duly communicated to the other co-owners (Mariano vs.
De Vega, 148 SCRA 342 [1987]). Furthermore, an action to demand partition is imprescriptible and
cannot be barred by laches (Del Banco vs. IAC, 156 SCRA 55 [1987]). On the other hand, an action
for partition may be seen to be at once an action for declaration of co-ownership and for segregation
and conveyance of a determinate portion of the property involved (Roque vs. IAC, 165 SCRA 118
[1988]).
Petitioners contend that they have repudiated the co-ownership when they executed the extrajudicial
partition excluding the private respondents and registered the properties in their own names
(Petition, p. 16; Rollo, p. 20). However, no valid repudiation was made by petitioners to the prejudice
of private respondents. Assuming petitioners' registration of the subject lot in 1971 was an act of
repudiation of the co-ownership, prescription had not yet set in when private respondents filed in
1973 the present action for partition (Ceniza vs. C.A., 181 SCRA 552 [1990]).
In their complaint, private respondents averred that in spite of their demands, petitioners, except the
unwilling defendants in the lower court, failed and refused to acknowledge and convey their lawful
shares in the estate of their father (Record on Appeal, p. 6). This allegation, though denied by the
petitioners in their answer (Ibid, p. 14), was never successfully refuted by them. Put differently, in
spite of petitioners' undisputed knowledge of their relationship to private respondents who are
therefore their co-heirs, petitioners fraudulently withheld private respondent's share in the estate of
Lupo Mariategui. According to respondent Jacinto, since 1962, he had been inquiring from petitioner
Maria del Rosario about their (respondents) share in the property left by their deceased father and
had been assured by the latter (Maria del Rosario) not to worry because they will get some shares.
As a matter of fact, sometime in 1969, Jacinto constructed a house where he now resides on Lot No.
163 without any complaint from petitioners.
Petitioners' registration of the properties in their names in 1971 did not operate as a valid repudiation
of the co-ownership. In Adille vs. Court of Appeals (157 SCRA 455, 461-462 [1988]), the Court held:
Prescription, as a mode of terminating a relation of co-ownership, must have been
preceded by repudiation (of the co-ownership). The act of repudiation, in turn, is
subject to certain conditions: (1) a co-owner repudiates the co-ownership; (2) such
an act of repudiation is clearly made known to the other co-owners; (3) the evidence
thereon is clear and conclusive; and (4) he has been in possession through open,
continuous, exclusive, and notorious possession of the property for the period
required by law.
xxx xxx xxx
It is true that registration under the Torrens system is constructive notice of title, but it
has likewise been our holding that the Torrens title does not furnish shield for fraud. It
is therefore no argument to say that the act of registration is equivalent to notice of
repudiation, assuming there was one, notwithstanding the long-standing rule that
registration operates as a universal notice of title.
Inasmuch as petitioners registered the properties in their names in fraud of their co-heirs prescription
can only be deemed to have commenced from the time private respondents discovered the
petitioners' act of defraudation (Adille vs. Court of Appeals, supra). Hence, prescription definitely
may not be invoked by petitioners because private respondents commenced the instant action
barely two months after learning that petitioners had registered in their names the lots involved.
WHEREFORE, the petition is DENIED and the assailed decision of the Court of Appeals dated
December 24, 1980 is Affirmed.
SO ORDERED.
Gutierrez, Jr., Feliciano, Davide, Jr. and Romero, JJ., concur.





















Republic of the Philippines
SUPREME COURT
Manila
EN BANC
G.R. No. L-22523 September 29, 1967
IN THE MATTER OF THE ADOPTION OF THE MINOR, EDWIN VILLA Y MENDOZA. LUIS E.
SANTOS, JR. and EDIPOLA V. SANTOS, petitioners-appellants,
vs.
REPUBLIC OF THE PHILIPPINES, oppositor-appellee.
A. E. Dacanay for petitioners-appellants.
Office of the Solicitor General for oppositor-appellee.

ANGELES, J .:
An appeal from the decision of the Juvenile and Domestic Relations Court, in Special
Proceeding No. 0001, dismissing the petition instituted by the spouses Luis R. Santos, Jr. and
Edipola V. Santos for the adoption of the minor Edwin Villa y Mendoza.
The issue before Us is, whether or not an elder sister may adopt a younger brother.
The trial court dismissed the petition reasoning thus:
A critical consideration in this case is the fact that the parents of the minor to be
adopted are also the parents of the petitioner-wife. The minor, therefore, is the latter's
legitimate brother.
In this proceeding, the adoption will result in an incongruous situation where the minor
Edwin Villa, a legitimate brother of the petitioner-wife, will also be her son. In the opinion of
the court, that incongruity not neutralized by other circumstances absent herein, should
prevent the adoption.
The petitioners moved to reconsider the decision but the same was denied. Hence, this
appeal.
The facts are not disputed.
The above-named spouses filed the petition before the court a quo on January 8, 1963,
praying that the minor Edwin Villa y Mendoza, 4 years old, be declared their (petitioner's) son by
adoption. Evidence was presented that the order setting the case for hearing has been duly
published, Exhibit A. There having been no opposition registered to the petition, the petitioners were
permitted to adduce their evidence.
It was established that the petitioners are both 32 years of age, Filipinos, residing in the City of
Manila. They were married in 1957 and have maintained a conjugal home of their own. They do not
have a child of their own blood. Neither spouse has any legitimate, legitimated, illegitimate,
acknowledged natural child, or natural child by legal fiction, nor has any one of them been convicted
of a crime involving moral turpitude. Edwin Villa y Mendoza, 4 years old, is a child of Francisco Villa
and Florencia Mendoza who are the common parents of the petitioner-wife Edipola Villa Santos and
the minor. Luis E. Santos, Jr., is a lawyer, with business interests in a textile development enterprise
and the IBA electric plant, and is the general manager of Medry Inc. and the secretary-treasurer of
Bearen Enterprises. His income is approximately P600.00 a month. His co-petitioner-wife, is a nurse
by profession, with an average monthly earning of about P300.00.
It was also shown that Edwin Villa y Mendoza was born on May 22, 1958, Exhibit C. He was a
sickly child since birth. Due to the child's impairing health his parents entrusted him to the petitioners
who reared and brought him up for the years thereafter, and as a result, there developed between
the petitioners and the child, a deep and profound love for each other. The natural parents of the
minor testified that they have voluntarily given their consent to the adoption of their son by the
petitioners, and submitted their written consent and conformity to the adoption, and that they fully
understand the legal consequences of the adoption of their child by the petitioners.
We are not aware of any provision in the law, and none has been pointed to Us by the Office
of the Solicitor General who argues for the State in this case, that relatives, by blood or by affinity,
are prohibited from adopting one another. The only objection raised is the alleged "incongruity" that
will result in the relation of the petitioner-wife and the adopted, in the circumstance that the adopted
who is the legitimate brother of the adopter, will also be her son by adoption. The theory is,
therefore, advanced that adoption among people who are related by nature should not be allowed, in
order that dual relationship should not result, reliance being made upon the views expressed by this
Court in McGee vs. Republic. L-5387, April 29, 1954, 94 Phil. 820.1awp hl.n t
In that case, an American citizen, Clyde E. McGee married to a Filipina by whom he had one
child, instituted a proceeding for the adoption of two minor children of the wife had by her first
husband. The lower court granted the petition of McGee to adopt his two minor step-children. On
appeal by the State. We reversed the decision. We said:
The purpose of adoption is to establish a relationship of paternity and filiation where
none existed before. Where therefore the relationship of parent and child already exists
whether by blood or by affinity as in the case of illegitimate and step-children, it would be
unnecessary and superfluous to establish and super impose another relationship of parent
and child through adoption. Consequently, an express authorization of law like article 338 is
necessary, if not to render it proper and legal, at least, to remove any and all doubt on the
subject matter. Under this view, article 338 may not be regarded as a surplusage. That may
have been the reason why in the old Code of Civil Procedure, particularly its provisions
regarding adoption, authority to adopt a step-child by a step-father was provided in section
766 notwithstanding the general authorization in section 765 extended to any inhabitant of
the Philippines to adopt a minor child. The same argument of surplusage could plausibly
have been advanced as regards section 766, that is to say, section 766 was unnecessary
and superfluous because without it a step-father could adopt a minor step-child anyway.
However, the inserting of section 766 was not entirely without reason. It seems to be an
established principle in American jurisprudence that a person may not adopt his own relative,
the reason being that it is unnecessary to establish a relationship where such already exists
(the same philosophy underlying our codal provisions on adoption). So some states have
special laws authorizing the adoption of relatives such as a grandfather adopting a
grandchild and a father adopting his illegitimate or natural-child.
Notwithstanding the views thus expressed, a study of American precedents would reveal that
there is a variance in the decisions of the courts in different jurisdictions regarding, the matter of
adoption of relatives. It cannot be stated as a general proposition that the adoption of a blood
relative is contrary to the policy of the law, for in many states of the Union, no restriction of that sort
is contained in the statutes authorizing adoption, although laws of other jurisdiction expressly provide
that adoption may not take place within persons within a certain degree of relationship (1 Am. Jur.
628-629). Courts in some states hold that in the absence of express statutory restriction, a blood
relationship between the parties is not a legal impediment to the adoption of one by the other, and
there may be a valid adoption where the relation of parent and child already exists by nature (2 Am.
Jur. 2d 869). Principles vary according to the particular adoption statute of a state under which any
given case is considered. It would seem that in those states originally influenced by the civil law
countries where adoption originated, the rules are liberally construed, while in other states where
common law principles predominate, adoption laws are more strictly applied because they are
regarded to be in derogation of the common law.
Article 335 of the Civil Code enumerates those persons who may not adopt, and it has been
shown that petitioners-appellants herein are not among those prohibited from adopting. Article 339
of the same code names those who cannot be adopted, and the minor child whose adoption is under
consideration, is not one of those excluded by the law. Article 338, on the other hand, allows the
adoption of a natural child by the natural father or mother, of other illegitimate children by their father
or mother, and of a step-child by the step-father or stepmother. This last article is, of course,
necessary to remove all doubts that adoption is not prohibited even in these cases where there
already exist a relationship of parent and child between them by nature. To say that adoption should
not be allowed when the adopter and the adopted are related to each other, except in these cases
enumerated in Article 338, is to preclude adoption among relatives no matter how far removed or in
whatever degree that relationship might be, which in our opinion is not the policy of the law. The
interest and welfare of the child to be adopted should be of paramount consideration. Adoption
statutes, being humane and salutary, and designed to provide homes, care and education for
unfortunate children, should be construed so as to encourage the adoption of such children by
person who can properly rear and educate them (In re Havsgord's Estate, 34 S.D. 131, 147 N.W.
378).
With respect to the objection that the adoption in this particular case will result in a dual
relationship between the parties, that the adopted brother will also be the son of the adopting elder
sister, that fact alone should not prevent the adoption. One is by nature, while the other is by fiction
of law. The relationship established by the adoption is limited to the adopting parents and does not
extend to their other relatives, except as expressly provided by law. Thus, the adopted child cannot
be considered as a relative of the ascendants and collaterals of the adopting parents, nor of the
legitimate children which they may have after the adoption except that the law imposes certain
impediments to marriage by reason of adoption. Neither are the children of the adopted considered
as descendants of the adopter (Tolentino, Civil Code, Vol. I, 1960 Ed., p. 652, citing 1 Oyuelos 284;
Perez, Gonzales and Castan; 4-11 Enneccerus, Kipp & Wolff 177; Muoz P. 104). So even
considered in relation to the rules on succession which are in pari materia, the adoption under
consideration would not be objectionable on the ground alone of the resulting relationship between
the adopter and the adopted. Similar dual relationships also result under our law on marriage when
persons who are already related, by blood or by affinity, marry each other. But as long as the
relationship is not within the degrees prohibited by law, such marriages are allowed notwithstanding
the resulting dual relationship. And as We do not find any provision in the law that expressly
prohibits adoption among relatives, they ought not to be prevented.
For all the foregoing considerations, the decision appealed from is set aside, and the petition
for the adoption of the subject minor, granted. No pronouncement as to costs.
Concepcion, C.J., Reyes, J.B.L., Dizon, Makalintal, Bengzon, J.P., Zaldivar, Sanchez, Castro and
Fernando, JJ., concur.
Republic of the Philippines
SUPREME COURT
Manila
SECOND DIVISION
G.R. No. 182367 December 15, 2010
CHERRYL B. DOLINA, Petitioner,
vs.
GLENN D. VALLECERA, Respondent.
D E C I S I O N
ABAD, J .:
This case is about a mothers claim for temporary support of an unacknowledged child, which she
sought in an action for the issuance of a temporary protection order that she brought against the
supposed father.
The Facts and the Case
In February 2008 petitioner Cherryl B. Dolina filed a petition with prayer for the issuance of a
temporary protection order against respondent Glenn D. Vallecera before the Regional Trial Court
(RTC) of Tacloban City in P.O. 2008-02-07
1
for alleged woman and child abuse under Republic Act
(R.A.) 9262.
2
In filling out the blanks in thepro-forma complaint, Dolina added a handwritten prayer
for financial support
3
from Vallecera for their supposed child. She based her prayer on the latters
Certificate of Live Birth which listed Vallecera as the childs father. The petition also asked the RTC
to order Philippine Airlines, Valleceras employer, to withhold from his pay such amount of support
as the RTC may deem appropriate.
Vallecera opposed the petition. He claimed that Dolinas petition was essentially one for financial
support rather than for protection against woman and child abuses; that he was not the childs father;
that the signature appearing on the childs Certificate of Live Birth is not his; that the petition is a
harassment suit intended to force him to acknowledge the child as his and give it financial support;
and that Vallecera has never lived nor has been living with Dolina, rendering unnecessary the
issuance of a protection order against him.
On March 13, 2008
4
the RTC dismissed the petition after hearing since no prior judgment exists
establishing the filiation of Dolinas son and granting him the right to support as basis for an order to
compel the giving of such support. Dolina filed a motion for reconsideration but the RTC denied it in
its April 4, 2008 Order,
5
with an admonition that she first file a petition for compulsory recognition of
her child as a prerequisite for support. Unsatisfied, Dolina filed the present petition for review directly
with this Court.
The Issue Presented
The sole issue presented in this case is whether or not the RTC correctly dismissed Dolinas action
for temporary protection and denied her application for temporary support for her child.
The Courts Ruling
Dolina evidently filed the wrong action to obtain support for her child. The object of R.A. 9262 under
which she filed the case is the protection and safety of women and children who are victims of abuse
or violence.
6
Although the issuance of a protection order against the respondent in the case can
include the grant of legal support for the wife and the child, this assumes that both are entitled to a
protection order and to legal support.
Dolina of course alleged that Vallecera had been abusing her and her child.1av vphil But it became apparent
to the RTC upon hearing that this was not the case since, contrary to her claim, neither she nor her
child ever lived with Vallecera. As it turned out, the true object of her action was to get financial
support from Vallecera for her child, her claim being that he is the father. He of course vigorously
denied this.
To be entitled to legal support, petitioner must, in proper action, first establish the filiation of the
child, if the same is not admitted or acknowledged. Since Dolinas demand for support for her son is
based on her claim that he is Valleceras illegitimate child, the latter is not entitled to such support if
he had not acknowledged him, until Dolina shall have proved his relation to him.
7
The childs remedy
is to file through her mother a judicial action against Vallecera for compulsory recognition.
8
If filiation
is beyond question, support follows as matter of obligation.
9
In short, illegitimate children are entitled
to support and successional rights but their filiation must be duly proved.
10

Dolinas remedy is to file for the benefit of her child an action against Vallecera for compulsory
recognition in order to establish filiation and then demand support. Alternatively, she may directly file
an action for support, where the issue of compulsory recognition may be integrated and resolved.
11

It must be observed, however, that the RTC should not have dismissed the entire case based solely
on the lack of any judicial declaration of filiation between Vallecera and Dolinas child since the main
issue remains to be the alleged violence committed by Vallecera against Dolina and her child and
whether they are entitled to protection. But of course, this matter is already water under the bridge
since Dolina failed to raise this error on review. This omission lends credence to the conclusion of
the RTC that the real purpose of the petition is to obtain support from Vallecera.
While the Court is mindful of the best interests of the child in cases involving paternity and filiation, it
is just as aware of the disturbance that unfounded paternity suits cause to the privacy and peace of
the putative fathers legitimate family.
12
Vallecera disowns Dolinas child and denies having a hand in
the preparation and signing of its certificate of birth. This issue has to be resolved in an appropriate
case.
ACCORDINGLY, the Court DENIES the petition and AFFIRMS the Regional Trial Court of Tacloban
Citys Order dated March 13, 2008 that dismissed petitioner Cherryl B. Dolinas action in P.O. 2008-
02-07, and Order dated April 4, 2008, denying her motion for reconsideration dated March 28, 2008.
SO ORDERED.
ROBERTO A. ABAD
Associate Justice



Republic of the Philippines
SUPREME COURT
Manila
FIRST DIVISION
G.R. No. L-48219 February 28, 1979
MANUEL J. C. REYES, petitioner,
vs.
HON. LEONOR INES-LUCIANO, as Judge of the Juvenile & Domestic Relations Court, Quezon
City, COURT OF APPEALS and CELIA ILUSTRE-REYES, respondents.
Eriberto D. Ignacio for petitioner.
Gonzalo D. David for private respondent.

FERNANDEZ, J .:
This is a petition for certiorari to review the decision of the Court of Appeals in CA-G.R. No. 06928-
SP entitled "Manuel J. C. Reyes, petitioner, versus, The Hon. Leonor Ines-Luciano as Judge of the
Juvenile & Domestic Relations Court (Quezon City) and Celia Ilustre-Reyes, Respondents",
dismissing the petition to annul the order of the respondent Judge directing the petitioner to give
support pendente lite to his wife, Celia Ilustre-Reyes, private respondent herein, in the amount of
P40,000.00 a month.
1

The private petitioner, Celia Ilustre-Reyes, filed in the Juvenile and Domestic Relations Court of
Quezon City a complaint dated June 3, 1976 against her husband, Manuel J. C. Reyes, for legal
separation on the ground that the defendant had attempted to kill plaintiff. The pertinent allegations
of the complaint are:
6.8 On March 10, 1976, defendant went to V. Ilustre and attacked plaintiff. He
pummeled her with fist blows that floored her, then held her head and, with intent to
kill, bumped it several times against the cement floor. When she ran upstairs to her
father for protection, he pushed her at the stairway of 13 flights and she fell sliding to
the ground floor. Determined to finish her off, he again gave her a strong swing at her
abdomen which floored her half unconscious. Were it not for plaintiff's father, he
would have succeeded killing her;
6.9. On May 26, 1976, although on May 11 previous she ceased holding office with
defendant at Bel-Air Apartments elsewhere adverted to, she went thereto to get her
overnight bag. Upon seeing her, defendant yelled at her to get out of the office. When he
did not mind him, he suddenly doused her with a glass of grape juice, kicked her several
times that landed at her back and nape, and was going to hit her with a steel tray as her
driver, Ricardo Mancera, came due to her screams for help. For fear of further injury and
for life, she rushed to Precinct 5 at united Nations Avenue, Manila Metropolitan Police, for
assistance and protection;
2

The plaintiff asked for support pendente lite for her and her three children. The defendant, petitioner
herein, opposed the application for support pendente lite on the ground that his wife had committed
adultery with her physician.
The application for support pendente lite was set for hearing and submitted for resolution on the
basis of the pleadings and the documents attached thereto by the parties.
The respondent Judge issued an order dated March 15, 1977 granting plaintiff's prayer for
alimony pendente litein the amount of P5,000.00 a month commencing from June 1976.
3

The petitioner filed a motion for reconsideration reiterating that his wife is not entitled to support
during the pendency of the case, and, alleging that even if she entitled, the amount awarded was
excessive. The respondent Judge reduced the amount from P5,000.00 to P44,00.00 a month in an
order dated June 17, 1977.
4

Manuel J. C. Reyes filed a petition for certiorari in the Court of Appeals dated July 25, 1977 asking
that the order granting support pendente lite to private respondent. Celia Ilustre-Reyes, be annulled
on the ground that the respondent Judge, Leonor Ines-Luciano, had committed a grave abuse of
discretion or that said order be modified inasmuch as the amount awarded as support pendente
lite is excessive.
The Court of Appeals dismissed the petition because:
Considering the plight of the wife during the pendency of the case for legal separation
and that the husband appears to be financially capable of giving the support, We believe
that the petitioner has not presented a clear case of grave abuse of discretion on the part
of the respondent in issuing the questioned orders. We see no compelling reason to give
it due course.
5

The petitioner contends that the Court of Appeal committed the following error:
THE HON. COURT OF APPEALS GRIEVOUSLY ERRED IN A MANNER
AMOUNTING IT CAN ERROR OF LAW AND A DEPARTURE FROM THE
ACCEPTED NORMS LAID DOWN BY THIS HON. COURT IN THE CASES WE
SHALL LATER ON DISCUSS, IN REFUSING TO GIVE DUE COURSE TO THE
ORIGINAL PETITION FOR certiorari HEREIN AGAINST RESPONDENTS-
APPELLEES, AND IN AFFIRMING THE ORDERS FOR SUPPORT PENDENTE
LITE ANNEXES "F" AND "H" OF THIS PETITION WHEN HELD THAT
RESPONDENT-APPELLEE JUDGE DID NOT COMMIT ANY ABUSE OF
DISCRETION IN ISSUING SAID ORDERS, FOR THE REASONS THAT:
A. IN ACTIONS FOR LEGAL SEPARATION THE WIFE IS ENTITLED TO
SUPPORT FROM THE HUSBAND DESPITE THE FACT THAT A CASE FOR
ADULTERY HAD BEEN FILED BY THE HUSBAND AGAINST HER; AND
B. IN DETERMINING THE AMOUNT OF SUPPORT PENDENTE LITE, IT IS ENOUGH
THAT THE COURT ASCERTAIN THE KIND AND AMOUNT OF EVIDENCE EVEN BY
AFFIDAVITS ONLY OR OTHER DOCUMENTARY EVIDENCE APPEARING IN THE
RECORDS.
6

It is true that the adultery of the wife is a defense in an action for support however, the alleged
adultery of wife must be established by competent evidence. The allegation that the wife has
committed adultery will not bar her from the right receive support pendente lite. Adultery is a good
defense and if properly proved and sustained wig defeat the action.
7

In the instant case, at the hearing of the application for support pendente lite before the Juvenile and
Domestic Relations Court presided by the respondent Judge, Hon. Leonor Ines-Luciano the
petitioner did not present any evidence to prove the allegation that his wife, private respondent Celia
Ilustre-Reyes, had committed adultery with any person.
The petitioner has still the opportunity to adduce evidence on the alleged adultery of his wife when
the action for legal separation is heard on the merits before the Juvenile and Domestic Relations
Court of Quezon City. It is to be noted however, that as pointed out by the respondents in their
comment, the "private respondent was not asking support to be taken from petitioner's personal
funds or wherewithal, but from the conjugal propertywhich, was her documentary evidence ...".
8
It
is, therefore, doubtful whether adultery will affect her right to alimony pendente lite. In Quintana vs.
Lerma,
9
the action for support was based on the obligation of the husband to support his wife.
The contention of the petitioner that the order of the respondent Judge granting the private
respondent support pendente lite in the amount of P4,000.00 a month is not supported by the
allegations of the complaint for legal separation and by competent evidence has no merit.
The complaint or legal separation contains allegations showing that on at least two occasions the
defendant, petitioner herein, had made attempts to kill the private respondent. Thus it is alleged that
on March 10, 1976, the defendant attacked plaintiff, pummeled her with fist blows that floored her,
held her head and with intent to kill, bumped it several times against the cement floor and when she
ran upstairs to her father for protection, the petitioner pushed her at the stairway of thirteen (13)
flights and she fell sliding to the ground floor and defendant gave her a strong swing at her abdomen
which floored her half unconscious and were it not for plaintiff's father, defendant would have
succeeded in killing her. 10 It is also alleged that on May 26, 1976, the defendant doused Celia
Ilustre-Reyes with a glass of grape juice, kicked her several times at her back and nape and was
going to hit her with a steel tray if it were not for her driver who came due to her creams for help."
11

In fixing the amount of monthly support pendente lite of P4,000,00, the respondent judge did not act
capriciously and whimsically. When she originally fixed the amount of P5,000.00 a month, the
respondent Judge considered the following:
On record for plaintiff's cause are the following: that she and defendant were married
on January 18, 1958; that she is presently unemployed and without funds, thus, she
is being supported by her father with whom she resides: that defendant had been
maltreating her and Cried to kill her; that all their conjugal properties are in the
possession of defendant who is also president, Manager and Treasurer of their
corporation namely:
1. Standard Mineral Products, which was incorporated on February 9, 1959:
presently with paid-in capital of P295,670.00; assets and liabilities of P757,108.52;
Retained Earnings of P85,654.61: and majority stockholder is defendant;
2. Development and Technology Consultant Inc. incorporated on July 12, 1971, with
paid-in capital of P200,000.00; Assets and liabilities of P831,669.34; defendant owns
99% of the stocks; and last Retained Earnings is P98,879.84.
3. The Contra-Prop Marine Philippines, Inc. which was incorporated on October 3,
1975, with paid-in capital of P100,000 defendant owns 99% of the stocks.
To secure some of the of said Agreement of Counter-Guaranty Mortgage with Real
Estate, and Real Estate Mortgage were undertaken by plaintiff of their properties outside
of other accommodations; and that she needs of P5,000.00 a month for her support in
accordance with their station in life.
12

The amount of support pendente lite was reduced to P4,000.00 inasmuch as the children are in the
custody of the petitioner and are being supported by him.
It is thus seen that the respondent judge acted with due deliberation before fixing the amount of
support pendente lite in the amount of P4,000.00 a month.
In determining the amount to be awarded as support pendente lite it is not necessary to go fully into
the merits of the case, it being sufficient that the court ascertain the kind and amount of evidence
which it may deem sufficient to enable it to justly resolve the application, one way or the other, in
view of the merely provisional character of the resolution to be entered. Mere affidavits may satisfy
the court to pass upon the application for support pendente lite.
13
It is enough the the facts be
established by affidavits or other documentary evidence appearing in the record.
14

The private respondent has submitted documents showing that the corporations controlled by the
petitioner have entered into multi-million contracts in projects of the Ministry of Public Highways.
Considering the high cost of living due to inflation and the financial ability of the petitioner as shown
by the documents of record, We find that the amount of P4,000.00 a month granted by the
respondent Judge as alimonypendente lite to the private respondent is not excessive. There is no
showing that the respondent Judge has committed a grave abuse of discretion in granting said
support.
In a resolution dated July 31, 1978, this Court issued a temporary restraining order effective
immediately against the enforcement of the lower court's order giving support pendente lite to private
respondent in the sum of P4,000.00 monthly commencing June 1976 and in lieu thereof to allow
such support only to the extent of P1,000.00 a month.
15

Later the petitioner was required to pay the support at the rate of P1,000.00 a month which had
accumulated since June 1976 within ten (10) days from notice of the resolution:
16

The private respondent acknowledged on November 20, 1978 having received from the petitioner,
through his counsel a check in the amount of P30,000.00 as payment of support for the period from
June 1976 to November 1978 or thirty (30) months at P1,000.00 a month in compliance with the
resolution of this Court dated October 9, 1978.
In view of the foregoing, the support of P4,000.00 should be made to commence or, March 1, 1979.
WHEREFORE, the petition for certiorari is hereby denied and the decision of the Council of Appeals
sought to be reviewed is affirmed with the modification that the support pendente lite at the rate of
Four Thousand Pesos (P4.000.00) a month should commence from March 1, 1979 without
pronouncement as to costs.
SO ORDERED.
Teehankee (Chairman), Makasiar, Guerrero, De Castro and Melencio-Herrera, JJ., concur.

[ G. R. No. L- 10028, May 23, 1958 ]
HIPOLITA ALMACEN, PLAINTIFF AND APPELLEE, VS. TEODORO N. BALTAZAR, DEFENDANT AND
APPELLANT.

D E C I S I O N
ENDENCIA, J.:
This is an appeal from a decision rendered by the Court of First Instance of Manila ordering the
defendant-appellant to pay plaintiff-appellee a monthly support of P50.00 beginning with the month of
August, 1955. Despite the meagre amount involved in the case, it was brought to this Court on appeal
because the appellant only raised questions of law.
The facts of the case as found by the lower court are as follows: that plaintiff and defendant were legally
married on March 24, 1923; that in 1937, plaintiff committed adultery with one named Jose Navarro, a
cousin of defendant; that prior to the infidelity of the wife, the defendant himself has not been loyal to her,
he having been once confined at the hospital suffering from venereal disease; that the defendant
separated from the plaintiff after the latter's infidelity and while estranged from her he lived maritally with
another woman by the name of Lourdes Alvarez; that after their separation there has been a
reconciliation between them or at least a condonation by defendant of the acts committed by the wife as
shown by the fact that he has been sending her money for her support; that husband and wife were
in pari delicto and, therefore, defendant is bound to support the plaintiff because he has likewise been
unfaithful to her.
Appellant contends that the lower court erred:
1. in not taking plaintiff's adulterous act of infidelity as defense against her claim for support and in
not exempting him from the obligation to give such support; and
2. in finding that the evidence on record was sufficient to establish a condonation of plaintiff's
adulterous act and reconciliation between plaintiff and defendant.
As to the first assignment of error, we find that by the provisions of Art. 303 of the new Civil Code, the
obligation to support shall cease "when the recipient has committed some act which gives rise to
disinheritance;" that under Art. 921 (k) of the same Code, a spouse may be disinherited when she has
given cause for legal separation," and under Art. 97, one of the causes for legal separation is "adultery on
the part of the wife and concubinage on the part of the husband", as defined in the Penal Code.
Accordingly, if the plaintiff was the only one who committed adultery which is a good cause for
disinheritance and legal separation, defendant's theory would seem to be correct; but, in the present
case, we agree with the lower court's ruling that defendant is still bound to support his wife, firstly,
because plaintiff and defendant were both guilty of infidelity and before the filing of the action they had a
reconciliation or, at least, defendant had pardoned plaintiff's unfaithfulness, for which reason we may
apply Art. 922 of the aforesaid Code which provides that "a subsequent reconciliation between the
offender and the offended person deprives the latter of the right to disinherit, aid renders ineffectual any
disinheritance that may have been made;" secondly, the law on support (Title IX, Book I, Arts. 290-304,
Civil Code) contains no provision squarely applicable to the present case in which both parties had
committed infidelity, neither is there any provision to the effect that when both spouses committed marital
offenses against one another, one can no longer ask support from the other; thirdly, there is the general
principle that when two persons acted in bad faith, they should be considered as having acted in good
faith, which principle may be applied to the instant case to the effect that plaintiff and defendant being
in pari delicto, the latter cannot claim the adultery of the former as defense to evade the obligation to give
her support.
As to the second assignment of error, we find it also groundless, for the lower court declared that there
has been a condonation by the defendant of the acts committed by the wife because of the latter's
testimony and documentary evidence submitted (Exhibits A, B, C, F and J), which show that the
defendant had given money to the plaintiff on several occasions through third persons and, in our opinion,
such evidence is really sufficient to show condonation or reconciliation between plaintiff and defendant,
for had there been no condonation of plaintiff's infidelity and no reconciliation between her and defendant,
the latter would not certainly have given any amount of money for her support.
Defendant argues, however, that he and plaintiff never resumed their conjugal relationship and, therefore,
there has been no legal condonation of the acts of the erring wife; but it cannot be disputed that the act of
giving money to an erring wife and the fact proven in the case that no action was taken against her before
the courts of justice are sufficient to establish forgiveness amounting to condonation, for "condonation is
the forgiveness of one of the married parties of an offense which he knows the other has committed
against the other." (Words & Phrases 8A, pp. 19-20} At any rate, pardon or condonation does not require
sexual intercourse and it may be express or implied.
"Wherefore, finding no errors in the decision appealed from, the same is hereby affirmed with costs
against the defendant.
Paras, C. J., Bengzon, Montemayor, Reyes, A., Bautista Angelo, Labrador, Concepcion, Reyes, J.B.L.,
Endencia,and Felix, JJ., concur.




Republic of the Philippines
SUPREME COURT
Manila
THIRD DIVISION

G.R. No. 85044 June 3, 1992
MACARIO TAMARGO, CELSO TAMARGO and AURELIA TAMARGO, petitioners,
vs.
HON. COURT OF APPEALS, THE HON. ARISTON L. RUBIO, RTC Judge, Branch 20, Vigan,
Ilocos Sur; VICTOR BUNDOC; and CLARA BUNDOC, respondents.

FELICIANO, J .:
On 20 October 1982, Adelberto Bundoc, then a minor of 10 years of age, shot Jennifer Tamargo
with an air rifle causing injuries which resulted in her death. Accordingly, a civil complaint for
damages was filed with the Regional Trial Court, Branch 20, Vigan, Ilocos Sur, docketed as Civil
Case No. 3457-V, by petitioner Macario Tamargo, Jennifer's adopting parent, and petitioner spouses
Celso and Aurelia Tamargo, Jennifer's natural parents against respondent spouses Victor and Clara
Bundoc, Adelberto's natural parents with whom he was living at the time of the tragic incident. In
addition to this case for damages, a criminal information or Homicide through Reckless Imprudence
was filed [Criminal Case No. 1722-V] against Adelberto Bundoc. Adelberto, however, was acquitted
and exempted from criminal liability on the ground that he bad acted without discernment.
Prior to the incident, or on 10 December 1981, the spouses Sabas and Felisa Rapisura had filed a
petition to adopt the minor Adelberto Bundoc in Special Proceedings No. 0373-T before the then
Court of First Instance of Ilocos Sur. This petition for adoption was grunted on, 18 November 1982,
that is, after Adelberto had shot and killed Jennifer.
In their Answer, respondent spouses Bundoc, Adelberto's natural parents, reciting the result of the
foregoing petition for adoption, claimed that not they, but rather the adopting parents, namely the
spouses Sabas and Felisa Rapisura, were indispensable parties to the action since parental
authority had shifted to the adopting parents from the moment the successful petition for adoption
was filed.
Petitioners in their Reply contended that since Adelberto Bundoc was then actually living with his
natural parents, parental authority had not ceased nor been relinquished by the mere filing and
granting of a petition for adoption.
The trial court on 3 December 1987 dismissed petitioners' complaint, ruling that respondent natural
parents of Adelberto indeed were not indispensable parties to the action.
Petitioners received a copy of the trial court's Decision on 7 December 1987. Within the 15-day
reglementary period, or on 14 December 1987, petitioners filed a motion for reconsideration followed
by a supplemental motion for reconsideration on 15 January 1988. It appearing, however, that the
motions failed to comply with Sections 4 and 5 of Rule 15 of the Revised Rules of Court that
notice of the motion shall be given to all parties concerned at least three (3) days before the hearing
of said motion; and that said notice shall state the time and place of hearing both motions were
denied by the trial court in an Order dated 18 April 1988. On 28 April 1988, petitioners filed a notice
of appeal. In its Order dated 6 June 1988, the trial court dismissed the notice at appeal, this time
ruling that the notice had been filed beyond the 15-day reglementary period ending 22 December
1987.
Petitioners went to the Court of Appeals on a petition for mandamus and certiorari questioning the
trial court's Decision dated 3 December 1987 and the Orders dated 18 April 1988 and 6 June 1988,
The Court of Appeals dismissed the petition, ruling that petitioners had lost their right to appeal.
In the present Petition for Review, petitioners once again contend that respondent spouses Bundoc
are the indispensable parties to the action for damages caused by the acts of their minor child,
Adelberto Bundoc. Resolution of this Petition hinges on the following issues: (1) whether or not
petitioners, notwithstanding loss of their right to appeal, may still file the instant Petition; conversely,
whether the Court may still take cognizance of the case even through petitioners' appeal had been
filed out of time; and (2) whether or not the effects of adoption, insofar as parental authority is
concerned may be given retroactive effect so as to make the adopting parents the indispensable
parties in a damage case filed against their adopted child, for acts committed by the latter, when
actual custody was yet lodged with the biological parents.
1. It will be recalled that, petitioners' motion (and supplemental motion) for reconsideration filed
before the trial court, not having complied with the requirements of Section 13, Rule 41, and Section
4, Rule 15, of the Revised Rules of Court, were considered pro forma and hence did not interrupt
and suspend the reglementary period to appeal: the trial court held that the motions, not having
contained a notice of time and place of hearing, had become useless pieces of paper which did not
interrupt the reglementary period.
1
As in fact repeatedly held by this Court, what is mandatory is the
service of the motion on the opposing counsel indicating the time and place of hearing.
2

In view, however, of the nature of the issue raised in the instant. Petition, and in order that
substantial justice may be served, the Court, invoking its right to suspend the application of technical
rules to prevent manifest injustice, elects to treat the notice of appeal as having been seasonably
filed before the trial court, and the motion (and supplemental motion) for reconsideration filed by
petitioner in the trial court as having interrupted the reglementary period for appeal. As the Court
held in Gregorio v. Court of Appeals:
3

Dismissal of appeal; purely on technical grounds is frowned upon where the policy of the
courts is to encourage hearings of appeal on their merits. The rules of procedure ought
not be applied in a very rigid technical sense, rules of procedure are used only to help
secure not override, substantial justice. if d technical and rigid enforcement of the rules is
made their aim would be defeated.
4

2. It is not disputed that Adelberto Bundoc's voluntary act of shooting Jennifer Tamargo with an air
rifle gave rise to a cause of action on quasi-delict against him. As Article 2176 of the Civil Code
provides:
Whoever by act or omission causes damage to another, there being fault or
negligence, is obliged to pay for the damage done. Such fault or negligence, if there
is no pre-existing contractual relation between the parties, is called a quasi-delict . . .
Upon the other hand, the law imposes civil liability upon the father and, in case of his death or
incapacity, the mother, for any damages that may be caused by a minor child who lives with them.
Article 2180 of the Civil Code reads:
The obligation imposed by article 2176 is demandable not only for one's own acts or
omissions, but also for those of persons for whom one is responsible.
The father and, in case of his death or incapacity, the mother, are responsible for the
damages caused by the minor children who live in their company.
xxx xxx xxx
The responsibility treated of in this Article shall cease when the person herein
mentioned prove that they observed all the diligence of a good father of a family to
prevent damage. (Emphasis supplied)
This principle of parental liability is a species of what is frequently designated as vicarious liability, or
the doctrine of "imputed negligence" under Anglo-American tort law, where a person is not only liable
for torts committed by himself, but also for torts committed by others with whom he has a certain
relationship and for whom he is responsible. Thus, parental liability is made a natural or logical
consequence of the duties and responsibilities of parents their parental authority which
includes the instructing, controlling and disciplining of the child.
5
The basis for the doctrine of
vicarious liability was explained by the Court in Cangco v. Manila Railroad Co.
6
in the following
terms:
With respect to extra-contractual obligation arising from negligence, whether of act or
omission, it is competent for the legislature to elect and our Legislature has so elected
to limit such liability to cases in which the person upon whom such an obligation is
imposed is morally culpable or, on the contrary, for reasons of public policy. to extend
that liability, without regard to the lack of moral culpability, so as to include responsibility
for the negligence of those persons whose acts or omissions are imputable, by a legal
fiction, to others who are in a position to exercise an absolute or limited control over
them. The legislature which adopted our Civil Code has elected to limit extra-contractual
liability with certain well-defined exceptions to cases in which moral culpability can
be directly imputed to the persons to be charged. This moral responsibility may consist in
having failed to exercise due care in one's own acts, or in having failed to exercise due
care in the selection and control of one's agent or servants, or in the control of persons
who, by reasons of their status, occupy a position of dependency with respect to the
person made liable for their conduct.
7
(Emphasis Supplied)
The civil liability imposed upon parents for the torts of their minor children living with them,
may be seen to be based upon the parental authority vested by the Civil Code upon such
parents. The civil law assumes that when an unemancipated child living with its parents
commits a tortious acts, the parents were negligent in the performance of their legal and
natural duty closely to supervise the child who is in their custody and control. Parental liability
is, in other words, anchored upon parental authority coupled with presumed parental
dereliction in the discharge of the duties accompanying such authority. The parental
dereliction is, of course, only presumed and the presumption can be overtuned under Article
2180 of the Civil Code by proof that the parents had exercised all the diligence of a good
father of a family to prevent the damage.
In the instant case, the shooting of Jennifer by Adelberto with an air rifle occured when parental
authority was still lodged in respondent Bundoc spouses, the natural parents of the minor Adelberto.
It would thus follow that the natural parents who had then actual custody of the minor Adelberto, are
the indispensable parties to the suit for damages.
The natural parents of Adelberto, however, stoutly maintain that because a decree of adoption was
issued by the adoption court in favor of the Rapisura spouses, parental authority was vested in the
latter as adopting parents as of the time of the filing of the petition for adoption that
is, before Adelberto had shot Jennifer which an air rifle. The Bundoc spouses contend that they were
therefore free of any parental responsibility for Adelberto's allegedly tortious conduct.
Respondent Bundoc spouses rely on Article 36 of the Child and Youth Welfare Code 8 which reads
as follows:
Art. 36. Decree of Adoption. If, after considering the report of the Department of
Social Welfare or duly licensed child placement agency and the evidence submitted
before it, the court is satisfied that the petitioner is qualified to maintain, care for, and
educate the child, that the trial custody period has been completed, and that the best
interests of the child will be promoted by the adoption, a decree of adoption shall be
entered, which shall be effective he date the original petition was filed. The decree
shall state the name by which the child is thenceforth to be known. (Emphasis
supplied)
The Bundoc spouses further argue that the above Article 36 should be read in relation to
Article 39 of the same Code:
Art. 39. Effect of Adoption. The adoption shall:
xxx xxx xxx
(2) Dissolve the authority vested in the natural parents, except where the adopter is
the spouse of the surviving natural parent;
xxx xxx xxx
(Emphasis supplied)
and urge that their Parental authority must be deemed to have been dissolved as of the time the
Petition for adoption was filed.
The Court is not persuaded. As earlier noted, under the Civil Code, the basis of parental liability for
the torts of a minor child is the relationship existing between the parents and the minor child living
with them and over whom, the law presumes, the parents exercise supervision and control. Article
58 of the Child and Youth Welfare Code, re-enacted this rule:
Article 58 Torts Parents and guardians are responsible for the damage caused by
the child under their parental authority in accordance with the civil Code. (Emphasis
supplied)
Article 221 of the Family Code of the Philippines
9
has similarly insisted upon the requisite that the
child, doer of the tortious act, shall have beer in the actual custody of the parents sought to be held
liable for the ensuing damage:
Art. 221. Parents and other persons exercising parental authority shall be civilly liable
for the injuries and damages caused by the acts or omissions of their unemancipated
children living in their companyand under their parental authority subject to the
appropriate defenses provided by law. (Emphasis supplied)
We do not believe that parental authority is properly regarded as having been retroactively
transferred to and vested in the adopting parents, the Rapisura spouses, at the time the air rifle
shooting happened. We do not consider that retroactive effect may be giver to the decree of
adoption so as to impose a liability upon the adopting parents accruing at a time when adopting
parents had no actual or physically custody over the adopted child. Retroactive affect may perhaps
be given to the granting of the petition for adoption where such is essential to permit the accrual of
some benefit or advantage in favor of the adopted child. In the instant case, however, to hold that
parental authority had been retroactively lodged in the Rapisura spouses so as to burden them with
liability for a tortious act that they could not have foreseen and which they could not have prevented
(since they were at the time in the United States and had no physical custody over the child
Adelberto) would be unfair and unconscionable. Such a result, moreover, would be inconsistent with
the philosophical and policy basis underlying the doctrine of vicarious liability. Put a little differently,
no presumption of parental dereliction on the part of the adopting parents, the Rapisura spouses,
could have arisen since Adelberto was not in fact subject to their control at the time the tort was
committed.
Article 35 of the Child and Youth Welfare Code fortifies the conclusion reached above. Article 35
provides as follows:
Art. 35. Trial Custody. No petition for adoption shall be finally granted unless and
until the adopting parents are given by the courts a supervised trial custody period of
at least six months to assess their adjustment and emotional readiness for the legal
union. During the period of trial custody, parental authority shall be vested in the
adopting parents. (Emphasis supplied)
Under the above Article 35, parental authority is provisionally vested in the adopting parents during
the period of trial custody, i.e., before the issuance of a decree of adoption, precisely because the
adopting parents are given actual custody of the child during such trial period. In the instant case,
the trial custody period either had not yet begun or bad already been completed at the time of the air
rifle shooting; in any case, actual custody of Adelberto was then with his natural parents, not the
adopting parents.
Accordingly, we conclude that respondent Bundoc spouses, Adelberto's natural parents, were
indispensable parties to the suit for damages brought by petitioners, and that the dismissal by the
trial court of petitioners' complaint, the indispensable parties being already before the court,
constituted grave abuse of discretion amounting to lack or excess of jurisdiction.
WHEREFORE, premises considered, the Petition for Review is hereby GRANTED DUE COURSE
and the Decision of the Court of Appeals dated 6 September 1988, in C.A.-G.R. No. SP-15016 is
hereby REVERSED and SET ASIDE. Petitioners' complaint filed before the trial court is hereby
REINSTATED and this case is REMANDED to that court for further proceedings consistent with this
Decision. Costs against respondent Bundoc spouses. This Decision is immediately executory.
SO ORDERED.
Gutierrez, Jr., Bidin, Davide, Jr. and Romero, concur.
Republic of the Philippines
SUPREME COURT
Manila
THIRD DIVISION

G.R. No. 113054 March 16, 1995
LEOUEL SANTOS, SR., petitioner-appellant,
vs.
COURT OF APPEALS, and SPOUSES LEOPOLDO and OFELIA BEDIA, respondents-appellees.

ROMERO, J .:
In this petition for review, we are asked to overturn the decision of the Court of Appeals
1
granting
custody of six-year old Leouel Santos, Jr. to his maternal grandparents and not to his father, Santos,
Sr. What is sought is a decision which should definitively settle the matter of the care, custody and
control of the boy.
Happily, unlike King Solomon, we need not merely rely on a "wise and understanding heart," for
there is man's law to guide us and that is, the Family Code.
The antecedent facts giving rise to the case at bench are as follows:
Petitioner Leouel Santos, Sr., an army lieutenant, and Julia Bedia a nurse by profession, were
married in Iloilo City in 1986. Their union beget only one child, Leouel Santos, Jr. who was born July
18, 1987.
From the time the boy was released from the hospital until sometime thereafter, he had been in the
care and custody of his maternal grandparents, private respondents herein, Leopoldo and Ofelia
Bedia.
Petitioner and wife Julia agreed to place Leouel Jr. in the temporary custody of the latter's parents,
the respondent spouses Bedia. The latter alleged that they paid for all the hospital bills, as well as
the subsequent support of the boy because petitioner could not afford to do so.
The boy's mother, Julia Bedia-Santos, left for the United States in May 1988 to work. Petitioner
alleged that he is not aware of her whereabouts and his efforts to locate her in the United States
proved futile. Private respondents claim that although abroad, their daughter Julia had been sending
financial support to them for her son.
On September 2, 1990, petitioner along with his two brothers, visited the Bedia household, where
three-year old Leouel Jr. was staying. Private respondents contend that through deceit and false
pretensions, petitioner abducted the boy and clandestinely spirited him away to his hometown in
Bacong, Negros Oriental.
The spouses Bedia then filed a "Petition for Care, Custody and Control of Minor Ward Leouel Santos
Jr.," before the Regional Trial Court of Iloilo City, with Santos, Sr. as respondent.
2

After an ex-parte hearing on October 8, 1990, the trial court issued an order on the same day
awarding custody of the child Leouel Santos, Jr. to his grandparents, Leopoldo and Ofelia Bedia.
3

Petitioner appealed this Order to the Court of Appeals.
4
In its decision dated April 30, 1992,
respondent appellate court affirmed the trial court's
order.
5
His motion for reconsideration having been denied,
6
petitioner now brings the instant petition
for review for a reversal of the appellate court's decision.
The Court of Appeals erred, according to petitioner, in awarding custody of the boy to his
grandparents and not to himself. He contends that since private respondents have failed to show
that petitioner is an unfit and unsuitable father, substitute parental authority granted to the boy's
grandparents under Art. 214 of the Family Code is inappropriate.
Petitioner adds that the reasons relied upon by the private respondents in having custody over the
boy, are flimsy and insufficient to deprive him of his natural and legal right to have custody.
On the other hand, private respondents aver that they can provide an air-conditioned room for the
boy and that petitioner would not be in a position to take care of his son since he has to be assigned
to different places. They also allege that the petitioner did not give a single centavo for the boy's
support and maintenance. When the boy was about to be released from the hospital, they were the
ones who paid the fees because their daughter and petitioner had no money. Besides, Julia Bedia
Santos, their daughter, had entrusted the boy to them before she left for the United States.
Furthermore, petitioner's use of trickery and deceit in abducting the child in 1990, after being
hospitably treated by private respondents, does not speak well of his fitness and suitability as a
parent.
The Bedias argue that although the law recognizes the right of a parent to his child's custody,
ultimately the primary consideration is what is best for the happiness and welfare of the latter. As
maternal grandparents who have amply demonstrated their love and affection for the boy since his
infancy, they claim to be in the best position to promote the child's welfare.
The issue to be resolved here boils down to who should properly be awarded custody of the minor
Leouel Santos, Jr.
The right of custody accorded to parents springs from the exercise of parental authority. Parental
authority orpatria potestas in Roman Law is the juridical institution whereby parents rightfully assume
control and protection of their unemancipated children to the extent required by the latter' s
needs.
7
It is a mass of rights and obligations which the law grants to parents for the purpose of the
children's physical preservation and development, as well as the cultivation of their intellect and the
education of their heart and senses.
8
As regards parental authority, "there is no power, but a task;
no complex of rights, but a sum of duties; no sovereignty but a sacred trust for the welfare of the
minor."
9

Parental authority and responsibility are inalienable and may not be transferred or renounced except
in cases authorized by law.
10
The right attached to parental authority, being purely personal, the law
allows a waiver of parental authority only in cases of adoption, guardianship and surrender to a
children's home or an orphan institution.
11
When a parent entrusts the custody of a minor to another,
such as a friend or godfather, even in a document, what is given is merely temporary custody and it
does not constitute a renunciation of parental authority.
12
Even if a definite renunciation is manifest,
the law still disallows the same.
13

The father and mother, being the natural guardians of unemancipated children, are duty-bound and
entitled to keep them in their custody and
company.
14
The child's welfare is always the paramount consideration in all questions concerning
his care and custody.
15

The law vests on the father and mother joint parental authority over the persons of their common
children.
16
In case of absence or death of either parent, the parent present shall continue exercising
parental authority.
17
Only in case of the parents' death, absence or unsuitability may substitute
parental authority be exercised by the surviving grandparent.
18
The situation obtaining in the case at
bench is one where the mother of the minor Santos, Jr., is working in the United States while the
father, petitioner Santos, Sr., is present. Not only are they physically apart but are also emotionally
separated. There has been no decree of legal separation and petitioner's attempt to obtain an
annulment of the marriage on the ground of psychological incapacity of his wife has failed.
19

Petitioner assails the decisions of both the trial court and the appellate court to award custody of his
minor son to his parents-in-law, the Bedia spouses on the ground that under Art. 214 of the Family
Code, substitute parental authority of the grandparents is proper only when both parents are dead,
absent or unsuitable. Petitioner's unfitness, according to him, has not been successfully shown by
private respondents.
The Court of Appeals held that although there is no evidence to show that petitioner (Santos Sr.) is
"depraved, a habitual drunkard or poor, he may nevertheless be considered, as he is in fact so
considered, to be unsuitable to be allowed to have custody of minor Leouel Santos Jr."
20

The respondent appellate court, in affirming the trial court's order of October 8, 1990, adopted as its
own the latter's observations, to wit:
From the evidence adduced, this Court is of the opinion that it is to be (sic) best interest
of the minor Leouel Santos, Jr. that he be placed under the care, custody, and control of
his maternal grandparents the petitioners herein. The petitioners have amply
demonstrated their love and devotion to their grandson while the natural father,
respondent herein, has shown little interest in his welfare as reflected by his conduct in
the past. Moreover the fact that petitioners are well-off financially, should be carefully
considered in awarding to them the custody of the minor herein, lest the breaking of such
ties with his maternal grandparents might deprive the boy of an eventual college
education and other material advantages (Consaul vs. Consaul, 63 N.Y.S. 688).
Respondent had never given any previous financial support to his son, while, upon the
other hand, the latter receives so much bounty from his maternal grandparents and his
mother as well, who is now gainfully employed in the United States. Moreover, the fact
that respondent, as a military personnel who has to shuttle from one assignment to
another, and, in these troubled times, may have pressing and compelling military duties
which may prevent him from attending to his son at times when the latter needs him
most, militates strongly against said respondent. Additionally, the child is sickly and
asthmatic and needs the loving and tender care of those who can provide for it.
21

We find the aforementioned considerations insufficient to defeat petitioner's parental authority and
the concomitant right to have custody over the minor Leouel Santos, Jr., particularly since he has not
been shown to be an unsuitable and unfit parent. Private respondents' demonstrated love and
affection for the boy, notwithstanding, the legitimate father is still preferred over the
grandparents.
22
The latter's wealth is not a deciding factor, particularly because there is no proof
that at the present time, petitioner is in no position to support the boy. The fact that he was unable to
provide financial support for his minor son from birth up to over three years when he took the boy
from his in-laws without permission, should not be sufficient reason to strip him of his permanent
right to the child's custody. While petitioner's previous inattention is inexcusable and merits only the
severest criticism, it cannot be construed as abandonment. His appeal of the unfavorable decision
against him and his efforts to keep his only child in his custody may be regarded as serious efforts to
rectify his past misdeeds. To award him custody would help enhance the bond between parent and
son. It would also give the father a chance to prove his love for his son and for the son to experience
the warmth and support which a father can give.
His being a soldier is likewise no bar to allowing him custody over the boy. So many men in uniform
who are assigned to different parts of the country in the service of the nation, are still the natural
guardians of their children. It is not just to deprive our soldiers of authority, care and custody over
their children merely because of the normal consequences of their duties and assignments, such as
temporary separation from their families.
Petitioner's employment of trickery in spiriting away his boy from his in-laws, though unjustifiable, is
likewise not a ground to wrest custody from him.
Private respondents' attachment to the young boy whom they have reared for the past three years is
understandable. Still and all, the law considers the natural love of a parent to outweigh that of the
grandparents, such that only when the parent present is shown to be unfit or unsuitable may the
grandparents exercise substitute parental authority, a fact which has not been proven here.
The strong bonds of love and affection possessed by private respondents as grandparents should
not be seen as incompatible with petitioner' right to custody over the child as a father. Moreover, who
is to say whether the petitioner's financial standing may improve in the future?
WHEREFORE, the petition is GRANTED. The decision of the respondent Court of Appeals dated
April 30, 1992 as well as its Resolution dated November 13, 1992 are hereby REVERSED and SET
ASIDE. Custody over the minor Leouel Santos Jr. is awarded to his legitimate father, herein
petitioner Leouel Santos, Sr.
SO ORDERED.
Feliciano, Melo, Vitug and Francisco, JJ., concur.








SECOND DIVISION
G.R. No. L-24803 May 26, 1977
PEDRO ELCANO and PATRICIA ELCANO, in their capacity as Ascendants of Agapito Elcano,
deceased,plaintiffs-appellants,
vs.
REGINALD HILL, minor, and MARVIN HILL, as father and Natural Guardian of said
minor, defendants-appellees.
Cruz & Avecilla for appellants.
Marvin R. Hill & Associates for appellees.

BARREDO, J .:
Appeal from the order of the Court of First Instance of Quezon City dated January 29, 1965 in Civil
Case No. Q-8102, Pedro Elcano et al. vs. Reginald Hill et al. dismissing, upon motion to dismiss of
defendants, the complaint of plaintiffs for recovery of damages from defendant Reginald Hill, a
minor, married at the time of the occurrence, and his father, the defendant Marvin Hill, with whom he
was living and getting subsistence, for the killing by Reginald of the son of the plaintiffs, named
Agapito Elcano, of which, when criminally prosecuted, the said accused was acquitted on the ground
that his act was not criminal, because of "lack of intent to kill, coupled with mistake."
Actually, the motion to dismiss based on the following grounds:
1. The present action is not only against but a violation of section 1, Rule 107, which
is now Rule III, of the Revised Rules of Court;
2. The action is barred by a prior judgment which is now final and or in res-
adjudicata;
3. The complaint had no cause of action against defendant Marvin Hill, because he
was relieved as guardian of the other defendant through emancipation by marriage.
(P. 23, Record [p. 4, Record on Appeal.])
was first denied by the trial court. It was only upon motion for reconsideration of the defendants of
such denial, reiterating the above grounds that the following order was issued:
Considering the motion for reconsideration filed by the defendants on January 14,
1965 and after thoroughly examining the arguments therein contained, the Court
finds the same to be meritorious and well-founded.
WHEREFORE, the Order of this Court on December 8, 1964 is hereby reconsidered
by ordering the dismissal of the above entitled case.
SO ORDERED.
Quezon City, Philippines, January 29, 1965. (p. 40, Record [p. 21, Record on
Appeal.)
Hence, this appeal where plaintiffs-appellants, the spouses Elcano, are presenting for Our resolution
the following assignment of errors:
THE LOWER COURT ERRED IN DISMISSING THE CASE BY UPHOLDING THE
CLAIM OF DEFENDANTS THAT -
I
THE PRESENT ACTION IS NOT ONLY AGAINST BUT ALSO A VIOLATION OF
SECTION 1, RULE 107, NOW RULE 111, OF THE REVISED RULES OF COURT,
AND THAT SECTION 3(c) OF RULE 111, RULES OF COURT IS APPLICABLE;
II
THE ACTION IS BARRED BY A PRIOR JUDGMENT WHICH IS NOW FINAL OR
RES-ADJUDICTA;
III
THE PRINCIPLES OF QUASI-DELICTS, ARTICLES 2176 TO 2194 OF THE CIVIL
CODE, ARE INAPPLICABLE IN THE INSTANT CASE; and
IV
THAT THE COMPLAINT STATES NO CAUSE OF ACTION AGAINST DEFENDANT
MARVIN HILL BECAUSE HE WAS RELIEVED AS GUARDIAN OF THE OTHER
DEFENDANT THROUGH EMANCIPATION BY MARRIAGE. (page 4, Record.)
It appears that for the killing of the son, Agapito, of plaintiffs-appellants, defendant- appellee
Reginald Hill was prosecuted criminally in Criminal Case No. 5102 of the Court of First Instance of
Quezon City. After due trial, he was acquitted on the ground that his act was not criminal because of
"lack of intent to kill, coupled with mistake." Parenthetically, none of the parties has favored Us with a
copy of the decision of acquittal, presumably because appellants do not dispute that such indeed
was the basis stated in the court's decision. And so, when appellants filed their complaint against
appellees Reginald and his father, Atty. Marvin Hill, on account of the death of their son, the
appellees filed the motion to dismiss above-referred to.
As We view the foregoing background of this case, the two decisive issues presented for Our
resolution are:
1. Is the present civil action for damages barred by the acquittal of Reginald in the criminal case
wherein the action for civil liability, was not reversed?
2. May Article 2180 (2nd and last paragraphs) of the Civil Code he applied against Atty. Hill,
notwithstanding the undisputed fact that at the time of the occurrence complained of. Reginald,
though a minor, living with and getting subsistenee from his father, was already legally married?
The first issue presents no more problem than the need for a reiteration and further clarification of
the dual character, criminal and civil, of fault or negligence as a source of obligation which was firmly
established in this jurisdiction in Barredo vs. Garcia, 73 Phil. 607. In that case, this Court postulated,
on the basis of a scholarly dissertation by Justice Bocobo on the nature of culpa aquiliana in relation
to culpa criminal or delito and mereculpa or fault, with pertinent citation of decisions of the Supreme
Court of Spain, the works of recognized civilians, and earlier jurisprudence of our own, that the same
given act can result in civil liability not only under the Penal Code but also under the Civil Code.
Thus, the opinion holds:
The, above case is pertinent because it shows that the same act machinist. come under
both the Penal Code and the Civil Code. In that case, the action of the agent killeth
unjustified and fraudulent and therefore could have been the subject of a criminal action.
And yet, it was held to be also a proper subject of a civil action under article 1902 of the
Civil Code. It is also to be noted that it was the employer and not the employee who was
being sued. (pp. 615-616, 73 Phil.).
1

It will be noticed that the defendant in the above case could have been prosecuted in a
criminal case because his negligence causing the death of the child was punishable by
the Penal Code. Here is therefore a clear instance of the same act of negligence being a
proper subject matter either of a criminal action with its consequent civil liability arising
from a crime or of an entirely separate and independent civil action for fault or negligence
under article 1902 of the Civil Code. Thus, in this jurisdiction, the separate individuality of
a cuasi-delito or culpa aquiliana, under the Civil Code has been fully and clearly
recognized, even with regard to a negligent act for which the wrongdoer could have been
prosecuted and convicted in a criminal case and for which, after such a conviction, he
could have been sued for this civil liability arising from his crime. (p. 617, 73 Phil.)
2

It is most significant that in the case just cited, this Court specifically applied article 1902
of the Civil Code. It is thus that although J. V. House could have been criminally
prosecuted for reckless or simple negligence and not only punished but also made civilly
liable because of his criminal negligence, nevertheless this Court awarded damages in
an independent civil action for fault or negligence under article 1902 of the Civil Code. (p.
618, 73 Phil.)
3

The legal provisions, authors, and cases already invoked should ordinarily be
sufficient to dispose of this case. But inasmuch as we are announcing doctrines that
have been little understood, in the past, it might not he inappropriate to indicate their
foundations.
Firstly, the Revised Penal Code in articles 365 punishes not only reckless but also
simple negligence. If we were to hold that articles 1902 to 1910 of the Civil Code
refer only to fault or negligence not punished by law, accordingly to the literal import
of article 1093 of the Civil Code, the legal institution of culpa aquiliana would have
very little scope and application in actual life. Death or injury to persons and damage
to property- through any degree of negligence - even the slightest - would have to be
Idemnified only through the principle of civil liability arising from a crime. In such a
state of affairs, what sphere would remain for cuasi-delito or culpa aquiliana? We are
loath to impute to the lawmaker any intention to bring about a situation so absurd and
anomalous. Nor are we, in the interpretation of the laws, disposed to uphold the letter
that killeth rather than the spirit that giveth life. We will not use the literal meaning of
the law to smother and render almost lifeless a principle of such ancient origin and
such full-grown development as culpa aquiliana or cuasi-delito, which is conserved
and made enduring in articles 1902 to 1910 of the Spanish Civil Code.
Secondary, to find the accused guilty in a criminal case, proof of guilt beyond
reasonable doubt is required, while in a civil case, preponderance of evidence is
sufficient to make the defendant pay in damages. There are numerous cases of
criminal negligence which can not be shown beyond reasonable doubt, but can be
proved by a preponderance of evidence. In such cases, the defendant can and
should be made responsible in a civil action under articles 1902 to 1910 of the Civil
Code. Otherwise. there would be many instances of unvindicated civil wrongs. "Ubi
jus Idemnified remedium." (p. 620,73 Phil.)
Fourthly, because of the broad sweep of the provisions of both the Penal Code and
the Civil Code on this subject, which has given rise to the overlapping or concurrence
of spheres already discussed, and for lack of understanding of the character and
efficacy of the action for culpa aquiliana, there has grown up a common practice to
seek damages only by virtue of the civil responsibility arising from a crime, forgetting
that there is another remedy, which is by invoking articles 1902-1910 of the Civil
Code. Although this habitual method is allowed by, our laws, it has nevertheless
rendered practically useless and nugatory the more expeditious and effective remedy
based on culpa aquiliana or culpa extra-contractual. In the present case, we are
asked to help perpetuate this usual course. But we believe it is high time we pointed
out to the harms done by such practice and to restore the principle of responsibility
for fault or negligence under articles 1902 et seq. of the Civil Code to its full rigor. It is
high time we caused the stream of quasi-delict or culpa aquiliana to flow on its own
natural channel, so that its waters may no longer be diverted into that of a crime
under the Penal Code. This will, it is believed, make for the better safeguarding or
private rights because it realtor, an ancient and additional remedy, and for the further
reason that an independent civil action, not depending on the issues, limitations and
results of a criminal prosecution, and entirely directed by the party wronged or his
counsel, is more likely to secure adequate and efficacious redress. (p. 621, 73 Phil.)
Contrary to an immediate impression one might get upon a reading of the foregoing excerpts from
the opinion in Garcia that the concurrence of the Penal Code and the Civil Code therein referred to
contemplate only acts of negligence and not intentional voluntary acts - deeper reflection would
reveal that the thrust of the pronouncements therein is not so limited, but that in fact it actually
extends to fault or culpa. This can be seen in the reference made therein to the Sentence of the
Supreme Court of Spain of February 14, 1919, supra, which involved a case of fraud or estafa, not a
negligent act. Indeed, Article 1093 of the Civil Code of Spain, in force here at the time of Garcia,
provided textually that obligations "which are derived from acts or omissions in which fault or
negligence, not punishable by law, intervene shall be the subject of Chapter II, Title XV of this book
(which refers to quasi-delicts.)" And it is precisely the underline qualification, "not punishable by law",
that Justice Bocobo emphasized could lead to an ultimo construction or interpretation of the letter of
the law that "killeth, rather than the spirit that giveth lift- hence, the ruling that "(W)e will not use the
literal meaning of the law to smother and render almost lifeless a principle of such ancient origin and
such full-grown development as culpa aquiliana orquasi-delito, which is conserved and made
enduring in articles 1902 to 1910 of the Spanish Civil Code." And so, because Justice Bacobo was
Chairman of the Code Commission that drafted the original text of the new Civil Code, it is to be
noted that the said Code, which was enacted after the Garcia doctrine, no longer uses the term, 11
not punishable by law," thereby making it clear that the concept of culpa aquiliana includes acts
which are criminal in character or in violation of the penal law, whether voluntary or matter. Thus, the
corresponding provisions to said Article 1093 in the new code, which is Article 1162, simply says,
"Obligations derived from quasi-delicto shall be governed by the provisions of Chapter 2, Title XVII of
this Book, (on quasi-delicts) and by special laws." More precisely, a new provision, Article 2177 of
the new code provides:
ART. 2177. Responsibility for fault or negligence under the preceding article is
entirely separate and distinct from the civil liability arising from negligence under the
Penal Code. But the plaintiff cannot recover damages twice for the same act or
omission of the defendant.
According to the Code Commission: "The foregoing provision (Article 2177) through at first sight
startling, is not so novel or extraordinary when we consider the exact nature of criminal and civil
negligence. The former is a violation of the criminal law, while the latter is a "culpa aquiliana" or
quasi-delict, of ancient origin, having always had its own foundation and individuality, separate from
criminal negligence. Such distinction between criminal negligence and "culpa extracontractual" or
"cuasi-delito" has been sustained by decision of the Supreme Court of Spain and maintained as
clear, sound and perfectly tenable by Maura, an outstanding Spanish jurist. Therefore, under the
proposed Article 2177, acquittal from an accusation of criminal negligence, whether on reasonable
doubt or not, shall not be a bar to a subsequent civil action, not for civil liability arising from criminal
negligence, but for damages due to a quasi-delict or 'culpa aquiliana'. But said article forestalls a
double recovery.", (Report of the Code) Commission, p. 162.)
Although, again, this Article 2177 does seem to literally refer to only acts of negligence, the same
argument of Justice Bacobo about construction that upholds "the spirit that giveth lift- rather than that
which is literal that killeth the intent of the lawmaker should be observed in applying the same. And
considering that the preliminary chapter on human relations of the new Civil Code definitely
establishes the separability and independence of liability in a civil action for acts criminal in character
(under Articles 29 to 32) from the civil responsibility arising from crime fixed by Article 100 of the
Revised Penal Code, and, in a sense, the Rules of Court, under Sections 2 and 3 (c), Rule 111,
contemplate also the same separability, it is "more congruent with the spirit of law, equity and justice,
and more in harmony with modern progress"- to borrow the felicitous relevant language in Rakes vs.
Atlantic. Gulf and Pacific Co., 7 Phil. 359, to hold, as We do hold, that Article 2176, where it refers to
"fault or negligencia covers not only acts "not punishable by law" but also acts criminal in character,
whether intentional and voluntary or negligent. Consequently, a separate civil action lies against the
offender in a criminal act, whether or not he is criminally prosecuted and found guilty or acquitted,
provided that the offended party is not allowed, if he is actually charged also criminally, to recover
damages on both scores, and would be entitled in such eventuality only to the bigger award of the
two, assuming the awards made in the two cases vary. In other words, the extinction of civil liability
referred to in Par. (e) of Section 3, Rule 111, refers exclusively to civil liability founded on Article 100
of the Revised Penal Code, whereas the civil liability for the same act considered as a quasi-
delict only and not as a crime is not estinguished even by a declaration in the criminal case that the
criminal act charged has not happened or has not been committed by the accused. Briefly stated,
We here hold, in reiteration of Garcia, thatculpa aquiliana includes voluntary and negligent acts
which may be punishable by law.
4

It results, therefore, that the acquittal of Reginal Hill in the criminal case has not extinguished his
liability for quasi-delict, hence that acquittal is not a bar to the instant action against him.
Coming now to the second issue about the effect of Reginald's emancipation by marriage on the
possible civil liability of Atty. Hill, his father, it is also Our considered opinion that the conclusion of
appellees that Atty. Hill is already free from responsibility cannot be upheld.
While it is true that parental authority is terminated upon emancipation of the child (Article 327, Civil
Code), and under Article 397, emancipation takes place "by the marriage of the minor (child)", it is,
however, also clear that pursuant to Article 399, emancipation by marriage of the minor is not really
full or absolute. Thus "(E)mancipation by marriage or by voluntary concession shall terminate
parental authority over the child's person. It shall enable the minor to administer his property as
though he were of age, but he cannot borrow money or alienate or encumber real property without
the consent of his father or mother, or guardian. He can sue and be sued in court only with the
assistance of his father, mother or guardian."
Now under Article 2180, "(T)he obligation imposed by article 2176 is demandable not only for one's
own acts or omissions, but also for those of persons for whom one is responsible. The father and, in
case of his death or incapacity, the mother, are responsible. The father and, in case of his death or
incapacity, the mother, are responsible for the damages caused by the minor children who live in
their company." In the instant case, it is not controverted that Reginald, although married, was living
with his father and getting subsistence from him at the time of the occurrence in question. Factually,
therefore, Reginald was still subservient to and dependent on his father, a situation which is not
unusual.
It must be borne in mind that, according to Manresa, the reason behind the joint and solidary liability
of presuncion with their offending child under Article 2180 is that is the obligation of the parent to
supervise their minor children in order to prevent them from causing damage to third persons.
5
On
the other hand, the clear implication of Article 399, in providing that a minor emancipated by
marriage may not, nevertheless, sue or be sued without the assistance of the parents, is that such
emancipation does not carry with it freedom to enter into transactions or do any act that can give rise
to judicial litigation. (See Manresa, Id., Vol. II, pp. 766-767, 776.) And surely, killing someone else
invites judicial action. Otherwise stated, the marriage of a minor child does not relieve the parents of
the duty to see to it that the child, while still a minor, does not give answerable for the borrowings of
money and alienation or encumbering of real property which cannot be done by their minor married
child without their consent. (Art. 399; Manresa, supra.)
Accordingly, in Our considered view, Article 2180 applies to Atty. Hill notwithstanding the
emancipation by marriage of Reginald. However, inasmuch as it is evident that Reginald is now of
age, as a matter of equity, the liability of Atty. Hill has become milling, subsidiary to that of his son.
WHEREFORE, the order appealed from is reversed and the trial court is ordered to proceed in
accordance with the foregoing opinion. Costs against appellees.
Fernando (Chairman), Antonio, and Martin, JJ., concur.
Concepcion Jr., J, is on leave.
Martin, J, was designated to sit in the Second Division.


Separate Opinions

AQUINO, J , concurring:
Article 2176 of the Civil Code comprehends any culpable act, which is blameworthy, when judged by
accepted legal standards. "The Idea thus expressed is undoubtedly board enough to include any
rational conception of liability for the tortious acts likely to be developed in any society." (Street, J. in
Daywalt vs. Corporacion de PP. Agustinos Recoletos, 39 Phil. 587, 600). See article 38, Civil Code
and the ruling that "the infant tortfeasor is liable in a civil action to the injured person in the same
manner and to the same extent as an adult" (27 Am. Jur. 812 cited by Bocobo, J., in Magtibay vs.
Tiangco, 74 Phil. 576, 579).


Separate Opinions
AQUINO, J , concurring:
Article 2176 of the Civil Code comprehends any culpable act, which is blameworthy, when judged by
accepted legal standards. "The Idea thus expressed is undoubtedly board enough to include any
rational conception of liability for the tortious acts likely to be developed in any society." (Street, J. in
Daywalt vs. Corporacion de PP. Agustinos Recoletos, 39 Phil. 587, 600). See article 38, Civil Code
and the ruling that "the infant tortfeasor is liable in a civil action to the injured person in the same
manner and to the same extent as an adult" (27 Am. Jur. 812 cited by Bocobo, J., in Magtibay vs.
Tiangco, 74 Phil. 576, 579).
Footnotes
1 Referring to Sentence of the Supreme Court of Spain of February 14, 1919.
2 Referring to Manzanares vs. Moreta, 38 Phil. 821.
3 Referring to Bernal et al, vs. House et al., 54 Phil. 327.
4 Parenthetically, Manresa seemingly holds. the contrary view thus:
"Sin embargo, para no ineurrir en error hay que tener en cuenta que los lineage. del
precepts contenido en el presente articulo son bastante mas reducidos, pues no se
hallan comprendidos en el todos los datios que pues tener por causa la culpa o la
negligencia.
"En efecto, examinando detenidamente la terminos general de la culpa y de la
negligencia. se observe que, tanto en una como en otra de dichas causas, hay tres
generoso o tres especies distintas, a saber:
1. La que represents una accion u omision voluntaria por la que results incumplida
una obligacion anteriormente constituida.
2. La que sin existencia de una obligacion anterior produce un dano o perjuicio que,
teniendo su origen en un hecho ilicito, no reviste los caracteres de delito o falta; y
3. La que teniendo por origen un hecho que constituya delito o falta produce una
responsabilidad civil como accesoria de la responsabilidad criminal.
"La primera de estas tres especies de culpa o negligencia es siempre accesoria de
una obligacion principal, cuyo incumplimiento da origen a la terminos especial de la
culpa en materia de contratos, y el eatudio de esta debe harms al examinar cada
contrato, en especial, como lo hicimos asi, analizando entoces los peculiares efectos
de dicha culpa en cada uno de ellos.
"La tercera de las especies citadas es accesoria tambien, pues no puede concebirse
su existencia sin la de un delicto o falts que la produzca. Es decir, que solo al lado
de la responsabilidad criminal puede supuesto esa responsabilidad civil y la
obligacion proveniente de la culpa, ineurrir como una consecuencia de la
responsabilidad criminal, y, por consiguente, su examen y regulacion perusal. al
Derecho penal.
"Como consecuencia de ello, results que la unica especie de culpa y omisiones o
negligencia que puede ser y es meanwhile.' del presente capitulo, es la separability,
o sea la que sin la existencia de una obligacion anterior, y sin ningun antecedents
contractual, produce un dano o perjuico que tiene su origen en una accion u omision
culpable solo civilmente; as decir, que siendo ilicita, no reviste sin embargo, los
caracteres de un delito o falta por no estar penada por la ley. Y aun dentro de estos
lineage hay que restringir aun mas los terminos o la materia propria de este articulo,
el cual se refiere unicamente a la culpa o negligencia personates del obligado, pero
no a las que prudencia de actos o de omisiones de persons., distintas de este." (pp.
642-643, Vol. XII, Manresa, Codigo Civil Espanol.)
5 "Nuestro Codigo no ha seguido la escuela italiana, sino que mas bien se ha
instantaneous, en el criterio de la doctrina full-grown puesto que impone la
obligacion de reparar, el dano causado en virtud de una presuncion juris tecum de
culpa por parte del que tiene bajo su autoridad o dependecia al causante del daho,
derivada del hicimos de no haber puesto el cuidado y la vinculos debida en los actos
de sus subordinados para evitar dicho resultado. Asi es que, segun el parrafo ultimo
del art. 1,903, cesa dicha responsabilidad cuando se prueba que los obligados por
los actos ajenos emplearon toda la diligencia de un buen padre de familia. Luego no
es la causa de la obligacion impuesta la representacion, ni el interes, ni la necesidad
de que haya quienes responda del dano causado por el que no tiene personalidad in
garantias de specialist. para responsabilidad por siendo sino el incumplimiento
implicito o supuesto de los deberes de precaucion y de prudencia que impuesta los
vinculos civiles que unicamente al obligado con las persons., por quienes debe
representacion, el mal causado, Por ese motivo coloca dicha obligacion entre las
que prudencia de la culpa of negligentj (pp. 670671, Manresa, Codigo Civil Espanol,
Vol. XII.)



































SECOND DIVISION


MARIA VIRGINIA V. REMO, G.R. No. 169202
Petitioner,
Present:

CARPIO, J ., Chairperson,
-versus- BRION,
DEL CASTILLO,
ABAD, and
PEREZ, J J .
THE HONORABLE SECRETARY
OF FOREIGN AFFAIRS, Promulgated:
Respondent. March 5, 2010
x-----------------------------------------------------------------------------------------x


D E C I S I O N

CARPIO, J .:

The Case

Before the Court is a petition for review
[1]
of the 27 May 2005 Decision
[2]
and 2 August 2005
Resolution
[3]
of the Court of Appeals in CA-G.R. SP No. 87710. The Court of Appeals affirmed the
decision of the Office of the President, which in turn affirmed the decision of the Secretary of Foreign
Affairs denying petitioners request to revert to the use of her maiden name in her replacement passport.


The Facts

Petitioner Maria Virginia V. Remo is a married Filipino citizen whose Philippine passport was then
expiring on 27 October 2000. Petitioner being married to Francisco R. Rallonza, the following entries
appear in her passport: Rallonza as her surname, Maria Virginia as her given name, and Remo as
her middle name. Prior to the expiry of the validity of her passport, petitioner, whose marriage still
subsists, applied for the renewal of her passport with the Department of Foreign Affairs (DFA) office in
Chicago, Illinois, U.S.A., with a request to revert to her maiden name and surname in the replacement
passport.

Petitioners request having been denied, Atty. Manuel Joseph R. Bretana III, representing petitioner,
wrote then Secretary of Foreign Affairs Domingo Siason expressing a similar request.

On 28 August 2000, the DFA, through Assistant Secretary Belen F. Anota, denied the request,
stating thus:

This has reference to your letter dated 17 August 2000 regarding one Ms. Maria
Virginia V. Remo who is applying for renewal of her passport using her maiden name.

This Office is cognizant of the provision in the law that it is not obligatory for a married woman to
use her husbands name. Use of maiden name is allowed in passport application only if the married
name has not been used in previous application. The Implementing Rules and Regulations for
Philippine Passport Act of 1996 clearly defines the conditions when a woman applicant may revert to her
maiden name, that is, only in cases of annulment of marriage, divorce and death of the husband. Ms.
Remos case does not meet any of these conditions.
[4]
(Emphasis supplied)


Petitioners motion for reconsideration of the above-letter resolution was denied in a letter dated 13
October 2000.
[5]

On 15 November 2000, petitioner filed an appeal with the Office of the President.

On 27 July 2004, the Office of the President dismissed the appeal
[6]
and ruled that Section 5(d) of
Republic Act No. 8239 (RA 8239) or the Philippine Passport Act of 1996offers no leeway for any other
interpretation than that only in case of divorce, annulment, or declaration [of nullity] of marriage may a
married woman revert to her maiden name for passport purposes. The Office of the President further
held that in case of conflict between a general and special law, the latter will control the former regardless
of the respective dates of passage. Since the Civil Code is a general law, it should yield to RA 8239.

On 28 October 2004, the Office of the President denied the motion for reconsideration.
[7]


Petitioner filed with the Court of Appeals a petition for review under Rule 43 of the Rules of Civil
Procedure.

In its Decision of 27 May 2005, the Court of Appeals denied the petition and affirmed the ruling of
the Office of the President. The dispositive portion of the Court of Appeals decision reads:

WHEREFORE, premises considered, the petition is DENIED, and the resolution
dated July 27, 2004, and the order dated October 28, 2004 of the Office of the President
in O.P. Case No. 001-A-9344 are hereby AFFIRMED.

SO ORDERED.
[8]



Petitioner moved for reconsideration which the Court of Appeals denied in its Resolution dated 2
August 2005.

Hence, this petition.


The Court of Appeals Ruling


The Court of Appeals found no conflict between Article 370 of the Civil Code
[9]
and Section 5(d) of
RA 8239.
[10]
The Court of Appeals held that for passport application and issuance purposes, RA 8239
limits the instances when a married woman applicant may exercise the option to revert to the use of her
maiden name such as in a case of a divorce decree, annulment or declaration of nullity of marriage. Since
there was no showing that petitioner's marriage to Francisco Rallonza has been annulled, declared void
or a divorce decree has been granted to them, petitioner cannot simply revert to her maiden name in the
replacement passport after she had adopted her husbands surname in her old passport. Hence,
according to the Court of Appeals, respondent was justified in refusing the request of petitioner to revert
to her maiden name in the replacement passport.


The Issue


The sole issue in this case is whether petitioner, who originally used her husbands surname in her
expired passport, can revert to the use of her maiden name in the replacement passport, despite the
subsistence of her marriage.

The Ruling of the Court


The petition lacks merit.

Title XIII of the Civil Code governs the use of surnames. In the case of a married woman,
Article 370 of the Civil Code provides:

ART. 370. A married woman may use:

(1) HER MAIDEN FIRST NAME AND SURNAME AND ADD HER HUSBANDS
SURNAME, OR
(2) HER MAIDEN FIRST NAME AND HER HUSBAND'S SURNAME, OR
(3) HER HUSBANDS FULL NAME, BUT PREFIXING A WORD INDICATING
THAT SHE IS HIS WIFE, SUCH AS MRS.


We agree with petitioner that the use of the word may in the above provision indicates that the use
of the husbands surname by the wife is permissive rather than obligatory. This has been settled in the
case of Yasin v. Honorable Judge Sharia District Court.
[11]


In Yasin,
[12]
petitioner therein filed with the Sharia District Court a Petition to resume the use of
maiden name in view of the dissolution of her marriage by divorce under the Code of Muslim Personal
Laws of the Philippines, and after marriage of her former husband to another woman. In ruling in favor of
petitioner therein, the Court explained that:

When a woman marries a man, she need not apply and/or seek judicial
authority to use her husbands name by prefixing the word Mrs. before her
husbands full name or by adding her husbands surname to her maiden first
name. The law grants her such right (Art. 370, Civil Code). Similarly, when the
marriage ties or vinculum no longer exists as in the case of death of the husband
or divorce as authorized by the Muslim Code, the widow or divorcee need not seek
judicial confirmation of the change in her civil status in order to revert to her
maiden name as use of her former husbands is optional and not obligatory for her
(Tolentino, Civil Code, p. 725, 1983 ed.; Art. 373, Civil Code). When petitioner
married her husband, she did not change her but only her civil status. Neither was
she required to secure judicial authority to use the surname of her husband after
the marriage as no law requires it. (Emphasis supplied)



Clearly, a married woman has an option, but not a duty, to use the surname of the husband in any
of the ways provided by Article 370 of the Civil Code.
[13]
She is therefore allowed to use not only any of
the three names provided in Article 370, but also her maiden name upon marriage. She is not prohibited
from continuously using her maiden name once she is married because when a woman marries, she
does not change her name but only her civil status. Further, this interpretation is in consonance with the
principle that surnames indicate descent.
[14]


In the present case, petitioner, whose marriage is still subsisting and who opted to use her
husbands surname in her old passport, requested to resume her maiden name in the replacement
passport arguing that no law prohibits her from using her maiden name. Petitioner cites Yasin as the
applicable precedent. However, Yasin is not squarely in point with this case. Unlike in Yasin, which
involved a Muslim divorcee whose former husband is already married to another woman, petitioners
marriage remains subsisting. Another point, Yasin did not involve a request to resume ones maiden
name in a replacement passport, but a petition to resume ones maiden name in view of the dissolution of
ones marriage.

The law governing passport issuance is RA 8239 and the applicable provision in this case is
Section 5(d), which states:

Sec. 5. Requirements for the Issuance of Passport. No passport shall be issued
to an applicant unless the Secretary or his duly authorized representative is satisfied that
the applicant is a Filipino citizen who has complied with the following requirements: x x x

(D) IN CASE OF A WOMAN WHO IS MARRIED, SEPARATED, DIVORCED OR
WIDOWED OR WHOSE MARRIAGE HAS BEEN ANNULLED OR
DECLARED BY COURT AS VOID, A COPY OF THE CERTIFICATE OF
MARRIAGE, COURT DECREE OF SEPARATION, DIVORCE OR
ANNULMENT OR CERTIFICATE OF DEATH OF THE DECEASED
SPOUSE DULY ISSUED AND AUTHENTICATED BY THE OFFICE OF THE
CIVIL REGISTRAR GENERAL: PROVIDED, THAT IN CASE OF A
DIVORCE DECREE, ANNULMENT OR DECLARATION OF MARRIAGE AS
VOID, THE WOMAN APPLICANT MAY REVERT TO THE USE OF HER
MAIDEN NAME: PROVIDED, FURTHER, THAT SUCH DIVORCE IS
RECOGNIZED UNDER EXISTING LAWS OF THE PHILIPPINES; X X X
(EMPHASIS SUPPLIED)


The Office of the Solicitor General (OSG), on behalf of the Secretary of Foreign Affairs, argues that
the highlighted proviso in Section 5(d) of RA 8239 limits the instances when a married woman may be
allowed to revert to the use of her maiden name in her passport. These instances are death of husband,
divorce decree, annulment or nullity of marriage. Significantly, Section 1, Article 12 of the Implementing
Rules and Regulations of RA 8239 provides:

The passport can be amended only in the following cases:

A) AMENDMENT OF WOMANS NAME DUE TO MARRIAGE;
B) AMENDMENT OF WOMANS NAME DUE TO DEATH OF SPOUSE, ANNULMENT OF MARRIAGE
OR DIVORCE INITIATED BY A FOREIGN SPOUSE; OR
C) CHANGE OF SURNAME OF A CHILD WHO IS LEGITIMATED BY VIRTUE OF A
SUBSEQUENT MARRIAGE OF HIS PARENTS.


Since petitioners marriage to her husband subsists, placing her case outside of the purview of
Section 5(d) of RA 8239 (as to the instances when a married woman may revert to the use of her maiden
name), she may not resume her maiden name in the replacement passport.
[15]
This prohibition, according
to petitioner, conflicts with and, thus, operates as an implied repeal of Article 370 of the Civil Code.

PETITIONER IS MISTAKEN. THE CONFLICT BETWEEN ARTICLE 370 OF THE CIVIL CODE
AND SECTION 5(D) OF RA 8239 IS MORE IMAGINED THAN REAL. RA 8239, INCLUDING ITS
IMPLEMENTING RULES AND REGULATIONS, DOES NOT PROHIBIT A MARRIED WOMAN FROM
USING HER MAIDEN NAME IN HER PASSPORT. IN FACT, IN RECOGNITION OF THIS RIGHT, THE
DFA ALLOWS A MARRIED WOMAN WHO APPLIES FOR A PASSPORT FOR THE FIRST TIME TO
USE HER MAIDEN NAME. SUCH AN APPLICANT IS NOT REQUIRED TO ADOPT HER HUSBAND'S
SURNAME.
[16]


In the case of renewal of passport, a married woman may either adopt her husbands surname or
continuously use her maiden name. If she chooses to adopt her husbands surname in her new passport,
the DFA additionally requires the submission of an authenticated copy of the marriage certificate.
Otherwise, if she prefers to continue using her maiden name, she may still do so. The DFA will not
prohibit her from continuously using her maiden name.
[17]


HOWEVER, ONCE A MARRIED WOMAN OPTED TO ADOPT HER HUSBANDS SURNAME IN
HER PASSPORT, SHE MAY NOT REVERT TO THE USE OF HER MAIDEN NAME, EXCEPT IN THE
CASES ENUMERATED IN SECTION 5(D) OF RA 8239. THESE INSTANCES ARE: (1) DEATH OF
HUSBAND, (2) DIVORCE, (3) ANNULMENT, OR (4) NULLITY OF MARRIAGE. SINCE PETITIONERS
MARRIAGE TO HER HUSBAND SUBSISTS, SHE MAY NOT RESUME HER MAIDEN NAME IN THE
REPLACEMENT PASSPORT. OTHERWISE STATED, A MARRIED WOMAN'S REVERSION TO THE
USE OF HER MAIDEN NAME MUST BE BASED ONLY ON THE SEVERANCE OF THE MARRIAGE.

EVEN ASSUMING RA 8239 CONFLICTS WITH THE CIVIL CODE, THE PROVISIONS OF RA
8239 WHICH IS A SPECIAL LAW SPECIFICALLY DEALING WITH PASSPORT ISSUANCE MUST
PREVAIL OVER THE PROVISIONS OF TITLE XIII OF THE CIVIL CODE WHICH IS THE GENERAL
LAW ON THE USE OF SURNAMES. A BASIC TENET IN STATUTORY CONSTRUCTION IS THAT A
SPECIAL LAW PREVAILS OVER A GENERAL LAW,
[18]
THUS:

[I]t is a familiar rule of statutory construction that to the extent of any necessary
repugnancy between a general and a special law or provision, the latter will control
the former without regard to the respective dates of passage.
[19]



Moreover, petitioners theory of implied repeal must fail. Well-entrenched is the rule that an implied
repeal is disfavored. T he apparently conflicting provisions of a law or two laws should be harmonized as
much as possible, so that each shall be effective.
[20]
For a law to operate to repeal another law, the two
laws must actually be inconsistent. The former must be so repugnant as to be irreconcilable with the latter
act.
[21]
This petitioner failed to establish.

The Court notes that petitioner would not have encountered any problems in the replacement
passport had she opted to continuously and consistently use her maiden name from the moment she was
married and from the time she first applied for a Philippine passport. However, petitioner consciously
chose to use her husbands surname before, in her previous passport application, and now desires to
resume her maiden name. If we allow petitioners present request, definitely nothing prevents her in the
future from requesting to revert to the use of her husbands surname. Such unjustified changes in one's
name and identity in a passport, which is considered superior to all other official documents,
[22]
cannot be
countenanced. Otherwise, undue confusion and inconsistency in the records of passport holders will
arise. Thus, for passport issuance purposes, a married woman, such as petitioner, whose marriage
subsists, may not change her family name at will.

THE ACQUISITION OF A PHILIPPINE PASSPORT IS A PRIVILEGE. THE LAW RECOGNIZES
THE PASSPORT APPLICANTS CONSTITUTIONAL RIGHT TO TRAVEL. HOWEVER, THE STATE IS
ALSO MANDATED TO PROTECT AND MAINTAIN THE INTEGRITY AND CREDIBILITY OF THE
PASSPORT AND TRAVEL DOCUMENTS PROCEEDING FROM IT
[23]
AS A PHILIPPINE PASSPORT
REMAINS AT ALL TIMES THE PROPERTY OF THE GOVERNMENT. THE HOLDER IS MERELY A
POSSESSOR OF THE PASSPORT AS LONG AS IT IS VALID AND THE SAME MAY NOT BE
SURRENDERED TO ANY PERSON OR ENTITY OTHER THAN THE GOVERNMENT OR ITS
REPRESENTATIVE.
[24]


As the OSG correctly pointed out:

[T]he issuance of passports is impressed with public interest. A passport is an official
document of identity and nationality issued to a person intending to travel or sojourn in
foreign countries. It is issued by the Philippine government to its citizens requesting
other governments to allow its holder to pass safely and freely, and in case of need, to
give him/her aid and protection. x x x

Viewed in the light of the foregoing, it is within respondents competence to regulate any
amendments intended to be made therein, including the denial of unreasonable and whimsical requests
for amendments such as in the instant case.
[25]



WHEREFORE, we DENY the petition. We AFFIRM the 27 May 2005 Decision and 2 August 2005
Resolution of the Court of Appeals in CA-G.R. SP No. 87710.

SO ORDERED.

ANTONIO T. CARPIO
Associate Justice












Republic of the Philippines
SUPREME COURT
Manila
FIRST DIVISION
G.R. No. 174689 October 22, 2007
ROMMEL JACINTO DANTES SILVERIO, petitioner,
vs.
REPUBLIC OF THE PHILIPPINES, respondent.
D E C I S I O N
CORONA, J .:
When God created man, He made him in the likeness of God; He created them male and
female. (Genesis 5:1-2)
Amihan gazed upon the bamboo reed planted by Bathala and she heard voices coming from
inside the bamboo. "Oh North Wind! North Wind! Please let us out!," the voices said. She
pecked the reed once, then twice. All of a sudden, the bamboo cracked and slit open. Out
came two human beings; one was a male and the other was a female. Amihan named the
man "Malakas" (Strong) and the woman "Maganda" (Beautiful). (The Legend of Malakas and
Maganda)
When is a man a man and when is a woman a woman? In particular, does the law recognize the
changes made by a physician using scalpel, drugs and counseling with regard to a persons sex?
May a person successfully petition for a change of name and sex appearing in the birth certificate to
reflect the result of a sex reassignment surgery?
On November 26, 2002, petitioner Rommel Jacinto Dantes Silverio filed a petition for the change of
his first name and sex in his birth certificate in the Regional Trial Court of Manila, Branch 8. The
petition, docketed as SP Case No. 02-105207, impleaded the civil registrar of Manila as respondent.
Petitioner alleged in his petition that he was born in the City of Manila to the spouses Melecio
Petines Silverio and Anita Aquino Dantes on April 4, 1962. His name was registered as "Rommel
Jacinto Dantes Silverio" in his certificate of live birth (birth certificate). His sex was registered as
"male."
He further alleged that he is a male transsexual, that is, "anatomically male but feels, thinks and acts
as a female" and that he had always identified himself with girls since childhood.
1
Feeling trapped in
a mans body, he consulted several doctors in the United States. He underwent psychological
examination, hormone treatment and breast augmentation. His attempts to transform himself to a
"woman" culminated on January 27, 2001 when he underwent sex reassignment surgery
2
in
Bangkok, Thailand. He was thereafter examined by Dr. Marcelino Reysio-Cruz, Jr., a plastic and
reconstruction surgeon in the Philippines, who issued a medical certificate attesting that he
(petitioner) had in fact undergone the procedure.
From then on, petitioner lived as a female and was in fact engaged to be married. He then sought to
have his name in his birth certificate changed from "Rommel Jacinto" to "Mely," and his sex from
"male" to "female."
An order setting the case for initial hearing was published in the Peoples Journal Tonight, a
newspaper of general circulation in Metro Manila, for three consecutive weeks.
3
Copies of the order
were sent to the Office of the Solicitor General (OSG) and the civil registrar of Manila.
On the scheduled initial hearing, jurisdictional requirements were established. No opposition to the
petition was made.
During trial, petitioner testified for himself. He also presented Dr. Reysio-Cruz, Jr. and his American
fianc, Richard P. Edel, as witnesses.
On June 4, 2003, the trial court rendered a decision
4
in favor of petitioner. Its relevant portions read:
Petitioner filed the present petition not to evade any law or judgment or any infraction thereof
or for any unlawful motive but solely for the purpose of making his birth records compatible
with his present sex.
The sole issue here is whether or not petitioner is entitled to the relief asked for.
The [c]ourt rules in the affirmative.
Firstly, the [c]ourt is of the opinion that granting the petition would be more in consonance
with the principles of justice and equity. With his sexual [re-assignment], petitioner, who has
always felt, thought and acted like a woman, now possesses the physique of a female.
Petitioners misfortune to be trapped in a mans body is not his own doing and should not be
in any way taken against him.
Likewise, the [c]ourt believes that no harm, injury [or] prejudice will be caused to anybody or
the community in granting the petition. On the contrary, granting the petition would bring the
much-awaited happiness on the part of the petitioner and her [fianc] and the realization of
their dreams.
Finally, no evidence was presented to show any cause or ground to deny the present petition
despite due notice and publication thereof. Even the State, through the [OSG] has not seen
fit to interpose any [o]pposition.
WHEREFORE, judgment is hereby rendered GRANTING the petition and ordering the Civil
Registrar of Manila to change the entries appearing in the Certificate of Birth of [p]etitioner,
specifically for petitioners first name from "Rommel Jacinto" to MELY and petitioners gender
from "Male" to FEMALE.
5

On August 18, 2003, the Republic of the Philippines (Republic), thru the OSG, filed a petition for
certiorari in the Court of Appeals.
6
It alleged that there is no law allowing the change of entries in the
birth certificate by reason of sex alteration.
On February 23, 2006, the Court of Appeals
7
rendered a decision
8
in favor of the Republic. It ruled
that the trial courts decision lacked legal basis. There is no law allowing the change of either name
or sex in the certificate of birth on the ground of sex reassignment through surgery. Thus, the Court
of Appeals granted the Republics petition, set aside the decision of the trial court and ordered the
dismissal of SP Case No. 02-105207. Petitioner moved for reconsideration but it was
denied.
9
Hence, this petition.
Petitioner essentially claims that the change of his name and sex in his birth certificate is allowed
under Articles 407 to 413 of the Civil Code, Rules 103 and 108 of the Rules of Court and RA 9048.
10

The petition lacks merit.
A Persons First Name Cannot Be Changed On the Ground of Sex Reassignment
Petitioner invoked his sex reassignment as the ground for his petition for change of name and sex.
As found by the trial court:
Petitioner filed the present petition not to evade any law or judgment or any infraction thereof
or for any unlawful motive but solely for the purpose of making his birth records
compatible with his present sex. (emphasis supplied)
Petitioner believes that after having acquired the physical features of a female, he became entitled to
the civil registry changes sought. We disagree.
The State has an interest in the names borne by individuals and entities for purposes of
identification.
11
A change of name is a privilege, not a right.
12
Petitions for change of name are
controlled by statutes.
13
In this connection, Article 376 of the Civil Code provides:
ART. 376. No person can change his name or surname without judicial authority.
This Civil Code provision was amended by RA 9048 (Clerical Error Law). In particular, Section 1 of
RA 9048 provides:
SECTION 1. Authority to Correct Clerical or Typographical Error and Change of First Name
or Nickname. No entry in a civil register shall be changed or corrected without a judicial
order, except for clerical or typographical errors and change of first name or nickname which
can be corrected or changed by the concerned city or municipal civil registrar or consul
general in accordance with the provisions of this Act and its implementing rules and
regulations.
RA 9048 now governs the change of first name.
14
It vests the power and authority to entertain
petitions for change of first name to the city or municipal civil registrar or consul general concerned.
Under the law, therefore, jurisdiction over applications for change of first name is now primarily
lodged with the aforementioned administrative officers. The intent and effect of the law is to exclude
the change of first name from the coverage of Rules 103 (Change of Name) and 108 (Cancellation
or Correction of Entries in the Civil Registry) of the Rules of Court, until and unless an administrative
petition for change of name is first filed and subsequently denied.
15
It likewise lays down the
corresponding venue,
16
form
17
and procedure. In sum, the remedy and the proceedings regulating
change of first name are primarily administrative in nature, not judicial.
RA 9048 likewise provides the grounds for which change of first name may be allowed:
SECTION 4. Grounds for Change of First Name or Nickname. The petition for change of
first name or nickname may be allowed in any of the following cases:
(1) The petitioner finds the first name or nickname to be ridiculous, tainted with dishonor or
extremely difficult to write or pronounce;
(2) The new first name or nickname has been habitually and continuously used by the
petitioner and he has been publicly known by that first name or nickname in the community;
or
(3) The change will avoid confusion.
Petitioners basis in praying for the change of his first name was his sex reassignment. He intended
to make his first name compatible with the sex he thought he transformed himself into through
surgery. However, a change of name does not alter ones legal capacity or civil status.
18
RA 9048
does not sanction a change of first name on the ground of sex reassignment. Rather than avoiding
confusion, changing petitioners first name for his declared purpose may only create grave
complications in the civil registry and the public interest.
Before a person can legally change his given name, he must present proper or reasonable cause or
any compelling reason justifying such change.
19
In addition, he must show that he will be prejudiced
by the use of his true and official name.
20
In this case, he failed to show, or even allege, any
prejudice that he might suffer as a result of using his true and official name.
In sum, the petition in the trial court in so far as it prayed for the change of petitioners first name was
not within that courts primary jurisdiction as the petition should have been filed with the local civil
registrar concerned, assuming it could be legally done. It was an improper remedy because the
proper remedy was administrative, that is, that provided under RA 9048. It was also filed in the
wrong venue as the proper venue was in the Office of the Civil Registrar of Manila where his birth
certificate is kept. More importantly, it had no merit since the use of his true and official name does
not prejudice him at all. For all these reasons, the Court of Appeals correctly dismissed petitioners
petition in so far as the change of his first name was concerned.
No Law Allows The Change of Entry In The Birth Certificate As To Sex On the Ground of Sex
Reassignment
The determination of a persons sex appearing in his birth certificate is a legal issue and the court
must look to the statutes.
21
In this connection, Article 412 of the Civil Code provides:
ART. 412. No entry in the civil register shall be changed or corrected without a judicial order.
Together with Article 376 of the Civil Code, this provision was amended by RA 9048 in so far
as clerical or typographical errors are involved. The correction or change of such matters can now
be made through administrative proceedings and without the need for a judicial order. In effect, RA
9048 removed from the ambit of Rule 108 of the Rules of Court the correction of such errors.
22
Rule
108 now applies only to substantial changes and corrections in entries in the civil register.
23

Section 2(c) of RA 9048 defines what a "clerical or typographical error" is:
SECTION 2. Definition of Terms. As used in this Act, the following terms shall mean:
xxx xxx xxx
(3) "Clerical or typographical error" refers to a mistake committed in the performance
of clerical work in writing, copying, transcribing or typing an entry in the civil register
that is harmless and innocuous, such as misspelled name or misspelled place of
birth or the like, which is visible to the eyes or obvious to the understanding, and can
be corrected or changed only by reference to other existing record or
records: Provided, however, That no correction must involve the change
of nationality, age, status or sex of the petitioner. (emphasis supplied)
Under RA 9048, a correction in the civil registry involving the change of sex is not a mere clerical or
typographical error. It is a substantial change for which the applicable procedure is Rule 108 of the
Rules of Court.
The entries envisaged in Article 412 of the Civil Code and correctable under Rule 108 of the Rules of
Court are those provided in Articles 407 and 408 of the Civil Code:
24

ART. 407. Acts, events and judicial decrees concerning the civil status of persons shall be
recorded in the civil register.
ART. 408. The following shall be entered in the civil register:
(1) Births; (2) marriages; (3) deaths; (4) legal separations; (5) annulments of marriage; (6)
judgments declaring marriages void from the beginning; (7) legitimations; (8) adoptions; (9)
acknowledgments of natural children; (10) naturalization; (11) loss, or (12) recovery of
citizenship; (13) civil interdiction; (14) judicial determination of filiation; (15) voluntary
emancipation of a minor; and (16) changes of name.
The acts, events or factual errors contemplated under Article 407 of the Civil Code include even
those that occur after birth.
25
However, no reasonable interpretation of the provision can justify the
conclusion that it covers the correction on the ground of sex reassignment.
To correct simply means "to make or set aright; to remove the faults or error from" while to change
means "to replace something with something else of the same kind or with something that serves as
a substitute."
26
The birth certificate of petitioner contained no error. All entries therein, including those
corresponding to his first name and sex, were all correct. No correction is necessary.
Article 407 of the Civil Code authorizes the entry in the civil registry of certain acts (such as
legitimations, acknowledgments of illegitimate children and naturalization), events (such as births,
marriages, naturalization and deaths) and judicial decrees (such as legal separations, annulments of
marriage, declarations of nullity of marriages, adoptions, naturalization, loss or recovery of
citizenship, civil interdiction, judicial determination of filiation and changes of name). These acts,
events and judicial decrees produce legal consequences that touch upon the legal capacity, status
and nationality of a person. Their effects are expressly sanctioned by the laws. In contrast, sex
reassignment is not among those acts or events mentioned in Article 407. Neither is it recognized
nor even mentioned by any law, expressly or impliedly.
"Status" refers to the circumstances affecting the legal situation (that is, the sum total of capacities
and incapacities) of a person in view of his age, nationality and his family membership.
27

The status of a person in law includes all his personal qualities and relations, more or less
permanent in nature, not ordinarily terminable at his own will, such as his being
legitimate or illegitimate, or his being married or not. The comprehensive term status
include such matters as the beginning and end of legal personality, capacity to have rights in
general, family relations, and its various aspects, such as birth, legitimation, adoption,
emancipation, marriage, divorce, and sometimes even succession.
28
(emphasis supplied)
A persons sex is an essential factor in marriage and family relations. It is a part of a persons legal
capacity and civil status. In this connection, Article 413 of the Civil Code provides:
ART. 413. All other matters pertaining to the registration of civil status shall be governed by
special laws.
But there is no such special law in the Philippines governing sex reassignment and its effects. This is
fatal to petitioners cause.
Moreover, Section 5 of Act 3753 (the Civil Register Law) provides:
SEC. 5. Registration and certification of births. The declaration of the physician or midwife
in attendance at the birth or, in default thereof, the declaration of either parent of the
newborn child, shall be sufficient for the registration of a birth in the civil register. Such
declaration shall be exempt from documentary stamp tax and shall be sent to the local civil
registrar not later than thirty days after the birth, by the physician or midwife in attendance at
the birth or by either parent of the newborn child.
In such declaration, the person above mentioned shall certify to the following facts: (a) date
and hour of birth; (b) sex and nationality of infant; (c) names, citizenship and religion of
parents or, in case the father is not known, of the mother alone; (d) civil status of parents; (e)
place where the infant was born; and (f) such other data as may be required in the
regulations to be issued.
xxx xxx xxx (emphasis supplied)
Under the Civil Register Law, a birth certificate is a historical record of the facts as they existed at
the time of birth.
29
Thus, the sex of a person is determined at birth, visually done by the birth
attendant (the physician or midwife) by examining the genitals of the infant. Considering that there is
no law legally recognizing sex reassignment, the determination of a persons sex made at the time of
his or her birth, if not attended by error,
30
is immutable.
31

When words are not defined in a statute they are to be given their common and ordinary meaning in
the absence of a contrary legislative intent. The words "sex," "male" and "female" as used in the Civil
Register Law and laws concerning the civil registry (and even all other laws) should therefore be
understood in their common and ordinary usage, there being no legislative intent to the contrary. In
this connection, sex is defined as "the sum of peculiarities of structure and function that distinguish a
male from a female"
32
or "the distinction between male and female."
33
Female is "the sex that
produces ova or bears young"
34
and male is "the sex that has organs to produce spermatozoa for
fertilizing ova."
35
Thus, the words "male" and "female" in everyday understanding do not include
persons who have undergone sex reassignment. Furthermore, "words that are employed in a statute
which had at the time a well-known meaning are presumed to have been used in that sense unless
the context compels to the contrary."
36
Since the statutory language of the Civil Register Law was
enacted in the early 1900s and remains unchanged, it cannot be argued that the term "sex" as used
then is something alterable through surgery or something that allows a post-operative male-to-
female transsexual to be included in the category "female."
For these reasons, while petitioner may have succeeded in altering his body and appearance
through the intervention of modern surgery, no law authorizes the change of entry as to sex in the
civil registry for that reason. Thus, there is no legal basis for his petition for the correction or change
of the entries in his birth certificate.
Neither May Entries in the Birth Certificate As to First Name or Sex Be Changed on the
Ground of Equity
The trial court opined that its grant of the petition was in consonance with the principles of justice
and equity. It believed that allowing the petition would cause no harm, injury or prejudice to anyone.
This is wrong.
The changes sought by petitioner will have serious and wide-ranging legal and public policy
consequences. First, even the trial court itself found that the petition was but petitioners first step
towards his eventual marriage to his male fianc. However, marriage, one of the most sacred social
institutions, is a special contract of permanent union between a man and a woman.
37
One of its
essential requisites is the legal capacity of the contracting parties who must be a male and a
female.
38
To grant the changes sought by petitioner will substantially reconfigure and greatly alter the
laws on marriage and family relations. It will allow the union of a man with another man who has
undergone sex reassignment (a male-to-female post-operative transsexual). Second, there are
various laws which apply particularly to women such as the provisions of the Labor Code on
employment of women,
39
certain felonies under the Revised Penal Code
40
and the presumption of
survivorship in case of calamities under Rule 131 of the Rules of Court,
41
among others. These laws
underscore the public policy in relation to women which could be substantially affected if petitioners
petition were to be granted.
It is true that Article 9 of the Civil Code mandates that "[n]o judge or court shall decline to render
judgment by reason of the silence, obscurity or insufficiency of the law." However, it is not a license
for courts to engage in judicial legislation. The duty of the courts is to apply or interpret the law, not
to make or amend it.
In our system of government, it is for the legislature, should it choose to do so, to determine what
guidelines should govern the recognition of the effects of sex reassignment. The need for legislative
guidelines becomes particularly important in this case where the claims asserted are statute-based.
To reiterate, the statutes define who may file petitions for change of first name and for correction or
change of entries in the civil registry, where they may be filed, what grounds may be invoked, what
proof must be presented and what procedures shall be observed. If the legislature intends to confer
on a person who has undergone sex reassignment the privilege to change his name and sex to
conform with his reassigned sex, it has to enact legislation laying down the guidelines in turn
governing the conferment of that privilege.
It might be theoretically possible for this Court to write a protocol on when a person may be
recognized as having successfully changed his sex. However, this Court has no authority to fashion
a law on that matter, or on anything else. The Court cannot enact a law where no law exists. It can
only apply or interpret the written word of its co-equal branch of government, Congress.
Petitioner pleads that "[t]he unfortunates are also entitled to a life of happiness, contentment and
[the] realization of their dreams." No argument about that. The Court recognizes that there are
people whose preferences and orientation do not fit neatly into the commonly recognized
parameters of social convention and that, at least for them, life is indeed an ordeal. However, the
remedies petitioner seeks involve questions of public policy to be addressed solely by the
legislature, not by the courts.
WHEREFORE, the petition is hereby DENIED.
Costs against petitioner.
SO ORDERED.
Puno, C.J., Chairperson, Sandoval-Gutierrez, Azcuna, Garcia, JJ., concur.
























Republic of the Philippines
Supreme Court
Baguio City

EN BANC

ANG LADLAD LGBT PARTY G.R. No. 190582
represented herein by its Chair,
DANTON REMOTO,
Petitioner, Present:

PUNO, C. J.,
CARPIO,
CORONA,
CARPIO MORALES,
VELASCO, JR.,
NACHURA,
LEONARDO-DE CASTRO,
- versus - BRION,
PERALTA,
BERSAMIN,
DEL CASTILLO,
ABAD,
VILLARAMA, JR.,
PEREZ, and
MENDOZA, JJ.

COMMISSION ON ELECTIONS, Promulgated:
Respondent. April 8, 2010
x - - - - - - - - - - - - - - - - - - - - - - - - - - - - - - - - - - - - - - - - - - - - - - - - - - - - - - - - x

D E C I S I O N

DEL CASTILLO, J .:

... [F]reedom to differ is not limited to things that do not matter much. That would be a mere
shadow of freedom. The test of its substance is the right to differ as to things that touch the heart
of the existing order.

Justice Robert A. Jackson
West Virginia State Board of Education v. Barnette
[1]


One unavoidable consequence of everyone having the freedom to choose is that others may make
different choices choices we would not make for ourselves, choices we may disapprove of, even choices that may
shock or offend or anger us. However, choices are not to be legally prohibited merely because they are different,
and the right to disagree and debate about important questions of public policy is a core value protected by our Bill
of Rights. Indeed, our democracy is built on genuine recognition of, and respect for, diversity and difference in
opinion.

Since ancient times, society has grappled with deep disagreements about the definitions and demands of
morality. In many cases, where moral convictions are concerned, harmony among those theoretically opposed is
an insurmountable goal. Yet herein lies the paradox philosophical justifications about what is moral are
indispensable and yet at the same time powerless to create agreement. This Court recognizes, however, that
practical solutions are preferable to ideological stalemates; accommodation is better than intransigence; reason
more worthy than rhetoric. This will allow persons of diverse viewpoints to live together, if not harmoniously, then, at
least, civilly.

Factual Background

This is a Petition for Certiorari under Rule 65 of the Rules of Court, with an application for a writ of
preliminary mandatory injunction, filed by Ang Ladlad LGBT Party (Ang Ladlad) against the Resolutions of the
Commission on Elections (COMELEC) dated November 11, 2009
[2]
(the First Assailed Resolution) and December
16, 2009
[3]
(the Second Assailed Resolution) in SPP No. 09-228 (PL) (collectively, the Assailed Resolutions). The
case has its roots in the COMELECs refusal to accredit Ang Ladlad as a party-list organization under Republic Act
(RA) No. 7941, otherwise known as the Party-List System Act.
[4]


Ang Ladlad is an organization composed of men and women who identify themselves as lesbians, gays,
bisexuals, or trans-gendered individuals (LGBTs). Incorporated in 2003, Ang Ladlad first applied for registration with
the COMELEC in 2006. The application for accreditation was denied on the ground that the organization had no
substantial membership base. On August 17, 2009, Ang Ladlad again filed a Petition
[5]
for registration with the
COMELEC.

Before the COMELEC, petitioner argued that the LGBT community is a marginalized and under-
represented sector that is particularly disadvantaged because of their sexual orientation and gender identity; that
LGBTs are victims of exclusion, discrimination, and violence; that because of negative societal attitudes, LGBTs are
constrained to hide their sexual orientation; and that Ang Ladlad complied with the 8-point guidelines enunciated by
this Court in Ang Bagong Bayani-OFW Labor Party v. Commission on Elections.
[6]
Ang Ladlad laid out its national
membership base consisting of individual members and organizational supporters, and outlined its platform of
governance.
[7]


On November 11, 2009, after admitting the petitioners evidence, the COMELEC (Second Division)
dismissed the Petition on moral grounds, stating that:
x x x This Petition is dismissible on moral grounds. Petitioner defines the Filipino Lesbian,
Gay, Bisexual and Transgender (LGBT) Community, thus:

x x x a marginalized and under-represented sector that is particularly
disadvantaged because of their sexual orientation and gender identity.

and proceeded to define sexual orientation as that which:

x x x refers to a persons capacity for profound emotional, affectional and sexual
attraction to, and intimate and sexual relations with, individuals of a different
gender, of the same gender, or more than one gender.

This definition of the LGBT sector makes it crystal clear that petitioner tolerates
immorality which offends religious beliefs. In Romans 1:26, 27, Paul wrote:

For this cause God gave them up into vile affections, for even their
women did change the natural use into that which is against nature: And
likewise also the men, leaving the natural use of the woman, burned in their lust
one toward another; men with men working that which is unseemly, and
receiving in themselves that recompense of their error which was meet.

In the Koran, the hereunder verses are pertinent:

For ye practice your lusts on men in preference to women ye are
indeed a people transgressing beyond bounds. (7.81) And we rained down on
them a shower (of brimstone): Then see what was the end of those who
indulged in sin and crime! (7:84) He said: O my Lord! Help Thou me against
people who do mischief (29:30).

As correctly pointed out by the Law Department in its Comment dated October 2, 2008:

The ANG LADLAD apparently advocates sexual immorality as
indicated in the Petitions par. 6F: Consensual partnerships or relationships by
gays and lesbians who are already of age. It is further indicated in par. 24 of the
Petition which waves for the record: In 2007, Men Having Sex with Men or
MSMs in the Philippines were estimated as 670,000 (Genesis 19 is the history
of Sodom and Gomorrah).

Laws are deemed incorporated in every contract, permit, license,
relationship, or accreditation. Hence, pertinent provisions of the Civil Code and
the Revised Penal Code are deemed part of the requirement to be complied
with for accreditation.

ANG LADLAD collides with Article 695 of the Civil Code which defines
nuisance as Any act, omission, establishment, business, condition of property,
or anything else which x x x (3) shocks, defies; or disregards decency or
morality x x x

It also collides with Article 1306 of the Civil Code: The contracting
parties may establish such stipulations, clauses, terms and conditions as they
may deem convenient, provided they are not contrary to law, morals, good
customs, public order or public policy. Art 1409 of the Civil Code provides that
Contracts whose cause, object or purpose is contrary to law,morals, good
customs, public order or public policy are inexistent and void from the
beginning.

Finally to safeguard the morality of the Filipino community, the Revised Penal Code, as
amended, penalizes Immoral doctrines, obscene publications and exhibitions and indecent
shows as follows:

Art. 201. Immoral doctrines, obscene publications and exhibitions, and
indecent shows. The penalty of prision mayor or a fine ranging from six
thousand to twelve thousand pesos, or both such imprisonment and fine, shall
be imposed upon:

1. Those who shall publicly expound or proclaim doctrines
openly contrary to public morals;

2. (a) The authors of obscene literature, published with their
knowledge in any form; the editors publishing such literature; and the
owners/operators of the establishment selling the same;

(b) Those who, in theaters, fairs, cinematographs or any other
place, exhibit indecent or immoral plays, scenes, acts or shows, it being
understood that the obscene literature or indecent or immoral plays, scenes,
acts or shows, whether live or in film, which are prescribed by virtue hereof, shall
include those which: (1) glorify criminals or condone crimes; (2) serve no other
purpose but to satisfy the market for violence, lust or pornography; (3) offend
any race or religion; (4) tend to abet traffic in and use of prohibited drugs; and (5)
are contrary to law, public order, morals, good customs, established policies,
lawful orders, decrees and edicts.

3. Those who shall sell, give away or exhibit films, prints, engravings,
sculpture or literature which are offensive to morals.

Petitioner should likewise be denied accreditation not only for advocating immoral
doctrines but likewise for not being truthful when it said that it or any of its nominees/party-list
representatives have not violated or failed to comply with laws, rules, or regulations relating to the
elections.

Furthermore, should this Commission grant the petition, we will be exposing our youth to an
environment that does not conform to the teachings of our faith. Lehman Strauss, a famous bible
teacher and writer in the U.S.A. said in one article that older practicing homosexuals are a threat
to the youth. As an agency of the government, ours too is the States avowed duty under Section
13, Article II of the Constitution to protect our youth from moral and spiritual degradation.
[8]



When Ang Ladlad sought reconsideration,
[9]
three commissioners voted to overturn the First Assailed
Resolution (Commissioners Gregorio Y. Larrazabal, Rene V. Sarmiento, and Armando Velasco), while three
commissioners voted to deny Ang Ladlads Motion for Reconsideration (Commissioners Nicodemo T. Ferrer,
Lucenito N. Tagle, and Elias R. Yusoph). The COMELEC Chairman, breaking the tie and speaking for the majority
in his Separate Opinion, upheld the First Assailed Resolution, stating that:

I. The Spirit of Republic Act No. 7941

Ladlad is applying for accreditation as a sectoral party in the party-list system. Even
assuming that it has properly proven its under-representation and marginalization, it cannot be
said thatLadlads expressed sexual orientations per se would benefit the nation as a whole.

Section 2 of the party-list law unequivocally states that the purpose of the party-list
system of electing congressional representatives is to enable Filipino citizens belonging to
marginalized and under-represented sectors, organizations and parties, and who lack well-defined
political constituencies but who could contribute to the formulation and enactment of appropriate
legislation that will benefit the nation as a whole, to become members of the House of
Representatives.

If entry into the party-list system would depend only on the ability of an organization to
represent its constituencies, then all representative organizations would have found themselves
into the party-list race. But that is not the intention of the framers of the law. The party-list system is
not a tool to advocate tolerance and acceptance of misunderstood persons or groups of persons.
Rather, the party-list system is a tool for the realization of aspirations of marginalized
individuals whose interests are also the nations only that their interests have not been
brought to the attention of the nation because of their under representation. Until the time comes
when Ladlad is able to justify that having mixed sexual orientations and transgender
identities is beneficial to the nation, its application for accreditation under the party-list
system will remain just that.

II. No substantial differentiation

In the United States, whose equal protection doctrine pervades Philippine jurisprudence,
courts do not recognize lesbians, gays, homosexuals, and bisexuals (LGBT) as a special class
of individuals. x x x Significantly, it has also been held that homosexuality is not a constitutionally
protected fundamental right, and that nothing in the U.S. Constitution discloses a comparable
intent to protect or promote the social or legal equality of homosexual relations, as in the case of
race or religion or belief.

x x x x

Thus, even if societys understanding, tolerance, and acceptance of LGBTs is elevated,
there can be no denying that Ladlad constituencies are still males and females, and they will
remain either male or female protected by the same Bill of Rights that applies to all
citizens alike.

x x x x

IV. Public Morals

x x x There is no question about not imposing on Ladlad Christian or Muslim religious
practices. Neither is there any attempt to any particular religious groups moral rules on Ladlad.
Rather, what are being adopted as moral parameters and precepts are generally accepted public
morals. They are possibly religious-based, but as a society, the Philippines cannot ignore its
more than 500 years of Muslim and Christian upbringing, such that some moral precepts
espoused by said religions have sipped [sic] into society and these are not publicly
accepted moral norms.

V. Legal Provisions

But above morality and social norms, they have become part of the law of the land.
Article 201 of the Revised Penal Code imposes the penalty of prision mayor upon Those who
shall publicly expound or proclaim doctrines openly contrary to public morals. It penalizes
immoral doctrines, obscene publications and exhibition and indecent shows. Ang
Ladlad apparently falls under these legal provisions. This is clear from its Petitions paragraph
6F: Consensual partnerships or relationships by gays and lesbians who are already of age It is
further indicated in par. 24 of the Petition which waves for the record: In 2007, Men Having Sex
with Men or MSMs in the Philippines were estimated as 670,000. Moreoever, Article 694 of the
Civil Code defines nuisance as any act, omission x x x or anything else x x x which shocks,
defies or disregards decency or morality x x x. These are all unlawful.
[10]



On January 4, 2010, Ang Ladlad filed this Petition, praying that the Court annul the Assailed Resolutions
and direct the COMELEC to grant Ang Ladlads application for accreditation. Ang Ladlad also sought the
issuance ex parte of a preliminary mandatory injunction against the COMELEC, which had previously announced
that it would begin printing the final ballots for the May 2010 elections by January 25, 2010.

On January 6, 2010, we ordered the Office of the Solicitor General (OSG) to file its Comment on behalf of
COMELEC not later than 12:00 noon of January 11, 2010.
[11]
Instead of filing a Comment, however, the OSG filed a
Motion for Extension, requesting that it be given until January 16, 2010 to Comment.
[12]
Somewhat surprisingly, the
OSG later filed a Comment in support of petitioners application.
[13]
Thus, in order to give COMELEC the opportunity
to fully ventilate its position, we required it to file its own comment.
[14]
The COMELEC, through its Law Department,
filed its Comment on February 2, 2010.
[15]


In the meantime, due to the urgency of the petition, we issued a temporary restraining order on January 12,
2010, effective immediately and continuing until further orders from this Court, directing the COMELEC to cease and
desist from implementing the Assailed Resolutions.
[16]


Also, on January 13, 2010, the Commission on Human Rights (CHR) filed a Motion to Intervene or to
Appear as Amicus Curiae, attaching thereto its Comment-in-Intervention.
[17]
The CHR opined that the denial of Ang
Ladlads petition on moral grounds violated the standards and principles of the Constitution, the Universal
Declaration of Human Rights (UDHR), and the International Covenant on Civil and Political Rights
(ICCPR). On January 19, 2010, we granted the CHRs motion to intervene.

On January 26, 2010, Epifanio D. Salonga, Jr. filed his Motion to Intervene
[18]
which motion was granted
on February 2, 2010.
[19]


The Parties Arguments

Ang Ladlad argued that the denial of accreditation, insofar as it justified the exclusion by using religious
dogma, violated the constitutional guarantees against the establishment of religion. Petitioner also claimed that the
Assailed Resolutions contravened its constitutional rights to privacy, freedom of speech and assembly, and equal
protection of laws, as well as constituted violations of the Philippines international obligations against discrimination
based on sexual orientation.

The OSG concurred with Ang Ladlads petition and argued that the COMELEC erred in denying petitioners
application for registration since there was no basis for COMELECs allegations of immorality. It also opined that
LGBTs have their own special interests and concerns which should have been recognized by the COMELEC as a
separate classification. However, insofar as the purported violations of petitioners freedom of speech, expression,
and assembly were concerned, the OSG maintained that there had been no restrictions on these rights.

In its Comment, the COMELEC reiterated that petitioner does not have a concrete and genuine national
political agenda to benefit the nation and that the petition was validly dismissed on moral grounds. It also argued for
the first time that the LGBT sector is not among the sectors enumerated by the Constitution and RA 7941, and that
petitioner made untruthful statements in its petition when it alleged its national existence contrary to actual
verification reports by COMELECs field personnel.

Our Ruling

We grant the petition.

Compliance with the Requirements of the Constitution
and Republic Act No. 7941


The COMELEC denied Ang Ladlads application for registration on the ground that the LGBT sector is
neither enumerated in the Constitution and RA 7941, nor is it associated with or related to any of the sectors in the
enumeration.

Respondent mistakenly opines that our ruling in Ang Bagong Bayani stands for the proposition that only
those sectors specifically enumerated in the law or related to said sectors (labor, peasant, fisherfolk, urban poor,
indigenous cultural communities, elderly, handicapped, women, youth, veterans, overseas workers, and
professionals) may be registered under the party-list system. As we explicitly ruled in Ang Bagong Bayani-OFW
Labor Party v. Commission on Elections,
[20]
the enumeration of marginalized and under-represented sectors is not
exclusive. The crucial element is not whether a sector is specifically enumerated, but whether a particular
organization complies with the requirements of the Constitution and RA 7941.

Respondent also argues that Ang Ladlad made untruthful statements in its petition when it alleged that it
had nationwide existence through its members and affiliate organizations. The COMELEC claims that upon
verification by its field personnel, it was shown that save for a few isolated places in the country, petitioner does not
exist in almost all provinces in the country.
[21]

This argument that petitioner made untruthful statements in its petition when it alleged its national
existence is a new one; previously, the COMELEC claimed that petitioner was not being truthful when it said that it
or any of its nominees/party-list representatives have not violated or failed to comply with laws, rules, or regulations
relating to the elections. Nowhere was this ground for denial of petitioners accreditation mentioned or even alluded
to in the Assailed Resolutions. This, in itself, is quite curious, considering that the reports of petitioners alleged non-
existence were already available to the COMELEC prior to the issuance of the First Assailed Resolution. At best,
this is irregular procedure; at worst, a belated afterthought, a change in respondents theory, and a serious violation
of petitioners right to procedural due process.

Nonetheless, we find that there has been no misrepresentation. A cursory perusal of Ang Ladlads initial
petition shows that it never claimed to exist in each province of thePhilippines. Rather, petitioner alleged that the
LGBT community in the Philippines was estimated to constitute at least 670,000 persons; that it had 16,100 affiliates
and members around the country, and 4,044 members in its electronic discussion group.
[22]
Ang Ladlad also
represented itself to be a national LGBT umbrella organization with affiliates around the Philippines composed of
the following LGBT networks:

Abra Gay Association
Aklan Butterfly Brigade (ABB) Aklan
Albay Gay Association
Arts Center of Cabanatuan City Nueva Ecija
Boys Legion Metro Manila
Cagayan de Oro People Like Us (CDO PLUS)
Cant Live in the Closet, Inc. (CLIC) Metro Manila
Cebu Pride Cebu City
Circle of Friends
Dipolog Gay Association Zamboanga del Norte
Gay, Bisexual, & Transgender Youth Association (GABAY)
Gay and Lesbian Activists Network for Gender Equality (GALANG)
Metro Manila
Gay Mens Support Group (GMSG) Metro Manila
Gay United for Peace and Solidarity (GUPS) Lanao del Norte
Iloilo City Gay Association Iloilo City
Kabulig Writers Group Camarines Sur
Lesbian Advocates Philippines, Inc. (LEAP)
LUMINA Baguio City
Marikina Gay Association Metro Manila
Metropolitan Community Church (MCC) Metro Manila
Naga City Gay Association Naga City
ONE BACARDI
Order of St. Aelred (OSAe) Metro Manila
PUP LAKAN
RADAR PRIDEWEAR
Rainbow Rights Project (R-Rights), Inc. Metro Manila
San Jose del Monte Gay Association Bulacan
Sining Kayumanggi Royal Family Rizal
Society of Transexual Women of the Philippines (STRAP) Metro Manila
Soul Jive Antipolo, Rizal
The Link Davao City
Tayabas Gay Association Quezon
Womens Bisexual Network Metro Manila
Zamboanga Gay Association Zamboanga City
[23]



Since the COMELEC only searched for the names ANG LADLAD LGBT or LADLAD LGBT, it is no
surprise that they found that petitioner had no presence in any of these regions. In fact, if COMELECs findings are
to be believed, petitioner does not even exist in Quezon City, which is registered as Ang Ladlads principal place of
business.

Against this backdrop, we find that Ang Ladlad has sufficiently demonstrated its compliance with the legal
requirements for accreditation. Indeed, aside from COMELECs moral objection and the belated allegation of non-
existence, nowhere in the records has the respondent ever found/ruled that Ang Ladlad is not qualified to register as
a party-list organization under any of the requisites under RA 7941 or the guidelines in Ang Bagong Bayani. The
difference, COMELEC claims, lies in Ang Ladlads morality, or lack thereof.

Religion as the Basis for Refusal to Accept Ang Ladlads
Petition for Registration


Our Constitution provides in Article III, Section 5 that [n]o law shall be made respecting an establishment of
religion, or prohibiting the free exercise thereof. At bottom, what our non-establishment clause calls for is
government neutrality in religious matters.
[24]
Clearly, governmental reliance on religious justification is
inconsistent with this policy of neutrality.
[25]
We thus find that it was grave violation of the non-establishment clause
for the COMELEC to utilize the Bible and the Koran to justify the exclusion ofAng Ladlad.

Rather than relying on religious belief, the legitimacy of the Assailed Resolutions should depend, instead,
on whether the COMELEC is able to advance some justification for its rulings beyond mere conformity to religious
doctrine. Otherwise stated, government must act for secular purposes and in ways that have primarily secular
effects. As we held inEstrada v. Escritor:
[26]


x x x The morality referred to in the law is public and necessarily secular, not religious as
the dissent of Mr. Justice Carpio holds. "Religious teachings as expressed in public debate may
influence the civil public order but public moral disputes may be resolved only on grounds
articulable in secular terms." Otherwise, if government relies upon religious beliefs in formulating
public policies and morals, the resulting policies and morals would require conformity to what
some might regard as religious programs or agenda. The non-believers would therefore be
compelled to conform to a standard of conduct buttressed by a religious belief, i.e., to a
"compelled religion," anathema to religious freedom. Likewise, if government based its actions
upon religious beliefs, it would tacitly approve or endorse that belief and thereby also tacitly
disapprove contrary religious or non-religious views that would not support the policy. As a result,
government will not provide full religious freedom for all its citizens, or even make it appear that
those whose beliefs are disapproved are second-class citizens.
In other words, government action, including its proscription of immorality as expressed
in criminal law like concubinage, must have a secular purpose. That is, the government proscribes
this conduct because it is "detrimental (or dangerous) to those conditions upon which depend the
existence and progress of human society" and not because the conduct is proscribed by the
beliefs of one religion or the other. Although admittedly, moral judgments based on religion might
have a compelling influence on those engaged in public deliberations over what actions would be
considered a moral disapprobation punishable by law. After all, they might also be adherents of a
religion and thus have religious opinions and moral codes with a compelling influence on them;
the human mind endeavors to regulate the temporal and spiritual institutions of society in a
uniform manner, harmonizing earth with heaven. Succinctly put, a law could be religious or
Kantian or Aquinian or utilitarian in its deepest roots, but it must have an articulable and discernible
secular purpose and justification to pass scrutiny of the religion clauses. x x x Recognizing the
religious nature of the Filipinos and the elevating influence of religion in society, however, the
Philippine constitution's religion clauses prescribe not a strict but a benevolent neutrality.
Benevolent neutrality recognizes that government must pursue its secular goals and interests but
at the same time strive to uphold religious liberty to the greatest extent possible within flexible
constitutional limits. Thus, although the morality contemplated by laws is secular, benevolent
neutrality could allow for accommodation of morality based on religion, provided it does not offend
compelling state interests.
[27]



Public Morals as a Ground to Deny Ang Ladlads Petition
for Registration


Respondent suggests that although the moral condemnation of homosexuality and homosexual conduct
may be religion-based, it has long been transplanted into generally accepted public morals. The COMELEC
argues:

Petitioners accreditation was denied not necessarily because their group consists of
LGBTs but because of the danger it poses to the people especially the youth. Once it is
recognized by the government, a sector which believes that there is nothing wrong in having
sexual relations with individuals of the same gender is a bad example. It will bring down the
standard of morals we cherish in our civilized society. Any society without a set of moral precepts
is in danger of losing its own existence.
[28]



We are not blind to the fact that, through the years, homosexual conduct, and perhaps homosexuals
themselves, have borne the brunt of societal disapproval. It is not difficult to imagine the reasons behind this
censure religious beliefs, convictions about the preservation of marriage, family, and procreation, even dislike or
distrust of homosexuals themselves and their perceived lifestyle. Nonetheless, we recall that the Philippines has not
seen fit to criminalize homosexual conduct. Evidently, therefore, these generally accepted public morals have not
been convincingly transplanted into the realm of law.
[29]


The Assailed Resolutions have not identified any specific overt immoral act performed by Ang Ladlad. Even the
OSG agrees that there should have been a finding by the COMELEC that the groups members have committed or
are committing immoral acts.
[30]
The OSG argues:

x x x A person may be sexually attracted to a person of the same gender, of a different
gender, or more than one gender, but mere attraction does not translate to immoral acts. There is
a great divide between thought and action. Reduction ad absurdum. If immoral thoughts could be
penalized, COMELEC would have its hands full of disqualification cases against both the
straights and the gays. Certainly this is not the intendment of the law.
[31]



Respondent has failed to explain what societal ills are sought to be prevented, or why special protection is
required for the youth. Neither has the COMELEC condescended to justify its position that petitioners admission
into the party-list system would be so harmful as to irreparably damage the moral fabric of society. We, of course,
do not suggest that the state is wholly without authority to regulate matters concerning morality, sexuality, and
sexual relations, and we recognize that the government will and should continue to restrict behavior considered
detrimental to society. Nonetheless, we cannot countenance advocates who, undoubtedly with the loftiest of
intentions, situate morality on one end of an argument or another, without bothering to go through the rigors of legal
reasoning and explanation. In this, the notion of morality is robbed of all value. Clearly then, the bare invocation of
morality will not remove an issue from our scrutiny.

We also find the COMELECs reference to purported violations of our penal and civil laws flimsy, at best;
disingenuous, at worst. Article 694 of the Civil Code defines a nuisance as any act, omission, establishment,
condition of property, or anything else which shocks, defies, or disregards decency or morality, the remedies for
which are a prosecution under the Revised Penal Code or any local ordinance, a civil action, or abatement without
judicial proceedings.
[32]
A violation of Article 201 of the Revised Penal Code, on the other hand, requires proof
beyond reasonable doubt to support a criminal conviction. It hardly needs to be emphasized that mere allegation of
violation of laws is not proof, and a mere blanket invocation of public morals cannot replace the institution of civil or
criminal proceedings and a judicial determination of liability or culpability.

As such, we hold that moral disapproval, without more, is not a sufficient governmental interest to justify
exclusion of homosexuals from participation in the party-list system. The denial of Ang Ladlads registration on
purely moral grounds amounts more to a statement of dislike and disapproval of homosexuals, rather than a tool to
further any substantial public interest. Respondents blanket justifications give rise to the inevitable conclusion that
the COMELEC targets homosexuals themselves as a class, not because of any particular morally reprehensible
act. It is this selective targeting that implicates our equal protection clause.

Equal Protection

Despite the absolutism of Article III, Section 1 of our Constitution, which provides nor shall any person be
denied equal protection of the laws, courts have never interpreted the provision as an absolute prohibition on
classification. Equality, said Aristotle, consists in the same treatment of similar persons.
[33]
The equal protection
clause guarantees that no person or class of persons shall be deprived of the same protection of laws which is
enjoyed by other persons or other classes in the same place and in like circumstances.
[34]


Recent jurisprudence has affirmed that if a law neither burdens a fundamental right nor targets a suspect
class, we will uphold the classification as long as it bears a rational relationship to some legitimate government
end.
[35]
In Central Bank Employees Association, Inc. v. Banko Sentral ng Pilipinas,
[36]
we declared that [i]n our
jurisdiction, the standard of analysis of equal protection challenges x x x have followed the rational basis test,
coupled with a deferential attitude to legislative classifications and a reluctance to invalidate a law unless there is a
showing of a clear and unequivocal breach of the Constitution.
[37]


The COMELEC posits that the majority of the Philippine population considers homosexual conduct as
immoral and unacceptable, and this constitutes sufficient reason to disqualify the petitioner. Unfortunately for the
respondent, the Philippine electorate has expressed no such belief. No law exists to criminalize homosexual
behavior or expressions or parties about homosexual behavior. Indeed, even if we were to assume that public
opinion is as the COMELEC describes it, the asserted state interest here that is, moral disapproval of an
unpopular minority is not a legitimate state interest that is sufficient to satisfy rational basis review under the equal
protection clause. The COMELECs differentiation, and its unsubstantiated claim that Ang Ladlad cannot contribute
to the formulation of legislation that would benefit the nation, furthers no legitimate state interest other than
disapproval of or dislike for a disfavored group.

From the standpoint of the political process, the lesbian, gay, bisexual, and transgender have the same
interest in participating in the party-list system on the same basis as other political parties similarly situated. State
intrusion in this case is equally burdensome. Hence, laws of general application should apply with equal force to
LGBTs, and they deserve to participate in the party-list system on the same basis as other marginalized and under-
represented sectors.

It bears stressing that our finding that COMELECs act of differentiating LGBTs from heterosexuals insofar
as the party-list system is concerned does not imply that any other law distinguishing between heterosexuals and
homosexuals under different circumstances would similarly fail. We disagree with the OSGs position that
homosexuals are a class in themselves for the purposes of the equal protection clause.
[38]
We are not prepared to
single out homosexuals as a separate class meriting special or differentiated treatment. We have not received
sufficient evidence to this effect, and it is simply unnecessary to make such a ruling today. Petitioner itself has
merely demanded that it be recognized under the same basis as all other groups similarly situated, and that the
COMELEC made an unwarranted and impermissible classification not justified by the circumstances of the case.

Freedom of Expression and Association

Under our system of laws, every group has the right to promote its agenda and attempt to persuade society
of the validity of its position through normal democratic means.
[39]
It is in the public square that deeply held
convictions and differing opinions should be distilled and deliberated upon. As we held in Estrada v. Escritor:
[40]


In a democracy, this common agreement on political and moral ideas is distilled in the
public square. Where citizens are free, every opinion, every prejudice, every aspiration, and every
moral discernment has access to the public square where people deliberate the order of their life
together. Citizens are the bearers of opinion, including opinion shaped by, or espousing religious
belief, and these citizens have equal access to the public square. In this representative
democracy, the state is prohibited from determining which convictions and moral judgments may
be proposed for public deliberation. Through a constitutionally designed process, the people
deliberate and decide. Majority rule is a necessary principle in this democratic governance. Thus,
when public deliberation on moral judgments is finally crystallized into law, the laws will largely
reflect the beliefs and preferences of the majority, i.e., the mainstream or median
groups.

Nevertheless, in the very act of adopting and accepting a constitution and the limits it
specifies including protection of religious freedom "not only for a minority, however small not
only for a majority, however large but for each of us" the majority imposes upon itself a self-
denying ordinance. It promises not to do what it otherwise could do: to ride roughshod over the
dissenting minorities.


Freedom of expression constitutes one of the essential foundations of a democratic society, and this
freedom applies not only to those that are favorably received but also to those that offend, shock, or disturb. Any
restriction imposed in this sphere must be proportionate to the legitimate aim pursued. Absent any compelling state
interest, it is not for the COMELEC or this Court to impose its views on the populace. Otherwise stated, the
COMELEC is certainly not free to interfere with speech for no better reason than promoting an approved message
or discouraging a disfavored one.

This position gains even more force if one considers that homosexual conduct is not illegal in this country. It
follows that both expressions concerning ones homosexuality and the activity of forming a political association that
supports LGBT individuals are protected as well.
Other jurisdictions have gone so far as to categorically rule that even overwhelming public perception that
homosexual conduct violates public morality does not justify criminalizing same-sex conduct.
[41]
European and
United Nations judicial decisions have ruled in favor of gay rights claimants on both privacy and equality grounds,
citing general privacy and equal protection provisions in foreign and international texts.
[42]
To the extent that there is
much to learn from other jurisdictions that have reflected on the issues we face here, such jurisprudence is certainly
illuminating. These foreign authorities, while not formally binding on Philippine courts, may nevertheless have
persuasive influence on the Courts analysis.

In the area of freedom of expression, for instance, United States courts have ruled that existing free speech
doctrines protect gay and lesbian rights to expressive conduct. In order to justify the prohibition of a particular
expression of opinion, public institutions must show that their actions were caused by something more than a mere
desire to avoid the discomfort and unpleasantness that always accompany an unpopular viewpoint.
[43]


With respect to freedom of association for the advancement of ideas and beliefs, in Europe, with its vibrant
human rights tradition, the European Court of Human Rights (ECHR) has repeatedly stated that a political party may
campaign for a change in the law or the constitutional structures of a state if it uses legal and democratic means and
the changes it proposes are consistent with democratic principles. The ECHR has emphasized that political ideas
that challenge the existing order and whose realization is advocated by peaceful means must be afforded a proper
opportunity of expression through the exercise of the right of association, even if such ideas may seem shocking or
unacceptable to the authorities or the majority of the population.
[44]
A political group should not be hindered solely
because it seeks to publicly debate controversial political issues in order to find solutions capable of satisfying
everyone concerned.
[45]
Only if a political party incites violence or puts forward policies that are incompatible with
democracy does it fall outside the protection of the freedom of association guarantee.
[46]


We do not doubt that a number of our citizens may believe that homosexual conduct is distasteful,
offensive, or even defiant. They are entitled to hold and express that view. On the other hand, LGBTs and their
supporters, in all likelihood, believe with equal fervor that relationships between individuals of the same sex are
morally equivalent to heterosexual relationships. They, too, are entitled to hold and express that view. However, as
far as this Court is concerned, our democracy precludes using the religious or moral views of one part of the
community to exclude from consideration the values of other members of the community.

Of course, none of this suggests the impending arrival of a golden age for gay rights litigants. It well may be
that this Decision will only serve to highlight the discrepancy between the rigid constitutional analysis of this Court
and the more complex moral sentiments of Filipinos. We do not suggest that public opinion, even at its most liberal,
reflect a clear-cut strong consensus favorable to gay rights claims and we neither attempt nor expect to affect
individual perceptions of homosexuality through this Decision.

The OSG argues that since there has been neither prior restraint nor subsequent punishment imposed
on Ang Ladlad, and its members have not been deprived of their right to voluntarily associate, then there has been
no restriction on their freedom of expression or association. The OSG argues that:

There was no utterance restricted, no publication censored, or any assembly denied.
[COMELEC] simply exercised its authority to review and verify the qualifications of petitioner as a
sectoral party applying to participate in the party-list system. This lawful exercise of duty cannot be
said to be a transgression of Section 4, Article III of the Constitution.

x x x x

A denial of the petition for registration x x x does not deprive the members of the
petitioner to freely take part in the conduct of elections. Their right to vote will not be hampered by
said denial. In fact, the right to vote is a constitutionally-guaranteed right which cannot be limited.

As to its right to be elected in a genuine periodic election, petitioner contends that the
denial of Ang Ladlads petition has the clear and immediate effect of limiting, if not outrightly
nullifying the capacity of its members to fully and equally participate in public life through
engagement in the party list elections.

This argument is puerile. The holding of a public office is not a right but a privilege subject
to limitations imposed by law. x x x
[47]

The OSG fails to recall that petitioner has, in fact, established its qualifications to participate in the party-list
system, and as advanced by the OSG itself the moral objection offered by the COMELEC was not a limitation
imposed by law. To the extent, therefore, that the petitioner has been precluded, because of COMELECs action,
from publicly expressing its views as a political party and participating on an equal basis in the political process with
other equally-qualified party-list candidates, we find that there has, indeed, been a transgression of petitioners
fundamental rights.

Non-Discrimination and International Law


In an age that has seen international law evolve geometrically in scope and promise, international human
rights law, in particular, has grown dynamically in its attempt to bring about a more just and humane world order. For
individuals and groups struggling with inadequate structural and governmental support, international human rights
norms are particularly significant, and should be effectively enforced in domestic legal systems so that such norms
may become actual, rather than ideal, standards of conduct.

Our Decision today is fully in accord with our international obligations to protect and promote human rights.
In particular, we explicitly recognize the principle of non-discrimination as it relates to the right to electoral
participation, enunciated in the UDHR and the ICCPR.

The principle of non-discrimination is laid out in Article 26 of the ICCPR, as follows:

Article 26

All persons are equal before the law and are entitled without any discrimination to the
equal protection of the law. In this respect, the law shall prohibit any discrimination and guarantee
to all persons equal and effective protection against discrimination on any ground such as race,
colour, sex, language, religion, political or other opinion, national or social origin, property, birth or
other status.


In this context, the principle of non-discrimination requires that laws of general application relating to
elections be applied equally to all persons, regardless of sexual orientation. Although sexual orientation is not
specifically enumerated as a status or ratio for discrimination in Article 26 of the ICCPR, the ICCPR Human Rights
Committee has opined that the reference to sex in Article 26 should be construed to include sexual
orientation.
[48]
Additionally, a variety of United Nations bodies have declared discrimination on the basis of sexual
orientation to be prohibited under various international agreements.
[49]


The UDHR provides:

Article 21.

(1) Everyone has the right to take part in the government of his country, directly or
through freely chosen representatives.

Likewise, the ICCPR states:

Article 25
Every citizen shall have the right and the opportunity, without any of the distinctions
mentioned in article 2 and without unreasonable restrictions:

(a) To take part in the conduct of public affairs, directly or through freely chosen
representatives;

(b) To vote and to be elected at genuine periodic elections which shall be by universal
and equal suffrage and shall be held by secret ballot, guaranteeing the free expression of the will
of the electors;

(c) To have access, on general terms of equality, to public service in his country.


As stated by the CHR in its Comment-in-Intervention, the scope of the right to electoral participation is
elaborated by the Human Rights Committee in its General Comment No. 25 (Participation in Public Affairs and the
Right to Vote) as follows:

1. Article 25 of the Covenant recognizes and protects the right of every citizen to take
part in the conduct of public affairs, the right to vote and to be elected and the right to have access
to public service. Whatever form of constitution or government is in force, the Covenant requires
States to adopt such legislative and other measures as may be necessary to ensure that citizens
have an effective opportunity to enjoy the rights it protects. Article 25 lies at the core of democratic
government based on the consent of the people and in conformity with the principles of the
Covenant.

x x x x

15. The effective implementation of the right and the opportunity to stand for elective
office ensures that persons entitled to vote have a free choice of candidates. Any restrictions on
the right to stand for election, such as minimum age, must be justifiable on objective and
reasonable criteria. Persons who are otherwise eligible to stand for election should not be
excluded by unreasonable or discriminatory requirements such as education, residence or
descent, or by reason of political affiliation. No person should suffer discrimination or disadvantage
of any kind because of that person's candidacy. States parties should indicate and explain the
legislative provisions which exclude any group or category of persons from elective office.
[50]


We stress, however, that although this Court stands willing to assume the responsibility of giving effect to
the Philippines international law obligations, the blanket invocation of international law is not the panacea for all
social ills. We refer now to the petitioners invocation of the Yogyakarta Principles (the Application of International
Human Rights Law In Relation to Sexual Orientation and Gender Identity),
[51]
which petitioner declares to reflect
binding principles of international law.

At this time, we are not prepared to declare that these Yogyakarta Principles contain norms that are
obligatory on the Philippines. There are declarations and obligations outlined in said Principles which are not
reflective of the current state of international law, and do not find basis in any of the sources of international law
enumerated under Article 38(1) of the Statute of the International Court of Justice.
[52]
Petitioner has not undertaken
any objective and rigorous analysis of these alleged principles of international law to ascertain their true status.

We also hasten to add that not everything that society or a certain segment of society wants or
demands is automatically a human right. This is not an arbitrary human intervention that may be added to or
subtracted from at will. It is unfortunate that much of what passes for human rights today is a much broader context
of needs that identifies many social desires as rights in order to further claims that international law obliges states to
sanction these innovations. This has the effect of diluting real human rights, and is a result of the notion that if
wants are couched in rights language, then they are no longer controversial.

Using even the most liberal of lenses, these Yogyakarta Principles, consisting of a declaration formulated
by various international law professors, are at best de lege ferenda and do not constitute binding obligations on
the Philippines. Indeed, so much of contemporary international law is characterized by the soft law
nomenclature, i.e., international law is full of principles that promote international cooperation, harmony, and respect
for human rights, most of which amount to no more than well-meaning desires, without the support of either State
practice or opinio juris.
[53]


As a final note, we cannot help but observe that the social issues presented by this case are emotionally
charged, societal attitudes are in flux, even the psychiatric and religious communities are divided in opinion. This
Courts role is not to impose its own view of acceptable behavior. Rather, it is to apply the Constitution and laws as
best as it can, uninfluenced by public opinion, and confident in the knowledge that our democracy is resilient enough
to withstand vigorous debate.

WHEREFORE, the Petition is hereby GRANTED. The Resolutions of the Commission on Elections
dated November 11, 2009 and December 16, 2009 in SPP No. 09-228 (PL) are hereby SET ASIDE. The
Commission on Elections is directed to GRANT petitioners application for party-list accreditation.
SO ORDERED.



MARIANO C. DEL CASTILLO
Associate Justice









Republic of the Philippines
SUPREME COURT
Manila
EN BANC
G.R. No. L-23482 August 30, 1968
ALFONSO LACSON, petitioner,
vs.
CARMEN SAN JOSE-LACSON and THE COURT OF APPEALS, respondents.
-----------------------------
G.R. No. L-23767 August 30, 1968
CARMEN SAN JOSE-LACSON, plaintiff-appellant,
vs.
ALFONSO LACSON, defendant-appellee.
-----------------------------
G.R. No. L-24259 August 30, 1968
ALFONSO LACSON, petitioner-appellee,
vs.
CARMEN SAN JOSE-LACSON, petitioner-appellant.
Paredes, Poblador, Cruz and Nazareno for respondent-appellant Carmen San Jose-Lacson.
Norberto Quisumbing for petitioner-appellee Alfonso Lacson.
CASTRO, J .:
These three cases (G.R. L-23482, L-23767 and L-24259) involving the same parties pose a common
fundamental issue the resolution of which will necessarily and inescapably resolve all the other
issues. Thus their joinder in this decision.
The antecedent facts are not disputed.
Alfonso Lacson (hereinafter referred to as the petitioner spouse) and Carmen San Jose-Lacson
(hereinafter referred to as the respondent spouse) were married on February 14, 1953. To them
were born four children, all alive.
On January 9, 1963 the respondent spouse left the conjugal home in Santa Clara Subdivision,
Bacolod City, and commenced to reside in Manila. She filed on March 12, 1963 a complaint
docketed as civil case E-00030 in the Juvenile and Domestic Relations Court of Manila (hereinafter
referred to as the JDRC) for custody of all their children as well as support for them and herself.
However, the spouses, thru the assistance of their respective attorneys, succeeded in reaching an
amicable settlement respecting custody of the children, support, and separation of property. On April
27, 1963 they filed a joint petition dated April 21, 1963, docketed as special proceeding 6978 of the
Court of First Instance of Negros Occidental (hereinafter referred to as the CFI).
The important and pertinent portions of the petition, embodying their amicable settlement, read as
follows:
3. Petitioners have separated last January 9, 1963 when petitioner Carmen San Jose-
Lacson left their conjugal home at the Santa Clara Subdivision, Bacolod City, did not return,
and decided to reside in Manila.
4. Petitioners have mutually agreed upon the dissolution of their conjugal partnership subject
to judicial approval as required by Article 191 of the Civil Code of the Philippines the
particular terms and conditions of their mutual agreement being as follows:
(a) There will be separation of property petitioner Carmen San Jose-Lacson
hereby waiving any and all claims for a share in property that may be held by
petitioner Alfonso Lacson since they have acquired no property of any consequence.
(b) Hereafter, each of them shall own, dispose of, possess, administer and enjoy
such separate estate as they may acquire without the consent of the other and all
earnings from any profession, business or industry as may be derived by each
petitioner shall belong to that petitioner exclusively.
(c) The custody of the two elder children named Enrique and Maria Teresa shall be
awarded to petitioner Alfonso Lacson and the custody of the younger children named
Gerrard and Ramon shall be awarded to petitioner Carmen San Jose-Lacson.
(d) Petitioner Alfonso Lacson shall pay petitioner Carmen San Jose-Lacson a
monthly allowance of P300.00 for the support of the children in her custody.
(e) Each petitioner shall have reciprocal rights of visitation of the children in the
custody of the other at their respective residences and, during the summer months,
the two children in the custody of each petitioner shall be given to the other except
that, for this year's summer months, all four children shall be delivered to and remain
with petitioner Carmen San Jose-Lacson until June 15, 1963 on which date, she
shall return the two elder children Enrique and Maria Teresa to petitioner Alfonso
Lacson this judgment of course being subject to enforcement by execution writ
and contempt.
5. Petitioners have no creditors.
WHEREFORE, they respectfully pray that notice of this petition be given to creditors and
third parties pursuant to Article 191 of the Civil Code of the Philippines and thereafter that the
Court enter its judicial approval of the foregoing agreement for the dissolution of their
conjugal partnership and for separation of property, except that the Court shall immediately
approve the terms set out in paragraph 4 above and embody the same in a judgment
immediately binding on the parties hereto to the end that any non-compliance or violation of
its terms by one party shall entitle the other to enforcement by execution writ and contempt
even though the proceedings as to creditors have not been terminated.".
Finding the foregoing joint petition to be "conformable to law," the CFI (Judge Jose F. Fernandez,
presiding) issued an order on April 27, 1963, rendering judgment (hereinafter referred to as the
compromise judgment) approving and incorporating in toto their compromise agreement. In
compliance with paragraph 4 (e) of their mutual agreement (par. 3[e] of the compromise judgment),
the petitioner spouse delivered all the four children to the respondent spouse and remitted money for
their support.
On May 7, 1963 the respondent spouse filed in the JDRC a motion wherein she alleged that she
"entered into and signed the ... Joint Petition as the only means by which she could have immediate
custody of the ... minor children who are all below the age of 7," and thereafter prayed that she "be
considered relieved of the ... agreement pertaining to the custody and visitation of her minor children
... and that since all the children are now in her custody, the said custody in her favor be
confirmed pendente lite." On May 24, 1963 the petitioner spouse opposed the said motion and
moved to dismiss the complaint based, among other things, on the grounds of res judicata and lis
pendens. The JDRC on May 28, 1963, issued an order which sustained the petitioner spouse's plea
of bar by prior judgment and lis pendens, and dismissed the case. After the denial of her motion for
reconsideration, the respondent spouse interposed an appeal to the Court of Appeals (CA-G.R. No.
32608-R) wherein she raised, among others, the issue of validity or legality of the compromise
agreement in connection only with the custody of their minor children. On October 14, 1964 the
Court of Appeals certified the said appeal to the Supreme Court (G.R. No. L-23767), since "no
hearing on the facts was ever held in the court below no evidence, testimonial or documentary,
presented only a question of law pends resolution in the appeal." .
The respondent spouse likewise filed a motion dated May 15, 1963 for reconsideration of the
compromise judgment dated April 27, 1963 rendered in special proceeding 6978 of the CFI, wherein
she also alleged, among others, that she entered into the joint petition as the only means by which
she could have immediate custody of her minor children, and thereafter prayed the CFI to reconsider
its judgment pertaining to the custody and visitation of her minor children and to relieve her from the
said agreement. The petitioner spouse opposed the said motion and, on June 1, 1963, filed a motion
for execution of the compromise judgment and a charge for contempt. The CFI (Judge Jose R.
Querubin, presiding), in its order dated June 22, 1963, denied the respondent spouse's motion for
reconsideration, granted the petitioner spouse's motion for execution, and ordered that upon "failure
on the part of Carmen San Jose-Lacson to deliver the said children [i.e., to return the two older
children Enrique and Maria Teresa in accordance with her agreement with Alfonso Lacson] to the
special sheriff on or before June 29, 1963, she may be held for contempt pursuant to the provisions
of Rule 39 sections 9 and 10, and Rule 64 section 7 of the (old) Rules of Court." From the aforesaid
compromise judgment dated April 27, 1963 and execution order dated June 22, 1963, the
respondent spouse interposed an appeal to the Court of Appeals (CA-G.R. No. 32798-R) wherein
she likewise questioned the validity or legality of her agreement with the petitioner spouse respecting
custody of their children. On February 11, 1965 the Court of Appeals also certified the said appeal to
the Supreme Court (G.R. No. L-24259), since "no evidence of any kind was introduced before the
trial court and ... appellant did not specifically ask to be allowed to present evidence on her behalf." .
The respondent spouse also instituted certiorari proceedings before the Court of Appeals (CA-G.R.
No. 32384R), now the subject of an appeal by certiorari to this Court (G.R. No. L-23482). In her
petition for certiorari dated June 27, 1963, she averred that the CFI (thru Judge Querubin) committed
grave abuse of discretion and acted in excess of jurisdiction in ordering the immediate execution of
the compromise judgment in its order of June 22, 1963, thus in effect depriving her of the right to
appeal. She prayed for (1) the issuance of a writ of preliminary injunction enjoining the respondents
therein and any person acting under them from enforcing, by contempt proceedings and other
means, the writ of execution issued pursuant to the order of the respondent Judge Querubin dated
June 22, 1963 in special proceeding 6978 of the CFI, (2) the setting aside, after hearing, of the
compromise judgment dated April 27, 1963 and the order dated June 22, 1963, and (3) the awarding
of the custody of Enrique and Maria Teresa to her, their mother. As prayed for, the Court of Appeals
issued ex parte a writ of preliminary injunction enjoining the enforcement of the order dated June 22,
1963 for execution of the compromise judgment rendered in special proceeding 6978. The petitioner
spouse filed an urgent motion dated July 5, 1963 for the dissolution of the writ of preliminary
injunction ex parte which urgent motion was denied by the Court of Appeals in its resolution dated
July 9, 1963. The petitioner spouse likewise filed his answer. After hearing, the Court of Appeals on
May 11, 1964 promulgated in said certiorari case (CA-G.R. No. 32384-R) its decision granting the
petition for certiorari and declaring null and void both (a) the compromise judgment dated April 27,
1963 in so far as it relates to the custody and right of visitation over the two children, Enrique and
Teresa, and (b) the order dated June 22, 1963 for execution of said judgment. The petitioner spouse
moved to reconsider, but his motion for reconsideration was denied by the Court of Appeals in its
resolution dated July 31, 1964. From the decision dated May 11, 1964 and the resolution dated July
31, 1964, the petitioner spouse interposed an appeal to this Court, as abovestated, and assigned the
following errors:
(1) The Court of Appeals erred in annulling thru certiorari the lower court's order of execution
of the compromise judgment.
(2) The Court of Appeals erred in resolving in the certiorari case the issue of the legality of
the compromise judgment which is involved in two appeals, instead of the issue of grave
abuse of discretion in ordering its execution.
(3) The Court of Appeals erred in ruling that the compromise agreement upon which the
judgment is based violates article 363 of the Civil Code. 1 wph 1. t
As heretofore adverted, the aforecited three appeals converge on one focal issue: whether the
compromise agreement entered into by the parties and the judgment of the CFI grounded on the
said agreement, are conformable to law.
We hold that the compromise agreement and the judgment of the CFI grounded on the said
agreement are valid with respect to the separation of property of the spouses and the dissolution of
the conjugal partnership.
The law allows separation of property of the spouses and the dissolution of their conjugal
partnership provided judicial sanction is secured beforehand. Thus the new Civil Code provides:
In the absence of an express declaration in the marriage settlements, the separation of
property between spouses during the marriage shall not take place save in virtue of a judicial
order. (Art. 190, emphasis supplied)
The husband and the wife may agree upon the dissolution of the conjugal partnership during
the marriage, subject to judicial approval. All the creditors of the husband and of the wife, as
well as of the conjugal partnership, shall be notified of any petition for judicial approval of the
voluntary dissolution of the conjugal partnership, so that any such creditors may appear at
the hearing to safeguard his interests. Upon approval of the petition for dissolution of the
conjugal partnership, the court shall take such measures as may protect the creditors and
other third persons. (Art. 191, par. 4, emphasis supplied).
In the case at bar, the spouses obtained judicial imprimatur of their separation of property and the
dissolution of their conjugal partnership. It does not appeal that they have creditors who will be
prejudiced by the said arrangements.
It is likewise undisputed that the couple have been separated in fact for at least five years - the wife's
residence being in Manila, and the husband's in the conjugal home in Bacolod City. Therefore,
inasmuch as a lengthy separation has supervened between them, the propriety of severing their
financial and proprietary interests is manifest.
Besides, this Court cannot constrain the spouses to live together, as
[I]t is not within the province of the courts of this country to attempt to compel one of the
spouses to cohabit with, and render conjugal rights to, the other. .. At best such an order can
be effective for no other purpose than to compel the spouse to live under the same roof; and
the experience of those countries where the courts of justice have assumed to compel the
cohabitation of married couple shows that the policy of the practice is extremely
questionable. (Arroyo v. Vasquez de Arroyo, 42 Phil. 54, 60).
However, in so approving the regime of separation of property of the spouses and the dissolution of
their conjugal partnership, this Court does not thereby accord recognition to nor legalize the de facto
separation of the spouses, which again in the language of Arroyo v. Vasquez de Arroyo, supra is
a "state which is abnormal and fraught with grave danger to all concerned." We would like to douse
the momentary seething emotions of couples who, at the slightest ruffling of domestic tranquility
brought about by "mere austerity of temper, petulance of manners, rudeness of language, a want of
civil attention and accommodation, even occasional sallies of passion" without more would be
minded to separate from each other. In this jurisdiction, the husband and the wife are obliged to live
together, observe mutual respect and fidelity, and render mutual help and support (art. 109, new
Civil Code). There is, therefore, virtue in making it as difficult as possible for married couples
impelled by no better cause than their whims and caprices to abandon each other's company.
'... For though in particular cases the repugnance of the law to dissolve the obligations of
matrimonial cohabitation may operate with great severity upon individuals, yet it must be
carefully remembered that the general happiness of the married life is secured by its
indissolubility. When people understand that they must live together, except for a very few
reasons known to the law, they learn to soften by mutual accommodation that yoke which
they know they cannot shake off; they become good husbands and good wives from the
necessity of remaining husbands and wives; for necessity is a powerful master in teaching
the duties which it imposes ..." (Evans vs. Evans, 1 Hag. Con., 35; 161 Eng. Reprint, 466,
467.) (Arroyo vs. Vasquez de Arroyo, Id., pp. 58-59).
We now come to the question of the custody and support of the children.
It is not disputed that it was the JDRC which first acquired jurisdiction over the matter of custody and
support of the children. The complaint docketed as civil case E-00030 in the JDRC was filed by the
respondent spouse on March 12, 1963, whereas the joint petition of the parties docketed as special
proceeding 6978 in the CFI was filed on April 27, 1963. However, when the respondent spouse
signed the joint petition on the same matter of custody and support of the children and filed the same
with the CFI of Negros Occidental, she in effect abandoned her action in the JDRC. The petitioner
spouse who could have raised the issue of lis pendens in abatement of the case filed in the CFI,
but did not do so - had the right, therefore, to cite the decision of the CFI and to ask for the dismissal
of the action filed by the respondent spouse in the JDRC, on the grounds of res judicata and lis
pendens. And the JDRC acted correctly and justifiably in dismissing the case for custody and
support of the children based on those grounds. For it is no defense against the dismissal of the
action that the case before the CFI was filed later than the action before the JDRC, considering:.
... [T]hat the Rules do not require as a ground for dismissal of a complaint that there is a prior
pending action. They provide only that there is a pending action, not a pending prior action.
1

We agree with the Court of Appeals, however, that the CFI erred in depriving the mother, the
respondent spouse, of the custody of the two older children (both then below the age of 7).
The Civil Code specifically commands in the second sentence of its article 363 that "No mother shall
be separated from her child under seven years of age, unless the court finds compelling reasons for
such measure." The rationale of this new provision was explained by the Code Commission thus:
The general rule is recommended in order to avoid many a tragedy where a mother has seen
her baby torn away from her. No man can sound the deep sorrows of a mother who is
deprived of her child of tender age. The exception allowed by the rule has to be for
"compelling reasons" for the good of the child: those cases must indeed be rare, if the
mother's heart is not to be unduly hurt. If she has erred, as in cases of adultery, the penalty
of imprisonment and the (relative) divorce decree will ordinarily be sufficient punishment for
her. Moreover, her moral dereliction will not have any effect upon the baby who is as yet
unable to understand the situation." (Report of the Code Commission, p. 12).
The use of the word shall
2
in article 363 of the Civil Code, coupled with the observations made by the
Code Commission in respect to the said legal provision, underscores its mandatory character. It
prohibits in no uncertain: terms the separation of a mother and her child below seven years, unless
such separation is grounded upon compelling reasons as determined by a court.
The order dated April 27, 1963 of the CFI, in so far as it awarded custody of the two older children
who were 6 and 5 years old, respectively, to the father, in effect sought to separate them from their
mother. To that extent therefore, it was null and void because clearly violative of article 363 of the
Civil Code.
Neither does the said award of custody fall within the exception because the record is bereft of
any compelling reason to support the lower court's order depriving the wife of her minor children's
company. True, the CFI stated in its order dated June 22, 1963, denying the respondent spouse's
motion for reconsideration of its order dated April 27, 1963, that .
... If the parties have agreed to file a joint petition, it was because they wanted to avoid the
exposure of the bitter truths which serve as succulent morsel for scandal mongers and idle
gossipers and to save their children from embarrassment and inferiority complex which may
inevitably stain their lives. ..
If the parties agreed to submit the matter of custody of the minor children to the Court for
incorporation in the final judgment, they purposely suppressed the "compelling reasons for such
measure" from appearing in the public records. This is for the sake and for the welfare of the minor
children.".
But the foregoing statement is at best a mere hint that there were compelling reasons. The lower
court's order is eloquently silent on what these compelling reasons are. Needless to state, courts
cannot proceed on mere insinuations; they must be confronted with facts before they can properly
adjudicate.
It might be argued and correctly that since five years have elapsed since the filing of these
cases in 1963, the ages of the four children should now be as follows: Enrique 11, Maria Teresa
10, Gerrard 9, and Ramon 5. Therefore, the issue regarding the award of the custody of
Enrique and Maria Teresa to the petitioner spouse has become moot and academic. The passage of
time has removed the prop which supports the respondent spouse's position.
Nonetheless, this Court is loath to uphold the couple's agreement regarding the custody of the
children. 1wp h1. t
Article 356 of the new Civil Code provides:
Every child:
(1) Is entitled to parental care;
(2) Shall receive at least elementary education;
(3) Shall be given moral and civic training by the parents or guardian;
(4) Has a right to live in an atmosphere conducive to his physical, moral and
intellectual development.
It is clear that the abovequoted legal provision grants to every child rights which are not and should
not be dependent solely on the wishes, much less the whims and caprices, of his parents. His
welfare should not be subject to the parents' say-so or mutual agreement alone. Where, as in this
case, the parents are already separated in fact, the courts must step in to determine in whose
custody the child can better be assured the right granted to him by law. The need, therefore, to
present evidence regarding this matter, becomes imperative. A careful scrutiny of the records
reveals that no such evidence was introduced in the CFI. This latter court relied merely on the
mutual agreement of the spouses-parents. To be sure, this was not a sufficient basis to determine
the fitness of each parent to be the custodian of the children.
Besides, at least one of the children Enrique, the eldest is now eleven years of age and should
be given the choice of the parent he wishes to live with. This is the clear mandate of sec. 6, Rule 99
of the Rules of Court which, states, inter alia:
... When husband and wife are divorced or living separately and apart from each other, and
the question as to the care, custody, and control of a child or children of their marriage is
brought before a Court of First Instance by petition or as an incident to any other proceeding,
the court, upon hearing testimony as may be pertinent, shall award the care, custody and
control of each such child as will be for its best interestpermitting the child to choose which
parent it prefers to live with if it be over ten years of age, unless the parent so chosen be
unfit to take charge of the child by reason of moral depravity, habitual drunkenness,
incapacity, or poverty... (Emphasis supplied).
One last point regarding the matter of support for the children assuming that the custody of any or
more of the children will be finally awarded to the mother. Although the spouses have agreed upon
the monthly support of P150 to be given by the petitioner spouse for each child, still this Court must
speak out its mind on the insufficiency of this amount. We, take judicial notice of the devaluation of
the peso in 1962 and the steady skyrocketing of prices of all commodities, goods, and services, not
to mention the fact that all the children are already of school age. We believe, therefore, that the CFI
may increase this amount of P150 according to the needs of each child.
With the view that we take of this case, we find it unnecessary to pass upon the other errors
assigned in the three appeals.
ACCORDINGLY, the decision dated May 11, 1964 and the resolution dated July 31, 1964 of the
Court of Appeals in CA-G.R. 32384-R (subject matter of G.R. L-23482), and the orders dated May
28, 1963 and June 24, 1963 of the Juvenile and Domestic Relations Court (subject matter of G.R. L-
23767) are affirmed. G.R. L-24259 is hereby remanded to the Court of First Instance of Negros
Occidental for further proceedings, in accordance with this decision. No pronouncement as to costs.
Concepcion, C.J., Reyes, J.B.L., Dizon, Makalintal, Zaldivar and Angeles, JJ., concur.
Sanchez and Fernando, JJ., took no part.





















Republic of the Philippines
SUPREME COURT
Manila
EN BANC
G.R. No. L-23678 June 6, 1967
TESTATE ESTATE OF AMOS G. BELLIS, deceased.
PEOPLE'S BANK and TRUST COMPANY, executor.
MARIA CRISTINA BELLIS and MIRIAM PALMA BELLIS, oppositors-appellants,
vs.
EDWARD A. BELLIS, ET AL., heirs-appellees.
Vicente R. Macasaet and Jose D. Villena for oppositors appellants.
Paredes, Poblador, Cruz and Nazareno for heirs-appellees E. A. Bellis, et al.
Quijano and Arroyo for heirs-appellees W. S. Bellis, et al.
J. R. Balonkita for appellee People's Bank & Trust Company.
Ozaeta, Gibbs and Ozaeta for appellee A. B. Allsman.
BENGZON, J.P., J .:
This is a direct appeal to Us, upon a question purely of law, from an order of the Court of First
Instance of Manila dated April 30, 1964, approving the project of partition filed by the executor in
Civil Case No. 37089 therein.1 wph 1. t
The facts of the case are as follows:
Amos G. Bellis, born in Texas, was "a citizen of the State of Texas and of the United States." By his
first wife, Mary E. Mallen, whom he divorced, he had five legitimate children: Edward A. Bellis,
George Bellis (who pre-deceased him in infancy), Henry A. Bellis, Alexander Bellis and Anna Bellis
Allsman; by his second wife, Violet Kennedy, who survived him, he had three legitimate children:
Edwin G. Bellis, Walter S. Bellis and Dorothy Bellis; and finally, he had three illegitimate children:
Amos Bellis, Jr., Maria Cristina Bellis and Miriam Palma Bellis.
On August 5, 1952, Amos G. Bellis executed a will in the Philippines, in which he directed that after
all taxes, obligations, and expenses of administration are paid for, his distributable estate should be
divided, in trust, in the following order and manner: (a) $240,000.00 to his first wife, Mary E. Mallen;
(b) P120,000.00 to his three illegitimate children, Amos Bellis, Jr., Maria Cristina Bellis, Miriam
Palma Bellis, or P40,000.00 each and (c) after the foregoing two items have been satisfied, the
remainder shall go to his seven surviving children by his first and second wives, namely: Edward A.
Bellis, Henry A. Bellis, Alexander Bellis and Anna Bellis Allsman, Edwin G. Bellis, Walter S. Bellis,
and Dorothy E. Bellis, in equal shares.1w ph1 .t
Subsequently, or on July 8, 1958, Amos G. Bellis died a resident of San Antonio, Texas, U.S.A. His
will was admitted to probate in the Court of First Instance of Manila on September 15, 1958.
The People's Bank and Trust Company, as executor of the will, paid all the bequests therein
including the amount of $240,000.00 in the form of shares of stock to Mary E. Mallen and to the
three (3) illegitimate children, Amos Bellis, Jr., Maria Cristina Bellis and Miriam Palma Bellis, various
amounts totalling P40,000.00 each in satisfaction of their respective legacies, or a total of
P120,000.00, which it released from time to time according as the lower court approved and allowed
the various motions or petitions filed by the latter three requesting partial advances on account of
their respective legacies.
On January 8, 1964, preparatory to closing its administration, the executor submitted and filed its
"Executor's Final Account, Report of Administration and Project of Partition" wherein it reported, inter
alia, the satisfaction of the legacy of Mary E. Mallen by the delivery to her of shares of stock
amounting to $240,000.00, and the legacies of Amos Bellis, Jr., Maria Cristina Bellis and Miriam
Palma Bellis in the amount of P40,000.00 each or a total of P120,000.00. In the project of partition,
the executor pursuant to the "Twelfth" clause of the testator's Last Will and Testament divided
the residuary estate into seven equal portions for the benefit of the testator's seven legitimate
children by his first and second marriages.
On January 17, 1964, Maria Cristina Bellis and Miriam Palma Bellis filed their respective oppositions
to the project of partition on the ground that they were deprived of their legitimes as illegitimate
children and, therefore, compulsory heirs of the deceased.
Amos Bellis, Jr. interposed no opposition despite notice to him, proof of service of which is
evidenced by the registry receipt submitted on April 27, 1964 by the executor.
1

After the parties filed their respective memoranda and other pertinent pleadings, the lower court, on
April 30, 1964, issued an order overruling the oppositions and approving the executor's final account,
report and administration and project of partition. Relying upon Art. 16 of the Civil Code, it applied
the national law of the decedent, which in this case is Texas law, which did not provide for legitimes.
Their respective motions for reconsideration having been denied by the lower court on June 11,
1964, oppositors-appellants appealed to this Court to raise the issue of which law must apply
Texas law or Philippine law.
In this regard, the parties do not submit the case on, nor even discuss, the doctrine of renvoi, applied
by this Court in Aznar v. Christensen Garcia, L-16749, January 31, 1963. Said doctrine is usually
pertinent where the decedent is a national of one country, and a domicile of another. In the present
case, it is not disputed that the decedent was both a national of Texas and a domicile thereof at the
time of his death.
2
So that even assuming Texas has a conflict of law rule providing that the
domiciliary system (law of the domicile) should govern, the same would not result in a reference
back (renvoi) to Philippine law, but would still refer to Texas law. Nonetheless, if Texas has a
conflicts rule adopting the situs theory (lex rei sitae) calling for the application of the law of the place
where the properties are situated, renvoi would arise, since the properties here involved are found in
the Philippines. In the absence, however, of proof as to the conflict of law rule of Texas, it should not
be presumed different from ours.
3
Appellants' position is therefore not rested on the doctrine of
renvoi. As stated, they never invoked nor even mentioned it in their arguments. Rather, they argue
that their case falls under the circumstances mentioned in the third paragraph of Article 17 in relation
to Article 16 of the Civil Code.
Article 16, par. 2, and Art. 1039 of the Civil Code, render applicable the national law of the decedent,
in intestate or testamentary successions, with regard to four items: (a) the order of succession; (b)
the amount of successional rights; (e) the intrinsic validity of the provisions of the will; and (d) the
capacity to succeed. They provide that
ART. 16. Real property as well as personal property is subject to the law of the country
where it is situated.
However, intestate and testamentary successions, both with respect to the order of
succession and to the amount of successional rights and to the intrinsic validity of
testamentary provisions, shall be regulated by the national law of the person whose
succession is under consideration, whatever may he the nature of the property and
regardless of the country wherein said property may be found.
ART. 1039. Capacity to succeed is governed by the law of the nation of the decedent.
Appellants would however counter that Art. 17, paragraph three, of the Civil Code, stating that
Prohibitive laws concerning persons, their acts or property, and those which have for their
object public order, public policy and good customs shall not be rendered ineffective by laws
or judgments promulgated, or by determinations or conventions agreed upon in a foreign
country.
prevails as the exception to Art. 16, par. 2 of the Civil Code afore-quoted. This is not correct.
Precisely, Congressdeleted the phrase, "notwithstanding the provisions of this and the next
preceding article" when they incorporated Art. 11 of the old Civil Code as Art. 17 of the new Civil
Code, while reproducing without substantial change the second paragraph of Art. 10 of the old Civil
Code as Art. 16 in the new. It must have been their purpose to make the second paragraph of Art.
16 a specific provision in itself which must be applied in testate and intestate succession. As further
indication of this legislative intent, Congress added a new provision, under Art. 1039, which decrees
that capacity to succeed is to be governed by the national law of the decedent.
It is therefore evident that whatever public policy or good customs may be involved in our System of
legitimes, Congress has not intended to extend the same to the succession of foreign nationals. For
it has specifically chosen to leave, inter alia, the amount of successional rights, to the decedent's
national law. Specific provisions must prevail over general ones.
Appellants would also point out that the decedent executed two wills one to govern his Texas
estate and the other his Philippine estate arguing from this that he intended Philippine law to
govern his Philippine estate. Assuming that such was the decedent's intention in executing a
separate Philippine will, it would not alter the law, for as this Court ruled in Miciano v. Brimo, 50 Phil.
867, 870, a provision in a foreigner's will to the effect that his properties shall be distributed in
accordance with Philippine law and not with his national law, is illegal and void, for his national law
cannot be ignored in regard to those matters that Article 10 now Article 16 of the Civil Code
states said national law should govern.
The parties admit that the decedent, Amos G. Bellis, was a citizen of the State of Texas, U.S.A., and
that under the laws of Texas, there are no forced heirs or legitimes. Accordingly, since the intrinsic
validity of the provision of the will and the amount of successional rights are to be determined under
Texas law, the Philippine law on legitimes cannot be applied to the testacy of Amos G. Bellis.
Wherefore, the order of the probate court is hereby affirmed in toto, with costs against appellants. So
ordered.
Concepcion, C.J., Reyes, J.B.L., Dizon, Regala, Makalintal, Zaldivar, Sanchez and Castro, JJ.,
concur.

You might also like